Download as pdf or txt
Download as pdf or txt
You are on page 1of 517

Taras Shevchenko National University of Kyiv

The Institute of Biology and Medicine

THE TEST BASE


for the preparation of the exam
on discipline “Biological and bioorganic chemistry”
(2nd semester)
for the studens of a 2nd course
with English of educating

Compilers – the candidate of biological sciences,


the associate professor
Synelnyk T.B.
the candidate of biological sciences,
the assistant professor
Konopelniuk V.V.

Readers:

It is ratified to printing by meeting of scientific advice of The Institute of Biology


and Medicine (“____”________________ 2018, protocol №____)

Kyiv-2018
2
1. Cyanide is a poison that causes instant death of the organism. What
enzymes found in mitochondria are affected by cyanide?
A. Flavin enzymes
B. Cytochrome oxidase (aa3)
C. Cytochrome b5
D. NAD-dependent dehydrogenase
E. Cytochrome P-450

2. It has been found out that one of a pesticide components is sodium


arsenate that blocks lipoic acid. Which enzyme activity is impaired by
this pesticide?
A. Pyruvate dehydrogenase complex
B. Microsomal oxidation
C. Methemoglobin reductase
D. Glutathione peroxidase
E. Glutathione reductase

3. The central intermediate which is common for the catabolic pathways


of proteins, carbohydrates and lipids is:
A. Succinyl-CoA.
B. Oxaloacetate.
C. Lactate.
D. Acetyl-CoA.
E. Citrate.

4. During the necropsy of a 20-year old girl a pathologist concluded that


the death of the patient had resulted from poisoning by cyanides. The
activity of what enzyme is mostly inhibited by cyanides?
A. Malate dehydrogenase.
B. Heme synthase.
C. Cytochrome oxidase.
D. Aspartate aminotransferase.
E. Carbamoyl phosphate synthetase.

5. Pain along large nervous stems and increased amount of pyruvate in


the blood were revealed in the patient. Insufficiency of which vitamin can
cause such change?
A. В2.
B. РР.
C. Pantothenic acid.
D. Biotin
E. В1.

6. A 9-month-old infant is fed with artificial formulas with unbalanced


vitamin B6 concentration. The infant presents with pellagral dermatitis,
3
convulsions, anaemia. Convulsion development might be caused by the
disturbed formation of:
A. Histamine.
B. Serotonin.
C. DOPA.
D. GABA.
E. Dopamine.

7. A woman who has been keeping to a clean-rice diet for a long time was
diagnosed with polyneuritis (beriberi). Which vitamin deficit results in
development of this disease?
A. Ascorbic acid.
B. Thiamine.
C. Pyridoxine.
D. Folic acid.
E. Riboflavin.

8. In case of enterobiasis acrihine - the structural analogue of vitamin B2


- is administered. The disorder of the synthesis of which enzymes does
this medicine cause in microorganisms?
A. Cytochrome oxidases.
B. Peptidases.
C. NAD-dependet dehydrogenases.
D. FAD-dependent dehydrogenases.
E. Aminotransferases.

9. A 10-year-old girl often experiences acute respiratory infections with


multiple spotty haemorrages in the places of clothes friction.
Hypovitaminosis of which vitamin is present at the girl?
A. В6.
B. В1.
C. С.
D. А.
E. В2.

10. Hydroxylation of endogenous substrates and xenobiotics requires a


donor of protons. Which of the following vitamins can play this role?
A. Vitamin P.
B. Vitamin B6.
C. Vitamin E.
D. Vitamin A.
E. Vitamin C.

11. Examination of a patient with frequent hemorrhages from internals


and mucous membranes revealed proline and lysine being a part of
4
collagen fibers. Which vitamin absence caused disturbance of their
hydroxylation?
A. Vitamin C.
B. Vitamin K.
C. Vitamin A.
D. Thiamine.
E. Vitamin E.

12. Examination of a patient suffering from frequent haemorrhages in the


inner organs and mucous membranes revealed proline and lysine being
included in collagen fibers. Impairment of their hydroxylation is caused
by lack of the following vitamin:
A. E.
B. K.
C. C.
D. A.
E. D.

13. A patient consulted a doctor about symmetric dermatitis of open skin


areas. It was found out that the patient lived mostly on cereals and ate
too little meat, milk and eggs. Which vitamin deficiency is the most
evident?
A. Calciferol.
B. Folic acid.
C. Biotin.
D. Nicotinamide.
E. Tocopherol.

14. Examination of a child who hasn't got fresh fruit and vegetables
during winter revealed numerous subcutaneous hemorrhages, gingivitis,
carious cavities in teeth. Which vitamin combination should be prescribed
in this case?
A. Thiamine and pyridoxine.
B. Folic acid and cobalamin.
C. Riboflavin and nicotinamide.
D. Calciferol and ascorbic acid.
E. Ascorbic acid and rutin.

15. Concentration of pyruvate is increased in the patient's blood, the


most of which is excreted with urine. Which avitaminosis is observed in
the patient?
A. Avitaminosis E.
B. Avitaminosis В1.
C. Avitaminosis В3.
D. Avitaminosis B6.
5
E. Avitaminosis В2.

16. While examining the child the doctor revealed symmetric cheeks
roughness, diarrhea, disfunction of the nervous system. Lack of which
food components caused it?
A. Lysine, ascorbic acid.
B. Threonine, pantothenic acid.
C. Methionine, lipoic acid.
D. Nicotinic acid, tryptophane
E. Phenylalanine, pangamic acid.

17. In clinical practice tuberculosis is treated with izoniazid preparation -


that is an antivitamin able to penetrate into the tuberculosis bacillus.
Tuberculostatic effect is induced by the interference with replication
processes and oxidation-reduction reactions due to the buildup of pseudo-
coenzyme:
A. FAD.
B. FMN.
C. NAD.
D. TDP.
E. CoQ.

18. A newborn child has convulsions that have been observed after
prescription of vitamin B6. This most probable cause of this effect is that
vitamin B6 is a component of the following enzyme:
A. Pyruvate dehydrogenase.
B. Glutamate decarboxylase.
C. Ketoglutarate dehydrogenase.
D. Aminolevulinate synthase.
E. Glycogen phosphorylase.

19. A 3 year old child with symptoms of stomatitis, gingivitis and


dermatitis of open skin areas was delivered to a hospital. Examination
revealed inherited disturbance of neutral amino acid transporting in the
bowels. These symptoms were caused by the deficiency of the following
vitamin:
A. Pantothenic acid.
B. Niacin.
C. Vitamin A.
D. Cobalamin.
E. Biotin.

20. Increased breaking of vessels, enamel and dentine destruction in


scurvy patients are caused by disorder of collagen maturing. Which stage
of modification of procollagen is disordered in this avitaminosis?
6
A. Formation of polypeptide chains.
B. Glycosylation of hydroxylysine residues.
C. Removal of C-ended peptide from procollagen.
D. Detaching of N-ended peptide.
E. Hydroxylation of proline.

21. Vitamin B1 deficiency results in disturbance of oxidative


decarboxylation of α-ketoglutaric acid. This will disturb synthesis of the
following coenzyme:
A. Thiamine pyrophosphate.
B. Nicotinamide adenine dinucleotide.
C. Flavine adenine dinucleotide.
D. Lipoic acid.
E. Coenzyme A.

22. Surgical removal of a part of stomach resulted in disturbed absorption


of vitamin B12, it is excreted with feces. The patient was diagnosed with
anemia. Which factor is necessary for absorption of this vitamin?
A. Gastrin.
B. Hydrochloric acid.
C. Pepsin.
D. Gastromucoprotein.
E. Folic acid.

23. Izoniazid preparation is used in clinical practice to treat tuberculosis.


Tuberculostatic effect is induced by the interference with replication
processes and oxidation-reduction reactions due to the buildup of pseudo-
coenzyme:
A. TMP.
B. NAD.
C. FMN.
D. THF.
E. CoQ.

24. Vitamin B1 deficiency results in disturbance of oxidative


decarboxylation of α-ketoglutaric acid. This will disturb synthesis of the
following coenzyme:
NAD.
FAD.
TPP.
THF.
CoA.

25. Most participants of Magellan expedition to America died from


avitaminosis. This disease declared itself by general weakness,
7
subcutaneous hemmorhages, falling of teeth, gingival hemmorhages. What
is the name of this avitaminosis?
A. Pellagra.
B. Rachitis.
C. Scurvy.
D. Polyneuritis (beriberi).
E. Biermer's anemia.

26. A patient with hypochromic anemia has splitting hair and loss of hair,
increased nail brittling and taste alteration. Which is the mechanism of
the development of these symptoms?
A. Decreased production of parathyrin.
B. Deficiency of vitamin А.
C. Deficiency of vitamin В12.
D. Decreased production of thyroid hormones.
E. Deficiency of iron-containing enzymes.

27. A 3 year old child with symptoms of stomatitis, gingivitis and


dermatitis of open skin areas was delivered to a hospital. Examination
revealed inherited disturbance of neutral amino acid transporting in the
bowels. These symptoms were caused by the deficiency of the following
vitamin:
A. Niacin.
B. Cobalamin.
C. Vitamin A.
D. Biotin.
E. Pantothenic acid.

28. A clinic observes a 49 year old patient with significant prolongation of


coagulation time, gastrointestinal haemorrhages, subcutaneous
hematomas. These symptoms might be explained by the deficiency of the
following vitamin:
A. H.
B. B6.
C. B1.
D. K.
E. E

29. Blood test of a patient suffering from atrophic gastritis gave the
following results: RBCs - 2,0*1012/l, Hb- 87 g/l, color index - 1,3, WBCs -
4,0*109/l, thrombocytes – 180*109/l. Anemia might have been caused by
the following substance deficiency:
A. Vitamin K.
B. Vitamin B12.
C. Zinc.
8
D. Vitamin A.
E. Iron.

30. Removal of gall bladder of a patient has disturbed processes of Ca


absorption through the intestinal wall. Which vitamin will stimulate this
process?
A. D3.
B. PP.
C. C.
D. B12.
E. K.

31. Vitamin A together with specific cytoreceptors penetrates through the


nuclear membranes, induces transcription processes that stimulate
growth and differentiation of cells. This biological function is realized by
the following form of vitamin A:
A. Trans-retinal.
B. Cis-retinal.
C. Trans-retinoic acid.
D. Retinol.
E. Carotin.

32. In patients with the biliary tract obstruction the blood coagulation is
inhibited; the patients have frequent haemorrhages caused by the
subnormal assimilation of the following vitamin:
A. A.
B. D.
C. K.
D. E.
E. C.

33. Examination of a man who hadn't been consuming fats but had been
getting enough carbohydrates and proteins for a long time revealed
dermatitis, poor wound healing, vision impairment. What is the probable
cause of metabolic disorder?
A. Lack of palmitic acid.
B. Lack of linoleic acid, vitamins A, D, E, K.
C. Lack of vitamins PP, H.
D. Low caloric value of diet.
E. Lack of oleic acid.

34. To prevent postoperative bleeding a 6 y.o. child was administered


vicasol that is a synthetic analogue of vitamin K. Name post-translational
changes of blood coagulation factors that will be activated by vicasol:
A. Phosphorylation of serine radicals.
9
B. Partial proteolysis.
C. Carboxylation of glutamic acid.
D. Polymerization.
E. Glycosylation.

35. As a result of posttranslational modifications some proteins taking


part in blood coagulation, particularly prothrombin, become capable of
calcium binding. The following vitamin takes part in this process:
A. C.
B. A.
C. B1.
D. K.
E. B2.

36. Posttranslational modifications of some proteins taking part in blood


coagulation, lead to ability of calcium binding. Which vitamin takes part
in this process?
A. C.
B. A.
C. B1.
D. B2.
E. K.

37. During examination of an 11-month-old infant a pediatrician revealed


osteoectasia of the lower extremities and delayed mineralization of
cranial bones. Such pathology is usually provoked by the deficit of the
following vitamin:
A. Cholecalciferol.
B. Thiamine.
C. Pantothenic acid.
D. Bioflavonoids.
E. Riboflavin.

38. A doctor examined a child and revealed symptoms of rachitis.


Development of this disease was caused by deficiency of the following
compound:
A. Biotin.
B. 1,25 [ОН]-dihydroxycholecalciferol.
C. Tocopherol.
D. Naphtaquinone.
E. Retinol.

39. A patient who was previously ill with mastectomy as a result of breast
cancer was prescribed radiation therapy. Which vitamin preparation has
marked radioprotective action caused by antioxidant activity?
10
A. Thiamine chloride.
B. Ergocalciferol.
C. Tocopherol acetate.
D. Folic acid.
E. Riboflavin.

40. A patient suffers from vision impairment - hemeralopy (night


blindness). Which vitamin preparation should be administered the patient
in order to restore his vision?
A. Pyridoxine.
B. Thiamine chloride.
C. Vicasol.
D. Retinol acetate.
E. Tocopherol acetate.

41. A patient underwent an operation on account of gall bladder excision


that resulted in obstruction of Ca absorption through the bowels wall.
Which vitamin will stimulate this process?
A. B12.
B. K.
C. PP.
D. C.
E. D3.

42. A 64 year old woman has impairment of twilight vision (hemeralopy).


Which vitamin should be recommended in the first place?
A. Vitamin A.
B. Vitamin E.
C. Vitamin C.
D. Vitamin B6.
E. Vitamin B2.

43. The obstruction of Ca absorption through the bowels wall resulted


after the operation on account of gall bladder. What vitamin will stimulate
this process?
A. Cobalamine.
B. Niacine.
C. Folic acid.
D. Riboflavine.
E. Cholecalciferol.

44. A 2-year-old child has got intestinal dysbacteriosis, which resulted in


hemorrhagic syndrome. What is the most likely cause of hemorrhage of
the child?
A. Fibrinogen deficiency
11
B. Vitamin K deficiency
C. Hypocalcemia
D. Activation of tissue thromboplastin
E. PP hypovitaminosis

45. A girl of 10 years old frequently is ill with respiratory infections after
which plural hemorrhages in places of friction of clothes are observed.
Which hypovitaminosis takes place at the girl?
A. С
B. В6
C. В1
D. A
E. В2

46. At the woman of 35 years with chronic disease of kidneys the


osteoporosis has developed. Specify what deficiency from below listed
substances is a principal cause of this complication?
A. 25OH-D3
B. D3
C. D2
D. 1,25(OH)2D3
E. Cholesterol

47. Pediatrist has examined the baby after an epileptiform fit, which
receives artificial feeding. The baby has dermatitis also. At laboratory
inspection decrease alanine- and aspartate aminotransferases activities of
erythrocytes is established. Which vitamin deficiency can be assumed?
A. Ascorbic acid
B. Cobalamin
C. Riboflavin
D. Calciferol
E. Pyridoxine

48. To the pregnant woman, which had in the anamnesis some stillborn
foetus, the therapy is prescripted which contains vitamins. Indicate
vitamin, which promotes carrying of a pregnancy.
A. Folic acid.
B. Cyanocobalamin
C. Alfa-tocopherol.
D. Pyridoxal phosphate
E. Rutin.

49. Patients complained of the general weakness and a bleeding from gum.
Insufficiency of which vitamin can be assumed?
A. Vitamin Е
12
B. Vitamin РР
C. Vitamin D
D. Vitamin В1
E. Vitamin C

50. At inspection of the patient dermatitis, diarrhea, dementia are


revealed. The absence of which vitamin is the reason of this condition?
A. Ascorbic acid.
B. Folic acid.
C. Biotin.
D. Nicotinamide.
E. Rutin.

51. The patient has pellagra. At interrogation it became known, that for a
long time he ate mainly corn, not enough using meat. What became the
reason of pellagra?
A. Deficiency of tyrosine in corn.
B. Deficiency of proline in corn
C. Deficiency of tryptophan in corn
D. Deficiency of alanine in corn
E. Deficiency of histidine in corn

52. At the patient after removal of a gall-bladder process of Са intestinal


absorption through a wall of intestines is complicated. Which vitamin will
stimulate this process?
A. Vitamin D3
B. Vitamin РР
C. Vitamin C
D. Vitamin В12
E. Vitamin K

53. Institute of gerontology advises to people of old age to use a complex


of vitamins which contains vitamin Е. What main function it carries out?
A. Antihemorrhagic.
B. Antioxidant.
C. Antiscorbutic.
D. Antineuritis.
E. Antidermatitis.

54. Doctor-dietician advises the patient during treatment of the


pernicious anemia to use in a diet a half-baked liver. The presence of
which vitamin in this product promotes treatment of an anemia?
A. Vitamin В1
B. Vitamin В12.
C. Vitamin В2.
13
D. Vitamin C
E. Vitamin Н.

55. At the patient such changes are marked: infringement of sight in


twilight, dryness of the conjunctiva and a cornea. Such infringements can
be at shortage of:
A. Vitamin B
B. Vitamin C
C. Vitamin D
D. Vitamin A
E. Vitamin В12

56. After removal 2/3 of the stomach the amount of erythrocytes has
decreased in blood, the level of hemoglobin has decreased. What
deficiency of vitamin leads to such changes of a picture of blood?
A. C
B. Р
C. В12
D. В6
E. РР

57. To patient with recurrent thromboembolism, artificial anticoagulant


pelentan is appointed. What vitamin antagonist is it?
A. Vitamin Е
B. Vitamin K
C. Vitamin A
D. Vitamin D
E. Vitamin C

58. At the patient of 43 years with chronic atrophic gastritis,


megaloblastic anemia observes. Urinalysis shows increasing of
methylmalonic acid. Which type of hypovitaminosis occurrence of the
specified infringement caused?
A. Vitamin В2
B. Vitamin В3
C. Vitamin В5
D. Vitamin В12
E. Vitamin В1

59. For diagnostics of some diseases activity of the transaminases in the


blood is defined. Which vitamin is a cofactor part of these enzymes?
A. В2
B. В1
C. В8
D. В5
14
E. В6

60. Сertain infections caused by bacteria are treated with sulphanilamides


that block the synthesis of bacterial growth factor. What is the
mechanism of these drugs action?
A. They are antivitamins of p-aminobenzoic acid.
B. They inhibit the folic acid absorption
C. They are allosteric enzyme inhibitors
D. They are involved in redox processes
E. They are allosteric enzymes

61. Hepatic dysfunctions accompanied by insufficient inflow of bile to the


bowels result in coagulation failure. This phenomenon can be explained
by:
A. Iron deficiency
B. Vitamin K deficiency.
C. Thrombocytopenia
D. Erythropenia
E. Leukopenia

62. A patient has increased permeability of blood-vessel walls, increased


gingival hemorrhage, small punctate hematomas on his skin, falling of
teeth. What disturbance of vitamin metabolism can account for these
symptoms?
A. Hypervitaminosis D
B. Hypervitaminosis C
C. Hypovitaminosis D
D. Hypovitaminosis C.
E. Hypovitaminosis A

63. A patient has the following changes: disorder of twilight vision, drying
out of conjunctiva and cornea. Such disorders may be caused by
deficiency of vitamin:
A. Vitamin A.
B. Vitamin B
C. Vitamin C
D. Vitamin D
E. Vitamin B12

64. Preoperative examination revealed prothrombin deficiency in the


blood of the patient. What drug should be preliminarily prescribed to
mitigate blood loss in the patient during the surgery?
A. Thrombin
B. Aminocapronic acid
C. Vicasol (Menadione)
15
D. Phenylin (Phenindione)
E. Contrykal (Aprotinin)

65. A hereditary disease - homocystinuria - is caused by disturbed


transformation of homocysteine into methionine. Accumulated
homocysteine forms its dimer (homocystine) that can be found in urine.
What vitamin preparation can decrease homocysteine production?
A. Vitamin C
B. Vitamin B1
C. Vitamin B2
D. Vitamin B12
E. Vitamin PP

66. One of the causes of pernicious anemia is disturbed synthesis of


transcorrin - Castle’s intrinsic factor - by the parietal cells of the stomach.
Which substance is called Castle’s extrinsic factor?
A. Folic acid
B. Cobalamin
C. Pyridoxine
D. Riboflavin
E. Biotin

67. Researchers isolated 5 isoenzymic forms of lactate dehydrogenase


from the human blood serum and studied their properties. Which property
indicates that the isoenzymic forms were isolated from the same
enzyme?
A. The same molecular weight.
B. The same physico-chemical properties.
C. Tissue localization.
D. The same electrophoretic mobility.
E. Catalyzation of the same reaction.

68. Profuse foam appeared when dentist put hydrogen peroxide on the
mucous of the oral cavity. Which enzyme caused such activity?
A. Catalase.
B. Cholinesterase.
C. Acetyltransferase.
D. Glucose-6-phosphate dehydrogenase.
E. Methemoglobin reductase.

69. A 46-year-old female patient has a continuous history of progressive


muscular (Duchenne's) dystrophy. Which blood enzyme changes will be of
diagnostic value in this case?
A. Lactate dehydrogenase.
B. Creatine phosphokinase.
16
C. Pyruvate dehydrogenase.
D. Glutamate dehydrogenase.
E. Adenylate cyclase.

70. A patient presents high activity of LDH1,2, aspartate aminotransferase,


creatine phosphokinase. In which organ (organs) is the development of a
pathological process the most probable?
A. In the heart muscle (initial stage of myocardium infarction).
B. In skeletal muscles (dystrophy, atrophy).
C. In kidneys and adrenals.
D. In connective tissue.
E. In liver and kidneys.

71. Marked increase of activity of МВ-forms of CPK (creatine


phosphokinase) and LDH-1 were revealed on the examination of the
patient's blood. What is the most likely pathology?
A. Hepatitis.
B. Rheumatism.
C. Pncreatitis.
D. Cholecystitis.
E. Myocardium infarction.

72. 12 hours after an acute attack of retrosternal pain a patient presented


a jump of aspartate aminotransferase activity in blood serum. Which
pathology is this deviation typical for?
A. Viral hepatitis.
B. Myocardium infarction.
C. Collagenosis.
D. Diabetes mellitus.
E. Diabetes insipidus.

73. A 49-year-old driver complains about unbearable constricting pain


behind the breastbone irradiating to the neck. The pain arose 2 hours ago.
Objectively: the patient’s condition is grave, he is pale, heart tones are
decrease. Laboratory studies revealed high activity of creatine kinase and
LDH. Which disease are these symptoms typical for?
A. Acute pancreatitis.
B. Stenocardia.
C. Cholelithiasis.
D. Diabetes mellitus.
E. Acute myocardium infarction.

74. Desulfiram is widely used in medical practice to prevent alcoholism. It


inhibits aldehyde dehydrogenase. Increased level of which metabolite
causes aversion to alcohol?
17
A. Acetaldehyde.
B. Methanol.
C. Malonyl aldehyde.
D. Ethanol.
E. Propionic aldehyde.

75. There are several groups of molecular mechanisms playing important


part in pathogenesis of insult to cells which contributes to the pathology
development. Which process is stimulated by mechanisms of protein
damage?
A. Acidosis.
B. Phospholipase activation.
C. Enzyme inhibition.
D. Osmotic membrane distension.
E. Lipid peroxidation.

76. A 30-year-old male patient with acute pancreatitis has been found to
have a disorder of digestion of protein in the gut cavity. The reason for
such condition can be the hyposynthesis and hyposecretion of the
following enzyme:
A. Dipeptidase.
B. Amylase.
C. Trypsin.
D. Pepsin.
E. Lipase.

77. At the patient with an acute pancreatitis at the analysis of blood and
urine activity of one of the specified enzymes which confirms the
diagnosis of disease is sharply raised:
A. Pepsin
B. Dipeptidase
C. Saccharase
D. Alfa-amylase
E. Lactase

78. The patient has the diagnosis of beriberi. What enzyme activity is
broken at the patient?
A. Citrate synthase
B. Malate dehydrogenase
C. Succinate dehydrogenase
D. Fumarase
E. Pyruvate dehydrogenase

79. At the patient of 36 years old with chronic alcoholism, in blood


pyruvate accumulation is developed, in erythrocytes - decrease in activity
18
transketolase takes place. Name the coenzyme form of vitamin which
insufficiency the specified changes are caused?
A. Thiamine pyrophosphate
B. Carboxybiotin
C. Methylcobalamin
D. Pyridoxal phosphate
E. Tetrahydrofolate

80. Oral mucosa of a patient was treated with hydrogen peroxide. Instead
of foaming, the blood turned brown. That is possible in case of reduced
concentration of the following enzyme:
A. Pseudocholinesterase
B. Glucose-6-phosphate dehydrogenase
C. Catalase.
D. Acetyltransferase
E. Methemoglobin reductase

81. One of the means of regulating enzyme activity in a human body is


the covalent modification. Glycogen phosphorylase and glycogen
synthetase activity is regulated by the following type of covalent
modification:
A. ADP-ribosylation
B. Methylation
C. Phosphorylation-dephosphorylation.
D. Hydrolysis
E. Sulfonation

82. In order to speed up healing of the thermal injury it is required to


prescribe a drug that facilitates epithelization of skin and mucous
membranes. What drug is it?
A. Retinol acetate.
B. Tocopherol acetate
C. Nicotinic acid
D. Ergocalciferol
E. Ascorbic acid

83. Malaria is treated with structural analogs of vitamin B2 (riboflavin).


These drugs disrupt the synthesis of the following enzymes in
plasmodium:
A. Cytochrome oxidase
B. Peptidase
C. FAD-dependent dehydrogenase
D. NAD-dependent dehydrogenase
E. Aminotransferase
19
84. During endotoxemia active forms of the oxygen including superoxide
anion radical are formed in the human body. With help of what enzyme is
this anion deactivated?
A. Catalase
B. Peroxidase
C. Glutathioneperoxidase
D. Glutathionereductase
E. Superoxide dismutase

85. The living organisms that did develop the system of defence against
the unfavorable action of H2O2 during the evolution can exist only in
anaerobic conditions. Which of the enzymes can destroy hydrogen
peroxide?
A. Oxygenases and hydroxylases
B. Peroxidase and catalase
C. Cytochrome oxidase, cytochrome b.
D. Oxygenase and catalase.
E. Flavin-linked oxidases.

86. When blood circulation in the damaged tissue is restored, then lactate
accumulation comes to a stop and glucose consumption decelerates.
These metabolic changes are caused by activation of the following
process:
A. Anaerobic glycolysis
B. Aerobic glycolysis
C. Lipolysis
D. Gluconeogenesis
E. Glycogen biosynthesis

87. Buffer capacity of blood was decreased in the worker due to


exhausting muscular work. Entry of which acid substance to the blood can
this state be explained?
A. Pyruvate
B. 1,3-bisphosphoglycerate
C. α-ketoglutarate
D. Lactate
E. 3-phosphoglycerate

88. As a result of exhausting muscular work a worker has largely reduced


buffer capacity of blood. Which acidic substance that came to blood
caused this phenomenon?
A. Pyruvate
B. Lactate
C. 1,3-bisphosphoglycerate
D. 3-phosphoglycerate
20
E. –

89. Myocyte cytoplasm contains a big number of dissolved metabolites of


glucose oxidation. Name one of them that turns directly into a lactate:
A. Oxaloacetate
B. Glycerophosphate
C. Glucose 6-phosphate
D. Pyruvate
E. Fructose 6-phosphate.

90. A child has got galactosemia. Concentration of glucose in blood has


not considerably changed. Deficiency of which enzyme caused this
illness?
A. Phosphoglucomutase
B. Amylo-1,6-glucosidase
C. Galactose-1-phosphate uridyltransferase
D. Galactokinase
E. Hexokinase

91. Some students developed myodynia after continuous physical activity


during physical education. The reason for such condition was
accumulation of lactic acid in the skeletal muscles. It was generated in
the students' bodies after activation of the following process:
A. Glycolysis
B. Gluconeogenesis
C. Glycogenolysis
D. Pentose-phosphate cycle
E. Lipolysis

92. A nontrained man has usually muscular hypoxia after a sprint. Which
metabolite accumulates in the muscles as a result of it?
A. Ketone bodies
B. Glucose-6-phosphate
C. Oxaloacetate
D. Lactate
E. -

93. Chronic overdosage of glucocorticoids leads to the development of


hyperglycemia. Which process of carbohydrate metabolism is responsible
for this effect?
A. Glycogenolysis
B. Aerobic glycolisis
C. Pentose-phosphate cycle
D. Gluconeogenesis
E. Glycogenesis
21

94. During starvation normal rate of glucose is maintained by means of


gluconeogenesis activation. What substance can be used as a substrate for
this process?
A. Alanine
B. Ammonia
C. Adenine
D. Urea
E. Guanine

95. Medical ambulance delivered a 2 year old girl to the children's


department. Objectively: the child is inert, apathetic. Liver is enlarged,
study of biopsy material revealed glycogen excess. Blood glucose rate is
below normal. The most probable cause of hypoglycemia is:
A. High activity of glucokinase
B. Low activity of glycogen phosphorylase
C. Low activity of glucose 6-phosphatase
D. Low activity of glucose 1-phosphate uridine transferase
E. Low activity of glycogen synthase

96. Clinical examination enabled to make a provisional diagnosis:


stomach cancer. Gastric juice contained lactic acid. What type of glucose
catabolism turns up in the cancerous cells?
A. Pentose Phosphate Pathway
B. Gluconeogenesis
C. Aerobic glycolysis
D. Anaerobic glycolysis
E. Glucose alanine cycle

97. A 22 year old woman has been taking sulfanilamides for a long time
that led to symptoms of hemolytic anemia caused by hereditary
disturbance of synthesis of glucose-6-phosphate dehydrogenase. This
enzyme of pentose phosphate cycle is responsible for generation of:
A. NADPH
B. NAD
C. FAD
D. FMN
E. ATP

98. It has been revealed that intense physical exercise causes activation
of gluconeogenesis in liver of experimental rats. Which substance is
glucose precursor in this case?
A. Glycogen
B. Urea
C. Stearate
22
D. Palmitate
E. Pyruvate

99. It is known that the pentose phosphate pathway that occurs in the
adipocytes of adipose tissue acts as a cycle. Which is the main function of
this cycle in the adipose tissue?
A. Xenobiotic detoxification
B. Energy generation
C. Glucose oxidation to end products
D. Ribose-phosphate production
E. NADPH generation

100. A blood sample is taken from a 40-year-old man has been fasting
completely for a week, drinking only water. Which of the following will be
at a higher concentration than after a normal overnight fast?
A. Glucose
B. Insulin
C. Ketone bodies
D. Nonesterified fatty acids
E. Triacylglycerol

101. A 25-year-old man visits his hospital complaining of abdominal


cramps and diarrhea after drinking milk. What is the most likely cause of
his problem?
A. Bacterial and yeast overgrowth in the large intestine
B. Infection with the intestinal parasite Giardia lamblia
C. Lack of small intestinal lactase
D. Lack of pancreatic amylase
E. Lack of small intestinal sucrase-isomaltase

102. Which of the following will provide the main fuel for muscle
contraction during short-term maximum exertion?
A. Muscle reserves of triacylglycerol
B. Plasma glucose
C. Plasma nonesterified fatty acids
D. Muscle glycogen
E. Triacylglycerol in plasma very low density lipoprotein

103. Galactosemia is revealed in the child. Concentration of glucose in


the blood is not considerably changed. Deficiency of what enzyme caused
this illness?
A. Galactose-1-phosphate uridyltransferase
B. Amylo-1,6-glucosidase
C. Phosphoglucomutase
D. Galactokinase
23
E. Hexokinase

104. Characteristic sign of glycogenosis is muscle pain during physical


work. Blood examination reveals usually hypoglycemia. This pathology is
caused by congenital deficiency of the following enzyme:
A. Glucose-6-phosphate dehydrogenase
B. Alpha amylase
C. Glycogen phosphorylase
D. Gamma amylase
E. Lysosomal glycosidase

105. A 34-year-old patient's resistance to heavy physical load is reduced


while the skeletal muscles glycogen level is increased. By decreasing of
the activity of what enzyme can this phenomenon be explained?
A. Phosphofructokinase
B. Glucose-6-phosphate dehydrogenase
C. Glycogen synthetase
D. Glycogen phosphorylase
E. Glucose-6-phosphatase

106. A patient is ill with diabetes mellitus that is accompanied by


hyperglycemia of over 7,2 mmol/l on an empty stomach. The level of
what blood plasma protein allows estimating the glycemia rate
retrospectively (4-8 weeks before examination)?
A. Albumin
B. Fibrinogen
C. C-reactive protein
D. Glycated hemoglobin
E. Ceruloplasmin

107. A 62-year-old female patient has developed a cataract (lenticular


opacity) secondary to the diabetes mellitus. What type of protein
modification is observed in case of diabetic cataract?
A. Glycosylation
B. Phosphorylation
C. ADP-ribosylation
D. Methylation
E. Limited proteolysis

108. The B cells of endocrine portion of pancreas are selectively damaged


by alloxan poisoning. How will it be reflected in blood plasma?
A. The content of fibrinogen decrease
B. The content of sugar increases
C. The level of sugar decreases
D. The content of globulins decreases
24
E. The content of albumins decreases

109. Untrained people often have muscle pain after sprints as a result of
lactate accumulation. This might be caused by intensification of the
following biochemical process:
A. Gluconeogenesis
B. Pentose phosphate pathway
C. Lipogenesis
D. Glycogenesis
E. Glycolysis

110. A patient was delivered to the hospital by an emergency team.


Objectively: grave condition, unconscious, adynamy. Cutaneous surfaces
are dry, eyes are sunken, face is cyanotic. There is tachycardia and smell
of acetone from the mouth. Analysis results: blood glucose - 20,1 mmol/l,
urine glucose - 3,5% . What is the most probable diagnosis?
A. Hypoglycemic coma
B. Acute heart failure
C. Acute alcoholic intoxication
D. Hyperglycemic coma
E. Anaphylactic shock

111. Patient with diabetes mellitus experienced loss of consciousness and


convulsions after injection of insulin. Which is the result of biochemical
blood analysis for concentration of the sugar?
A. 8,0 mmol/L
B. 10,0 mmol/L
C. 3,3 mmol/L
D. 5,5 mmol/L
E. 1,5 mmol/L

112. On the empty stomach in the patients blood glucose level was 5,65
mmol/L, in an hour after usage of sugar it was 8,55 mmol/L, in a 2 hours
- 4,95 mmol/L. Such indicators are typical for:
A. Patient with hidden diabetes mellitus
B. Patient with insulin-dependent diabetes mellitus
C. Patient with non-insulin dependent diabetes mellitus
D. Healthy person
E. Patient with thyrotoxicosis

113. A child is languid, apathetic. Liver is enlarged and liver biopsy


revealed a significant excess of glycogen. Glucose concentration in the
blood stream is below normal. What is the cause of low glucose
concentration?
A. Low (absent) activity of glycogene phosphorylase in liver
25
B. Low (absent) activity of hexokinase
C. High activity of glycogen synthetase
D. Low (absent) activity of alfa-1,4- glucosidase
E. Deficit of glucose 1-phosphaturidine transferase

114. After a sprint an untrained person develops muscle hypoxia. This


leads to the accumulation of the following metabolite in muscles:
A. Ketone bodies
B. Lactate
C. Acetyl CoA
D. Glucose-6-phosphate
E. Oxaloacetate

115. A child's blood presents high content of galactose, glucose


concentration is low. There are such presentations as cataract, mental
deficiency, adipose degeneration of liver. What disease is it?
A. Diabetes mellitus
B. Lactosemia
C. Galactosemia
D. Steroid diabetes
E. Fructosemia

116. A 45 y.o. woman suffers from Cushing's syndrome - steroid diabetes.


Biochemical examination revealed: hyperglycemia, hypochloremia. Which
of the under-mentioned processes is the first to be activated?
A. Glycogenolysis
B. Glucose reabsorption
C. Gluconeogenesis
D. Glucose transport to the cell
E. Glycolysis

117. The patient with complaints of permanent thirst applied to the


doctor. Hyperglycemia, polyuria and increased concentration of 17-
ketosteroids in the urine were revealed. What disease is the most likely?
A. Insulin-dependent diabetes mellitus
B. Myxoedema
C. Type I glycogenosis
D. Steroid diabetes
E. Addison's disease

118. A newborn develops dyspepsia after the milk feeding. When the milk
is substituted by the glucose solution the dyspepsia symptoms disappear.
The newborn has the subnormal activity of the following enzyme:
A. Invertase
B. Maltase
26
C. Lactase
D. Amylase
E. Isomaltase

119. After taking sulfonamides and aspirin by a 38-year-old patient,


hemolysis of erythrocytes caused by the insufficiency of glucose-6-
phosphate dehydrogenase developed. The disturbance of what coenzyme
formation does this pathology result from?
A. Ubiquinone
B. FADH2
C. Pyridoxal phosphate
D. FMNH2
E. NADPH

120. A child with point mutation has the absence of glucose-6


phosphatase in body tissues, hypoglycemia and hepatomegaly detected.
Define the type of pathology which these symptoms are characteristic of:
A. Measles
B. Addison's disease
C. Girke's disease
D. Parkinson's disease
E. McArdle's disease

121. The concentration of glucose in the blood plasma of a healthy man


varies within the following limits:
A. 2.0-4.0 mM/L
B. 10.0-25.0 mM/L
C. 6.0-9.5 mM/L
D. 3.3-5.5 mM/L
E. 1.0-2.0 mM/L

122. Some hours after an intensive physical training a sportsman showed


activated gluconeogenesis. Which of the following is the basic substrate of
gluconeogenesis?
A. Serine
B. Lactate
C. Aspartate
D. Glutamate
E. α-Ketoglutarate

123. A newborn child had dyspepsia phenomena (diarrhea, vomiting)


detected after feeding with milk. After additional feeding with glucose the
morbid symptoms disappeared. The insufficient activity of what enzyme
that takes part in the carbohydrates breakdown causes the indicated
disorders?
27
A. Saccharase
B. Amylase
C. lsomaltase
D. Lactase
E. Maltase

124. A 2-year-old boy has the increase of liver and spleen sizes detected
and eye cataract present. The total sugar level in blood is increased, but
glucose tolerance is within the normal range. The inherited disturbance of
the metabolism of what substance is the cause of the indicated state?
A. Glucose
B. Galactose
C. Fructose
D. Maltose
E. Saccharose

125. A 57-year-old patient, suffering from insulin dependent diabetes


mellitus, showed the development of ketoacidosis. The biochemical
mechanism of the development of this pathology is decreasing of acetyl-
CoA utilization due to the deficiency of:
A. 2-oxoglutarate
B. Glutamate
C. Oxaloacetate
D. Aspartate
E. Succinate

126. A 38-year-old man is receiving treatment for schizophrenia in


hospital. The initial levels of glucose, ketone bodies and urea in the blood
are within the normal range. Shock therapy put into practice by regular
insulin injections resulted in the development of the comatose state
which improved the clinical status of the patient. What is the most
probable cause of insulin coma?
A. Hypoglycemia
B. Hyperglycemia
C. Dehydratation of tissues
D. Metabolic acidosis
E. Ketonemia

127. A 7-year-old girl manifests obvious signs of anemia. Laboratory tests


showed the deficiency of pyruvate kinase activity in erythrocytes. The
disorder of what biochemical process is a major factor in the development
of anemia?
A. Deamination of amino acid
B. Oxidative phosphorylation
C. Tissue respiration
28
D. Breaking up of peroxides
E. Anaerobic glycolysis

128. A 45-year-old woman does not have any symptoms of insulin


dependent diabetes mellitus but testing on an empty stomach showed the
increase of the blood glucose level (7.5 mM/L). What additional laboratory
test needs to be done to substantiate the diagnosis?
A. Determination of tolerance to glucose
B. Determination of tolerance to glucose on an empty stomach
C. Determination of ketone bodies concentration in the urine
D. Determination of rest nitrogen level in the blood
E. Determination of glycosylated hemoglobin level

129. What biochemical process is stimulated in the liver and kidneys of a


patient exhausted by starvation?
A. Synthesis of bilirubin
B. Gluconeogenesis
C. Synthesis of urea
D. Formation of hippuric acid
E. Synthesis of uric acid

130. During intensive physical exertion, one of the energy sources for the
working muscles is glucose produced as the result of gluconeogenesis.
This process is the most intensive in the following organ:
A. Brain
B. Lungs
C. Muscles
D. Liver
E. Stomach

131. In a young man during exercise, the minute oxygen uptake and
carbon dioxide emission equalled to 1000 ml. What substrates are
oxidized in the cells of his body?
A. Proteins
B. Fats
C. Carbohydrates and fats
D. Carbohydrates
E. Carbohydrates and proteins

132. Human red blood cells do not contain mitochondria. What is the
main pathway for ATP production in these cells?
A. Anaerobic glycolysis
B. Aerobic glycolysis
C. Oxidative phosphorylation
D. Creatine kinase reaction
29
E. Cyclase reaction

133. A child has a history of hepatomegaly, hypoglycemia, seizures,


especially on an empty stomach and in stressful situations. The child is
diagnosed with Gierke disease. This disease is caused by the genetic
defect of the following enzyme:
A. Amyloid-1,6-glycosidase
B. Phosphoglucomutase
C. Glucose-6-phosphatase
D. Glycogen phosphorylase
E. Glucokinase

134. Which of the following is NOT a hemoprotein?


A. Albumin
B. Myoglobin
C. Cytochrome c
D. Catalase
E. Cytochrome P450

135. A 30-year-old man presented at clinic with a history of intermittent


abdominal pain and episodes of confusion and psychiatric problems.
Laboratory tests revealed increases of urinary δ-aminolevulinate and
porphobilinogen. Mutational analysis revealed a mutation in the gene for
uroporphyrinogen I synthase (porphobilinogen deaminase). The probable
diagnosis was:
A. X-linked sideroblastic anemia.
B. Congenital erythropoietic porphyria.
C. Porphyria cutanea tarda.
D. Variegate porphyria.
E. Acute intermittent porphyria.

136. Select the one of the following statements that is NOT CORRECT:
A. Bilirubin is a cyclic tetrapyrrole.
B. Albumin-bound bilirubin is transported to the liver.
C. High levels of bilirubin can cause damage to the brains of newborn
infants.
D. Bilirubin contains methyl and vinyl groups.
E. Bilirubin does not contain iron.

137. Patient with encephalopathy was admitted to the neurological in-


patient department. Correlation of increasing of encephalopathy and
substances absorbed by the bloodstream from the intestines was revealed.
Which substances that are created in the intestines can cause
endotoxemia?
A. Butyrate
30
B. Indole
C. Acetacetate
D. Biotin
E. Ornithine

138. Examination of a patient suffering from cancer of urinary bladder


revealed high rate of serotonin and hydroxyanthranilic acid. It is caused
by excess of the following amino acid in the organism:
A. Alanine
B. Histidine
C. Tryptophan
D. Methionine
E. Tyrosine

139. A 4 y.o. child with signs of durative protein starvation was admitted
to the hospital. The signs were as follows: growth inhibition, anemia,
edema, mental deficiency. Choose a cause of edema development:
A. Reduced synthesis of globulins
B. Reduced synthesis of hemoglobin
C. Reduced synthesis of lipoproteins
D. Reduced synthesis of albumins
E. Reduced synthesis of glycoproteins

140. The concentration of albumins in human blood sample is lower than


normal. This leads to edema of tissues. Which blood function is damaged?
A. Maintaining the oncotic blood pressure
B. Maintaining the pH level
C. Maintaining the body temperature
D. Maintaining the blood sedimentation system
E. All answers are correct

141. Ammonia is a very toxic substance, especially for nervous system.


Which substance takes the most active part in ammonia detoxication in
brain tissues?
A. Lysine
B. Proline
C. Histidine
D. Alanine
E. Glutamic acid

142. A patient has pellagra. Interrogation revealed that he had lived


mostly on maize for a long time and eaten little meat. This disease had
been caused by the deficit of the following substance in the maize:
A. Tryptophan
B. Tyrosine
31
C. Proline
D. Alanine
E. Histidine

143. Nappies of a newborn have dark spots that witness of formation of


homogentisic acid. Metabolic imbalance of which substance is it
connected with?
A. Galactose
B. Methionine
C. Cholesterine
D. Tryptophane
E. Tyrosine

144. A 1,5-year-old child presents with both mental and physical lag,
decolorizing of skin and hair, decrease in catecholamine concentration in
blood. When a few drops of 5% solution of trichloroacetic iron had been
added to the child’s urine it turned olive green. Such alterations are
typical for the following pathology of the amino acid metabolism:
A. Alkaptonuria
B. Tyrosinosis
C. Albinism
D. Phenylketonuria
E. Xanthinuria

145. The greater amount of nitrogen is excreted from the organism in


form of urea. Inhibition of urea synthesis and accumulation of ammonia
in blood and tissues are induced by the decreased activity of the following
liver enzyme:
A. Aspartate aminotransferase
B. Urease
C. Carbamoyl phosphate synthetase
D. Amylase
E. Pepsin

146. After a serious viral infection a 3-year-old child has repeated


vomiting, loss of consciousness, convulsions. Examination revealed
hyperammoniemia. What may have caused changes of biochemical blood
indices of this child?
A. Activated processes of amino acids decarboxylation
B. Disorder of ammonia neutralization in ornithinic cycle
C. Disorder of biogenic amines neutralization
D. Increased putrefaction of proteins in intestines
E. Inhibited activity of transamination enzymes
32
147. Albinos can't stand sun impact - they don't aquire sun-tan but get
sunburns. Disturbed metabolism of which amino acid underlies this
phenomenon?
A. Phenylalanine
B. Methionine
C. Tryptophan
D. Glutamic acid
E. Histidine

148. Glutamate decarboxylation results in formation of inhibitory


transmitter in CNS. Name it:
A. Glutathione
B. Histamine
C. Serotonin
D. Asparagine
E. GABA

149. In course of histidine catabolism a biogenic amine is formed that has


powerful vasodilatory effect. Name it:
A. Serotonin
B. Dioxyphenylalanine
C. Noradrenalin
D. Histamine
E. Dopamine

150. A patient diagnosed with carcinoid of bowels was admitted to the


hospital. Analysis revealed high production of serotonin. It is known that
this substance is formed of tryptophan amino acid. What biochemical
mechanism underlies this process?
A. Desamination
B. Microsomal oxydation
C. Decarboxylation
D. Transamination
E. Formation of paired compounds

151. During hypersensitivity test a patient got subcutaneous injection of


an antigen which caused reddening of skin, edema, pain as a result of
histamine action. This biogenic amine is generated as a result of the
following transformation of histidine amino acid:
A. Methylation
B. Decarboxylation
C. Phosphorylation
D. Isomerization
E. Deaminization
33
152. A patient complained about dizziness, memory impairment,
periodical convulsions. It was revealed that these changes were caused by
a product of decarboxylation of glutamic acid. Name this product:
A. GABA
B. Pyridoxal phosphate
C. TDP
D. ATP
E. THFA

153. Laboratory examination of a child revealed increased concentration


of leucine, valine, isoleucine and their ketoderivatives in blood and urine.
Urine smelt of maple syrup. This disease is characterized by the deficit of
the following enzyme:
A. Aminotransferase
B. Dehydrogenase of branched amino acids
C. Glucose-6-phosphatase
D. Phosphofructokinase
E. Phosphofructomutase

154. A newborn child was found to have reduced intensity of sucking,


frequent vomiting, hypotonia. In urine and blood exhibit increased
concentration of citrulline. Which metabolic process is disturbed?
A. Ornithinic cycle
B. Tricarboxylic acid cycle
C. Glycolysis
D. Glyconeogenesis
E. Cori cycle

155. Plasmic factors of blood coagulation are exposed to post-


translational modification with the participation of vitamin K. It is
necessary as a cofactor in the enzyme system of gamma-carboxylation of
protein factors of blood coagulation due to the increased affinity of their
molecules with calcium ions. Which amino acid is carboxylated in these
proteins?
A. Glutamate
B. Valine
C. Serine
D. Phenylalanine
E. Arginine

156. Pharmacological effects of antidepressants are connected with


inhibition of an enzyme catalyzing biogenic amines noradrenaline and
serotonine in the mitochondrion of cerebral neurons. Which enzyme
participates in this process?
A. Transaminase
34
B. Decarboxylase
C. Peptidase
D. Lyase
E. Monoamine oxidase

157. A child manifests epileptic seizures caused by vitamin B6 deficiency.


This is conditioned by the decrease of the gamma-aminobutyrate level in
the nervous tissue which acts as an inhibiting neurotransmitter. The
activity of which enzyme is decreased in this case?
A. Pyridoxal kinase
B. Alanine aminotransferase
C. Glutamate decarboxylase
D. Glutamate dehydrogenase
E. Glutamate synthetase

158. A child has an acute renal failure. Which biochemical factor found in
saliva can confirm this diagnosis?
A. Increase in glucose concentration.
B. Increase in urea concentration.
C. Decrease in glucose concentration.
D. Increase in concentration of higher fatty acids.
E. Decrease in nucleic acid concentration.

159. After severe viral hepatitis a 4 year old boy presents with vomiting,
occasional loss of consciousness, convulsions. Blood test revealed
hyperammoniemia. Such condition is caused by a disorder of the
following biochemical hepatic process:
A. Disorder of ammonia neutralization.
B. Disorder of biogenic amines neutralization.
C. Protein synthesis inhibition.
D. Activation of amino acid decarboxylation.
E. Inhibition of transamination enzymes.

160. A 4 y.o. boy has had recently serious viral hepatitis. Now there are
such clinical presentations as vomiting, loss of consciousness,
convulsions. Blood analysis revealed hyperammoniemia. Disturbance of
which biochemical process causes such pathological condition of the
patient?
A. Disturbed neutralization of biogenic amines.
B. Disturbed neutralization of ammonia in liver.
C. Increased putrefaction of proteins in bowels.
D. Activation of amino acid decarboxylation.
E. Inhibition of transamination enzymes.
35
161. Cerebral trauma caused increase of ammonia formation. Which
amino acid takes part in removal of ammonia from cerebral tissue?
A. Tryptophan.
B. Lysine.
C. Glutamic acid.
D. Valine.
E. Tyrosine.

162. A patient presents with dysfunction of cerebral cortex accompanied


by epileptic seizures. He has been administered a biogenic amine
synthesized from glutamate and responsible for central inhibition. What
substance is it?
A. Serotonin.
B. Dopamine.
C. Acetylcholine.
D. Histamine.
E. Gamma-amino butyric acid.

163. A male patient has been diagnosed with acute radiation disease.
Laboratory examination revealed a considerable reduction of platelet
serotonin level. The likely cause of platelet serotonin reduction is the
disturbed metabolism of the following substance:
A. 5-oxytryptophane.
B. Tyrosine.
C. Histidine.
D. Phenylalanine.
E. Serine.

164. A patient with suspected diagnosis "progressing muscular dystrophy"


got his urine test. What compound will confirm this diagnosis if found in
urine?
A. Collagen.
B. Creatine.
C. Porphyrin.
D. Myoglobin.
E. Calmodulin.

165. According to clinical indications a patient was administered


pyridoxal phosphate. Which processes is this medication intended to
correct?
A. Transamination and decarboxylation of amino acids.
B. Oxidative decarboxylation of keto acids.
C. Desamination of purine nucleotides.
D. Synthesis of purine and pyrimidine bases.
E. Protein synthesis.
36

166. Depressions and emotional insanities result from the deficit of


noradrenalin, serotonin and other biogenic amines in the brain. Their
concentration in the synapses can be increased by means of the
antidepressants that inhibit the following enzyme:
A. Diamine oxidase.
B. L-amino-acid oxidase.
C. D-amino-acid oxidase.
D. Monoamine oxidase.
E. Phenylalanine-4-monooxygenase.

167. A 13-year-old boy complains of general weakness, dizziness,


tiredness. He is mentally retarded. Increased level of valine, isoleucine,
leucine is in the blood and urine. Urine has specific smell. What is the
diagnosis?
A. Addison's disease.
B. Maple syrup urine disease.
C. Tyrosinosis.
D. Histidinemia.
E. Graves' disease.

168. A 2-year-old child with mental and physical retardation has been
delivered to a hospital. He presents with frequent vomiting after having
meals. There is phenylpyruvic acid in urine. Which metabolism
abnormality is the reason for this pathology?
A. Lipid metabolism.
B. Carbohydrate metabolism.
C. Amino acid metabolism.
D. Water-salt metabolism.
E. Phosphoric calcium metabolism.

169. A patient has been diagnosed with alkaptonuria. Choose an enzyme


whose deficiency can be the reason for this pathology:
A. Homogentisic acid oxidase.
B. Phenylalanine hydroxylase.
C. Glutamate dehydrogenase.
D. Pyruvate dehydrogenase.
E. Dioxyphenylalanine decarboxylase.

170. Urine analysis of a 12-year-old boy reveals high concentration of all


aliphatic amino acids with the highest excretion of cystine and cysteine.
US of kidneys revealed kidney concrements. What is the most likely
pathology?
A. Phenylketonuria.
B. Alkaptonuria.
37
C. Cystitis.
D. Cystinuria.
E. Hartnup’s disease.

171. Reason of disease pellagra can be a primary feed by corn and


decrease of products of an animal origin in a diet. Absence in a diet what
amino acid leads to this pathology?
A. Tryptophan
B. Isoleucine
C. Phenylalanine
D. Methionine
E. Histidine

172. Glycogenous amino acids belong to those that are transformed into:
A. Acetyl-CoA
B. Acetoacetate
C. Glycerin
D. Glucoronic acid
E. Pyruvic acid

173. A patient suffers hyperammoniemia. Name the enzyme which


decrease activity is the reason for this condition
A. Creatine phosphokinase of the skeletal muscle
B. Aspatate aminotransferase of the myocardium
C. Alanine aminotransferase of the liver
D. Leucine aminopeptidase of the liver
E. Ornithine carbomoyl phosphate transferase of the liver

174. The urine of a newborn baby is observed to have citrulline and high
level of ammonia. Which substance formation leads to the disturbance of
the child?
A. Urea
B. Uric acid
C. Ammonia
D. Creatinine
E. Creatine

175. Name the amino acid which participates in heme synthesis:


A. Serine
B. Glycine
C. Alanine
D. threonine
E. Valine
38
176. Which amino acid participates in toxic substances deactivation in
the liver?
A. Serine
B. Threonine
C. Tryptophan
D. Glycine
E. Histidine

177. Which amino acid participates in purine bases synthesis of


nucleotides?
A. Glycine
B. Threonine
C. Alanine
D. Serine
E. Tryptophan

178. As boiling water appeared on the skin, the injured part of it turned
red, swelled and started to hurt. Which substance can lead to such
reaction?
A. Lysine
B. Thiamine
C. Glutamine
D. Histamine
E. Asparagine

179. Glutathione is a component of the human organism antioxidant


system. Which of the amino acids included in its composition:
A. Threonine
B. Serine
C. Aspartic acid
D. Tyrosine
E. Glycine

180. For muscle functioning, a very important role is given to creatine


phosphate which is formed from creatine and ATP. Name the amino acids
necessary for creatine synthesis:
A. Cystine
B. Threonine
C. Serine
D. Glycine
E. Alanine

181. For muscle functioning, a very important role is given creatine


phosphate which is formed from creatine and ATP. Name the amino acids
necessary for creatine synthesis:
39
A. Glycine, Proline, Cystine
B. Methionine, Leucine, phenylalanine
C. Glycine, Arginine, Methionine
D. Arginine, Tryptophan, Lysine
E. Valine, leucine, isoleucine

182. Methyl groups (-CH3) are used by organism to synthesize important


compounds such creatine, choline, adrenalin etc. Which amino acid has
it?
A. Methionine
B. Valine
C. Leucine
D. Isoleucine
E. Tryptophan

183. What biogenic amine relaxes vessels and formed during histidine
catabolism:
A. Serotonin
B. Histamine
C. Dopa
D. Noradrenalin
E. Dopamine

184. With a repeated action of the ultra violet rays, the skin gets dark
because of the melanine synthesis which protects cells from lesion .The
main mechanism of this defense introduction is:
A. Tyrosinase suppression
B. Activation of homogentisic acid oxidase
C. Tyrosinase activation
D. Suppression of homogentisic acid oxidase
E. Phenylalanine hydroxylase suppression

185. In psychiatry, to treat various CMS disease biogenic amines are used.
Name the medicine of this group that is an inhibition mediator:
A. Histamine
B. Serotonin
C. Dopamine
D. Taurine
E. Gamma-aminobutyric acid

186. Under the influence of ultra-violet irradiation, human skin gets dark.
Which defense substance is synthesized in cells under the influence of
ultra-violet?
A. Glycine
B. Serine
40
C. Aspartic acid
D. Melanin
E. Tyrosin

187. With alkaptonuria, patients urine contain high amount of


homogentisic acid (urine get dark in an open air). The inborn deficiency of
which enzyme takes place?
A. Phenylalanine hydroxylase
B. Glutamic transaminase
C. Homogentisinate oxidase
D. Asparate aminotransferase
E. Glutamate decarboxylase

188. An infant is noticed to have dimness of sclera, mucous membranes,


auricle: the executed urine gets dark in an open air. The blood and urine
are seen to contain homogentisic acid. What is the most probably
symptom?
A. Albinism
B. Cystinuria
C. Porphyria
D. Hemolytic anemia
E. Alkaptonuria

189. A 10-months old child, whose parents are black-haired, has fair hair,
very light skin and blue eyes, at birth she had a regular appearance, but
for 3 months she has developed cranial blood circulation disturbance and
mental retardation. What is the reason for this condition?
A. Phenylketonuria
B. Galactosemia
C. Glycogenosis
D. Acute porphyria
E. Histidinemia

190. A 6-day old infant has phenylpyruvate and phenylacetate


redundancy in his urine. Which amino acid metabolism is disturbed in the
child’s organism?
A. Tryptophan
B. Phenylalanine
C. Methionine
D. Histidine
E. Arginine

191. A hospital admits a 9 year old boy with mental and physical
retardation. The biochemical blood analysis revealed the elevated amount
41
of phenylalanine. The blocking of which enzyme can lead to this
situation?
A. Homogentisic acid oxidase
B. Glutamine transaminase
C. Aspartate aminotransferase
D. Phenylalanine 4-monooxigenase
E. Glutamate decarboxylase

192. A child has dark spots on his nappies, which shows presence of
homogentisic acid. The disturbance of metabolism of which substance is
connected with it?
A. Galactose
B. Tryptophan
C. Tyrosine
D. Cholesterol
E. Methionine

193. What is the main form of ammonia transport from the majority of
peripheral tissue to the liver?
A. Glutamine
B. Asparagine
C. Citrulline
D. Ornithine
E. Urea

194. In which form is ammonia transmitted from the muscles from the
liver?
A. As asparagine
B. As urea
C. As arginine
D. As ammonia salt
E. As alanine

195. An unconscious patient had been delivered to a hospital by the


ambulance. Objectively: absent reflexes, occasional convulsions, irregular
breathing. After a laboratory examination he was diagnosed with hepatic
coma. What metabolite accumulation is essential for the development of
the central nervous system disorders?
A. Ammonia
B. Histamine
C. Bilirubin
D. Urea
E. Glutamine.
42
196. Laboratory examination of a child revealed high content of leucine,
valine, isoleucine and their ketoderivates in blood and urine. Urine had
the typical smell of maple syrup. This disease was caused by deficiency of
the following enzyme:
A. Aminotransferase
B. Glucose-6-phosphatase
C. Phosphofructokinase
D. Dehydrogenase of branched amino acids.
E. Phosphofructomutase

197. A 50-year-old woman with myocardium infarction has been delivered


to the intensive care unit. Which enzyme's activity will be most increased
during the first two days?
A. LDH4
B. Aspartate aminotransferase
C. LDH5
D. Alanine aminotransferase
E. Alanine aminopeptidase

198. A lab rat has subcutaneously received mercury (II) chloride in the
amount of 5 mg/kg. 24 hours later the plasma creatinine concentration
increased several times. What mechanism of retention azotemia is
observed in this case?
A. Increased creatinine production in the muscles
B. Increased creatinine reabsorption
C. Increased glomerular filtration
D. Increased creatinine production in the renal tubules
E. Decreased glomerular filtration

199. A woman has been limiting the amount of products in her diet to
lose some weight. 3 months later she developed edemas and her diuresis
increased. What dietary component deficiency is the cause of this?
A. Fats
B. Carbohydrates
C. Proteins
D. Vitamins
E. Minerals

200. During intensive muscle work there is a large amount of ammonia


produced in the muscles. What amino acid plays the main role in the
transportation of ammonia to the liver and participates in
gluconeogenesis reactions?
A. Arginine
B. Lysine
C. Ornithine
43
D. Aspartate
E. Alanine

201. Encephalopathy has developed in a child with hemolytic disease of


the newborn. What substance had increased in the child’s blood, resulting
in damage to the CNS?
A. Unconjugated bilirubin
B. Bilirubin-albumin complex
C. Bilirubin glucuronide
D. Verdohemoglobin
E. Bile acids

202. Vascular endothelium is characterized by high metabolic activity and


synthesizes vasoactive substances. Among these substances there is a
potent vasodilator synthesized from L-arginine. Name this vasodilator:
A. Histamine
B. Nitrogen oxide
C. Bradykinin
D. Acetylcholine
E. Adrenaline

203. A patient, who suffers from congenital erythropoietic porphyria, has


skin photosensitivity. The accumulation of what compound in the skin
can cause it?
A. Uroporphyrinogen 1
B. Protoporphyrin
C. Uroporphyrinogen 2
D. Coproporphyrinogen 3
E. Heme

204. A patient with hereditary hyperammonemia due to a disorder of


ornithine cycle has developed secondary orotaciduria. The increased
synthesis of orotic acid is caused by an increase in the following
metabolite of ornithine cycle:
A. Citrulline
B. Ornithine
C. Urea
D. Argininosuccinate
E. Carbamoyl phosphate

205. A patient has yellow skin colour, dark urine, achromatic feces. What
substance will have strengthened concentration in the blood serum?
A. Conjugated bilirubin
B. Unconjugated bilirubin
C. Mesobilirubin
44
D. Verdoglobin
E. Biliverdin

206. A 46 year old woman suffering from cholelithiasis developed jaundice.


Her urine became dark-yellow and feces became colourless. Blood serum
will have the highest concentration of the following substance:
A. Unconjugated bilirubin
B. Biliverdin
C. Conjugated bilirubin
D. Mesobilirubin
E. Urobilinogen

207. Patient experienced increased susceptibility of the skin to the


sunlight. His urine after some time became dark-red. What is the most
likely cause of this?
A. Hemolytic jaundice
B. Porphyria
C. Albinism
D. Pellagra
E. Alkaptonuria

208. A 65 year old man suffering from gout complains of kidney pain.
Ultrasound examination revealed renal calculi. The most probable cause of
calculi formation is the strengthened concentration of the following
substance:
A. Cholesterol
B. Bilirubin
C. Urea
D. Uric acid
E. Cystine

209. A patient with high rate of obesity was advised to use carnitine as a
food additive in order to enhance "fat burning". What is the role of
carnitine in the process of fat oxidation?
A. Transport of FFA (free fatty acids) from cytosol to the mitochondria
B. Transport of FFA from fat depots to the tissues.
C. It takes part in one of reactions of FFA beta-oxidation.
D. FFA activation.
E. Activation of intracellular lipolysis.

210. A 70 year old man is ill with vascular atherosclerosis of lower


extremities and coronary heart disease. Examination revealed disturbance
of lipidic blood composition. The main factor of atherosclerosis
pathogenesis is the excess of the following lipoproteins:
A. Intermediate density lipoproteins.
45
B. Cholesterol.
C. High-density lipoproteins.
D. Chylomicrons.
E. Low-density lipoproteins.

211. An experimantal animal that was kept on protein-free diet developed


fatty liver infiltration, in particular as a result of deficiency of
methylating agents. This is caused by disturbed generation of the
following metabolite:
A. DOPA.
B. Cholesterol.
C. Choline.
D. Acetoacetate.
E. Linoleic acid.

212. Examination of cell culture got from a patient with lysosomal


pathology revealed accumulation of great quantity of lipids in the
lysosomes. What of the following diseases is this disturbance typical for?
A. Tay-Sachs disease.
B. Galactosemia.
C. Wilson disease.
D. Phenylketonuria.
E. Gout.

213. Diabetes mellitus causes ketosis as a result of activated oxidation of


fatty acids. What disorders of acid-base equilibrium may be caused by
excessive accumulation of ketone bodies in blood?
A. Metabolic alkalosis.
B. Any changes woun't happen.
C. Respiratory acidosis.
D. Respiratory alkalosis.
E. Metabolic acidosis.

214. Synthesis of phospholipids is disturbed as a result fatty infiltration


of liver. Indicate which of the following substances can enhance the
process of methylation during phospholipids synthesis?
A. Glucose.
B. Glycerin.
C. Methionine.
D. Ascorbic acid.
E. Citrate.

215. A sportsman needs to improve his sporting results. He was


recommended to take a preparation that contains carnitin. Which process
is activated the most by this compound?
46
A. Amino acids transporting.
B. Calcium ions transporting.
C. Glucose transporting.
D. Vitamin K transporting.
E. Fatty acids transporting.

216. A patient has symptoms of atherosclerosis. Which plasma lipid


transport forms should have an increased concentration?
A. Chylomicrons
B. IDL
C. HDL
D. LDL.
E. VLDL

217. Examination of an ill child's blood revealed inherited


hyperlipoproteinemia. Genetic defect of which enzyme synthesis causes
this phenomenon?
A. Glycosidase
B. Lipoprotein lipase.
C. Proteinase
D. Heme synthetase
E. Phenylalanine hydroxylase

218. A patient was diagnosed with seborrheic dermatitis associated with


vitamin H (biotin) deficiency. The patient has disturbed activity of the
following enzyme:
A. Pyruvate decarboxylase
B. Alcohol dehydrogenase
C. Acetyl-CoA-carboxylase.
D. Amino transferase
E. Carbomoyl phosphate synthetase

219. A 1-year-old child with the symptoms of affection of limb and trunk
muscles had been admitted to a hospital. Examination revealed muscle
carnitine deficiency. The biochemical basis of this pathology is a
disruption of the following process:
A. Regulation of Ca2 level in mitochondria
B. Substrate phosphorylation
C. Utilization of lactic acid
D. Oxidative phosphorylation
E. Transport of fatty acids to mitochondria.

220. After consumption of rich food a patient has nausea and heartburn,
steatorrhea. This condition might be caused by:
A. Bile acid deficiency
47
B. Increased lipase secretion
C. Disturbed trypsin synthesis
D. Amylase deficiency
E. Disturbed phospholipase synthesis

221. Increased amount of free fatty acids is observed in the blood of the
patients with diabetes mellitus. It can be caused by:
A. Storage of palmitoyl-CoA
B. Activation of the ketone bodies utilization
C. Increased activity of triglyceride lipase of adipocytes
D. Activation of the synthesis of the apolipoproteins
E. Decreased activity of phosphatidylcholine:cholesterol-acyltransferase in
blood plasma

222. An experimental animal that was kept on protein-free diet developed


fatty liver infiltration, in particular as a result of deficiency of
methylation agents. This is caused by disturbed generation of the
following metabolite:
A. Choline
B. DOPA
C. Cholesterol
D. Acetoacetate
E. Linoleic acid

223. Carnitine including drug was recommended to the sportsman for


improving results. What process is activated most of all with help of
carnitine?
A. Synthesis of steroid hormones
B. Transport of fatty acids to the mitochondria
C. Synthesis of ketone bodies
D. Synthesis of lipids
E. Tissue respiration

224. After intake of rich food a patient feels nausea and sluggishness;
with time there appeared signs of steatorrhea. Blood cholesterine
concentration is 9,2 micromole/l. This condition was caused by lack of:
A. Triglycerides
B. Fatty acids
C. Bile acids
D. Phospholipids
E. Chylomicrons

225. An experimental animal has been given excessive amount of carbon-


labeled glucose for a week. What compound can the label be found in?
A. Palmitic acid
48
B. Methionine
C. Vitamin A
D. Choline
E. Arachidonic acid

226. A sportsman was recommended to take a medication that contains


carnitine in order to improve his results. What process is activated by
carnitine the most?
A. Synthesis of steroid hormones
B. Synthesis of ketone bodies
C. Synthesis of lipids
D. Tissue respiration
E. Fatty acids transport to mitochondrion

227. Examination of a patient suffering from chronic hepatitis revealed a


significant decrease in the synthesis and secretion of bile acids. What
process will be mainly disturbed in the patient’s bowels?
A. Protein digestion
B. Carbohydrate digestion
C. Glycerin absorption
D. Fats emulsification
E. Amino acid absorption

228. A 6 year old child was delivered to a hospital. Examination revealed


that the child couldn't fix his eyes, didn't keep his eyes on toys, eye
ground had the cherry-red spot sign. Laboratory analyses showed that
brain, liver and spleen had high rate of ganglioside glycometide. What
congenital disease is the child ill with?
A. Wilson's syndrome
B. Tay-Sachs disease
C. Turner's syndrome
D. Niemann-Pick disease
E. MacArdle disease

229. NSAID blockade the utilization of arachidonic acid via


cyclooxygenase pathway, which results in formation of some bioactive
substances. They are:
A. Thyroxine
B. Biogenic amines
C. Prostaglandins
D. Somatomedins
E. Insulin-like growth factors

230. Arachidonic acid, an essential component of a human diet, acts as a


precursor of the vitally important physiologically active biomolecules.
49
Which substances are synthesized via cyclooxygenase pathway from
arachidonic acid?
A. Ethanolamine
B. Choline
C. Noradrenaline
D. Prostaglandins
E. Triiodothyronine

231. A 1-year-old child with symptoms of muscle involvement was


admitted to the hospital. Examination revealed carnitine deficiency in his
muscles. What process disturbance is the biochemical basis of this
pathology?
A. Regulation of Са2 level in mitochondrion
B. Substrate phosphorylation
C. Lactic acid utilization
D. Actin and myosin synthesis
E. Transporting of fatty acids to mitochondrion

232. Laboratory investigation of the patient's blood plasma, which was


performed 4 hours after a consumption of a fat diet, displayed a marked
increase of plasma turbidity. The most credible cause of this phenomenon
is the increase of in the plasma.
A. Chylomicrons
B. HDL
C. LDL
D. Cholesterol
E. Phospholipids

233. Patients who suffer from severe diabetes and don't receive insulin
have metabolic acidosis. This is caused by increased concentration of the
following metabolites:
A. Fatty acids
B. Ketone bodies
C. Unsaturated fatty acids
D. Triacylglycerols
E. Cholesterol

234. In a human body the adipose tissue is the basic location of


triacylglycerols (TAG) deposit. At the same time their synthesis takes
place in hepatocytes. In the form of what molecular complex are TAG
transported from the liver into the adipose tissue?
A. Chylomicrons
B. LDL
C. HDL
D. VLDL
50
E. Complexes with albumin

235. Laboratory investigation of a patient revealed a high level of plasma


LDL. What disease can be diagnosed?
A. Gastritis
B. Nephropathy
C. Atherosclerosis
D. Acute pancreatitis
E. Pneumonia

236. Aerobic oxidation of substrates is typical for cardiac myocytes.


Which of the following is the major oxidation substrate of cardiac
muscles?
A. Triacylglycerols
B. Glycerol
C. Glucose
D. Amino acids
E. Fatty acids

237. Which of the following enzymes accelerates the lipolysis under the
action of epinephrine in stress situations?
A. Triacylglycerol lipase
B. Lypoprotein lipase
C. Phospholipase A2
D. Phospholi pase C
E. Cholesterol esterase

238. Clinical signs and laboratory testing of a patient allow make the
assumption of gall-bladder inflammation, colloid properties of bile
disorder and occurrence of gall-stones. Which substances can underlie the
formation of gall-stones?
A. Oxalates
B. Urates
C. Chlorides
D. Phosphates
E. Cholesterol

239. Emotional stress causes activation of hormon-sensitive triglyceride


lipase in the adipocytes. Which secondary mediator takes part in this
process?
A. Cyclic guanosine monophosphate
B. Cyclic adenosine monophosphate
C. Adenosine monophosphate
D. Diacylglycerol
E. Ions of Са2
51

240. The insufficient secretion of which enzyme is the cause of


incomplete fats degradation in the digestive tract and appearance of great
quantity of neutral fats in feces?
A. Pepsin
B. Phospholipase
C. Enterokinase
D. Pancreatic lipase
E. Amylase

241. Which one of the following is NOT a phospholipid?


A. Sphingomyelin
B. Plasmalogen
C. Galactosylceramide
D. Cardiolipin
E. Lysolecithin

242. After they are produced from acetyl-CoA in the liver, ketone bodies
are mainly used for which one of the following processes?
A. Excretion as waste products
B. Generation of energy in the tissues
C. Energy generation in the liver
D. Conversion to fatty acids for storage of energy
E. Generation of energy in red blood cells

243. The subcellular site of the breakdown of long chain fatty acids to
acetyl-CoA via β-oxidation is:
A. The matrix of the mitochondria
B. The cytosol
C. The endoplasmic reticulum
D. The mitochondrial intermembrane space
E. The Golgi apparatus

244. The breakdown of one molecule of a C16 fully saturated fatty acid
(palmitic acid) by β-oxidation lead to the formation of:
A. 8 FADH2, 8 NADH and 8 acetyl CoA molecules
B. 7 FADH2, 7 NADH and 7 acetyl CoA molecules
C. 8 FADH2, 8 NADH and 7 acetyl CoA molecules
D. 7 FADH2, 8 NADH and 8 acetyl CoA molecules
E. 7 FADH2, 7 NADH and 8 acetyl CoA molecules

245. Which one of the following enzymes is inhibited by the nonsteroidal


anti-inflammatory drug (NSAID) aspirin?
A. Lipoxygenase
B. Thromboxane synthase
52
C. Prostacyclin synthase
D. Cyclooxygenase
E. ∆6 desaturase

246. Which one of the following is the major product of fatty acid
synthase?
A. Acetyl-CoA
B. Oleate
C. Palmitate
D. Palmitoyl-CoA
E. Acetoacetate

247. Which one of the following best describes the action of


phospholipase C?
A. It releases the fatty acyl chain from the sn-2 position of a phospholipid.
B. It releases the head group of a phospholipid, leaving phosphatidic acid.
C. It releases the fatty acyl chain from the sn-1 position of a phospholipid.
D. It cleaves a phospholipid into its phosphate-containing head group and a
diacylglycerol.
E. It releases the fatty acyl chains from the sn-1 and sn-2 positions of a
phospholipid.

248. Tay-Sachs disease is a lipid storage disease caused by a genetic


defect in deficiency which one of the following enzymes:
A. Hexosaminidase A
B. β-Galactosidase
C. Sphingomyelinase
D. Ceramidase
E. β-Glucosidase

249. Which of the plasma lipoproteins is best described as follows:


synthesized in the intestinal mucosa, containing a high concentration of
triacylglycerol and responsible for the transport of dietary lipids in the
circulation?
A. High-density lipoprotein
B. Intermediate density lipoprotein
C. Low-density lipoprotein
D. Very low density lipoprotein
E. Chylomicrons

250. Which of the plasma lipoproteins is best described as follows:


synthesized in the liver, containing a high concentration of
triacylglycerol and mainly cleared from the circulation by adipose tissue
and muscle?
A. Very low density lipoprotein
53
B. Chylomicrons
C. High-density lipoprotein
D. Intermediate density lipoprotein
E. Low-density lipoprotein

251. Which of the plasma lipoproteins is best described as follows: formed


in the circulation by removal of triacylglycerol from very low density
lipoprotein, contains cholesterol taken up from high-density lipoprotein
delivers cholesterol to extrahepatic tissues?
A. Chylomicrons
B. High-density lipoprotein
C. Low-density lipoprotein
D. Intermediate density lipoprotein
E. Very low density lipoprotein

252. Which of the following will be elevated in the bloodstream about 2


hours after eating a high-fat meal?
A. High-density lipoprotein
B. Chylomicrons
C. Ketone bodies
D. Nonesterified fatty acids
E. Very low density lipoprotein

253. Which of the following will be elevated in the bloodstream about 4


hours after eating a high-fat meal?
A. Chylomicrons
B. High-density lipoprotein
C. Ketone bodies
D. Very low density lipoprotein
E. Nonesterified fatty acids

254. Which one of the following statements concerning the biosynthesis


of cholesterol is CORRECT?
A. All the carbon atoms in the cholesterol synthesized originate from acetyl-
CoA.
B. The rate-limiting step is the formation of 3-hydroxy 3-methylglutaryl-CoA
(HMG-CoA) by the enzyme HMG-CoA synthase.
C. Synthesis occurs in the cytosol of the cell.
D. Squalene is the first cyclic intermediate in the pathway.
E. The initial substrate is mevalonate.

255. The class of drugs called statins have proved very effective against
hypercholesterolemia, a major cause of atherosclerosis and associated
cardiovascular disease. These drugs reduce plasma cholesterol levels by:
A. Preventing absorption of cholesterol from the intestine.
54
B. Increasing the excretion of cholesterol from the body via conversion to
bile acids.
C. Increasing the rate of degradation of 3-hydroxy-3- methylglutaryl CoA
reductase.
D. Stimulating the activity of the LDL receptor in the liver.
E. Inhibiting the conversion of 3-hydroxy-3-methylglutaryl-CoA to
mevalonate in the pathway for cholesterol biosynthesis.

256. Which of the following statements about bile acids (or bile salts) is
INCORRECT?
A. Primary bile acids are synthesized in the liver from cholesterol.
B. Secondary bile acids are produced by modification of primary bile acids
in the liver.
C. Bile acids are needed for the breakdown of fats by pancreatic lipase.
D. Bile acids facilitate the absorption of the products of lipid digestion in the
jejunum.
E. Bile acids are recirculated between the liver and the small intestine in
the enterohepatic circulation.

257. A 35-year-old man with severe hypercholesterolemia has a family


history of deaths at a young age from heart disease and stroke. Which of
the following genes is likely to be defective?
A. Apolipoprotein E
B. The LDL receptor
C. Lipoprotein lipase
D. PCSK9
E. LCAT

258. Stool test detects in the patient’s feces a large amount of undigested
fats. This patient is the most likely to have disturbed secretion of the
following enzymes:
A. Pancreatic amylase
B. Pancreatic proteases
C. Pancreatic lipases
D. Bile lipase
E. Gastric protease

259. A patient is diagnosed with glucocerebroside lipidosis (Gaucher’s


disease) that manifests as splenomegaly, liver enlargement, affected bone
tissue, and neuropathies. What enzyme of complex lipid catabolism is
deficient, thus causing this disease?
A. Hexosaminidase
B. Sphingomyelinase
C. β-galactosidase
D. Glucocerebrosidase
55
E. Hyaluronidase

260. A 3-year-old girl with mental retardation has been diagnosed with
sphingomyelin lipidosis (Niemann-Pick disease). In this condition the
synthesis of the following substance is disturbed:
A. Sphingomyelinase
B. Glycosyltransferase
C. Sphingosine
D. Ceramides
E. Gangliosides

261. A drycleaner’s worker has been found to have hepatic steatosis. This
pathology can be caused by the disruption of synthesis of the following
substance:
A. Tristearin
B. Urea
C. Phosphatidic acid
D. Cholic acid
E. Phosphatidylcholine

262. Increased HDL levels decrease the risk of atherosclerosis. What is


the mechanism of HDL anti-atherogenic action?
A. They supply tissues with cholesterol
B. They are involved in the breakdown of cholesterol
C. They activate the conversion of cholesterol to bile acids
D. They remove cholesterol from tissues
E. They promote absorption of cholesterol in the intestine

263. A 39-year-old female patient with a history of diabetes was


hospitalized in a precomatose state for diabetic ketoacidosis. This
condition had been caused by an increase in the following metabolite
level:
A. Citrate
B. Acetoacetate
C. Alpha-ketoglutarate
D. Malonate
E. Aspartate

264. Disruption of nerve fiber myelinogenesis causes neurological


disorders and mental retardation. These symptoms are typical for
hereditary and acquired alterations in the metabolism of:
A. Neutral fats
B. Higher fatty acids
C. Sphingolipids
D. Cholesterol
56
E. Phosphatidic acid

265. A doctor administered Allopurinol to a 26-year-old young man with


the symptoms of gout. What pharmacological action of Allopurinol
ensures therapeutical effect?
A. By inhibiting uric acid synthesis.
B. By inhibiting leukocyte migration into the joint.
C. By general analgesic effect.
D. By general anti-inflammatory effect.
E. By increasing uric acid excretion.

266. Blood of a 12 year old boy has a low concentration of uric acid and
accumulation of xanthine and hypoxanthine. This child has genetic defect
of the following enzyme:
A. Arginase.
B. Urease.
C. Ornithine carbamoyltransferase.
D. Glycerylkinase.
E. Xanthine oxidase.

267. A 48 year old patient complained about intense pain, slight swelling
and reddening of skin over the joints, temperature rise up to 38o. Blood
analysis revealed high concentration of urates. This condition might be
caused by disturbed metabolism of:
A. Collagen.
B. Cholesterol.
C. Pyrimidines.
D. Purines.
E. Carbohydrates.

268. A 46 year old patient applied to a doctor complaining about joint


pain that becomes stronger the day before weather changes. Blood
examination revealed strengthened concentration of uric acid. The most
probable cause of the disease is the intensified disintegration of the
following substance:
A. Cytidine monophosphate.
B. Uridine triphosphate.
C. Adenosine monophosphate.
D. Uridine monophosphate.
E. Thymidine monophosphate.

269. A 42-year man suffering from gout has increased level of urinary acid
in the blood. Allopurinol was prescribed to decrease the level of urinary
acid. Competitive inhibitor of which enzyme is allopurinol?
A. Adenosine deaminase.
57
B. Xanthine oxidase.
C. Adenine phosphoribosil transferase.
D. Hypoxantin phosphoribosil transferase.
E. Guaninedeaminase.

270. A 65-year-old suffering from the gout man complains of the pain in
the kidney's region. On ultrasonic examination the renal calculi were
revealed. As a result of what process were they formed?
A. Decay of purine nucleotides.
B. Protein catabolism.
C. Ornithine cycle.
D. Heme decay.
E. Restoration of cysteine.

271. Methotrexate (structural analogue of the folic acid which is


competitive inhibitor of the dihydrofolatreductase) is prescribed for
treatment of the malignant tumor. On which level does methotrexate
inhibit synthesis of the nucleic acids?
A. Replication.
B. Transcription.
C. Mononucleotide synthesis.
D. Reparation.
E. Processing.

272. An oncological patient had been administered methotrexate. With


time target cells of the tumor lost sensitivity to this drug. At the same
time the change in gene expression of the following enzyme is observed:
A. Thiaminase.
B. Dehydropholate reductase.
C. Deaminase.
D. Pholate oxidase.
E. Pholate decarboxylase.

273. The boy of 8 years old has Lesh-Nyhan disease. In blood


concentration of a uric acid is increased. Which process infringement is
the reason of this hereditary disease?
A. Decay of purine nucleotides
B. Synthesis of purine nucleotides
C. Synthesis of pyrimidine nucleotides
D. Dissociation of pyrimidine nucleotides
E. Formation of deoxynucleotids

274. For treatments of malignant tumours prescribed metatrexate -


structural analogue of a folic acid which is competitive inhibitor of
dehydrofolicreductase and consequently suppresses synthesis of:
58
A. Monosaccharides
B. Fatty acids
C. Glycerophosphatides
D. Glycogen
E. Nucleotides

275. The person of 58 years old has addressed to the doctor with the
complaint to a pain in joints. At inspection increase of concentration of a
uric acid in blood and urine is revealed. Specify, at what disintegration of
substances the uric acid is formed?
A. Pyrimidine nucleotide
B. Purines nucleotide
C. Amino acids
D. Proteins
E. chromoproteins

276. A patient has increased content of uric acid in his blood that is
clinically presented by pain syndrome as a result of urate deposition in
the joints. What process does this acid result from?
A. Lysis of pyrimidine nucleotides
B. Heme catabolism
C. Lysis of purine nucleotides.
D. Proteolysis
E. Reutilization of purine bases.

277. Continuous treatment of cancer patients with methotrexate over


time reduces the target cell's sensitivity to the drug. In this case gene
amplification of the following enzyme is observed:
A. Thioredoxin reductase
B. Thiaminase
C. Deaminase
D. Dihydrofolate reductase
E. -

278. DNA replication occurs during the cell division when a signal is
received from the cytoplasm, and a certain portion of the DNA helix is
unwound and divided into two chains. The helix is unwound by the
following enzyme:
A. Helicase
B. RNA polymerase
C. Ligase
D. Restrictase
E. DNA polymerase
59
279. RNA-polymerase B (II) is blocked due to amanitine poisoning (poison
of death-cup). It disturbes:
A. Synthesis of tRNA.
B. Synthesis of mRNA.
C. Reverse transcription.
D. Primers synthesis.
E. Maturation of mRNA.

280. An experiment proved that UV-radiated cells of patients with


xeroderma pigmentosum restore the native DNA structure slower than
cells of healthy individuals as a result of reparation enzyme defection.
What enzyme helps this process?
A. RNA ligase.
B. Primase.
C. Endonuclease.
D. DNA polymerase III.
E. DNA gyirase.

281. It was found out that some compounds, for instance fungi toxins and
some antibiotics can inhibit activity of RNA-polymerase. What process
will be disturbed in a cell in case of inhibition of this enzyme?
A. Processing.
B. Replication.
C. Translation.
D. Transcription.
E. Reparation.

282. Tuberculosis can be treated by means of combined chemotherapy


that includes substances with different mechanisms of action. Which
antituberculous medication inhibits transcription of RNA into DNA in
mycobacteria?
A. Streptomycin.
B. Isoniazid.
C. Ethionamide.
D. Para-aminosalicylic acid.
E. Rifampicin.

283. A patient has low rate of magnesium ions that are necessary for
affixion of ribosomes to the endoplasmic reticulum. It is known that it
causes disturbance of protein biosynthesis. At which stage is protein
biosynthesis impaired?
A. Translation.
B. Amino acid activation.
C. Replication.
D. Transcription.
60
E. Termination.

284. It was revealed that T-lymphocytes were affected by HIV. Virus


enzyme - reverse transcriptase (RNA-dependent DNA-polymerase) -
catalyzes the synthesis of:
A. Virus informational RNA on the template of DNA
B. DNA on the template of virus RNA. .
C. DNA on virus ribosomal RNA.
D. Viral DNA on DNA template.
E. mRNA on the template of virus protein.

285. A doctor prescribed a cephalosporin antibiotic to the patient after


appendectomy for infection prevention. Antimicrobial activity of this
group of antibiotics is based upon the disturbance of the following
process:
A. Choline esterase block.
B. Ribosome protein synthesis.
C. Microbial wall formation.
D. Energy metabolism.
E. Nucleic acid synthesis.

286. General structure of eukaryotic genes is as follows: exon-intron-exon.


Such functional structure of a gene leads to certain specifics of the
transcription process. Which sequence will correspond with precursor
mRNA (immature)?
A. Exon-exon-intron
B. Exon-exon
C. Exon-intron-exon
D. Intron-exon
E. Exon-intron

287. A mutation has occurred in a cell in the first exon of the structural
gene. The number of nucleotide pairs changed from 290 to 250. Name this
type of mutation:
A. Insertion
B. Deletion
C. Duplication
D. Translocation
E. Nullisomy

288. A man is a carrier of HIV that is an RNA virus. The cells of this
patient synthesize viral DNA. This process is based on:
A. Replication
B. Transcription
C. Reverse transcription
61
D. Repair
E. Translation

289. A 52-year-old man presents with fever and pain in the joints. Both of
his first metatarsophalangeal articulations are deformed, swollen, and
reddened. Blood urea is high. The patient is diagnosed with gout. What is
the main developmental factor in the pathogenesis of this disease?
A. Argininosuccinic aciduria
B. Hyperazotemia
C. Hyperaminoacidemia
D. Hyperuricemy
E. Citrullinuria

290. Part of the DNA chain turned about 180 degrees due to gamma
radiation. What type of mutation took place in the DNA chain?
A. Deletion
B. Doubling
C. Translocation
D. Replication
E. Inversion

291. During cell division, DNA replication occurs by a signal from the
cytoplasm, and a certain portion of the DNA helix unwinds and splits into
two individual strains. What enzyme facilitates this process?
A. Helicase
B. RNA polymerase
C. Ligase
D. Restrictase
E. DNA polymerase

292. In cancer patients who have been continuously receiving


methotrexate, the target cells of tumor with time become insensitive to
this drug. In this case, gene amplification of the following enzyme is
observed:
A. Thiaminase
B. Dihydrofolate reductase
C. Deaminase
D. Thioredoxin reductase
E. -

293. Examination of a patient revealed reduced contents of magnesium


ions that are necessary for attachment of ribosomes to the granular
endoplasmatic reticulum. It is known that it causes disturbance of protein
biosynthesis. What stage of protein biosynthesis will be disturbed?
A. Transcription
62
B. Translation
C. Replication
D. Aminoacid activation
E. Termination

294. Labeled aminoacids alanine and tryptophane were introducted to a


mouse in order to study localization of protein biosynthesis in its cells.
Around what organellas will the accumulation of labeled amino acids be
observed?
A. Agranular endoplasmic reticulum
B. Cell centre
C. Ribosomes
D. Lysosomes
E. Golgi apparatus

295. RNA that contains AIDS virus penetrated into a leukocyte and by
means of reverse transcriptase forced a cell to synthesize a viral DNA.
This process is based upon:
A. Operon repression
B. Reverse translation
C. Operon depression
D. Reverse transcription
E. Convariant replication

296. A patient has a disturbed absorbtion of fat hydrolysates. It might


have been caused by a deficit in the small intestine cavity:
A. Of sodium ions
B. Of liposoluble vitamins
C. Of bile pigments
D. Of lipolytic enzymes
E. Of bile acids

297. It was proved that a molecule of immature mRNA (precursor mRNA)


contained more triplets than amino acids found in the synthesized
protein. The reason for that is that translation is normally preceded by:
A. Initiation
B. Reparation
C. Mutation
D. Processing
E. Replication

298. Labelled amino acids alanine and tryptophane were injected to a


mouse in order to study localization of protein synthesis in its cells. The
labelled amino acids will be accumulated near the following organelles:
A. Smooth endoplasmic reticulum
63
B. Ribosomes
C. Cell centre
D. Lysosomes
E. Golgi apparatus

299. A mother consulted a doctor about her 5-year-old child who develops
erythemas, vesicular rash and skin itch under the influence of sun.
Laboratory studies revealed decreased iron concentration in the blood
serum, increased uroporphyrinogen I excretion with the urine. What is
the most likely inherited pathology in this child?
A. Methemoglobinemia
B. Hepatic porphyria
C. Erythropoietic porphyria
D. Coproporphyria
E. Intermittent porphyria

300. According to the model of double DNA helix that was suggested by
Watson and Creek, it was established that one of chains would not be lost
during replication and the second chain would be synthesized
complementary to the first one. What way of replication is it?
A. Analogous
B. Identical
C. Dispersed
D. Semiconservative
E. Conservative

301. A newborn child suffers from milk curdling in stomach, this means
that soluble milk proteins (caseins) transform to insoluble proteins
(paracaseins) by means of calcium ions and a certain enzyme. What
enzyme takes part in this process?
A. Renin
B. Pepsin
C. Gastrin
D. Secretin
E. Lipase

302. On an electron micrograph a scientist has identified a structure


formed by eight histone proteins and a part of DNA molecule which makes
about 1,75 revolutions around the molecules. Which structure has been
identified?
A. Elemetary fibril
B. Half-chromatid
C. Nucleosoma
D. Chromatid
E. Chromosome
64

303. Pterin derivatives (aminopterin and methotrexate) are the inhibitors


of dihydrofolate reductase, so that they inhibit the regeneration of
tetrahydrofolic acid from dihydrofolate. These drugs inhibit the
intermolecular tranfer of monocarbon groups, thus suppressing the
synthesis of the following polymer:
A. Protein
B. DNA
C. Homopolysaccharides
D. Gangliosides
E. Glycosaminoglycans

304. Glycogen polysaccharide is synthesized from the active form of


glucose. The immediate donor of glucose residues during the glycogenesis
is:
A. Glucose-1-phosphate
B. UDP-glucose
C. ADP-glucose
D. Glucose-6-phosphate
E. Glucose-3-phosphate

305. A patient has normally coloured stool including a large amount of


free fatty acids. The reason for this is a disturbance of the following
process:
A. Fat absorption
B. Fat hydrolysis
C. Biliary excretion
D. Choleresis
E. Lipase secretion

306. Deficiency of linoleic and linolenic acids in the body leads to the
skin damage, hair loss, delayed wound healing, thrombocytopenia, low
resistance to infections. These changes are most likely to be caused by
the impaired synthesis of the following substances:
A. Interleukins
B. Eicosanoids
C. Interferons
D. Catecholamines
E. Corticosteroids

307. Children with Lesch-Nyhan syndrome have a severe form of


hyperuricemia accompanied by the formation of tophi, urate calculi in the
urinary tracts, as well as serious neuro-psychiatric disorders. The cause of
this disease is the reduced activity of the following enzyme:
A. Xanthine oxidase
65
B. Hypoxanthine-guanine phosphoribosyltransferase
C. Dihydrofolate reductase
D. Thymidylate synthase
E. Karbamoyl phosphate synthetase

308. Due to the blockage of the common bile duct (which was
radiographically confirmed), the biliary flow to the duodenum was stopped.
We should expect the impairment of:
A. Protein absorption
B. Fat emulsification
C. Carbohydrate hydrolysis
D. Secretion of hydrochloric acid
E. Salivation inhibition

309. At the stage of translation in the rough endoplasmic reticulum, the


ribosome moves along the mRNA. Amino acids are joined together by
peptide bonds in a specific sequence, and thus polypeptide synthesis
takes place. The sequence of amino acids in a polypeptide corresponds to
the sequence of:
A. tRNA nucleotides
B. tRNA anticodons
C. rRNA nucleotides
D. mRNA codons
E. rRNA anticodons

310. A 12-year-old patient was found to have blood serum cholesterol at


the rate of 25 mmol/l. The boy has a history of hereditary familial
hypercholesterolemia, which is caused by the impaired synthesis of the
following protein receptors:
A. High density lipoproteins
B. Chylomicrons
C. Very low density lipoproteins
D. Intermediate density lipoproteins
E. Low density lipoproteins

311. A patient has a critical impairment of protein, fat and hydrocarbon


digestion. Most likely it has been caused by low secretion of the following
digestive juice:
A. Saliva
B. Pancreatic juice
C. Gastric juice
D. Bile
E. Intestinal juice
66
312. Patients with erythropoietic porphyria (Gunther’s disease) have teeth
that fluoresce with bright red color when subjected to ultraviolet
radiation; their skin is light-sensitive, urine is red-colored. What enzyme
can cause this disease, when it is deficient?
A. Uroporphyrinogen I synthase
B. Delta-aminolevulinate synthase
C. Uroporphyrinogen III cosynthase
D. Uroporphyrinogen decarboxylase
E. Ferrochelatase

313. A 4-year-old child with hereditary renal lesion has signs of rickets;
vitamin D concentration in blood is normal. What is the most probable
cause of rickets development?
A. Increased excretion of calcium
B. Impaired synthesis of calcitriol
C. Hyperfunction of parathyroid glands
D. Hypofunction of parathyroid glands
E. Lack of calcium in food

314. Diseases of respiratory system and circulatory disorders impair the


transport of oxygen, thus causing hypoxia. Under these conditions the
energy metabolism is carried out by anaerobic glycolysis. As a result, the
following substance is generated and accumulated in blood:
A. Pyruvic acid
B. Glutamic acid
C. Lactic acid
D. Citric acid
E. Fumaric acid

315. A 15-year-old patient has fasting plasma glucose level 4,8 mmol/l,
one hour after glucose challenge it becomes 9,0 mmol/l, in 2 hours it is
7,0 mmol/l, in 3 hours it is 4,8 mmol/l. Such parameters are
characteristic of:
A. Diabetes mellitus type 1
B. Diabetes mellitus type 2
C. Subclinical diabetes mellitus
D. Healthy person
E. Cushing’s disease

316. A 6-year-old child suffers from delayed growth, disrupted ossification


processes, decalcification of the teeth. What can be the cause?
A. Decreased glucagon production
B. Insulin deficiency
C. Hyperthyroidism
D. Vitamin D deficiency
67
E. Vitamin C deficiency

317. Obesity is a common disease. The aim of its treatment is to lower


content of neutral fats in the body. What hormone-sensitive enzyme is
the most important for intracellular lipolysis?
A. Protein kinase
B. Triacylglycerol lipase
C. Adenylate kinase
D. Diacylglycerol lipase
E. Monoacylglycerol lipase

318. A woman complains of visual impairment. Examination revealed


obesity in the patient and her fasting plasma glucose level is
hyperglycemic. What diabetes complication can cause visual
impairment/blindness?
A. Macroangiopathy
B. Microangiopathy
C. Atherosclerosis
D. Neuropathy
E. Glomerulopathy

319. A patient is diagnosed with chronic atrophic gastritis attended by


deficiency of Castle’s intrinsic factor.What type of anemia does the
patient have?
A. Iron refractory anemia
B. B12-deficiency anemia
C. Hemolytic anemia
D. Iron-deficiency anemia
E. Protein-deficiency anemia

320. The process of metabolism in the human body produces active forms
of oxygen, including superoxide anion radical O2•‒. This anion is
inactivated by the following enzyme:
A. Catalase
B. Peroxidase
C. Superoxide dismutase
D. Glutathione peroxidase
E. Glutathione reductase

321. Parents of a sick 5-year-old girl visited a genetic consultation.


Karyotype investigation revealed 46 chromosomes. One chromosome of
the 15th pair was abnormally long, having a part of the chromosome
belonging to the 21st pair attached to it. What mutation occurred in this
girl?
A. Translocation
68
B. Deletion
C. Insertion
D. Deficiency
E. Duplication

322. A 7-year-old child in the state of allergic shock caused by a bee sting
has been delivered into an emergency ward. High concentration of
histamine was observed in blood. Production of this amine was the result
of the following reaction:
A. Hydroxylation
B. Dehydrogenation
C. Deaminization
D. Decarboxylation
E. Reduction

323. A patient, who has been subsisting exclusively on polished rice, has
developed polyneuritis due to thiamine deficiency. What substance is an
indicator of such avitaminosis, when it is excreted with urine?
A. Malate
B. Methylmalonic acid
C. Pyruvic acid
D. Uric acid
E. Phenyl pyruvate

324. Cells of a person working in the Chornobyl Exclusion Zone have


undergone a mutation in DNA molecule. However, with time the damaged
interval of DNA molecule has been restored to its initial structure with a
specific enzyme. In this case the following occurred:
A. Replication
B. Transcription
C. Reverse transcription
D. Translation
E. Repair

325. After an extended treatment with sulfanilamides a patient has


developed macrocytic anemia. Production of active forms of the following
vitamin is disrupted in such a condition:
A. Thiamine
B. Riboflavin
C. Folic acid
D. Pyridoxine
E. Cyanocobalamin

326. A therapeutist has an appointment with a 40-year-old patient


complaining of recurrent pain attacks in his hallux joints and their
69
swelling. Urine analysis revealed its marked acidity and pink color. What
substances can cause such changes in urine?
A. Chlorides
B. Ammonium salts
C. Uric acid salt
D. Calcium phosphate
E. Magnesium sulfate

327. A patient presents with dry peeling skin, frequent cases of acute
respiratory diseases, xerophthalmia. What vitamin preparation should be
prescribed in this case?
A. Thiamine
B. Cyanocobalamin
C. Menadione (Vikasolum)
D. Retinol acetate
E. Ergocalciferol

328. Fructosuria is known to be connected with inherited deficiency of


fructose 1- phosphate aldolase. What product of fructose metabolism will
accumulate in the organism resulting in toxic action?
A. Glucose 1-phosphate
B. Fructose 1-phosphate
C. Glucose 6-phosphate
D. Fructose 1,6-biphosphate
E. Fructose 6-phosphate

329. A 6-year-old child suffers from delayed growth, disrupted ossification


processes, decalcification of the teeth. What can be the cause?
A. Decreased glucagon production
B. Insulin deficiency
C. Hyperthyroidism
D. Vitamin D deficiency
E. Vitamin C deficiency

330. Coenzyme A participates in numerous important metabolic reactions.


It is a derivative of the following vitamin:
A. Thiamine
B. Pantothenic acid
C. Niacin
D. Calciferol
E. Ubiquinone

331. When investigating human saliva it is necessary to assess its


hydrolytic properties. What substance should be used as a substrate in the
process?
70
A. Proteins
B. Fats
C. Fiber
D. Amino acids
E. Starch

332. A patient suffering from gout was prescribed allopurinol. What


pharmacological property of allopurinol provides therapeutic effect in this
case?
A. Acceleration of nitrogen-containing substances excretion
B. Competitive inhibition of xanthine oxidase
C. Acceleration of pyrimidine nucleotides catabolism
D. Deceleration of pyrimidine nucleotides salvage
E. Acceleration of nucleic acids synthesis

333. Blood of the patients with diabetes mellitus shows increased content
of free fatty acids. Name the most likely cause of this:
A. Accumulation of palmitoyl-CoA in cytosol
B. Activation of ketone bodies utilization
C. Activation of apoА1, apoА2, and apoА4 apolipoprotein synthesis
D. Increased activity of adipose triglyceride lipase
E. Decreased activity of plasma phosphatidylcholine-
cholesterolacyltransferase

334. During regular check-up a child is determined to have interrupted


mineralization of the bones. What vitamin deficiency can be the cause?
A. Riboflavin
B. Tocopherol
C. Folic acid
D. Cobalamin
E. Calciferol

335. Cytochrome oxidase is a hemeprotein that is an end component of


the mitochondrial respiratory chain. What reaction is catalyzed with this
enzyme?
A. Cytochrome synthesis
B. Transfer of reduced equivalents to molecular oxygen
C. Transfer of reduced equivalents to ubiquinone
D. Cytochrome splicing
E. Adenosine triphosphate synthesis

336. It is known that pentose-phosphate pathway actively functions in


the erythrocytes. What is the main function of this metabolic pathway in
the erythrocytes?
A. Counteraction to lipid peroxidation
71
B. Activation of microsomal oxidation
C. Neutralization of xenobiotics
D. Oxidation of glucose into lactate
E. Increase of lipid peroxidation

337. Streptomycin and other aminoglycosides prevent the joining of


formylmethionyl-tRNA by bonding with the 30S ribosomal subunit. This
effect leads to disruption of the following process:
A. Translation initiation in eukaryotes
B. Transcription initiation in prokaryotes
C. Translation initiation in prokaryotes
D. Transcription initiation in eukaryotes
E. Replication initiation in prokaryotes

338. An infant, who was on synthetic formula feeding, developed signs of


vitamin B1 deficiency. What reactions does this vitamin take part in?
A. Amino acids transamination
B. Keto acids oxidative decarboxylation
C. Amino acids decarboxylation
D. Proline hydroxylation
E. Redox reactions

339. Nitrogen is being excreted from the body mainly as urea. When
activity of a certain enzyme in the liver is low, it results in inhibition of
urea synthesis and nitrogen accumulation in blood and tissues. Name this
enzyme:
A. Aspartate aminotransferase
B. Urease
C. Carbamoyl phosphate synthetase
D. Amylase
E. Pepsin

340. A sick child presents with high content of phenyl pyruvate in urine
(normally it is practically absent). Blood phenylalanine level is 350 mg/L
(norm - 15 mg/L). What disease are these symptoms characteristic of?
A. Albinism
B. Phenylketonuria
C. Tyrosinosis
D. Alkaptonuria
E. Gout

341. A newborn presents with weak suckling, frequent vomiting, and


hypotonia. Blood and urine citrulline are very high. What metabolic
process is disturbed?
A. Tricarboxylic acid cycle
72
B. Glycolysis
C. Gluconeogenesis
D. Ornithine cycle
E. Cori cycle

342. Congenital pyruvate carboxylase deficiency causes physical and


mental retardation in children and leads to early death. It is characterized
by lactic acidemia, lactaciduria, and a number of metabolic disorders.
Among others, inhibition of the following occurs:
A. Citric acid cycle and gluconeogenesis
B. Glycolysis and glycogenolysis
C. Glycogenesis and glycogenolysis
D. Lipolysis and lipogenesis
E. Pentose-phosphate pathway and glycolysis

343. A 25-year-old young man came to the doctor complaining of general


weakness, rapid fatigability, irritability, reduced working ability, and
bleeding gums. What vitamin is likely to be deficient in this case?
A. Riboflavin
B. Thiamine
C. Retinol
D. Folic acid
E. Ascorbic acid

344. Dehydrogenases are enzymes that detach hydrogen atoms from the
substrate. What enzyme class is lactate dehydrogenase related to?
A. Isomerases
B. Oxidoreductases
C. Lipases
D. Transferases
E. Hydrolases

345. Low rate of vitamin B6 in the dietary intake leads to disturbance of


protein metabolism. What biochemical processes in the patient's organism
will become less active?
A. Hydrolysis
B. Phosphorilation
C. Transamination
D. Reduction-oxidation
E. Methylation

346. It is known, that some compounds are uncouplers for tissue


respiration and oxidative phosphorylation find out those one:
A. Carbon monooxide
B. Lactate
73
C. Antimycin A
D. Acetyl-CoA
E. 2,4-dinitrophenol

347. To what total ATP quantity is the full glucose oxidation and its
linking with phosphorylation equivalent?
A. 38
B. 8
C. 12
D. 52
E. 58

348. Formation of ribosome subunits in a cell was disturbed in course of


an experiment (by means of activated mutagenic factors). This will have
an effect on the following metabolic process:
A. Carbohydrate biosynthesis
B. Protein biosynthesis
C. ATP synthesis
D. Photosynthesis
E. Biological oxidation

349. Low level of albumins and fibrinogen was detected in the patient’s
blood. Decreased activity of what organelle of the liver hepatocytes can
most probably cause it?
A. Agranular endoplasmatic reticulum
B. Mitochondrions
C. Golgi complex
D. Granular endoplasmatic reticulum
E. Lysosomes

350. In evolution, molecular mechanism for correction of damaged DNA


molecules developed. This process is called:
A. Transcription
B. Translation
C. Replication
D. Processing
E. Repair

351. Genetic information is stored in DNA but does not participate


directly in protein synthesis within DNA cells. What process ensures
transfer of genetic information into polypeptide chain?
A. DNA replication
B. Formation of tRNA
C. Formation of iRNA
D. Translation
74
E. Formation of rRNA

352. Infectious diseases are treated with antibiotics (streptomycin,


erythromycin, chloramphenicol). They inhibit the following stage of
protein synthesis:
A. Transcription
B. Replication
C. Processing
D. Translation
E. Splicing

353. Inside a human cell the informational RNA containing both exons
and introns was delivered to the granular endoplasmic reticulum to the
ribosomes. What process does NOT take place?
A. Replication
B. Processing
C. Transcription
D. Translation
E. Prolongation

354. At the stage of translation in the rough endoplasmic reticulum, the


ribosome moves along the mRNA. Amino acids are joined together by
peptide bonds in a specific sequence, and thus polypeptide synthesis
takes place. The sequence of amino acids in a polypeptide corresponds to
the sequence of:
A. mRNA codons
B. tRNA nucleotides
C. tRNA anticodons
D. rRNA nucleotides
E. rRNA anticodons

355. In some areas of South Africa many people have sickle cell disease
characterized by red blood cells that assume an abnormal sickle shape
due to the substitution of glutamic acid for valine in the hemoglobin
molecule. What is the cause of this disease?
A. Disturbances of the mechanisms of genetic information transmission
B. Crossing-over
C. Gene mutation
D. Genomic mutation
E. Transduction

356. Mother had noticed her 5-year-old child’s urine to become dark in
colour. Bile pigments in urine were not detected. The diagnosis of
alkaptonouria was made. What pigment is deficient?
A. Phenylpyruvate decarboxylase
75
B. Phenylalanine hydroxylase
C. Homogentisic acid oxidase
D. Tyrosinase
E. Oxyphenylpyruvate oxidase

357. A 1,5 year old child was taken to the hospital. The examination
revealed dementia, disorder of motor functions regulation,
hypopigmentation of skin, high rate of phenylalanine in blood. What is
the most probable diagnosis?
A. Galactosemia
B. Tyrosinosis
C. Phenylketonuria
D. Down’s syndrome
E. Mucoviscidosis

358. In case of alkaptonuria, homogentisic acid is excreted in urine in


large amounts. The development of this disease is associated with a
disorder of metabolism of the following amino acid:
A. Phenylalanine
B. Alanine
C. Methionine
D. Tyrosine
E. Asparagine

359. A 2-year-old child experienced convulsions because of lowering


calcium ions concentration in the blood plasma. Function of what
structure is decreased?
A. Thymus
B. Hypophysis
C. Adrenal cortex
D. Pineal gland
E. Parathyroid glands

360. The alternate usage of dichlotiazide, etacrin acid and lasex did not
cause marked diuretic effect in the patient with marked peripheral edema.
The aldosterone level in the blood is increased. Indicate which medicine
should be prescribed:
A. Urea
B. Mannit
C. Clopamid
D. Spironolacton
E. Amilorid
76
361. The action of electric current on the exitable cell caused
depolarization of its membrane. Movement of what ions through the
membrane caused depolarization?
A. HCO3‒
B. Na+
C. Cl−
D. Ca2+
E. K+

362. There is only one hormone among the neurohormones which refers
to the derivatives of amino acids according to classification. Point it out:
A. Oxytocin
B. Thyroliberin
C. Vasopressin
D. Melatonin
E. Somatotropin

363. Some diseases reveal symptoms of aldosteronism with hypertension


and edema due to sodium retention in the organism. What organ of the
internal secretion is affected on aldosteronism?
A. Testicle
B. Adrenal glands
C. Ovaries
D. Pancreas
E. Hypophysis

364. There is an inhibited coagulation in the patients with bile ducts


obstruction, bleeding due to the low level of absorbtion of a vitamin. What
vitamin is in deficiency?
A. А
B. D
C. К
D. Е
E. Carotene

365. Substitution of the glutamic acid on valine was revealed while


examining initial molecular structure. For what inherited pathology is
this symptom typical?
A. Minkowsky-Shauffard disease
B. Sickle-cell anemia
C. Thalassemia
D. Favism
E. Hemoglobinosis
77
366. A worker has decreased buffer capacity of blood due to exhausting
muscular work The influx of what acid substance in the blood can cause
this symptom?
A. 3-phosphoglycerate
B. Pyruvate
C. 1,3-bisphosphoglycerate
D. α-ketoglutarate
E. Lactate

367. A 19-year-old female suffers from tachycardia in rest condition,


weight loss, excessive sweating, exophtalmos and irritability. What
hormone would you expect to find elevated in her serum?
A. ACTH
B. Cortisol
C. Mineralocorticoids
D. Thyroxine
E. Insulin

368. The concentration of albumins in human blood sample is lower than


normal. This leads to edema of tissues. What blood function is damaged?
A. Maintaining the body temperature
B. Maintaining the Ph level
C. Maintaining the oncotic blood pressure
D. Maintaining the blood sedimentation system
E. All answers are correct

369. The low specific gravity of the secondary urine (1,002) was found out
in the sick person. What is THE MOST DISTANT part of nephron where
concentration of secondary urine takes place?
A. In the collecting duck
B. In the nephron’s glomerulus
C. In proximal tubule of nephron
D. In ascending part of loop of Henle
E. In distal tubule of nephron

370. Periodic renal colics attacks are observed in a woman with primary
hyperparathyroidizm. Ultrasonic examination revealed small stones in the
kidneys. What is the most plausible reason of the stones’s formation?
A. Hyperphosphatemia
B. Hypercalcemia
C. Hypercholesterinemia
D. Hyperuricemia
E. Hyperkalemia
78
371. A man after 1,5 litre blood loss has suddenly reduced diuresis. The
increased secretion of what hormone caused such diuresis alteration?
A. Corticotropin
B. Natriuretic
C. Vasopressin
D. Cortisol
E. Parathormone

372. A 2-year-old child has got intestinal dysbacteriosis, which results in


hemorrhagic syndrome. What is the most likely cause of hemorrhage of
the child?
A. Fibrinogen deficiency
B. Activation of tissue thromboplastin
C. PP hypovitaminosis
D. Vitamin K insufficiency
E. Hypocalcemia

373. Intake of oral contraceptives containing sex hormones inhibits


secretion of the hypophysiae hormones. Secretion of which of the
indicated hormones is inhibited while taking oral contraceptives with sex
hormones?
A. Vasopressin
B. Thyrotropic
C. Somatotropic
D. Oxytocin
E. Follicle-stimulating

374. A patient with diabetes mellitus has been delivered in hospital in the
state of unconsciousness. Arterial pressure is low. The patient has
acidosis. Point substances, which accumulation in the blood results in
these manifestations:
A. Amino acids
B. Monosaccharides
C. Ketone bodies
D. High fatty acids
E. Cholesterol esters

375. Testosterone and it’s analogs increase the mass of skeletal muscles
that allows to use them for treatment of dystrophy. Due to interaction of
the hormone with what cell substrate is this action caused?
A. Membrane receptors
B. Nuclear receptors
C. Ribosomes
D. Chromatin
E. Proteins-activators of transcription
79

376. Increased fragility of vessels, enamel and dentine destruction


resulting from scurvy are caused by disorder of collagen maturation. What
stage of procollagen modification is disturbed under this avitaminosis?
A. Removal of C-ended peptide from procollagen
B. Hydroxylation of proline
C. Formation of polypeptide chains
D. Glycosylation of hydroxylysine residues
E. Detaching of N-ended peptide

377. A person has reduced diuresis, hypernatremia, hypokalemia.


Hypersecretion of what hormone can cause such changes?
A. Auricular sodiumuretic factor
B. Vasopressin
C. Adrenalin
D. Aldosterone
E. Parathormone

378. A considerable increase of activity of МВ-forms of CPK (creatine


phosphokinase) and LDH-1 was revealed on the examination of patient’s
blood. What is the most likely pathology?
A. Hepatitis
B. Rheumatism
C. Pancreatitis
D. Cholecystitis
E. Miocardial infarction

379. A patient with serious damage of muscular tissue was admitted to


the traumatological department. What biochemical urine index will be
increased in this case?
A. Glucose
B. Common lipids
C. Creatine
D. Mineral salts
E. Uric acid

380. A pregnant woman had been having toxicosis with severe repeated
vomiting for 24 hours. In the end of the day there appeared tetanic
convulsions and fluid loss. What shift of acid-base state caused these
changes?
A. Gaseous alkalosis
B. Gaseous acidosis
C. Excretory alkalosis
D. Metabolic acidosis
E. Excretory acidosis
80

381. A 40-year-old patient complains of intensive heartbeats, sweating,


nausea, visual impairment, arm tremor, hypertension. From his
anamnesis: 2 years ago he was diagnosed with pheochromocytoma.
Hyperproduction of what hormones causes the given pathology?
A. Catecholamines
B. Aldosterone
C. Glucocorticoids
D. ACTH
E. Thyroidal hormones

382. A 58-year-old patient with acute cardiac insufficiency has decreased


volume of daily urine - oligouria. What is the mechanism of this
phenomenon?
A. Decreased number of functioning glomeruli
B. Drop of oncotic blood pressure
C. Rise of hydrostatic blood pressure in capillaries
D. Decreased glomerular filtration
E. Reduced permeability of renal filter

383. Cerebral trauma caused increase of ammonia formation. What amino


acid takes part in removal of ammonia from cerebral tissue?
A. Tyrosine
B. Valine
C. Glutamic acid
D. Tryptophan
E. Lysine

384. Kidneys of a man under examination show increased resorbtion of


calcium ions and decreased resorbtion of phosphate ions. What hormone
causes this phenomenon?
A. Thyrocalcitonin
B. Hormonal form D3
C. Aldosterone
D. Parathormone
E. Vasopressin

385. Diabetes mellitus causes ketosis as a result of activated oxidation of


fatty acids. What disorders of acid-base equilibrium may be caused by
excessive accumulation of ketone bodies in blood?
A. Any changes won’t happen
B. Metabolic alkalosis
C. Metabolic acidosis
D. Respiratory acidosis
E. Respiratory alkalosis
81

386. A patient presents high activity of LDH1,2, aspartate


aminotransferase, creatine phosphokinase. In what organ (organs) is the
development of a pathological process the most probable?
A. In skeletal muscles (dystrophy, atrophy)
B. In the heart muscle (initial stage of myocardium infarction)
C. In kidneys and adrenals
D. In connective tissue
E. In liver and kidneys

387. In some regions of South Africa there is a spread sickle-shaped cell


anemia, in which erythrocytes have shape of a sickle as a result of
substitution of glutamin by valine in the hemoglobin molecule. What is
the cause of this disease?
A. Disturbance of mechanisms of genetic information realization
B. Crossingover
C. Transduction
D. Genomic mutations
E. Gene mutation

388. A 30 y.o. woman had been ill for a year when she felt pain in the area
of joints for the first time, they got swollen and skin above them became
reddened. Provisional diagnosis is rheumatoid arthritis. One of the most
probable causes of this disease is a structure alteration of a connective
tissue protein:
A. Collagen
B. Mucin
C. Myosin
D. Ovoalbumin
E. Troponin

389. As a result of exhausting muscular work a worker has largely reduced


buffer capacity of blood. What acidic substance that came to blood caused
this phenomenon?
A. Pyruvate
B. 1,3-bisphosphoglycerate
C. Lactate
D. 3-phosphoglycerate
E. -

390. A patient complains of frequent diarrheas, especially after


consumption of fattening food, and of body weight loss. Laboratory
examination revealed steatorrhea; hypocholic feces. What can be the
cause of this condition?
A. Mucous membrane inflammation of small intestine
82
B. Obturation of biliary tracts
C. Lack of pancreatic lipase
D. Lack of pancreatic phospholipase
E. Unbalanced diet

391. A 35 y.o. patient who often consumes alcohol was treated with
powerful diuretics. There appeared serious muscle and heart weakness,
vomiting, diarrhea, AP- 100/60 mm Hg, depression. This condition is
caused by intensified excretion with urine of:
A. Sodium
B. Chlorine
C. Calcium
D. Potassium
E. Phosphates

392. Analysis of blood serum of a patient revealed increase of alanine


aminotransferase and aspartate aminotransferase level. What cytological
changes can cause such a situation?
A. Disorder of enzyme systems of cells
B. Cellular breakdown
C. Disturbed function of energy supply of cells
D. Disturbance of genetic apparatus of cells
E. Disturbance of cellular interrelations

393. A patient suffers from hepatic cirrhosis. Examination of which of the


following substances excreted by urine can characterize the state of
antitoxic function of liver?
A. Ammonium salts
B. Hippuric acid
C. Kreatinine
D. Uric acid
E. Aminoacids

394. A 2 y.o. child has convulsions as a result of lowered concentration of


calcium ions in blood plasma. It is caused by reduced function of:
A. Hypophysis
B. Adrenal cortex
C. Pineal gland
D. Thymus
E. Parathyroid glands

395. Parents of a 10 y.o. boy consulted a doctor about extension of hair-


covering, growth of beard and moustache, low voice. Intensified secretion
of which hormone must be assumed?
A. Of progesterone
83
B. Of somatotropin
C. Of oestrogen
D. Of testosterone
E. Of cortisol

396. A 48 y.o. patient was admitted to the hospital with complaints about
weakness, irritability, sleep disturbance. Objectively: skin and scleras are
yellow. In blood: conjugated bilirubin, cholalemia. Feces are acholic. Urine
is of dark colour (bilirubin). What jaundice is it?
A. Parenchymatous
B. Mechanic
C. Hemolytic
D. Gilbert’s syndrome
E. Crigler-Najjar syndrome

397. To prevent postoperative bleeding a 6 y.o. child was administered


vicasol that is a synthetic analogue of vitamin K. Name post-translational
changes of blood coagulation factors that will be activated by vicasol:
A. Phosphorylation of serine radicals
B. Partial proteolysis
C. Polymerization
D. Glycosylation
E. Carboxylation of glutamic acid

398. Examination of a patient revealed hyperkaliemia and hyponatremia.


Low secretion of which hormone may cause such changes?
A. Vasopressin
B. Cortisol
C. Aldosteron
D. Parathormone
E. Natriuretic

399. A 4 y.o. boy has had recently serious viral hepatitis. Now there are
such clinical presentations as vomiting, loss of consciousness,
convulsions. Blood analysis revealed hyperammoniemia. Disturbance of
which biochemical process causes such pathological condition of the
patient?
A. Disturbed neutralization of biogenic amines
B. Increased putrefaction of proteins in bowels
C. Activation of aminoacid decarboxylation
D. Inhibition of transamination enzyms
E. Disturbed neutralization of ammonia in liver

400. Inhabitants of territories with cold climate have high content of an


adaptive thermoregulatory hormone. What hormone is meant?
84
A. Insulin
B. Glucagon
C. Thyroxin
D. Somatotropin
E. Cortisol

401. Examination of a 43 y.o. anephric patient revealed anemia


symptoms. What is the cause of these symptoms?
A. Reduced synthesis of erythropoietins
B. Enhanced destruction of erythrocytes
C. Iron deficit
D. Vitamin B12 deficit
E. Folic acid deficit

402. Glutamate decarboxylation results in formation of inhibitory


transmitter in CNS. Name it:
A. Histamine
B. Glutathione
C. GABA
D. Serotonin
E. Asparagine

403. Osmotic pressure of a man’s blood plasma is 350 mosmole/l


(standard pressure is 300 mosmole/l). First of all it will result in high
secretion of the following hormone:
A. Aldosteron
B. Cortisol
C. Adrenocorticotropin
D. Vasopressin
E. Natriuretic

404. A patient was ill with burn disease that was complicated by
disseminated intravascular coagulation syndrome (DIC syndrome). Which
stage of DIC syndrome can be suspected if it is known that the patient’s
blood coagulates in less than 3 minutes?
A. Hypocoagulation
B. Hypercoagulation
C. Transition phase
D. Fibrinolysis
E. Terminal

405. A patient who had been working hard under conditions of elevated
temperature of the environment, has now a changed quantity of blood
plasma proteins. What phenomenon is the case?
A. Disproteinemia
85
B. Absolute hypoproteinemia
C. Absolute hyperproteinemia
D. Relative hyperproteinemia
E. Paraproteinemia

406. An infant has apparent diarrhea resulting from improper feeding.


One of the main diarrhea effects is plentiful excretion of sodium
bicarbonate. What form of acid-base balance disorder is the case?
A. Metabolic alkalosis
B. Respiratory acidosis
C. Respiratory alkalosis
D. Metabolic acidosis
E. No disorders of acid-base balance will be observed

407. A 20 year old patient complains of general weakness, dizziness,


quick fatigability. Blood analysis results: Hb - 80 g/l. Microscopical
examination results: erythrocytes are of modified form. This condition
might be caused by:
A. Obstructive jaundice
B. Hepatocellular jaundice
C. Acute intermittent porphyria
D. Sickle-cell anemia
E. Addison’s disease

408. As a result of long-term starvation the glomerular filtration of a man


was accelerated by 20%. The most probable cause of filtration changes
under such conditions is:
A. Rise of systemic arterial pressure
B. Increased permeability of renal filter
C. Fall of oncotic pressure of blood plasma
D. Growth of filtration coefficient
E. Increase of renal plasma flow

409. A 38 year old patient suffers from rheumatism in its active phase.
What laboratory characteristic of blood serum is of diagnostic importance
in case of this pathology?
A. Uric acid
B. Urea
C. Creatinine
D. C-reactive protein
E. Transferrin

410. A 65 year old man suffering from gout complains of kidney pain.
Ultrasound examination revealed renal calculi. The most probable cause of
86
calculi formation is the strengthened concentration of the following
substance:
A. Cholesterol
B. Bilirubin
C. Uric acid
D. Urea
E. Cystine

411. A 44 year old woman complains of general weakness, heart pain, and
significant increase of body weight. Objectively: moon face, hirsutism, AP
is 165/100 mm Hg, height - 164 cm, weight - 103 kg; the fat is mostly
accumulated on her neck, thoracic girdle, belly. What is the main
pathogenetic mechanism of obesity?
A. Reduced production of thyroid hormones
B. Increased production of glucocorticoids
C. Increased insulin production
D. Reduced glucagon production
E. Increased mineralocorticoid production

412. A patient is ill with hepatocirrhosis. State of antitoxic liver function


can be characterized by examination of the following substance exreted
by urine:
A. Hippuric acid
B. Ammonium salts
C. Creatinine
D. Uric acid
E. Amino acids

413. A concentrated solution of sodium chloride was intravenously


injected to an animal. This caused decreased reabsorption of sodium ions
in the renal tubules. It is the result of the following changes of hormonal
secretion:
A. Vasopressin reduction
B. Vasopressin increase
C. Aldosterone reduction
D. Aldosterone increase
E. Reduction of atrial natriuretic factor

414. A patient has a decreased vasopressin synthesis that causes polyuria


and as a result of it evident organism dehydratation. What is the
mechanism of polyuria development?
A. Reduced tubular reabsorption of Na ions
B. Reduced tubular reabsorption of protein
C. Reduced tubular reabsorption of water
D. Reduced glucose reabsorption
87
E. Acceleration of glomerular filtration

415. As a result of posttranslative modifications some proteins taking


part in blood coagulation, particularly prothrombin, become capable of
calcium binding. The following vitamin takes part in this process:
A. C
B. K
C. A
D. B1
E. B2

416. A patient ill with essential hypertension was recommended a drug


that prevents thrombosis. It is to be taken parenterally. What drug is it?
A. Heparin
B. Amben
C. Protamine sulfate
D. Neodicumarin
E. Syncumar

417. A patient being treated for viral hepatitis type B got symptoms of
hepatic insufficiency. What blood changes indicative of protein
metabolism disorder will be observed in this case?
A. Absolute hypoalbuminemia
B. Absolute hyperalbuminemia
C. Absolute hyperfibrinogenemia
D. Proteinic blood composition is unchanged
E. Absolute hyperglobulinemia

418. A patient with suspected diagnosis "progressing muscular dystrophy"


got his urine tested. What compound will confirm this diagnosis if found
in urine?
A. Calmodulin
B. Creatine
C. Collagen
D. Porphyrin
E. Myoglobin

419. A driver who got a trauma in a road accident and is shocked has
reduction of daily urinary output down to 300 ml. What is the main
pathogenetic factor of such diuresis change?
A. Drop of oncotic blood pressure
B. Increased vascular permeability
C. Decreased number of functioning glomerules
D. Drop of arterial pressure
E. Secondary hyperaldosteronism
88

420. A patient takes digoxin for treatment of cardiac insufficiency. What


diuretic may increase digoxin toxicity due to the intensified excretion of
K ions?
A. Spironolactone
B. Panangine
C. Siliborum
D. Lisinopril
E. Hydrochlorothiazide

421. Ultramicroscopical examination of "dark" hepatocyte population in


the cell cytoplasm detected a developed granular endoplasmic reticulum.
What function has this organelle in these cells?
A. Carbohydrate synthesis
B. Deintoxicative function
C. Bile production
D. Synthesis of blood plasma proteins
E. Calcium ion depositing

422. A patient came to the hospital complaining about quick fatigability


and apparent muscle weakness. Examination revealed an autoimmune
disease that causes disorder of functional receptor condition in
neuromuscular synapses. What transmitter will be blocked?
A. Noradrenalin
B. Dopamine
C. Acetylcholine
D. Serotonin
E. Glycine

423. Plasmic factors of blood coagulation are exposed to post-


translational modification with the participation of vitamin K. It is
necessary as a cofactor in the enzyme system of γ-carboxylation of
protein factors of blood coagulation due to the increased affinity of their
molecules with calcium ions. What amino acid is carboxylated in these
proteins?
A. Valine
B. Arginine
C. Serine
D. Phenylalanine
E. Glutamic acid

424. Pharmacological effects of antidepressants are connected with


inhibition of an enzyme catalyzing inactivation of biogenic amines such
as noradrenaline and serotonine. What enzyme participates in this
process?
89
A. Transaminase
B. Decarboxylase
C. Peptidase
D. Monoamine oxidase
E. Lyase

425. A patient underwent a surgery for excision of a cyst on pancreas.


After this he developed haemorrhagic syndrome with apparent disorder of
blood coagulation. Development of this complication can be explained by:
A. Activation of fibrinolytic system
B. Insufficient fibrin production
C. Reduced number of thrombocytes
D. Activation of anticoagulation system
E. Activation of Christmas factor

426. Examination of a 27-year-old patient revealed pathological changes


in liver and brain. Blood plasma analysis revealed an abrupt decrease in
the copper concentration, urine analysis revealed an increased copper
concentration. The patient was diagnosed with Wilson’s degeneration. To
confirm the diagnosis it is necessary to study the activity of the following
enzyme in blood serum:
A. Carbonic anhydrase
B. Ceruloplasmin
C. Xanthine oxidase
D. Leucine aminopeptidase
E. Alcohol dehydrogenase

427. Examination of a patient with frequent haemorrhages from the


internal organs and mucous membranes revealed proline and lysine within
the collagen fibers. Disorder of their hydroxylation is caused by lack of
the following vitamin:
A. Vitamin K
B. Vitamin A
C. Vitamin C
D. Vitamin B1
E. Vitamin E

428. Products of some proteins hydrolysis and modification are the


biologically active substances called hormones. Lipotropin, corticotropin,
melanotropin and endorphins are synthesized in the hypophysis of the
following protein:
A. Neuroalbumin
B. Proopiomelanocortin (POMC)
C. Neurostromin
D. Neuroglobulin
90
E. Thyroglobulin

429. In patients with the biliary tract obstruction the blood coagulation is
inhibited; the patients have frequent haemorrhages caused by the
subnormal assimilation of the following vitamin:
A. A
B. D
C. E
D. C
E. K

430. A patient suffering from coronary artery disease had taken a certain
drug many times a day in order to arrest stenocardia attacks. Overdose of
this drug finally caused intoxication. Objectively: cyanotic skin and
mucous membranes, dramatic fall in the arterial pressure, tachycardia,
respiration inhibition. Blood has increased concentration of
methemoglobin. The drug the patient had taken relates to the following
group:
A. Organic nitrates
B. α-adrenoceptor blockers
C. Calcium channel blockers
D. Adenosine drugs
E. Myotropic spamolytics

431. A middle-aged man went to a foreign country because he had been


offered a job there. However he had been unemployed for quite a long
time. What endocrine glands were exhausted most of all in this man?
A. Parathyroid glands
B. Adrenal glands
C. Seminal glands
D. Substernal gland
E. Thyroid gland

432. After a sprint an untrained person develops muscle hypoxia. This


leads to the accumulation of the following metabolite in muscles:
A. Ketone bodies
B. Lactate
C. Acetyl CoA
D. Glucose 6-phosphate
E. Oxaloacetate

433. Examination of a patient revealed overgrowth of facial bones and soft


tissues, tongue enlargement, wide interdental spaces in the enlarged
dental arch. What changes of the hormonal secretion are the most likely?
A. Hyposecretion of the somatotropic hormone
91
B. Hypersecretion of insulin
C. Hyposecretion of thyroxin
D. Hypersecretion of the somatotropic hormone
E. Hyposecretion of insulin

434. A 32-year-old patient consulted a doctor about the absence of


lactation after parturition. Such disorder might be explained by the
deficit of the following hormone:
A. Somatotropin
B. Vasopressin
C. Prolactin
D. Thyrocalcitonin
E. Glucagon

435. Depressions and emotional insanities result from the deficit of


noradrenalin, serotonin and other biogenic amines in the brain. Their
concentration in the synapses can be increased by means of the
antidepressants that inhibit the following enzyme:
A. Diamine oxidase
B. L-amino-acid oxidase
C. D-amino-acid oxidase
D. Monoamine oxidase
E. Phenylalanine-4-monooxygenase

436. A 9-month-old infant is fed with artificial formulas with unbalanced


vitamin B6 concentration. The infant presents with pellagral dermatitis,
convulsions, anaemia. Convulsion development might be caused by the
disturbed formation of:
A. DOPA
B. GABA
C. Histamine
D. Serotonin
E. Dopamine

437. After a disease a 16-year-old boy is presenting with decreased


function of protein synthesis in the liver as a result of vitamin K
deficiency. This may cause disorder of:
A. Erythrocyte sedimentation rate
B. Blood coagulation
C. Anticoagulant production
D. Erythropoietin production
E. Osmotic blood pressure

438. As a result of a road accident a driver has gotten a trauma. Now he is


in shock condition and presents with a decrease in daily diuresis down to
92
300 ml. What is the main pathogenetic factor of such alteration in the
diuresis?
A. Oncotic blood pressure drop
B. Increase in vascular permeability
C. Decrease in number of the functioning glomerules
D. Arterial pressure drop
E. Secondary hyperaldosteronism

439. A patient suffers from the haemorrhagic syndrome that shows itself
in frequent nasal bleedings, posttraumatic and spontaneous
intracutaneous and intra-articular haemorrhages. After a laboratory study
a patient was diagnosed with the type B haemophilia. This disease is
provoked by the deficit of the following factor of blood coagulation:
A. VIII
B. XI
C. V
D. VII
E. IX

440. A full-term newborn child has yellowish skin and mucous membranes.
This might be probably caused by temporary deficiency of the following
enzyme:
A. Uridine transferase
B. UDP-glucoronyltransferase
C. Heme synthetase
D. Heme oxygenase
E. Biliverdin reductase

441. Examination of a patient revealed typical presentations of


collagenosis. This pathology is characterized by increase of the following
urine index:
A. Arginine
B. Glucose
C. Mineral salts
D. Hydroxyproline
E. Ammonium salts

442. A 48 year old patient complained about intense pain, slight swelling
and reddening of skin over the joints, temperature rise up to 38oC. Blood
analysis revealed high concentration of urates. This condition might be
caused by disturbed metabolism of:
A. Purines
B. Collagen
C. Cholesterol
D. Pyrimidines
93
E. Carbohydrates

443. A 46 year old woman suffering from cholelithiasis developed jaundice.


Her urine became dark-yellow and feces became colourless. Blood serum
will have the highest concentration of the following substance:
A. Unconjugated bilirubin
B. Biliverdin
C. Conjugated bilirubin
D. Mesobilirubin
E. Urobilinogen

444. Atria of an experimental animal were super-distended by blood that


resulted in decreased reabsorption of Na and water in renal tubules. This
can be explained by the influence of the following factor upon kidneys:
A. Aldosterone
B. Vasopressin
C. Renin
D. Angiotensin
E. Natriuretic hormone

445. A clinic observes a 49 year old patient with significant prolongation


of coagulation time, gastrointestinal haemorrhages, subcutaneous
hematomas. These symptoms might be explained by the deficiency of the
following vitamin:
A. B1
B. B6
C. H
D. K
E. E

446. Hepatitis has led to the development of hepatic failure. Mechanism


of edemata formation is activated by the impairment of the following liver
function:
A. Protein-synthetic
B. Barrier
C. Chologenetic
D. Antitoxic
E. Glycogen-synthetic

447. A viral infection has damaged cells that form walls of bile capillaries.
This stimulated conditions for inflow of bile into the blood of sinusoidal
capillaries. What cells are damaged?
A. Kupffer’s cells
B. Ito cells
C. Hepatocytes
94
D. Pit-cells
E. Endotheliocytes

448. Study of conversion of a food colouring agent revealed that


neutralization of this xenobiotic takes place only in one phase -
microsomal oxydation. Name a component of this phase:
A. Cytochrome A
B. Cytochrome B
C. Cytochrome C
D. Cytochrome Р-450
E. Cytochrome oxidase

449. A patient with hip fracture was prescribed a narcotic analgetic such
as morphine. Its anesthetic action is determined by interaction with the
following receptors:
A. Adrenoreceptors
B. Opiate receptors
C. Cholinoreceptors
D. Benzodiazepine receptors
E. GABA-ergic receptors

450. Urine analysis of a 12-year-old boy reveals high concentration of all


aliphatic amino acids with the highest excretion of cystine and cysteine.
US (ultrasound examination) of kidneys revealed kidney concrements.
What is the most likely pathology?
A. Alkaptonuria
B. Cystitis
C. Phenylketonuria
D. Cystinuria
E. Hartnup disease

451. A 36-year-old female patient has a history of collagen disease. Urine


analysis is likely to reveal an increased concentration of the following
metabolite:
A. Indican
B. Creatinine
C. Urea
D. Oxyproline
E. Urobilinogen

452. The secretion of which pituitary hormones will be inhibited after


taking the oral contraceptives containing sex hormones?
A. Vasopressin
B. Gonadotropic hormone
C. Thyrotrophic hormone
95
D. Somatotropic hormone
E. Ocytocin

453. Which of the listed diuretic agents WILL NOT have diuretic effect on
a patient with Addison’s disease?
A. Furosemide
B. Spironolactone
C. Hydrochlorothiazide
D. Triamterene
E. Ethacrynic acid

454. Jaundice treatment involves administration of barbiturates inducing


the synthesis of UDP-glucuronyl transferase. A medicinal effect is caused
by the production of:
A. Indirect reacting (unconjugated) bilirubin
B. Direct reacting (conjugated) bilirubin
C. Biliverdin
D. Protoporphyrin
E. Heme

455. Blood analysis of a patient with jaundice reveals conjugated


bilirubinemia, increased concentration of bile acids. There is no
stercobilinogen in urine. What type of jaundice is it?
A. Hepatocellular jaundice
B. Parenchymatous jaundice
C. Obstructive jaundice
D. Hemolytic jaundice
E. Cythemolytic jaundice

456. A female patient presents with endocrine dysfunction of follicular


cells of the ovarian follicles resulting from an inflammation. The
synthesis of the following hormone will be inhibited:
A. Progesterone
B. Estrogen
C. Lutropin
D. Follicle stimulating hormone
E. Follistatine

457. A patient complains of polyuria (7 liters per day) and polydipsia.


Examination reveals no disorders of carbohydrate metabolism. These
abnormalities might be caused by the dysfunction of the following
endocrine gland:
A. Adenohypophysis (anterior pituitary gland)
B. Islets of Langerhans (pancreatic islets)
C. Neurohypophysis (posterior pituitary gland)
96
D. Adrenal cortex
E. Adrenal medulla

458. Blood plasma of a healthy man contains several dozens of proteins.


During an illness new proteins can originate, namely the protein of "acute
phase". Select such protein from the listed below:
A. Prothrombin
B. Fibrinogen
C. C-reactive protein
D. G immunoglobulin
E. A immunoglobulin

459. A tooth extraction in a patient with chronic persistent hepatitis was


complicated with prolonged hemorrhage. What is the reason for the
haemorrhagic syndrome?
A. Increase in thromboplastin production
B. Decrease in fibrin production
C. Increase in fibrinogen synthesis
D. Fibrinolysis intensification
E. Decrease in thrombin production

460. A 46 year old female patient has a continuous history of progressive


muscular (Duchenne’s) dystrophy. Which blood enzyme changes will be of
diagnostic value in this case?
A. Lactate dehydrogenase
B. Pyruvate dehydrogenase
C. Glutamate dehydrogenase
D. Creatine phosphokinase
E. Adenylate cyclase

461. A child has abnormal formation of tooth enamel and dentin as a


result of low concentration of calcium ions in blood. Such abnormalities
might be caused by deficiency of the following hormone:
A. Thyrocalcitonin
B. Thyroxin
C. Somatotropic hormone
D. Triiodothyronine
E. Parathormone

462. Vegetative abnormalities in the sleep, heat regulation, all kinds of


metabolism, diabetes insipidus are developing in the patient due to
grouth of the tumour in the III ventricle of brain. Irritation of the nucleus
of what part of the brain can cause these symptoms?
A. Cerebral peduncles (cruces cerebri)
B. Mesencephalic tegmentum
97
C. Pons cerebelli
D. Hypothalamus
E. Medulla

463. Prior to glucose utilization in cells it is transported inside cells from


extracellular space through plasmatic membrane. This process is
stimulated by the following hormone:
A. Glucagon
B. Thyroxin
C. Insulin
D. Aldosterone
E. Adrenalin

464. Parodontitis is treated with calcium preparations and a hormone


that stimulates tooth mineralization and inhibits tissue resorption. What
hormone is it?
A. Calcitonin
B. Parathormone
C. Adrenalin
D. Aldosterone
E. Thyroxine

465. A 20 year old patient complains of morbid thirst and hyperdiuresis


(up to 10 l daily). Glucose concentration in blood is normal but it is
absent in urine. The patient has been diagnosed with diabetes insipidus.
What hormonal drug is the most appropriate for management of this
disorder?
A. Cortisol
B. Thyroxin
C. Oxytocin
D. Insulin
E. Vasopressin

466. From the group of children who were eating sweet sappy watermelon
two kids developed the signs of poisoning: rapid weakness, dizziness,
headache, vomiting, edema, tachycardia, cyanosis of mouth, ears, tips of
the fingers cyanosis. High concentration of nitrates was detected. What is
the leading mechanism of the pathogenesis of the poisoning in the two
children?
A. Insufficiency of met-Hb-reductase
B. Insufficiency of superoxiddismutase
C. Block cytochrome oxidase
D. Insufficiency glutathione pyroxidase
E. Insufficiency of catalase
98
467. To prevent postoperative bleeding a 6-year-old child was
administered vicasol that is a synthetic analogue of vitamin K. Name
post-translational changes of blood coagulation factors that will be
activated by vicasol:
A. Phosphorylation of serine radicals
B. Partial proteolysis
C. Carboxylation of glutamic acid
D. Polymerization
E. Glycosylation

468. A patient presented to a hospital with complaints about quick


fatigability and significant muscle weakness. Examination revealed an
autoimmune disease that causes functional disorder of receptors in the
neuromuscular synapses. This will result in the disturbed activity of the
following mediator:
A. Noradrenaline
B. Acetylcholine
C. Dopamine
D. Serotonin
E. Glycine

469. Toxic affection of liver results in dysfunction of protein synthesis. It


is usually accompanied by the following kind of dysproteinemia:
A. Relative hypoproteinemia
B. Absolute hyperproteinemia
C. Relative hyperproteinemia
D. Absolute hypoproteinemia
E. Paraproteinemia

470. After taking poor-quality food a patient developed repeated episodes


of diarrhea. On the next day he presented with decreased arterial pressure,
tachycardia, extrasystole. Blood pH is 7,18. These abnormalities were
caused by the development of:
A. Gaseous acidosis
B. Nongaseous alkalosis
C. Nongaseous acidosis
D. Gaseous alkalosis
E. Metabolic alkalosis

471. Hemoglobin catabolism results in release of iron which is


transported to the bone marrow by a certain transfer protein and used
again for the synthesis of hemoglobin. Specify this transfer protein:
A. Transcobalamin
B. Haptoglobin
C. Ceruloplasmin
99
D. Transferrin
E. Albumin

472. A patient with diabetes developed a diabetic coma due to the acid-
base imbalance. Specify the kind of this imbalance:
A. Metabolic alkalosis
B. Metabolic acidosis
C. Respiratory acidosis
D. Gaseous alkalosis
E. Non-gaseous alkalosis

473. Electrophoretic study of a blood serum sample, taken from the


patient with pneumonia, revealed an increase in one of the protein
fractions. Specify this fraction:
A. Albumins
B. α1-globulins
C. γ-globulins
D. α2-globulins
E. β-globulins

474. A patient with respiratory failure has blood pH of 7,35. pCO2 test
revealed hypercapnia. Urine pH test revealed an increase in the urine
acidity. What form of acid-base imbalance is the case?
A. Compensated metabolic acidosis
B. Decompensated metabolic acidosis
C. Compensated respiratory alkalosis
D. Compensated respiratory acidosis
E. Decompensated respiratory alkalosis

475. A patient with jaundice has high total bilirubin that is mainly
indirect (unconjugated), high concentration of stercobilin in the stool and
urine. The level of direct (conjugated) bilirubin in the blood plasma is
normal. What kind of jaundice can you think of?
A. Parenchymal (hepatic)
B. Mechanical
C. Hemolytic
D. Neonatal jaundice
E. Gilbert’s disease

476. Curariform substances introduced into a human body cause the


relaxation of all skeletal muscles. What changes in the neuromuscular
synapse cause this phenomenon?
A. Impaired acetylcholine release
B. Blockade of Ca2 channels of the presynaptic membrane
C. Impaired cholinesterase synthesis
100
D. Depolarization of the postsynaptic membrane
E. Blockade of cholinergic receptors of the synaptic membrane

477. By the decarboxylation of glutamate in the CNS an inhibitory


mediator is formed. Name it:
A. Glutathione
B. Histamine
C. GABA
D. Serotonin
E. Asparagine

478. A comatose patient was taken to the hospital. He has a history of


diabetes mellitus. Objectively: Kussmaul breathing, low blood pressure,
acetone odor of breath. After the emergency treatment the patient’s
condition improved. What drug had been administered to the patient?
A. Adrenaline
B. Isadrinum
C. Glibenclamide
D. Insulin
E. Furosemide

479. It is known that the monoamine oxidase (MAO) enzyme plays an


important part in the metabolism of catecholamine neurotransmitters. In
what way does the enzyme inactivate these neurotransmitters
(norepinephrine, epinephrine, dopamine)?
A. Addition of an amino group
B. Oxidative deamination
C. Removal of a methyl group
D. Carboxylation
E. Hydrolysis

480. Enzymatic jaundices are accompanied by decreased activity of UDP-


glucuronyl transferase. What compound is accumulated in blood serum in
case of these pathologies?
A. Conjugated bilrubin
B. Unconjugated bilirubin
C. Dehydrobilirubin
D. Hydrobilirubin
E. Choleglobin

481. Inherited diseases, such as mucopolysaccharidoses, are manifested


in metabolic disorders of connective tissue, bone and joint pathologies.
The sign of this disease is the excessive urinary excretion of the following
substance:
A. Amino acids
101
B. Glucose
C. Lipids
D. Urea
E. Glycosaminoglycans

482. A 19-year-old male was found to have an elevated level of potassium


in the secondary urine. These changes might have been caused by the
increase in the following hormone level:
A. Oxytocin
B. Aldosterone
C. Adrenaline
D. Glucagon
E. Testosterone

483. Due to the use of poor-quality measles vaccine for preventive


vaccination, a 1-year-old child developed an autoimmune renal injury.
The urine was found to contain macromolecular proteins. What process of
urine formation was disturbed?
A. Filtration
B. Reabsorption
C. Secretion
D. Reabsorption and secretion
E. Secretion and filtration

484. A 46-year-old female is scheduled for a maxillofacial surgery. It is


known that the patient is prone to high blood coagulation. What natural
anticoagulant can be used to prevent blood clotting?
A. Hirudin
B. Sodium citrate
C. Fibrinolysin
D. Heparin
E. None of the above-listed substances

485. A 50-year-old patient with food poisoning is on a drip of 10% glucose


solution. It not only provides the body with necessary energy, but also
performs the FUNCTION OF DETOXIFICATION by the production of a
metabolite that participates in the following conjugation reaction:
A. Sulfation
B. Glucuronidation
C. Methylation
D. Glycosylation
E. Hydroxylation

486. A 26-year-old woman at 40 weeks pregnant has been delivered to the


maternity ward. Objectively: the uterine cervix is opened, but the
102
contractions are absent. The doctor has administered her a hormonal
drug to stimulate the labor. Name this drug:
A. Hydrocortisone
B. Estrone
C. Testosterone
D. ACTH
E. Oxytocin

487. A patient with signs of osteoporosis and urolithiasis has been


admitted to the endocrinology department. Blood test revealed
hypercalcemia and hypophosphatemia. These changes are associated with
abnormal synthesis of the following hormone:
A. Calcitonin
B. Cortisol
C. Aldosterone
D. Parathyroid hormone
E. Calcitriol

488. A 30-year-old female exhibits signs of virilism (growth of body hair,


balding temples, menstrual disorders). This condition can be caused by
the overproduction of the following hormone:
A. Testosterone
B. Oestriol
C. Relaxin
D. Oxytocin
E. Prolactin

489. As a result of a home injury, a patient suffered a significant blood


loss, which led to a fall in blood pressure. Rapid blood pressure recovery
after the blood loss is provided by the following hormones:
A. Cortisol
B. Sex hormones
C. Adrenaline, vasopressin
D. Oxytocin
E. Aldosterone

490. A 53-year-old male patient is diagnosed with Paget’s disease. The


concentration of oxyproline in daily urine is sharply increased, which
primarily means intensified disintegration of:
A. Keratin
B. Albumin
C. Hemoglobin
D. Collagen
E. Fibrinogen
103
491. Feces of a patient contain high amount of undissociated fats and
have grayish-white color. Specify the cause of this phenomenon:
A. Hypoactivation of pepsin by hydrochloric acid
B. Obturation of bile duct
C. Hypovitaminosis
D. Enteritis
E. Irritation of intestinal epithelium

492. A biochemical urine analysis has been performed for a patient with
progressive muscular dystrophy. In the given case muscle disease can be
confirmed by the high content of the following substance in urine:
A. Hippuric acid
B. Porphyrin
C. Urea
D. Creatine
E. Creatinine

493. A patient with chronic heart failure with edema has increased level
of blood aldosterone. What diuretic would be most effective in this case?
A. Triamterene
B. Spironolactone
C. Acetazolamide
D. Hydrochlorothiazide
E. Furosemide

494. Human red blood cells do not contain mitochondria. What is the
main pathway for ATP production in these cells?
A. Aerobic glycolysis
B. Anaerobic glycolysis
C. Oxidative phosphorylation
D. Creatine kinase reaction
E. Cyclase reaction

495. Urine analysis has shown high levels of protein and erythrocytes in
urine. This can be caused by the following:
A. Hydrostatic blood pressure in glomerular capillaries
B. Effective filter pressure
C. Renal filter permeability
D. Hydrostatic primary urine pressure in capsule
E. Oncotic pressure of blood plasma

496. A patient is diagnosed with hereditary coagulopathy that is


characterized by factor VIII deficiency. Specify the phase of blood clotting
during which coagulation will be disrupted in the given case:
A. Thrombin formation
104
B. Fibrin formation
C. Clot retraction
D. Prothrombinase formation
E. -

497. A 49-year-old male patient with acute pancreatitis was likely to


develop pancreatic necrosis, while active pancreatic proteases were
absorbed into the blood stream and tissue proteins broke up. What
protective factors of the body can inhibit these processes?
A. α2-macroglobulin, α1-antitrypsin
B. Immunoglobulin
C. Cryoglobulin, interferon
D. Ceruloplasmin, transferrin
E. Hemoplexin, haptoglobin

498. A 53-year-old male patient complains of acute pain in the right


hypochondrium. Objective examination revealed scleral icterus.
Laboratory tests revealed increased ALT activity, and stercobilin was not
detected in the stool. What disease is characterized by these symptoms?
A. Hemolytic jaundice
B. Hepatitis
C. Chronic colitis
D. Cholelithiasis
E. Chronic gastritis

499. A patient has insufficient blood supply to the kidneys, which has
caused the development of pressor effect due to the constriction of
arterial resistance vessels. This is the result of the vessels being greately
affected by the following substance:
A. Angiotensinogen
B. Renin
C. Angiotensin II
D. Catecholamines
E. Norepinephrine

500. According to the results of glucose tolerance test, the patient has no
disorder of carbohydrate tolerance. Despite that, glucose is detected in
the patient’s urine (5 mmol/l). The patient has been diagnosed with renal
diabetes. What renal changes cause glucosuria in this case?
A. Increased activity of glucose reabsorption transporter
B. Exceeded glucose reabsorption threshold
C. Increased glucose secretion
D. Decreased activity of glucose reabsorption transporter
E. Increased glucose filtration
105
501. In the course of an experiment adenohypophysis (anterior pituitary)
of an animal has been removed. The resulting atrophy of thyroid gland
and adrenal cortex has been caused by deficiency of the following
hormone:
A. Thyroid hormones
B. Somatotropin
C. Tropic hormones
D. Cortisol
E. Thyroxin

502. A newborn baby has numerous hemorrhages. Blood coagulation tests


reveal increased prothrombin time. The child is most likely to have a
disorder of the following biochemical process:
A. Conversion of homocysteine to methionine
B. Conversion of methylmalonyl CoA to succinyl CoA
C. Degradation of glutathione
D. Hydroxylation of proline
E. Production of gamma-carboxyglutamate

503. Decarboxylation of glutamate induces production of gamma-


aminobutyric acid (GABA) neurotransmitter. After breakdown, GABA is
converted into a metabolite of the citric acid cycle, that is:
A. Succinate
B. Citric acid
C. Malate
D. Fumarate
E. Oxaloacetate

504. A patient has been admitted to the contagious isolation ward with
signs of jaundice caused by hepatitis virus. Which of the symptoms given
below is strictly specific for hepatocellular jaundice?
A. Hyperbilirubinemia
B. Bilirubinuria
C. Increase of ALT, AST level
D. Cholemia
E. Urobilinuria

505. Disruption of nerve fiber myelinogenesis causes neurological


disorders and mental retardation. These symptoms are typical for
hereditary and acquired alterations in the metabolism of:
A. Neutral fats
B. Sphingolipids
C. Higher fatty acids
D. Cholesterol
E. Phosphatidic acid
106

506. A patient with signs of emotional lability that result in troubled


sleep has been prescribed diazepam. Specify the sleep-inducing
mechanism of this drug:
A. Blockade of opiate receptors
B. Inhibition of stimulating amino acids
C. H1-histamine receptors stimulation
D. GABA-ergic system activation
E. Supression of serotonergic neurotransmission

507. Pancreas is known as a mixed gland. Endocrine functions include


production of insulin by beta cells. This hormone affects the metabolism
of carbohydrates. What is its effect upon the activity of glycogen
phosphorylase (GP) and glycogen synthase (GS)?
A. It activates both GP and GS
B. It inhibits GP and activates GS
C. It inhibits both GP and GS
D. It activates GP and inhibits GS
E. It does not affect the activity of GP and GS

508. Untrained people often have muscle pain after sprints as a result of
lactate accumulation. This can be caused by intensification of the
following biochemical process:
A. Gluconeogenesis
B. Pentose phosphate pathway
C. Lipogenesis
D. Glycogenesis
E. Glycolysis

509. A 16-year-old adolescent is diagnosed with hereditary UDP (uridine


diphosphate) glucuronyltransferase deficiency. Laboratory tests revealed
hyperbilirubinemia caused mostly by increased blood content of the
following substance:
A. Conjugated bilirubin
B. Unconjugated bilirubin
C. Urobilinogen
D. Stercobilinogen
E. Biliverdine

510. After implantation of a cardiac valve a young man systematically


takes indirect anticoagulants. His state was complicated by hemorrhage.
What substance content has decreased in blood?
A. Haptoglobin
B. Heparin
C. Prothrombin
107
D. Creatin
E. Ceruloplasmin

511. A 41-year-old man has a history of recurrent attacks of heartbeats


(paroxysms), profuse sweating, headaches. Examination revealed
hypertension, hyperglycemia, increased basal metabolic rate, and
tachycardia. These clinical presentations are typical for the following
adrenal pathology:
A. Hypofunction of the medulla
B. Hyperfunction of the adrenal cortex
C. Hypofunction of the adrenal cortex
D. Hyperfunction of the medulla
E. Primary aldosteronism

512. Along with normal hemoglobin types there can be pathological ones
in the organism of an adult. Name one of them:
A. HbF
B. HbA1
C. HbS
D. HbA2
E. HbO2

513. Monoamine oxidase inhibitors are widely used as


psychopharmacological drugs. They change the level of nearly all
neurotransmitters in synapses, with the following neurotransmitter being
the exception:
A. Acetylcholine
B. Noradrenaline
C. Adrenaline
D. Dopamine
E. Serotonin

514. A 30-year-old man with diabetes mellitus type I was hospitalized.


The patient is comatose. Laboratory tests revealed hyperglycemia and
ketonemia. What metabolic disorder can be detected in this patient?
A. Metabolic alkalosis
B. Metabolic acidosis
C. Respiratory acidosis
D. Respiratory alkalosis
E. Normal acid-base balance

515. A patient, who has been suffering for a long time from intestine
disbacteriosis, has increased hemorrhaging caused by disruption of
posttranslational modification of blood-coagulation factors II, VII, IХ, and
Х in the liver. What vitamin deficiency is the cause of this condition?
108
A. B12
B. B1
C. К
D. С
E. Р

516. A43-year-old patient suffers from acute pancreatitis with disrupted


common bile duct patency. What condition can develop in this case?
A. Hemolytic jaundice
B. Hepatocellular jaundice
C. Hepatic coma
D. Mechanical jaundice
E. Portal hypertension

517. Prolonged treatment of hypothyroidism has caused general


dystrophy, dental caries, tachycardia, tremor of extremities. What drug is
the cause of these side effects?
A. Humulin (Human insulin)
B. Parathyreoidinum
C. Thyrocalcitonin
D. L-thyroxin
E. Prednisolone

518. A patient is diagnosed with cardiac infarction. Blood test for


cardiospecific enzymes activity was performed. Which of the enzymes has
three isoforms?
A. Lactate dehydrogenase
B. Aspartate transaminase
C. Alanine transaminase
D. Pyruvate kinase
E. Creatine kinase

519. Parkinson’s disease is caused by disruption of dopamine synthesis.


What brain structure synthesizes this neurotransmitter?
A. Substantia nigra
B. Globus pallidus
C. Corpora quadrigemina
D. Red nucleus
E. Pituitary gland

520. During narcosis a patient developed a risk of cerebral edema. What


drug should be administered in this case?
A. Dopamine
B. Phenazepam
C. Triamterene
109
D. Furosemide
E. Sodium bromide

521. A patient suffers from disrupted patency of the airways at the level
of small and medium-sized bronchial tubes. What changes of acid-base
balance can occur in the patient?
A. Respiratory alkalosis
B. Metabolic acidosis
C. Metabolic alkalosis
D. Respiratory acidosis
E. Acid-base balance remains unchanged

522. Upon toxic damage of hepatic cells resulting in disruption of liver


function the patient developed edemas. What changes of blood plasma are
the main cause of edema development?
A. Increase of globulin content
B. Decrease of albumin content
C. Decrease of fibrinogen content
D. Increase of albumin content
E. Decrease of globulin content

523. A 15-year-old boy has been diagnosed with acute viral hepatitis.
What blood value should be determined to confirm acute affection of
hepatic cells?
A. Unconjugated and conjugated bilirubin content
B. Erythrocytes sedimentation rate (ESR)
C. Aminotransferase activity (AST, ALT)
D. Cholesterol content
E. Protein fraction content

524. An infant born prematurely 2 days ago presents with yellow coloring
of skin and mucosa. Such a condition in the infant is caused by
temporary deficiency of the following enzyme:
A. Aminolevulinate synthase
B. Heme oxygenase
C. Heme synthetase
D. Biliverdine reductase
E. UDP-glucuronyl transferase

525. Autopsy of a 40-year-old woman, who died of cerebral hemorrhage


during hypertensic crisis, revealed: upperbody obesity, hypertrichosis,
hirsutism, stretchmarks on the skin of thighs and abdomen. Pituitary
basophil adenoma is detected in the anterior lobe. What diagnosis is the
most likely?
A. Essential hypertension
110
B. Alimentary obesity
C. Simmonds’ disease
D. Cushing’s disease
E. Hypothalamic obesity

526. A patient suffers from acute cardiopulmonary failure with pulmonary


edema. What powerful diuretic should be prescribed in the given case?
A. Triamterene
B. Furosemide
C. Spironolactone
D. Hydrochlorothiazide (Dichlothiazidum)
E. Acetazolamide (Diacarb)

527. A 60-year-old man suffering from chronic hepatitis frequently


observes nasal and gingival hemorrhages, spontaneous hemorrhagic
rashes on the skin and mucosa. Such presentations result from:
A. Decreased synthesis of prothrombin and fibrinogen
B. Increased blood content of aminotransferases
C. Decreased synthesis of serum albumins
D. Increased blood content of macroglobulins and cryoglobulins
E. Decreased blood content of cholinesterase

528. Leading symptoms of primary hyperparathyroidism are osteoporosis


and renal damage resulting in urolithiasis development. What substances
are the basis of uroliths in such cases?
A. Calcium phosphate
B. Uric acid
C. Cystine
D. Bilirubin
E. Cholesterol

529. Activation of a number of hemostatic factors occurs through their


joining with calcium ions. What structural component allows for adjoining
of calcium ions?
A. Gamma-aminobutyric acid
B. Gamma-carboxyglutamic acid
C. Gamma-oxybutyric acid
D. Hydroxyproline
E. Monoamine-dicarboxylic acids

530. Blood test of the patient revealed albumine content of 20 g/l and
increased activity of lactate dehydrogenase isoenzyme 5 (LDH5). These
results indicate disorder of the following organ:
A. Kidneys
B. Heart
111
C. Lungs
D. Liver
E. Spleen

531. During removal of the hyperplastic thyroid gland of a 47-year-old


woman, the parathyroid gland was damaged. One month after the surgery
the patient developed signs of hypoparathyroidism: frequent convulsions,
hyperreflexia, laryngospasm. What is the most likely cause of the
patient’s condition?
A. Hyponatremia
B. Hyperchlorhydria
C. Hypocalcemia
D. Hypophosphatemia
E. Hyperkalemia

532. On examination the patient presents with hirsutism, moon-shaped


face, stretch marks on the abdomen. BP is 190/100 mm Hg, blood glucose
is 17,6 mmol/l. What pathology is such clinical presentation
characteristic of?
A. Adrenocortical hyperfunction
B. Hyperthyroidism
C. Hypothyroidism
D. Gonadal hypofunction
E. Hyperfunction of the insular apparatus

533. A 64-year-old woman presents with disturbed fine motor function of


her fingers, marked muscle rigidity, and tremor. The neurologist
diagnosed her with Parkinson’s disease. What brain structures are
damaged resulting in this disease?
A. Pituitary gland
B. Red nuclei
C. Cerebellum
D. Substantia nigra
E. Reticular formation

534. A patient with jaundice has high total bilirubin that is mainly
indirect (unconjugated), high concentration of stercobilin in the feces and
urine. The level of direct (conjugated) bilirubin in the blood plasma is
normal. What type of jaundice can be suspected?
A. Parenchymal (hepatic)
B. Mechanical
C. Hemolytic
D. Neonatal
E. Gilbert’s disease
112
535. During acute hemorrhage the body loses not only fluid but also
electrolytes. What substance solution can be used as a simple blood
substitute?
A. Sodium bromide
B. Albumin
C. Sodium nucleotide
D. Calcium chloride
E. Sodium chloride

536. Corticosteroid hormones regulate the adaptation processes of the


body as a whole to environmental changes and ensure the maintenance of
internal homeostasis. What hormone activates the hypothalamo-pituitary-
adrenal axis?
A. Somatoliberin
B. Somatostatin
C. Corticostatin
D. Corticoliberin
E. Thyroliberin

537. A 50-year-old inpatient during examination presents with glucosuria


and blood glucose of 3,0 mmol/l, which are the most likely to be caused
by:
A. Diabetes insipidus
B. Myxedema
C. Renal disorder
D. Essential hypertension
E. Pellagra

538. A patient visited a dentist to extract a tooth. After the tooth had
been extracted, bleeding from the tooth socket continued for 15 minutes.
Anamnesis states that the patient suffers from active chronic hepatitis.
What phenomenon can extend the time of hemorrhage?
A. Thrombocytopenia
B. Decrease of fibrinogen content in blood
C. Hypocalcemia
D. Increased activity of anticoagulation system
E. Decrease of albumine content in blood

539. A 40-year-old woman on examination presents with intensified basal


metabolic rate. What hormone present in excess leads to such condition?
A. Thyrocalcitonin
B. Glucagon
C. Aldosterone
D. Somatostatin
E. Triiodothyronine
113

540. In the process of hemoglobin catabolism iron is released and then as


a part of special transport protein is returned to the bone marrow, to be
used again for hemoglobin synthesis. Name this transport protein:
A. Transcobalamin
B. Transferrin
C. Haptoglobin
D. Ceruloplasmin
E. Albumin

541. After a case of sepsis a 27-year-old woman developed ”bronzed” skin


discoloration characteristic of Addison’s disease. Hyperpigmentation
mechanism in this case is based on increased secretion of:
A. Somatotropin
B. Gonadotropin
C. Melanocyte-stimulating hormone
D. β-lipotropin
E. Thyroid-stimulating hormone

542. A lab rat has subcutaneously received mercury(II) chloride in the


amount of 5 mg/kg. 24 hours later the plasma creatinine concentration
increased several times. What mechanism of retention azotemia is
observed in this case?
A. Increased creatinine production in the muscles
B. Increased creatinine reabsorption
C. Increased glomerular filtration
D. Decreased glomerular filtration
E. Increased creatinine production in the renal tubules

543. Collagenosis patients typically present with connective tissue


destruction processes. The presence of these processes can be confirmed
by the increase in:
A. Blood oxyproline and oxylysine
B. Blood creatine and creatinine
C. LDH-isoenzyme activity in the blood
D. Transaminase activity in the blood
E. Blood urates

544. A patient presents with acute attack of cholelithiasis. Laboratory


examination of the patient’s feces will show the following in this case:
A. Positive reaction to stercobilin
B. Connective tissue
C. Partially digested cellulose
D. Negative reaction to stercobilin
E. Starch granules
114

545. People, who for a long time remained in hypodynamic state, develop
intense pain in the muscles after a physical exertion. What is the most
likely cause of this pain?
A. Intensive breakdown of muscle proteins
B. Accumulation of creatinine in muscles
C. Decreased content of lipids in muscles
D. Increased content of ADP in muscles
E. Accumulation of lactic acid in muscles

546. Due to prolonged stay in the mountains at the altitude of 3000 m


above the sea level, a person developed increased oxygen capacity of
blood, which was directly caused by intensified production of:
A. Leukopoietins
B. Carbaminohemoglobin
C. Erythropoietins
D. Catecholamines
E. 2,3-bisphosphoglycerate

547. During intensive muscle work there is a large amount of ammonia


produced in the muscles. What amino acid plays the main role in the
transportation of ammonia to the liver and participates in
gluconeogenesis reactions?
A. Arginine
B. Alanine
C. Lysine
D. Ornithine
E. Aspartate

548. Encephalopathy has developed in a child with hemolytic disease of


the newborn. What substance had increased in the child’s blood, resulting
in damage to the CNS?
A. Bilirubin-albumin complex
B. Bilirubin glucuronide
C. Verdohemoglobin
D. Unconjugated bilirubin
E. Bile acids

549. Neutralization of xenobiotics and active endogenous metabolites


often occurs via introduction of an oxygen atom into the substrate
molecule. What process occurs as the result?
A. Decarboxylation
B. Transamination
C. Phosphorilation
D. Hydroxylation
115
E. Deaminization

550. During intensive physical exertion, one of the energy sources for the
working muscles is glucose produced as the result of gluconeogenesis.
This process is the most intensive in the following organ:
A. Liver
B. Brain
C. Lungs
D. Muscles
E. Stomach

551. Ketosis develops in the patients with diabetes mellitus, as the result
of activation of fatty acids oxidation processes. What acid/base imbalance
can result from accumulation of excessive ketone bodies in the blood?
A. Metabolic alkalosis
B. No imbalance occurs
C. Metabolic acidosis
D. Respiratory acidosis
E. Respiratory alkalosis

552. To stimulate the labor activity a parturient woman was prescribed a


drug - a posterior pituitary hormone that does not affect the blood
pressure. As the pregnancy progresses, the sensitivity to this hormone
increases. Name the prescribed drug:
A. Dinoprostone
B. Dinoprost
C. Pituitrin
D. Oxytocin
E. Ergotal

553. A 52-year-old man presents with fever and pain in the joints. Both of
his first metatarsophalangeal articulations are deformed, swollen, and
reddened. Blood urea is high. The patient is diagnosed with gout. What is
the main developmental factor in the pathogenesis of this disease?
A. Argininosuccinic aciduria
B. Hyperuricemy
C. Hyperazotemia
D. Hyperaminoacidemia
E. Citrullinuria

554. In human organism significant blood loss leads to decreased blood


pressure, tachycardia, and weakness. Eventually the sensation of thirst
appears. What hormone participates in the development of this
sensation?
A. Cortisol
116
B. Serotonin
C. Angiotensin II
D. Dopamine
E. Adrenalin

555. Wilson’s disease is a disorder of copper transport which leads to the


accumulation of this metal in brain and liver cells. It is associated with a
disturbance in the synthesis of the following protein:
A. Haptoglobin
B. Siderophilin
C. Metallothionein
D. Ceruloplasmin
E. Transcobalamin
THE MINISTRY OF PUBLIC HEALTH OF UKRAINE
VINNITSIA NATIONAL MEDICAL UNIVERSITY
named after M.I.PIROGOV

EDUCATIONAL-METHODICAL RECOMMENDATION
FROM PATHOPHYSIOLOGY
FOR THE 3RD YEAR
MEDICAL FACULTY STUDENTS

Module N 2 PATHOLOGY OF ORGANS


AND SYSTEMS

Practical training N 4 PATHOLOGY OF BLOOD

VINNITSIA - 2012

1
The manual from Pathophysiology is approved and confirmed for the
introduction into the educational process at Pathophysiology department meeting
according to the protocol of meeting №10 from 10.01.12.

Compilers: Head of Department, Dr. of Med.Sc. Rikalo N.A.


as.Grytsenko A.S.
as.Guminska O.Y.
as.Piliponova V.V

The manual is recommended for the foreign students of medical faculty (3rd
course).

2
THEME: PATHOLOGY OF RED BLOOD
(Determination of color index (CI)

Actuality of theme.
Anaemia - is a haematological syndrome or independent disease which is
characterized by the decline of red blood cells (RBC) and haemoglobin amount in the
unit of blood volume and accompanied by the qualitative changes of the RBC.
Anaemia accompanies the most various diseases of digestive system, kidneys,
urogenital and endocrine systems, infectious and parasitic diseases, malignant tumors
and other.
Knowledge of principal reasons, mechanisms of development, manifestations
of different types of anemias helps a doctor not only to make a diagnosis in time but
also choose faithful tactic of treatment, define measures on a prophylaxis.

General purpose of the lesson.


1. To define principal reasons and mechanisms of different types of anaemias
development.
2. To be able to classify the anemias according to pathogenesis, color index,
ability of the bone marrow to regeneration, type of hemopoiesis.
3. To be able to expose etiology and mechanisms of erythrocytosis development.

For this it is necessary to know (the concrete purposes):


1. To classify the anemias.
2. To be able to define the amount of haemoglobin, count up the amount of RBC
and color index.
3. To be able to define anaemia by the results of the blood test.
4. To know the signs of the bone marrow regeneration and degeneration.
5. To give definition of erythrocytosis and explain the reasons and mechanisms of
its development.
6. To pay attention to the quantitative parameters of blood in new-born and in
different age-old periods. To be able to explain these changes.

For realization of purposes of lesson it is necessary to have the base


knowledges-skills:
1. Functions of blood (Normal Physiology Department)
2. Physical and chemical properties of blood (Normal Physiology Department)
3. Hemopoiesis and its regulation (Normal Physiology Department, Histology
Department)
4. Amount of the RBC and haemoglobin in a norm, method of their calculation
(Normal Physiology Department)
5. Determination of color index (Normal Physiology Department)

The checking of primary level of knowledges.


Give the answers to the following questions:
1. Amount of RBC for male and female.
3
2. Amount of haemoglobin for male and female.
3. Color index in a norm.
4. Anemia, definition.
5. Qualitative changes of red corpuscles.
6. Cells which behave to the regenerative forms of red corpuscles.
7. Classification of anemias according to the etiology.
8. Classification of anemias according to the pathogenesis.
9. Classification of anemias according to the color index.
10. Classification of anemias according to the type of hemopoiesis.
11. Classification of anemias according to the ability of the bone marrow to
regeneration.
12. What cells of erythrocytic row behave to the cells of pathological regeneration?
13. What is erythrocytosis?
14. Types of erythrocytosis?

Standards of answers at the theoretical questions of initial level of


knowledges:
1. Amount of RBC for female: 3.9-4.7x1012/l; for male: 4.5-5.0x1012/l.
2. Amount of haemoglobin: for female: 120-140g/l; for male: 140-160g/l.
3. Color index in a norm 0,85-1,0
4. Anaemia - is a haematological syndrome or independent disease which is
characterized by the decline of red corpuscles and haemoglobin amount in the
unit of blood volume and accompanied by the qualitative changes of red
corpuscles.
5. Poikilocytosis, anisocytosis, anisochromia, pathological including (Jolly's
bodies, Cabot's rings, basophilic granules).
6. Reticulocytes, polychromatophiles.
7. Inherited, acquired.
8. Dyserythropoietic anemia, hemolytic anemia (inherited, acquired),
posthemorrhagic anemia (acute, chronic).
9. Normo-, hypo-, hyperchromic anemia.
10. With erythroblastic and megaloblastic type of hemopoiesis.
11. Regenerative, hyporegenerative, hyperregenerative and aregenerative.
12. Megaloblasts, megalocytes.
13. Erythrocytosis - is the absolute or relative increase of erythrocytes and
hemoglobin amount in unit of blood volume.
14. Primary (inherited, acquired), secondary.

4
Test control for verification of primary level of knowledges:

1. What violation of general blood volume is observed in the initial phase of


acute hemorrhage?
А. Simple hypovolemia.
В. Oligocythemic hypovolemia.
С. Polycythemic hypovolemia.
D. Simple hypervolemia.
Е. Oligocythemic hypervolemia.

2. In what cells destruction of red corpuscles takes place?


A. Fibroblasts.
B. Lymphocytes.
С. Phagocytic mononuclears.
D. Endotheliocytes
E. Parietal cells

3. The globular value shows:


А. Amount of haemoglobin in the blood.
В. Amount of haemoglobin in the one red corpuscles.
С. Amount of haemoglobin in the unit of blood volume.
D. Amount of red corpuscles in the blood.

4. Anisochromia - is:
А. Changes of red corpuscles forms.
В. Changes of red corpuscles sizes.
С. Increase of red corpuscles amount
D. Different degree of red corpuscles saturation haemoglobin
Е. Decrease of red corpuscles amount

5. Liquid, which is necessary for breeding of blood at the counting of red


corpuscles:
А. Solution of acetic acid.
В. Physiological solution.
С. Hypotonic solution
D. Solution of glucose
Е. Distilled water

6. Presence of what cells in the blood testifies to development of physiological


regeneration:
A. Erhytroblasts.
B. Megalocytes.
C. Megaloblasts.
D. Reticulocytes
Е. Erythrocytes
5
7. Name the increase of red corpuscles amount in unit of blood volume:
А. Erythrocytosis
В. Polycythemia
С. Poliglobulia
D. Erythremia
Е. Erythropenia

8. Erythropoiesis in adults healthy people takes place:


A. In a liver
B. In lymphatic nodes
C. In red bone marrow
D. In a spleen
E. In thymus

9. What is the norm of red corpuscles for the adult man?


A. 3,8 - 5.9 x of 1012/ l
B. 3.9 - 4.7 x 1012/ l
C. 4.5 - 5.0 x 1012/ l
D. 6.0 - 6.5 x of 1012/ l
E. 0,85-1,0

10. What is the norm of red corpuscles for the adult woman?
A. 3,8 - 5.9 x 1012/ l
B. 3.9 - 4.7 x 1012/ l
C. 4.5 - 5.5 x 1012/l
D. 6.0 - 6.5 x 1012/ l
E. 0,85-1,0

11. During a prophylactic inspection of young man it was discovered: Hb- of


48g/l, amount of red corpuscles - 3,0 х1012/l, GV - 0,7, leucocytes - 7,5 109/l,
thrombocytes- 280·109/l. Give description the state of the patient.
A. Thrombocytopenia
B. Leukopenia
C. Leukocytosis
D. Erythrocytosis
E. Anaemia

12. In a patient with pathology of liver Iron-deficiency anaemia developed. What


globular value is characteristic for this disease?
A. 1,2
B. 0,8
C. 1,0
D. 0,6
E. 1,4

6
13. B12-deficiency anemia was discovered at a pregnant woman. What globular
value is characteristic for this disease?
A. 0,85
B. 0,9
C. 1,0
D. 1,3
E. 0,6
14. What is a norm of globular value for adult healthy man?
A. 1-5
B. 0-1
C. 0.5-0.8
D. 0.85-1
E. 1-1.8
15. Poikilocytosis - is:
A. Red corpuscles of different form
B. Red corpuscles of different size
C. Pathological including in red corpuscles
D. Multiplying the amount of red corpuscles
E. Diminishing of amount of red corpuscles
16 Anisocytosis - is:
A. Red corpuscles of different form
B. Red corpuscles of different size
C. Pathological including in red corpuscles
D. Multiplying the amount of red corpuscles
E. Diminishing of amount of red corpuscles
Correct answers
1-A 2-C 3-B 4-D 5-E 6-D 7-A 8-C 9-C 10-B 11-E 12-D 13-D 14-D 15-A 16-B

Theoretical questions for the FMC № 2.


1. Anemia, definition.
2. Classification of anemias according to pathogenesis, color index, type of
hemopoiesis, ability of the bone marrow to regeneration.
3. Signs of regeneration and degeneration of the bone marrow.
4. Erythrocytosis, definition. Kinds. Reasons and mechanisms of development.
Picture of blood. Difference from erythremia.

Literature.
1. Handbook of general and Clinical Pathophysiology/ Edited by
prof.A.V.Kubyshkin, CSMU, 2005.p.192-196
2. Pathophysiology/ Edited by prof.Zaporozan, OSMU, 2005.p.167-171
3. General and clinical pathophysiology/ Edited by Anatoliy V/ Kubyshkin –
Vinnytsia: Nova Knuha Publishers – 2011. p.371-380

7
Testing according system “Krok-I”

Tests of an open database (2010)

1. Female, 55 years old, complaints of prolonged cyclical uterine bleeding during


the year, weakness, dizziness. Objective: pale skin. In the blood: Hb - 70 g/l, er.-
3,2x1012/l, CI - 0,6, leuk.-6,0x109 /l, reticulocytes - 1%; hypochromia of
erythrocytes. What is anemia in patients?
A. Chronic posthemorrhagic
B. Hemolytic
C. Iron-deficiency
D. Aplastic
E. B12-deficiency

2. Patient, 32 years old, brought to the clinic with massive blood loss due to road
trauma. Pulse - 110 / min., RR - 22 / min., AP - 100/60 mm Hg/ What will be the
typical in 1 hour after hemorrhage?
A. Erythropenia
B. Hypoproteinemia
C. Hypovolemia
D. Leukopenia
E. Hypochromia of erythrocytes

3. In the blood of patient, 36 years, who treated the respiratory viral infection
by the sulfanilamides, there is hyporegenerative normochromic anemia,
leukopenia, thrombocytopenia. In the bone marrow - reducing of
myelokariocytes. What is the kind of anemia?
A Hypoplastic
B Hemolytic
C Posthemorrhagic
D-B12 - deficiency
E Iron - deficiency

4. The patient entered into the hospital with complains on general weakness,
dizziness, shortness of breathing. Shortly before the going to the clinic he took
Levomicetin to prevent intestinal infections. Blood: red cells - 1,9 T/l, Hb - 58 g/l,
CI - 0,9, leukocytes - 2,2 G/ l. What is the kind of anemia it?
A Hypoplastic
B Metaplastic
C Aplastic
D Hemolytic
E Iron

8
5. The patient has chronic posthemorrhagic anemia, accompanied by lower
concentrations serum iron, hypochromia of erythrocytes, poikilocytosis and
anisocytosis. What is the value of color index at this anemia?
A 0,7
B 0,8
C 0,9
D 1,0
E 1,1

6. Anaemia was revealed in a patient with diffuse glomerulonephritis. What


underlies pathogenesis of this anaemia?
A. Decreasing production of erythropoietin
B. Suppression of red bone marrow function
C. Deficit of internal Kastl factor
D. Increased hemolysis
E. Presence of antibodies to the cell of peripheral blood.

7. A B-12 -deficiency anaemia developed in a patient following the stomach


resection. What colour index of blood is typical for this pathology?
A. 1.4
B. 1.0
C. 0.8
D. 0.5
E. 0.2

(2009 - 2004)

8. A 38-year-old patient with an uterine haemorrhage lasting for 2 days was


delivered to the admission ward. Which of the following will be revealed in the
patient's blood?
A. Leukocytosis
B. Increase in the colour index
C. Decrease in the haematocrite index
D. Deceleration in ESR
E. Eosinophilia

9. It was established that agglutination of the recipient's blood erythrocytes had


been caused by the standard sera from the I and II groups. Serum from the III
group as well as anti-Rh serum hadnt provoke any agglutination. Which blood
group and rhesus is allowed to be transfused this recipient?
A. B, α (III) Rh-
B. 0, α, β (I) Rh+
C. A, β (II) Rh –
D. AB (IV), Rh+
E. AB (IV), Rh-
9
10. Anisochromia is:
А. Changes of red corpuscles forms.
В. Changes of red corpuscles sizes.
С. Increase of red corpuscles amounts
D. Different degree of red corpuscles saturation by haemoglobin
Е. Decrease of red corpuscles amounts

11. The presence of what cells in blood testifies of physiological regeneration


development:
A. Erythroblasts.
B. Megalocytes.
C. Megaloblasts.
D. Reticulocytes
Е. Erythrocytes

12. Hb – 48 g/l, amount of red corpuscles - 3,0 х1012/l, GV - 0,7, leucocytes -


7,5x109/l, thrombocytes – 280x109/l were discovered for a young man during a
prophylactic inspection. Give description of the patient state.
A. Thrombocytopenia
B. Leukopenia
C. Leukocytosis
D. Erythrocytosis
E. Anaemia

13. Iron-deficiency anaemia developed at a patient with pathology of liver. What


globular value is characteristic for this disease?
A. 1,2
B. 0,8
C. 1,0
D. 0,6
E. 1,4

14. The amount of reticulocytes grew in peripheral blood at a man, 50 years old,
on 5th day after the acute gastric bleeding. What this phenomenon testify about?
А. About presence of oxygen deficiency in an organism
В. About presence of inflammatory reaction in a stomach
С. About multiplying the albumens synthesis in a liver
D. About increase of hemopoietic activity of bone marrow
Е. About presence of allergic reaction

15. Hypochromic anaemia is diagnosed at a woman after the complicated labor


which was accompanied the considerable bleeding. What pathological forms of
red corpuscles are characteristic for such anaemia?
А. Macrocytes
В. Targed red corpuscles
10
С. Sickle red corpuscles
D. Spherocytes
E. Anulocytes

16. Victim lost much blood as a result of accident. What violation of general
blood volume takes place?
А. Polycythemic normovolemia
В. Polycythemic hypovolemia
С. Oligocythemic normovolemia
D. Simple hypovolemia
Е. Oligocythemic hypovolemia

17. A child got thermal burns of 40% surface of body. What form of general
blood volume violation is observed here?
А. Polycythemic hypovolemia.
В. Polycythemic hypervolemia.
С. Oligocythemic hypovolemia.
D. Normocythemic hypovolemia.
Е. Oligocythemic hypervolemia.

18. What violation of general volume of blood does arise up in case of absolute
erythrocytosis?
А. Simple hypervolemia.
В. Oligocythemic hypervolemia.
С. Oligocythemic normovolemiya.
D. Policythemic hypervolemia.
Е. Simple hypovolemia.

19. A patient lost a 25% volume of blood as a result of wound. Name the urgent
mechanism of blood loss compensation.
А. Entry of intercellular liquid into the vessels
В. Restoration albuminous composition of blood
С. Increase of reticulocytes numbers
D. Restoration of red corpuscles number
Е. Activation of erythropoiesis

20. Signs of increase of bone marrow Regenerativey possibilities was found on a


6th day at a patient which is on stationary treatment concerning acute
posthemorrhagic anaemia. Name the organ which is chargeable for
erythropoiesis regulation (erythropoietins synthesis) in this case.
A. Adrenal gland
B. Kidneys
C. Spleen
D. Thyroid gland
E. Hypophysis
11
21. Blood test is done on 30th days after bleeding at a patient with the wound of
subclavian artery. What is testify about activation of erythropoiesis in the blood
smear?
A. Poikilocytosis
B. Anisocytosis
C. Reticulocytosis
D. Anisochromia
E. Hypochromia

22. ESR is promoted at young pregnant woman. What results the increase of
ISR?
A. Considerable diminishing of amount of red corpuscles
B.Increase of concentration of fibrinogen
C. Multiplying of lipoproteids concentration
D. Increase of imunoglobulins concentration
E. Increase of albumins concentration

23. Such indexes found out at a patient at the inspection of blood: erythrocytes -
2,0х1012/l, Hb-60 g/l, GV - 1,0, reticulocytes are absent, platelites. - 50х10 9/l,
leukocytes - 2,0х109/l. What is the type of anaemia?
A. Posthemorrhegic
B. Pernicious
C. Iron-deficiency
D. Aplastic
E. Hemolytic

24. Single oxyphyle normocytes appeared in the patient blood after one day after
a loss of 15% of blood. 25% of reticulocytes was discovered at the blood smear.
What type of anaemia at this patient by the ability of bone marrow to
regeneration?
A. Hyporegenerative
B. Aregenerative
C. Hyperregenerative
D. Regenerative
E. -

25. Signs of anaemia found out at a patient with chronic glomerulonephritis.


What stimulated their appearance?
A. Decline of erythropoetins synthesis
B. Loss of red corpuscles with urine
C. Destruction of normal red corpuscles
D. Hemolysis of red corpuscles
E. Insufficiency of iron for the haemoglobin synthesis

12
26. A man was living in the mountains for a long time. What changes in the
blood will he have?
A. Multiplying the amount of haemoglobin
B. Multiplying of leukocytes amount
C. Decline of leukocytes amount
D. Diminishing of blood vessels diameter
E. Multiplying of vessels diameter

27. A 58-year-old woman complains of fatigability, sleepiness, dyspnea when


walking fast. The analysis of the blood is as follows: erythrocytes — 4x1012/l,
hemoglobin — 92.0 g/1, Colour Index - 0.6, plenty of anulocytes and microcytes.
What anemia is it?
A. Thalassemia.
B. Iron deficiency.
C. Hemolytic.
D. Pernicious.
E. Sicklemia.

28. The erythrocytes of a patient with hypochromic anemia contain 45 % Hb S


and 55 % Hb A,. What form of anemia is it?
A. Microspherocytic.
B. α-Thalassemia.
C. Addison-Birmer illness.
D. Glucose-6-phosphatdehydrogenase deficiency.
E. Sicklemia.

29. Degenerative and regenerative forms of erythrocytes were found in the


peripheric blood of a patient with toxic hemolytic anemia. What cells refer to
regenerative?
A. Spherocytes.
B. Microcytes.
C. Reticulocytes.
D.Poikilocytes.
E. Hyperchromic erythrocytes.

30. A patient with chronic diffuse glomerulonephritis suffers from anemia.


What is the pathogenesis of this anemia?
A. Presence of antibodies to the cells of the peripheric blood.
B. Iron deficiency.
C. Increased deficiency of the internal Castle factor.
D. Hemolysis of erythrocytes.
E. Diminished production of erythropoietin.

13
31. A 40-year-old man was diagnosed with sicklemia. What is the mechanism of
decreasing the erythrocytes number in the patient's blood?
Α. Intracellular hemolysis.
B. Intravascular hemolysis.
C. Bleeding.
D. Inhibition of erythropoiesis.
E. Disturbance of DNA synthesis.

32. A 34-year-old woman is diagnosed with hereditary hemolytic microspherocytic


anemia (Minovski — Shoffar disease). What is the cause of hemolysis of
erythrocytes in this case?
A. Fermentopathia.
B. Endogenic intoxication.
C. Hemoglobinopathia.
D. Autoimmune lesion.
E. Membranopathia.

33. In the 6-th month of pregnancy a woman had the following blood findings:
the amount of erythrocytes and hemoglobin is reduced, Colour Index is 1.4,
megalocytes, oxyfile megaloblasts are present. What type of anemia is it?
A. Iron deficiency.
B. B12 and folic deficiency.
C. Myelotoxic.
D. Aplastic.
E. Metaplastic.

34. A woman complains of headache, giddiness, dyspnea during physical activity.


For the last 3 years she has been having extensive menstrual bleedings. The
patient has normal body type, the skin is pale and dry. The analysis of the
blood is as follows: hemoglobin — 90.0 g/1, erythrocytes-3.7-1012/l, Colour Index
- 0.7, ESR - 20 mm/h, significant hypochromia of erythrocytes, anisocytosis,
poikilocytosis. What type of anemia is it?
A. Hypoplastic.
B. Hemolytic.
C. Acute posthemorrhagic.
D.B12 and folic deficiency.
E. Chronic posthemorrhagic.

35. After the resection of the stomach a patient developed B)2 deficiency anemia.
What color index is characteristic of this anemia?
A. 0.4.
B. 0.5.
C. 0.8.
D. 1.0.
E. 1.4.
14
36. Microcythemia, poikilocytosis, and anulocytosis are found in the patient's
blood smear. What anemia are these changes characteristic of?
A. Microspherocytic.
B. B12 deficiency.
C. Hypoplastic.
D. Sicklemia
E. Iron deficiency.

Correct answers
1- 2- 3- 4- 5- 6- 7- 8- 9- 10- 11- 12-
A A A A A A A C A D D E
13- 14- 15- 16- 17- 18- 19- 20- 21- 22- 23- 24-
D D E D A D A B C B D C
25- 26- 27- 28- 29- 30- 31- 32- 33- 34- 35- 36-
A A B E C E A E B E E E

Situate task:

1. Patient, 36 years, delivered to the clinic with complaints about a general


weakness, dizziness. Objectively: the skin is pale. In a blood test: RBC - 3х1012/l,
Hb - 60 g/l, ESR -12 mm/hour.
1. What is the pathology at this patient?
__________________________________________________________________
2. Calculate the color index.
__________________________________________________________________
3. What is the mechanism of this pathology development?
____________________________________________________________________
____________________________________________________________________
____________________________________________________________________
4. Describe this pathology according to the ability of the bone marrow to
regeneration, color index, type of hemopoiesis.
____________________________________________________________________
____________________________________________________________________

2. Patient S., 45 years, hospitalized in a gynaecological clinic with the massive


uterine bleeding. Objectively: pale skin, rapid pulse. Picture of blood:
erythrocytes - 2,8х1012/l; Hb-65g/l, reticulocytes-3‰; ESR-15mm/h.
1. What is the pathology at this patient?
__________________________________________________________________
2. Calculate the color index.
__________________________________________________________________
3. What is the mechanism of this pathology development?
____________________________________________________________________
____________________________________________________________________
____________________________________________________________________
15
4. Describe this pathology according to the ability of bone marrow to regeneration,
color index, type of hemopoiesis.
____________________________________________________________________
____________________________________________________________________

3. Patient, 50 years, who was in the mountains during few months, entered into
the clinic with complaints about acute general weakness. Objectively: cherry-red
colouring of skin and mucuses. AT - 160/100 mm.Hg. PCV - 80/20. Blood test:
gemoglobin- 200g/lл, red corpuscles – 8x1012/l, globular value - 0,96, leucocytes –
10x109/l, thrombocytes – 500x109/l, ESR - 2 mm/h.
1. What pathology does this analysis testify about?
__________________________________________________________________
2. What are the reasons for its origin and the mechanisms of its development?
____________________________________________________________________
____________________________________________________________________
____________________________________________________________________
3. Explain the changes in the blood at this pathology.
____________________________________________________________________
____________________________________________________________________
____________________________________________________________________
4. Describe this pathology according to the ability of bone marrow to regeneration,
color index, type of hemopoiesis.
____________________________________________________________________
____________________________________________________________________
____________________________________________________________________

4. Haemoglobin – 49g/l, RBC - 2,0x1012/l, leucocytes - 3,0x109/l. Blood smear:


megalocytes, megaloblasts, anisocytosis, poikilocytosis, there are RBC with the
Jolly’s bodies and Cabot’s rings. There are atrophy of tongue papillae, shaky
wolk.
1. What is the pathology at this patient?
__________________________________________________________________
2. Calculate the color index.
__________________________________________________________________
3. What is the mechanism of this pathology development?
____________________________________________________________________
____________________________________________________________________
____________________________________________________________________
4. Describe this pathology according to the ability of bone marrow to regeneration,
color index, type of hemopoiesis.
____________________________________________________________________
____________________________________________________________________
____________________________________________________________________
_________________________________________________________________

16
5. Following blood picture was discovered at pregnant woman: RBC - 1,2x1012/l,
hemoglobin- 86 g/l, in the blood smear: anisocytosis, poikilocytosis,
megaloblasts, megalocytes.
1. What is the pathology at this patient?
__________________________________________________________________
2. Calculate the color index.
__________________________________________________________________
3. What is the mechanism of this pathology development?
____________________________________________________________________
____________________________________________________________________
____________________________________________________________________
4. Describe this pathology according to the ability of bone marrow to regeneration,
color index, type of hemopoiesis.
____________________________________________________________________
____________________________________________________________________
____________________________________________________________________

6. In a blood test of a patient: RBC - 2,5x1012/l, haemoglobin – 50g/l.


1. What is the pathology at this patient?
__________________________________________________________________
2. Calculate the color index.
__________________________________________________________________

7. In a blood test of a patient: RBC - 2,9x1012/l, haemoglobin – 65g/l. In the blood


smear: reticulocytes - 300‰. Maintenance of indirect bilirubin is multiply in the
blood.
1. What is the pathology at this patient?
__________________________________________________________________
2. Calculate the color index.
__________________________________________________________________
3. Explain appearance of so high number of reticulocytes in the blood.
____________________________________________________________________
____________________________________________________________________
____________________________________________________________________

Practical work: «PATHOLOGY OF HEMATOPOESIS. ANEMIA.


(Determination of color index)».

Object of work: to demonstrate the changes of hemoglobin and erythrocytes content


and globular value of blood at hemolitic anemia.
Summary: Anemia in a rabbit is caused by preliminary injection of lead acetate
subcutaneosly in the dose of 0,08-0,17 ml per 1 kg of body weight for 3 days.

17
EXPERIMENT № 1. Determination of hemoglobin according to Saly.
Fill the graduated test-tube of hemometer with 1/10 solution of HCІ up to the
mark 10. Take the blood up to the mark of 20 and to flow it out on the bottom of the
test-tube. Wash the test dropper twice with distilled water and blow it out on the
bottom of the test-tube. Wait for 5 minutes until the complete hemolysis occures.
Dilute the fluid in the test - tube with the distilled water until its colour coincides with
the colour of the standard.
Determined the amount of hemoglobin in the blood by the low border of the
meniscus of the fluid in the test – tube.

1 – graduated tube;
2 – test-tube;
3 - rack

Amount of hemoglobin __________________.

EXPERIMENT № 2. Calculation of the erythrocytes number.


Fill the melangeur for erythrocytes with 0,5 mm3 of blood. Delute it with 3%
solution of NaCІ up to the mark of 101. Mix it. Discharge the first drops on to the
cotton wool, and then a drop into the chamber for the erythrocytes calculation. Cover
the chamber with the glass, fitting its edges to the glass of the chamber.
Calculate the number of erythrocytes in 80 small squares. As to volume of one
small square is 1/4000 mm3 and the blood was taken in the delution of 1/200 then for
the calculation of the erythrocytes number it is necessary to multiply the number of
erythrocytes in 80 small squares by 4000 and by 200 and to divide by 80. That is to
multiply by 10000.
1 – big squares (100);
2 – big squares are divided into the
small one (by 16);
3 – big squares are divided by lines

Number of erythrocytes ______.

EXPERIMENT № 3. Calculation of color index.


Color index = 3 x Hb / the first 3 figures of erythrocytes.
For example. The number of hemoglobin – 80 g/l, of erythrocytes –3,2 x 1012/l.
Colour index = 3 x 80 /320 = 0,75
CI = ______________________________________ = _____________
Conclusion _______________________________________________________
____________________________________________________________________
_________________________________________________________________

18
THEME: PATHOLOGY OF RED BLOOD.
(Morphology of blood)

Actuality of theme.
The pathological changes of erythrocytes can arise up at violation of
erythropoiesis, promoted destruction of RBC in the hemopoietic organs and in the
peripheral blood, and also as a result of hemorrhage. Thus, in connection with
participating of haemoglobin in the transport of oxygen and carbon dioxide, the
respiratory function of blood changes foremost.
Anaemia as independent disease or symptom of other organs and systems
pathology is the most widespread pathology. Knowledge of etiology and
pathogenesis of anaemias is an important constituent in diagnostically - medical
process.

General purpose of the lesson.


To master etiology, pathogenesis and morphological composition of blood at
different anemias (according to the classification).

For this it is necessary to know (the concrete purposes):


1. To explain the reasons and mechanisms of anaemias development, related to
violations of erythropoiesis (Iron-deficiency, В12- deficiency).
2. To explain the reasons and mechanisms of hemolytic anemias `development.
3. To explain the reasons and mechanisms of posthemorrhagic anaemias
development
4. To be able to define the anaemias according to the morphology of blood.
5. To be able to explain the pathogenetic principles of anaemias treatment.
6. To be able to explain the etiology, mechanisms of development of the iron-
deficiency states at children. To expose anaemias of new-born.
7. To expose the most frequent reasons for В12-deficiency of anaemia origin at
children.
8. To know etiology, pathogenesis of hemolitic disease of new-born and principles
of it prophylaxis and pathogenetic treatment.

For realization of purposes of lesson it is necessary to have the base


knowledges-skills:
1. Structure of RBC and haemoglobin (Histology Department)
2. Life cycle of RBC (Normal Physiology Department)
3. Regulation of hemopoiesis. (Normal Physiology Department)

Theoretical questions for the FMC № 2.


1. Etiology, pathogenesis of Iron-deficiency anaemia. Picture of blood.
Pathogenetic treatment.
2. Iron-deficiency states at children. Anaemia of new-born. Green sickness.
3. Iron refractory anemia anaemia. Kinds. Reasons, mechanism of development.
Picture of blood. Differences from Iron-deficiency anaemia.
19
4. В12-deficiency anaemia. Reasons, mechanisms of development for children and
adults. Picture of blood. Pathogenetic treatment.
5. Hemolytic anaemias. Kinds. Reasons, mechanisms of development of the
inherited and aquired hemolytic anaemias.
6. Hemolytic disease of new-born. Reasons, mechanisms of development. Picture
of blood. Pathogenetic treatment.

Literature.
1. Handbook of general and Clinical Pathophysiology/ Edited by
prof.A.V.Kubyshkin, CSMU, 2005.p.197-203
2. Pathophysiology/ Edited by prof.Zaporozan, OSMU, 2005.p171-178
3. General and clinical pathophysiology/ Edited by Anatoliy V/ Kubyshkin –
Vinnytsia: Nova Knuha Publishers – 2011. p.381-409

Testing according system “Krok-I”


Tests of an open database (2010)

1. An immune haemolytic anaemia developed in a patient. What index of the


blood serum will increase the most significantly&
A. Indirect bilirubin
B. Direct bilirubin
C. Stercobilinogen
D. Mesobilinogen
E. Protoporphin

2. Hereditary spherocytosis (Minkowsky-Shauffard disease) was diagnosed in a


34-year-old woman. What mechanism caused haemolysis in this patient?
A. Membranopathy
B. Enzymopathy
C. Hemoglobinopathy
D. Autoimmune damage
E. Bone marrow hypoplasia

3. A patient complains that his hair split and fall out, his nails are brittle, his
taste is broken. What is the mechanism of the mentioned symptoms?
A. Iron-containing enzymes deficit
B. Vit B-12 deficit
C. Decreased paratyrin production
D. Vit A deficit
E. Decreased thyroid hormones production

4. A B-12 folio-deficit anaemia developed in a patient following the stomach


resection 5 years ago. What mechanism is typical for this pathology
development?
A. Absence of the internal Casl’s factor
20
B. Absence of the external Casl’s factor
C. Malabsorption of the vit B-12 in small intestine
D. Deficit of folic acid
E. Deficit of transcobalamin

5. Hypochromic anaemia was diagnosed in a 54-year-old patient, who had


permanent contact with lead due to professional activities. Treatment with iron
preparations during a month had no effect. In addition, increased level of iron
was detected in blood serum. What is the cause of the anaemia?
A. Disorders of porthyrin synthesis
B. Vit B-12 deficit
C. Deficit of folic acid
D. Bone marrow hypoplasia
E. Deficit of proteins

6. A 20-year-old man complains of periodic skin and sclera icteritiousness,


fatigue. Minkowsky-Shauffard disease was diagnosed. What is typical for blood
film in this case?
A. Microspherocytosis
B. Anulocytosis
C. Agranulocytosis
D. Macrocytosis
E. Thrombocytosis

7. A woman was hospitalised for examination. Decreasing of haemoglobin levels


down to 90-95 g/l has been taking place from childhood. Treatment with iron
medications had no effect. Blood test results at the moment of hospitalisation:
erythrocytes – 3.2 x 1012/l, Hb – 85 g/l, colour index of blood – 0.78, anisocytosis,
poikilocytosis, target cell erythrocytes, reticulocytes – 16%. Thalassemia was
diagnosed. What type of haemolytic anaemia takes place?
A. Hereditary hemoglobinopathy
B. Hereditary membranopathy
C. Acquired membranopathy
D. Hereditary enzymopathy
E. Acquired enzymopathy

8. A woman developed severe iron-deficiency anaemia on the 6th month of


pregnancy. It was diagnosed due to presence in her blood
A. Anulocytes
B. Macrocytes
C. Poikilocytes
D. Reticulocytes
E. Normocytes

21
9. A B-12 folio-deficit anaemia with disorder of haemopoiesis and appearance of
changed forms of erythrocytes in blood developed in a patient following the
stomach resection. As an illustration of this pathology might serve presence in
blood
A. Megalocytes
B. Microcytes
C. Ovalocytes
D. Normocytes
E. Anulocytes
10. A patient develops decreasing of haemoglobin level and erythrocyte count
from time to time. It was figured out from anamnesis that these episodes appear
after eating of field beans. What type of anaemia takes place?
A. Hereditary enzymopathy
B. Hereditary membranopathy
C. Iron-deficiency anaemia
D. Hereditary hemoglobinopathy
E. Acquired hemolytic anemia
11. During the study of globin molecule structure it was revealed that glutamic
acid was replaced by valine. What hereditary pathology takes place?
A. Sickle-cell anemia
B. Thalassemia
C. Minkowsky-Shauffard disease
D. Favism
E. Haemoglobinosis
12. A hypochromic anemia takes place in a patient with hypoacid gastritis.
Anulocytes, microanisocytosis, and poikilocytosis were revealed in a blood
smear. What type of anaemia takes place?
A. Iron-deficiency
B. Acute posthaemorrhagic
C. Thalassemia
D. Sickle-cell
E. Pernicious

(2009 - 2004)
13. A 36-year-old patient with respiratory viral infection was treated with
sulphanilamide drugs. Blood test findings revealed hyporegenerative
normochrome anemia, leukopenia, and thrombocytopenia. In the bone marrow
decreased amount of myelokariocytes was found. What anemia is it?
A. Posthemorrhagic.
B. Hemolytic.
C. Hypoplastic.
D. B12 and folic deficiency.
E. Iron deficiency.

22
14. A patient, who had arrived from Tunis, revealed α-thalassemia with hemolysis
of erythrocytes and icterus. The presence of what cells in the blood is typical
of this illness?
A. Target-like erythrocytes.
B. Granular erythrocytes.
C. Polychromatophilous erythrocytes.
D.Normocytes.
E. Reticulocytes.

15. A 3-year-old child was hospitalized with hemoglobin opathy (sicklemia).


What amino acid replaced glutamin acid in the β-chain of hemoglobin S?
A. Arginine.
B. Serine.
C. Thyrosin.
D. Phenylalanine.
E. Valine.

16. Beta-thalassemia was revealed in a patient, who came from Tunis. The
disease was accompanied by hemolysis and jaundice. The disease was diagnosed
on the base of presence in blood:
A. Target-like erythrocytes
B. Grained erythrocytes
C. Polychromatophil erythrocytes
D. Normocytes
E. Reticulocytes

17. During the examination of adolescents that reside in mounting region


increase in level of erythrocytes and hemoglobin in peripheral blood was found
out. What is the reason for indicated erythrocytosis?
A. Exogenous hypoxia
B. Diseases of lungs
C. Congenital heart disease
D. Condensation of blood due to large loss of water
E. Vakes’s disease

18. Content of hemoglobin and number of erythrocytes significantly decreases in


a patient’s blood from time to time. It was found out that such attacks appear
after taking some horse beans. What kind of anemia takes place in this case?
A. Enzymopathy
B. Membranopathy
C. Iron deficiency anemia
D. Hemoglobinopathy
E. Acquired hemolytic anemia

23
19. In blood analysis of 37-years-old woman following data were revealed:
content of hemoglobin is 60 g/L, number of erythrocytes is 3.0x10 12/L, and color
index of 0.6; differential count of leukocytes without any changes; number of
platelets is 200x109/L; reticulocytes count of 0.3%; ESR of 18 mm/hour;
microcytosis and poikilocytosis of erythrocytes. Indicate the most probable type
of anemia according to mechanisms of its development.
A. Hypoplastic anemia
B. Hemolytic anemia
C. Acute post-hemorrhagic anemia
D. Iron deficiency anemia
E. B12-folate deficiency anemia

20. A patient aged 20, has periodically yellowness of sclera and skin that is
accompanied by weakness. Minkovski-Shoffar disease is diagnosed. What is the
most typical for blood picture in this disease?
A. Reticulositosis
B. Agranulocytosis
C. Macrocytosis
D. Microspherocytosis
E. Thrombocytosis

21. Posthemorrhagic anemia has developed in a patient with periodical bleeding


due to uterus fribromyoma. What type of chronic post hemorrhagic anemia
takes place in this case?
A. Megaloblastic, hyperchromic, hyperegenerative.
B. Megaloblastic, hypochromic, hyporegenerative.
C. Erythroblastic, hyperchromic, hyporegenerative.
D. Erythroblastic, hypochromic, hyperregenerative.
E. Erythroblastic, hypochromic, hyporegenerative.

22. Hypochromic anemia was found out in a patient aged 54, who had a
prolonged contact with lead at his work. Treatment with iron preparations for a
month didn’t give any effect. The increased amount of iron was determined in
blood serum. What is this anemia due to?
A. Vitamin B12 deficiency
B. Porphirin synthesis impairment
C. Folic acid deficiency
D. Hypoplasia of bone marrow
E. Erythrocyte hemolysis

23. Excessive flow of estrogens into the blood due to follicle persistence (a state
when follicle does not reach complete maturation and ovulation does not take
place) often cause uterine bleeding. What anemia may develop in this case?
A. Iron deficiency
B. Sideroachrestic
24
C. Sickle cell
D. Hypoplastic
E. Metaplastic.

24. Which of the below named anemias relevant to hemoglobinopathies?


A. Minkovsky-Shoffar disease
B. Iron deficiency anemia
C. B12 deficiency
D. Thalassemia
E. Iron refractory anemia

25. Evaluate the blood analysis: erythrocytes 3.10; Hb 90g\L reticulocytes 0.5%.
In the smear there are poikilocytes hypochromatic erythrocytes. Blood serum
iron is 80 micromol/l. What pathology is this typical for?
A. Minkovsky-Shoffer disease
B. Iron deficiency anemia
C. B12 deficiency
D. Sickle cell anemia
E. Iron refractory anemia

26. The experiment was carried out on a rabbit. The increase of the number of
erythrocytes and hemoglobin in the blood due to the stimulation of
erythropoiesis by erythropoietin was determined 2 weeks later after the
narrowing of renal artery. What increases the formation of erythropoietin?
A. Hypoosmia
B. Hypercapnia
C. Hypoxemia
D. Hyperosmia
E. Hypovolemia

27. Hemolytic anemia with decrease of osmotic erythrocyte resistance that


averaged 0.6-0.5 was revealed in a smear of venous blood during microscopic
examination. What substance accumulation in the blood plasma may also
indicate the development oh hemolytic anemia?
A. Creatinine
B. Urea
C. Indirect bilirubin
D. Lactic acid
E. Inorganic phosphate

28. Hypochromia of erythrocytes, micro-, anisocytosis, poikilocytosis, are


determined in the blood in case of development of iron deficiency and iron
refractory anemia. What index must be determined to carry out differential
diagnosis of these anemias?
A. Serum chlorine
25
B. Serum phosphorous
C. Serum magnesium
D. Serum calcium
E. Serum iron

29. A patient aged 32 with massive hemorrhage due to car accident trauma was
admitted to the hospital. Pluse-100 beats per min, respiratory rate-22 per 2 min,
BP-100/60 mmHg. What blood change will be the most characteristic in an hour
after hemorrhage?
A. Erythropoenia
B. Hypoproteinemia
C. Hypovolemia
D. Leucopoenia
E. Erythrocyte hypochromia

30. Singular oxyphilic normoblasts appeared in the blood of a patient after acute
post-traumatic hemorrhage composing 15% of blood volume. On supravital
staining 25% of reticulocytes were found. What is the patient anemia according
to its ability of regeneration?
A. Hyperregenerative
B. Regenerative
C. Hyporegenerative
D. Aregenerative
E. Hypo- and aregenerative

31. What index of blood analysis is the most typical for beta-thalassemia?
A. Considerable decrease of erythrocytes and hemoglobin
B. Erythrocytes with basophilic stippling
C. Increase of fetal hemoglobin
D. Target-like erythrocytes
E. Increase of met-hemoglobin

32. A patient had anemia due to profuse blood loss. What blood changes are
typical at the beginning of development of acute post-hemorrhagic anemia?
A. Presence of megalocytes in the blood
B. Absence of reticulocytes
C. Poikilocytosis, anisocytosis
D. Hyperchromia
E. Normochromia

33. Erythropenia, hyperchromia, normocytes, macrocytes, megalocytes,


poikilocytosis were found out in a patient’s blood at examination. What is the
cause of this pathology?
A. Ascariasis
B. Deficiency of gastromucoprotein
26
C. Iron deficiency in food
D. Trichocephaliasis
E. Fequent loss of blood

34. In an infant, who is under an artificial nutrition with cow milk, severe
anemia has developed. At the blood count of the infant: number of erythrocytes
is 4x1012/L, content of hemoglobin is 68 g/L, reticulocytes of 0%. What kind of
anemia developed in the infant?
A. Sickle-cell anemia
B. Inborn hemolytic anemia
C. B12-deficiency anemia
D. Hypopastic anemia
E. Iron-deficiency anemia

35. Predomination of erythroblasts, normoblasts and megaloblasts was revealed


in blood analysis of a patient with anemia. The same cells were found in bone
marrow. What type of anemia do these changes characteristic for?
A. Post-hemorrhagic
B. Hemolytic
C. Aplastic
D. B12-folate deficiency anemia
E. Iron-deficiency anemia

36. Funicular myelosis and hyperchromic anemia developed in a man 7 years


later after the stomach resection due to ulcer. What pathogenic mechanism of
changes in spinal cord is the most possible one?
A. Hypoxia impairment in anemia
B. Accumulation of methylmalonic acid in cyanocobalamin deficiency
C. Impairment of DNA in cyanocobalamin deficiency
D. Deficiency of folic acid
E. Deficiency of iron containing enzyme

37. Examining the oral cavity of a patient, a dentist paid attention to the
presence of inflammatory-dystrophy process in the mucous membrane
(Gunter’s glossitis, atrophic stomatitis). Blood analysis revealed hyperchromic
anemia. What factor is a cause of this disease?
A. Hypovitaminosis B6
B. Hypovitaminosis A
C. Increase of stomach juice acidity
D. Hypovitaminosis B1
E. Hypovitaminosis B12

27
38. A female patient complains of malaise, weakness, breathlessness, rapid
fatigability, and dizziness. Her blood test data: erythrocytes-1.8x1012/L, Hb-80
g/L, leukocytes-3.2x109/L, color index-1.5. Anisocytosis, poikilocytosis,
megaloblasts, megalocytes were found in smear. What is the possible diagnosis?
A. B12-deficiency anemia
B. Posthemorragic anemia
C. Acute leukemia
D. Iron deficiency anemia
E. Immunohemolytic anemia

39. Anemia, leuko- and thrombocytopenia, color index-1.3, presence of


megaloblasts and megalocytes were determined in the laboratory analysis of
blood of a patient a year later after he was operated on for subtotal resection of
the stomach for the ulcer of lesser curvature of the stomach. What factor
deficiency results in these changes?
A. Gastromucoprotein
B. Gastrin
C. Pepsin
D. Chlorine hydrate
E. Mucin
40. Amino acids replacement in alpha and beta chains of hemoglobin takes place
in a number of hemoglobimopathies. Which of them is typical for HbS (sickle-
cell anemia)?
A. Glycine to serine
B. Aspartate to lysine
C. Methionine to histidine
D. Glutamate to valine
E. Alanine to serine

41. Hereditary microspherocytic hemolytic anemia (Mincovsky-Shoffar disease)


was diagnosed in a woman aged 34. What mechanism caused hemolysis of
erythrocytes in the patient?
A. Enzymopathy
B. Hemoglobinopathy
C. Autoimmune impairment
D. Membranopathy
E. Hypoplasia of bone marrow

42. Megaloblastic anemia was diagnosed in a patient. What substance deficiency


may cause the development of this disease?
A. Cyanocobalamin
B. Cholecalciferol
C. Magnesium
D. Glycine
E. Copper
28
43. Three years ago a man aged 45 was operated on for stomach resection. After
the operation the content of erythrocytes in the blood is 2.0x10 12, Hb 85 g/l, color
index-1.27. What vitamin absorption is impaired that causes the change of
erythropoiesis?
A. C
B. P
C. A
D. B6
E. B12

44. Substitution of the glutamic acid on valine was revealed while examining
initial molecular structure. For what iherited pathology is this typical?
A. Minkowsky-Shauffard disease
B. Sickle-cell anemia
C. Favism
D. Talassemia
E. Hemoglobinosis
45. A 58-year-old woman complaints of increased tiredness, decreased capasity for
work, somnolence and dyspnea during fast walking. Blood test revealed:
erythrocytes -4,6x1012/l, hemoglobin - 92 g/l, colour index – 0,6. Blood smear
demonstrated high contents of microcytes and anulocytes. What anemia is it
typical for?
A. Hemolytic
B. Iron deficiency
C. Acute posthemorragic
D. Penicious
E. Sickle cell
46. A 43-year-old man suffers from chronic atrophic gastritis and Megaloblastsc
hyperchromic anemia. He also has methylmalonic aciduria. Insufficiency of what
vitamin led to the development of such complex of symptoms?
A. Vitamin B5
B. Vitamin B2
C. Vitamin B12
D. Vitamin B3
E. Vitamin B1
47. A woman with III (B), Rh- blood group born a child with II (A) blood group.
The child is diagnosed with hemolytic disease of newborn as a result of rhesus
incompatibility. What blood group is the child΄s father likely to have?
A. I (0), Rh+
B. I (0), Rh-
C. II (A), Rh-
D. II (A), Rh+
E. III (B), Rh+

29
48. Examination of a 43 y.o. anephric patient revealed anemia symptoms. What
is the cause of this symptoms?
A. Iron deficiency
B. Vitamin B12 deficiency
C. Enchanced destruction of erythrocytes
D. Folic acid deficiency
E. Reduced synthesis of erythropoetins

49. Surgical removal of a part of stomach resulted in disturbed absorption of


vitamin B12, it is excreted with feces. The patient was diagnosed with anemia.
What factor is necessary for absorption of this vitamin/
A. Gastrin
B. Gastromucoprotein
C. Pepsin
D. Folic acid
E. Hydrochloric acid

50. A 56 year old patient came to a hospital with complaints about general
weakness, tongue pain and burning, sensation of limb numbness. In the past he
underwent resection of forestomach. In blood: Hb-80 g/l; erythrocytes-
2,0x1012/l; colour index-1,2, leucocytes-3,5x109/l. What anemia type is it?
A. B12-folate deficient
B. Iron-deficient
C. Posthemorragic
D. Aplastic
E. Hemolytic

51. Packed cell volume of a man was 40% before the trauma. What packed cell
volume will be observed 24 hours after blood loss of 750 ml?
A. 30%
B. 40%
C. 55%
D. 50%
E. 45%

52. A patient was diagnosed with autoimmune hemolitic cytotoxic anemia. What
substances are antigens in II type allergic reactions?
A. Hormones
B. Modified receptors of cell membranes
C. Antibiotics
D. Serum proteins
E. Inflammation modulators

30
53. A 55 y.o. woman consulted a doctor about having continuous cyclic uterine
hemorrhages for a year, weakness, dizziness. Examination revealed skin pallor.
Hemogram: Hb-70 g/l, erythrocytes-3,2x1012/l, color index-0,6, leucocytes-
6,0x109/l,reticulocytes-1%; erythrocyte hypochromia. What anemia is it?
A. Iron-deficiency anemia
B. Aplastic anemia
C. Hemolytic anemia
D. Chronic posthemorhagic anemia
E. B12-folate-deficiency anemia

54. Patient, 52 years old, had the operation of stomach resection three years. In
the blood: erythrocytes - 2,0х1012/l, Hb-85 g/l, GV - 1,27. Violation of what
vitamin uptake caused such changes?
A. P
B. B12
C. B6
D. A
E. C
55. Inkcretory function of kidneys is violated at a patient with chronic
glomerulonephritis. To the deficiency of what uniform elements of blood will it
lead?
A. Red corpuscles
B. Red corpuscles and leukocytes
C. Thrombocytes
D. Leukocytes
E. Leukocytes and thrombotcytes

56. Long-term starvation cure of a patirnt resulted in deminieshed ratio of


albumines and globulines in plasma. What of the following will be result of these
changes?
A. Decrease of ESR
B. Hypercoagulation
C. Increase of ESR
D. Increase of hematocrit
E. Decrease of hematocrit

57. 2 years ago a patient underwent resection of pyloric part of stomach. He


complains of weakness,periodical dark shadows beneath his eyes, dyspnea. In
blood: Hb:-70 g/l, erythrocytes-3,0x1012/l, colour index-0,7. What changes of
erythrocytes in blood smears are the most typical for this condition?
A. Microcytes
B. Shizocytes
C. Megalocytes
D. Ovalocytes
E. Macrocytes
31
58. A three-year child with the fever after taking an aspirin has increased
hemolysis of red corpuscles. Insufficiency of what enzyme could cause hemolytic
anaemia at a child?
A. Glucose 6-phosphate dehydrogenase
B. - glutamintransferase
C. Glycerophosphatedehydrogenase
D. Glucose 6-phosphatase
E. -

59. A 20 year old patient complains of general weakness, dizziness, quick


fatigability. Blood analysis results: Hb-80 g/l. Microscopical examination results:
erythrocytes are of modified form. This condition might be caused by:
A. Acute intermittent porphyria
B. Hepatocellular jaundice
C. Obturative jaundice
D. Addison’s disease
E. Sickle-cell anemia
Correct answers
1-A 2-A 3-A 4-A 5-A 6-A 7-A 8-A 9-A 10-A 11-A 12-A
13-C 14-A 15-E 16-A 17-A 18-A 19-D 20-D 21-E 22-D 23-A 24-D
25-E 26-C 27-C 28-E 29-C 30-A 31-C 32-E 33-B 34-E 35-D 36-B
37-E 38-A 39-A 40-D 41-D 42-A 43-E 44-B 45-B 46-C 47-D 48-E
49-B 50-A 51-A 52-B 53-D 54-B 55-A 56-C 57-A 58-A 59-E

Situate task:

1. Patient, 40 years, delivered to the surgical clinic with the knife wound of the
thorax. Objectively: the skin is pale, BP-70/40 mm Hg, tachycardia. In a blood
test: RBC - 3х1012/l, Hb – 60g/l; reticulocytes - 15‰, ESR -12 mm/h.
1. What is the pathology at this patient?
__________________________________________________________________
2. Calculate the color index.
__________________________________________________________________
3. What is the mechanism of this pathology development?
____________________________________________________________________
____________________________________________________________________
____________________________________________________________________
4. What does amount of reticuloctes testify about?
__________________________________________________________________
5. Describe this pathology according to the pathogenesis, color index, type of
hemopoiesis, ability of bone marrow to regeneration.
____________________________________________________________________
____________________________________________________________________
____________________________________________________________________

32
2. Patient S., 55 years, hospitalized in a gynaecological clinic with complaints
about the protracted and large uterine bleeding during the last year. A skin is
pale, rapid pulse. Picture of blood: erythrocytes. -2,8х1012/l; Hb-65g/l,
reticulocytes 3‰; ESR -15 mm/h. Iron of blood plasma - 6 micromole/l.
1. What is the pathology at this patient?
__________________________________________________________________
2. Calculate the color index.
__________________________________________________________________
3. What regenerative and degenerative changes of RBC is possible to find out in the
blood smear at this pathology?
____________________________________________________________________
____________________________________________________________________
____________________________________________________________________
4. Describe pathogenesis of this pathology.
____________________________________________________________________
____________________________________________________________________
____________________________________________________________________
5. Describe this pathology according to the ability of the bone marrow to
regeneration, color index, type of hemopoiesis.
____________________________________________________________________
____________________________________________________________________
____________________________________________________________________

3. Haemoglobin – 90g/l, RBC - 2,9x1012/л. In the blood smear: reticulocytes-


30‰. Maintenance of indirect bilirubin in the blood is multiplied.
1. What is the pathology at this patient?
__________________________________________________________________
2. Calculate the color index.
__________________________________________________________________
3. What is the mechanism of this pathology development?
____________________________________________________________________
____________________________________________________________________
____________________________________________________________________
4. Describe this pathology according to the ability of the bone marrow to
regeneration, color index, type of hemopoiesis.
____________________________________________________________________
____________________________________________________________________
____________________________________________________________________

4. Following picture of blood was founded at a patient: RBC - 1,4x1012/l,


hemoglobin- 82g/l, in the blood smear: anisocytosis, poikilocytosis, megaloblasts,
megalocytes.
1. What is the pathology at this patient?
__________________________________________________________________

33
2. What are the reasons for this anemia development?
____________________________________________________________________
____________________________________________________________________
____________________________________________________________________
3. What are the mechanisms of this anemia development?
____________________________________________________________________
____________________________________________________________________
____________________________________________________________________
4. Give the morphological description of megaloblasts and megalocytes?
____________________________________________________________________
____________________________________________________________________
____________________________________________________________________
5. What violations from the side of other organs and systems are possible at this
anemia? What are the mechanisms of their development?
____________________________________________________________________
____________________________________________________________________
____________________________________________________________________
____________________________________________________________________
____________________________________________________________________

5. Following blood picture was discovered at a patient: RBC-1,6x1012/l,


haemoglobin-70g/l. In a blood smear: anisocytosis, poikilocytosis, megalocytes,
reticulocytes are absent, neutrophyles polysegmentation, leukopenia.
1. What is the pathology at this patient?
__________________________________________________________________
2. Calculate the color index.
__________________________________________________________________
3. What is the mechanism of this pathology development?
____________________________________________________________________
____________________________________________________________________
4. Describe this pathology according to the ability of bone marrow to regeneration,
color index, type of hemopoiesis.
____________________________________________________________________
____________________________________________________________________

6. The girl of 3 month is hospitalized (by nationality - moroccan). Objectively:


skin and scleras are icteric, temperature of body - 390С, a spleen and liver is
increased, urine is black, contains of haemoglobin and haeemosiderin. Blood
test: haemoglobin - 70 g/l, RBC - 2,3x1012/l, leucocytes-15x109/l, neutrophylic
leukocytosis with a left nuclear shift, ESR – 25mm/h, poikilocytosis, anisocytosis,
single sickle-cells. Haemoglobin HBS and HBA was discovered.
1. What pathology is observed at a child?
__________________________________________________________________
2. Calculate the globular value.
__________________________________________________________________
34
3. Explain the mechanism of hemoglobinuria development.
____________________________________________________________________
4. Describe this anemia by 4 known classifications.
____________________________________________________________________
____________________________________________________________________

7. Patient N., 68 years, seven years ago carried the total resection of stomach
concerning ulcerous illness. Was hospitalized in a therapeutic clinic with
complaints about a general weakness, dizziness, pain and heartburn in the area
of tongue, feeling of numbness and “gooseflesh” in extremities. Objectively:
pallor of skin, smooth and laked, bright red tongue. Blood test: haemoglobin –
33g/l, red corpuscles - 0,8x1012/l, leucocytes – 3x109/l, thrombocytes – 90x109/l,
ESR – 14mm/h, anisocytosis, poikilocytosis, megalocytes with the Jolly’s bodies
and Cabot’s rings, hyperchromia of red corpuscles.
1. What pathology is observed at a patient?
__________________________________________________________________
2. Calculate the globular value.
__________________________________________________________________
3. What is the mechanism this blood pathology development?
____________________________________________________________________
___________________________________________________________________
4. Describe this anemia by 4 known classifications.
____________________________________________________________________
________________________________________________________________

35
Practical work: “PATHOLOGY OF HEMATOPOIESIS. ANAEMIA.
(Morphology of blood)”

Object of work: to study the changes of morphological blood picture at


different types of anaemia.

EXPERIMENT № 1. Preparation of blood smears of rabbit with hemolytic


anaemia.
2 days prior to experiment enter to the rabbit under a skin 0,08-0,17 ml lead
acetate on the 1 kg of the body weight.
Do the smears of the blood got from the ear of rabbit. Dry out the smears on
air, fix by mixture of Nikiforov and paint by Romanovsky 30-40 min.
Examine the smears and sketch the morphological changes of blood picture in
the protocols.

Conclusions__________________________________________________________
____________________________________________________________________
____________________________________________________________________
____________________________________________________________________

EXPERIMENT № 2. Microscopic examination of blood smears of patients


with В12-deficiency anaemia.
Smears of blood are of patients being on treatment in the hematological
department of regional hospital named after M.I. Pyrogov.
Sketch the morphological changes at this anaemia at the protocols.

1 – ____________________________
2 – ____________________________
3 – ____________________________
4 – ____________________________
5 – ____________________________
6 – ____________________________
7 – ____________________________
8 – ____________________________
9 – ____________________________
10 – ___________________________
11 – ___________________________
12 – ___________________________
13 - ___________________________

Hemolytic anemia

36
Iron-deficiency anemia
1 – ____________________________
2 – ____________________________
3 – ____________________________
4 – ____________________________
5 – ____________________________
6 – ____________________________
7 – ____________________________
8 – ____________________________

1 – ____________________________
2 – ____________________________
3 – ____________________________
4 – ____________________________
5 – ____________________________
6 – ____________________________
7 – ____________________________
8 – ____________________________
9 – ____________________________
10 – ___________________________
11 – ___________________________
12 – ___________________________
13 - ___________________________

В12-deficiency anemia

Conclusions__________________________________________________________
____________________________________________________________________
____________________________________________________________________
____________________________________________________________________
____________________________________________________________________
____________________________________________________________________
____________________________________________________________________

37
THEME: PATHOLOGY OF WHITE BLOOD.
LEUKOCYTOSIS. LEUKOPENIA.

Actuality of theme.
Pathological processes in an organism are not strictly isolated and the system
of blood reacts on the whole on them. There are quantitative and qualitative
pathological changes of leukocytes. These changes can be the result of primary
demage of leukocytic row cells at the hemopoietic tissue or at the circulatory system
by the various factors of external environment. The secondary changes of leukocytes
arise up as the proper, often protective reaction of organism on pathological
processes. Knowledges of general conformities to the law of leukocytes system
violations development are necessary for successful diagnostics and treatment of
many diseases.

General purpose of the lesson.


To learn reasons and mechanisms of leukocytosis and leukopenia development.
To learn the qualitative changes of leukocytes at different diseases.

For this it is necessary to know (the concrete purposes):


1. To select principal reasons of different types of leukocytosis and leukopenia
origin.
2. To explain the mechanisms of different types of leukocytosis and leukopenia
development.
3. To model leukocytosis and leukopenia.
4. To count the amount of leukocytes and explain its changes.
5. To define the qualitative changes of leukocytes.

For realization of purposes of lesson it is necessary to have the base


knowledges-skills:
1. Number of leukocytes, percent correlation of different forms of leukocytes.
(Normal Physiology Department)
2. Functions of leukocytes. (Normal Physiology Department)
3. Morphological features of different forms of leukocytes. (Normal Physiology
Department)

The checking of primary level of knowledges.


Give the answers to the following questions:
1. Leukocytosis, definition.
2. Types of leukocytosis according to the reasons of development.
3. What is absolute leukocytosis?
4. What is relative leukocytosis?
5. Types of leukocytosis according to the mechanism of development.
6. Types of leukocytosis according to the cellular composition.
7. What is the difference between physiological and pathological leukocytosis?
8. Examples of physiological leukocytosis.
38
9. What pathology is neutrophilic leukocytosis characteristic for?
10. What pathology is eosinophilic leukocytosis characteristic for?
11. What pathology is basophilic leukocytosis characteristic for?
12. What pathology is lymphocytosis characteristic for?
13. What pathology is monocytosis characteristic for?
14. Leukopenia, definition.
15. Types of leukopenia according to the mechanism of development.
16. Types of leukopenia according to cellular composition.
17. Agranulocytosis, definition.
18. Types of agranulocytosis according to pathogenesis.
19. What is a leukogram?
20. What is a nuclear shift?
21. What states is a nuclear shift to the left characteristic for?
22. What states is a nuclear shift to the right characteristic for?
23. Types of nuclear shift to the left.
24. Examples of degenerative changes of leukocytes.

Standards of answers at the theoretical questions of initial level of


knowledges:
1. It is multiplying the amount of leukocytes in the unit of blood volume higher
than 9х109/l.
2. Physiological and pathological.
3. Characterized by multiplying the absolute amount of leukocytes in unit of
blood volume.
4. Characterized by multiplying the separate forms of leukocytes in comparing to
other.
5. А) As a result of increased leukopoiesis (reactive, tumour hyperplasia)
B) As a result of the increased exit of leukocytes from the bone marrow to the
blood
C) Redistributive
6. А) Neutrophilic
B) Eosinophilic
C) Basophilic
D) Lymphocytosis
E) Monocytosis
7. Relative leukocytosis is short-term and it is not accompanied with the changes
of leukogram.
8. А) Alimentary
B) Myogenic
C) Emotional
D) Static
E) Leukocytosis of pregnant
F) Leukocytosis of new-born
9. А) Purulent inflammation
B) Aseptic inflammation
39
C) Chronic myeloleukemia
10. А) Allergic reactions
B) Helminthiasis
C) Chronic myeloleukemia
11. А) Autoallergy
B) Chronic myeloleukemia
C) Haemophilia
12. А) Acute infectious diseases (whooping-cough, viral hepatitis)
B) Some chronic infectious diseases (tuberculosis, syphilis, brucellosis)
C) Chronic lymphocytic leukemia
13. А) Chronic specific infections (tuberculosis, brucellosis)
B) Infectious mononucleosis
C) Infections caused by Rickettsia and protozoan (epidemic typhus, malaria)
14. It is diminishing of amount of leukocytes in unit of blood volume below
4х109/l.
15. Absolute and relative
16. А) Neutropenia
B) Lymphopenia
C) Eosinopenia
D) Monocytopenia
17. It is a clinical-hematological syndrome which is characterized by diminishing
or absence of granulocytes in unit of blood volume.
18. А) Myelotoxic
B) Immune
19. It is percent correlation of different forms of leukocytes.
20. It is violation of correlation between the different forms of neutrophiles.
21. А) Purulent inflammation
B) Chronic myeloleukemia
22. А) В12-deficiency anaemia
B) Folic acid-deficiency anaemia
C) Action of ionizing radiation
23. А) Regenerative
B) Hyperregenerative
C) Degenerative
D) Regenerative-degenerative
24. А) Anisocytosis
B) Presence of the toxic granulosity
C) Absence of physiological granulosity
D) Pyknosis of nuclei
E) Hyper- or hyposegmentation of nuclei

40
Theoretical questions for the FMC № 2.
1. Leukocytosis, definition. Classification of leukocytosis.
2. Reasons and mechanisms of different forms of leukocytosis development.
3. Shifts of leukogram. Types of shift. Value of the shifts determination. Reasons
of different types of shifts development.
4. Qualitative changes of leukocytes, their value in pathology.
5. Leukopenia, definition. Classification. Reasons and mechanisms of
development. Picture of blood at the different types of leukopenia.
6. Value of leukocytosis and leukopenia for an organism.
7. Agranulocytosis, definition. Kinds. Reasons. Mechanisms of development.

Literature.
1. Handbook of general and Clinical Pathophysiology/ Edited by
prof.A.V.Kubyshkin, CSMU, 2005.p.204-209
2. Pathophysiology/ Edited by prof.Zaporozan, OSMU, 2005p.179-187
3. General and clinical pathophysiology/ Edited by Anatoliy V/ Kubyshkin –
Vinnytsia: Nova Knuha Publishers – 2011. p.410-421

Testing according system “Krok-I”


Tests of an open database (2010)

1. A 30-year-old man got approximately 3 grays of radiation. What blood


changes will appear in 8 hours after exposure to radiation?
A. Lymphopenia
B. Leucopenia
C. Granulocytopenia
D. Thrombocytopenia
E. Anemia

2. Leucocytosis without changes in leukocyte formula was revealed in a student


after exam. Choose the most possible mechanism of relative leucocytosis
development.
A. Redistribution of leukocytes in a body
B. Strengthening of leucopoiesis
C. Decreasing of leukocytes destruction
D. Deceleration of leukocytes migration towards tissues
E. Acceleration of leucopoiesis

3. Hematologic syndrome is one among signs of acute radiation sickness. What


changes in peripheral blood are typical for period of primary reactions?
A. Leucocytosis, lymphopenia
B. Leukopenia, lymphopenia
C. Leukopenia, thrombocytopenia
D. Leucocytosis, thrombocytopenia
E. Leukopenia, anaemia, thrombocytopenia
41
4. A woman during a period of herb flowering developed acute inflammatory
disease of upper respiratory tract and eyes: hyperemia, oedema, mucosal
discharge. What kind of leucocytosis is the most typical?
A. Eosinophilia
B. Basophilia
C. Neutropenia
D. Lymphocytosis
E. Monocytosis

5. Increased leukocyte count was registered in blood smear taking from a patient
with appendicitis. What type of leucocytosis might be in this case?
A. Neutrophilic
B. Basophilic
C. Eosinophilic
D. Lymphocytosis
E. Monocytosis

6. A 30-year-old man got 3 grays of ionizing radiation. What changes might be


registered in 8 hours?
A. Lymphopenia
B. Leukopenia
C. Granulocytopenia
D. Thrombocytopenia
E. Anaemia

(2009 – 2004)
7. The presence of agranulocytosis was determined in the blood analysis of a
liquidator of accident at Chernobyl atomic power station who had got 5 Gr dose
of irradiation. What pathogenetic mechanism is the leading one in its
appearance?
A. Inhibition of leucopoiesis
B. Increased penetration of granulocytes into tissues
C. Increase of leucocytes destruction
D. Impairment of going out of mature leukocytes from bone marrow
E. Development of autoimmune process.

8. During the development of acute pulpitis a patient complained of paroxysm of


pain in the upper jaw, which is increasing at night, fever. At examination
leucocytosis was established in the blood. What kind of leucocytosis is possible in
this case?
A. Basophilic leukocytosis
B. Lymphocytosis
C. Eosinophilic leukocytosis
D. Monocytosis
E. Neutrophilic leukocytosis.
42
9. Considerable increase of the number of eosinophils in a unit of blood volume
was determined during the examination of a 5 years old boy. What may cause
eosinophilia in this patient?
A. Helminthic invasion
B. Obesity
C. Hypodynamia
D. Hypothermia
E. Physical exertion

10. Leucocytosis was found out in a person who didn’t complain of his health.
The cause of this may be that fact that the blood was taken for analysis after:
A. Physical load
B. Mental work
C. Rest at a health resort
D. Considerable use of water
E. Usage of alcohol

11. Lymphocytosis was revealed in a patient. What diseases may be


accompanied by lymphocytosis?
A. Sepsis
B. Helminthic invasion
C. Somatotropin insufficiency
D. Pertussis, chicken pox
E. Bronchial asthma

12. Neutropoenia is found out in a patient who has the manifestations of


immunodeficiency. What diseases may neutropoenia be determined in?
A. After profuse haemorrhage
B. Fuzarium fungus poisoning
C. Myeloleukemia
D. Insufficiency of sexual gland function
E. Septic process

13. Increase of the number of eosinophils was determined in a patient with


endocrine pathology during examination. Which of the named diseases may be
accompanied by eosinophilia?
A. Pheochromocytoma
B. Conn’s disease
C. Acromegaly
D. Cushing disease
E. Addison’s disease

43
14. Functional insufficiency of monocytes is accompanied by immunodeficiency.
What BAS produced by monocytes stimulate specific reaction?
A. Interleukin-2
B. Lysocime
C. Interleukin-1
D. Fibronectin
E. Myeloperoxidase

15. Relative neutropoenia with degenerative shift was revealed in the blood of a
patient with TB during examination. What change of differential blood count
corresponds to this state?
A. Decrease of the number of segmented forms and increase of band forms of
neutrophils
B. Decrease of lymphocyte number
C. Increase of monocyte number
D. Increase of basophil number
E. Decrease of eosinophil number

16. What conditions are accompanied by neutrophilic leukocytosis with shift of


differential count to the left?
A. Purulent inflammation
B. Tuberculosis
C. Infections mononucleiosis
D. Agranulocytosis
E. Alimentaray leukocytosis

17. Patient R. aged 12 was operated on for acute phlegmonous appendicitis. At


the examination of his blood, the amount of leukocytes is 12x10 9/L. On the blood
smear there are: basophils – 0%; eosinophils – 2%; monocytes – 2%; juvenile
neutrophils – 0%; stab neutrophils – 30%; segmented neutrophils – 43%;
lymphocytes – 23%; monocytes – 0%. Stab (immature) neutrophils have
pyknosis of nuclei. There is anisocytosis of neutrophils; some of them have toxic
granulation. What form of change of leukocyte blood composition takes place in
this case?
A. Neutrophilia with degenerative shift to the left.
B. Neutrophilia with regenerative shift to the left.
C. Neutrophilia with hyperegenerative shift to the left.
D. Leukemoid reaction of neutrophil type.
E. Neutrophilia with the shift to the right.

18. Neutrophil leukocytosis is determined in a patient with chronic myelogenous


leukemia. Which variant of nuclear shift of differential count to the left is the
most typical for chronic myelogenous leukemia?
A. Regenerative
B. Degenerative.
44
C. Hyperregenerative.
D. Regenerative and degenerative.
E. Hyporegenerative.

19. Acute pain in the lower jaw, swelling of the cheek, temperature of 37.6oC
occurred in the patient who had had dental caries for some years. What changes
in the patient’s blood may be observed in this case?
A. Neutrophilic leukocytosis;
B. Leucopoenia;
C. Moncytosis;
D. Anemia;
E. Eosinophilia.

20. A patient with acute pulpitis has an increased body temperature and a
number of leukocytes up to 14.109/L, differential count represents: basophils-
0%; eosinophils-2%; megakaryocytes-0%; juvenile neutrophils-2%; stab
neutrophils-8%; segmented neutrophils-58%; lymphocytes-26%; monocytes-
4%. How can you evaluate such changes in blood?
A. Neutrophilic lekocytosis with regenerative shift to the left
B. Neutrophilic lekocytosis degenerative shift to the left
C. Neutrophilic lekocytosis with hyperregenerative shift to the left
D. Lymphocytosis
E. Neutrophilic leukocytosis with shift to the right

21. A patient aged 30 took sulfaethidole for infectious process of mucous


membrane in the mouth locally (as powder). Preparation has hemotoxic action
and the treatment was complicated by the development of agranulocytosis. At
examination it was revealed:
A. Increase of agranulocytes in the blood;
B. Decrease of the number of granulocytes in the blood on the background of
leukocytosis;
C. Decrease of the number of granulocytes in the blood on the background of
leucopoenia;
D. Decrease of the number of neutrophylic granules with their simultaneous
increase;
E. Loss of their granules by granulocytes.

22. A women fell ill with purulent stomatitis. What index of complete blood
count is characteristic for this disease?
A. Lymphocytosis
B. Thrombocytosis
C. Leukocytosis
D. Anemia
E. Monocytosis

45
23. A patient addressed a dentist with complaints of affections of mucous
membrane of his mouth. During the examination of the patient ulcerous
stomatitis with necrosis in center was revealed at him in area of his palate. In the
history of disease of the patient recently endued pneumonia and taking of
medicines (sulfonilamides) were present. After administering treatment the
doctor pointed the patient for blood analysis. What pathology from below
mentioned does doctor suppose?
A. Immune agranulocytosis
B. Iron deficiency anemia
C. Thrombocytopoenia
D. Infectious mononucleosis
E. Infectious lymphocytosis

24. Myocardial infarction was diagnosed in 65-years-old man. Neutrophilic


leukocytosis with left shift is present in the blood of this patient. What factors
underlie this phenomenon?
A. Products of tissue decay
B. Elevation of mass of muscular fibers
C. Disorders of alveolar ventilation
D. Decrease in glycogen content in the myocardium
E. Increase of arterial pressure

25. Increased concentration of leukopoietins in blood was found in a patient with


acute appendicitis. What kind of lekocytosis occurs in these conditions?
A. Neutrophilic
B. Basophilic
C. Eosinophilic
D. Lymphocytosis
E. Monocytosis

26. A patient has deficiency of cyancobalamine and folic acid that leads to
disorder of leucopoiesis. What changes takes place in these conditions?
A. Leucopoenia
B. Eosinophilia
C. Basophilia
D. Hemophilia
E. Hyperemia

27. A patient, who was exposed to ionizing radiation, has panmyelophtisis and
secondary infections. What changes occur in blood analysis in this case?
A. Agranulocytosis
B. Leucocytosis
C. Eosinophilia
D. Basophilia
E. Hyperemia
46
28. When examining a blood in a patient, who endued bleeding three days ago,
following data was revealed: number of leukocytes is 12x10 9/L, basophils count
is 0%, eosinophils count is 3%, myelocytes count 0%, juvenile neutrophils count
is 3%, stab neutrophils count is 12%, segmented neutrophils count is 62%,
lymphocytes count 16%, and monocytes count is 4%. What kind of changes of
leukocyte differential count takes place in this case?
A. Neutrophilia with regenerative shift to the left
B. Neutrophilia with degenerative shift to the left
C. Neutrophilia with shift to the right
D. Absolute lymphopoenia
E. Absolute monocytopoenia

29. A 40-years-old patient, who was admitted to the surgical department with
diagnosis of phlegmona of thigh, had high temperature, tachycardia, and
breathlessness. On the blood test of this patient: number of leukocytes is
25x109/L; eosinophils count is 1%, myelocytes count is 1%, juvenile neutrophils
count is 15%, band neutrophils count is 25%, segmented neutrophils count is
40%, lymphocytes count 14%, monocytes count is 4%. What kind of shift in
differential count is present in this case?
A. Regenerative
B. Hyperregenerative
C. Degenerative
D. Regenerative-degenerative
E. Leukemoid

30. 24 hours after appendectomy blood of a patient presents neutrophilic


leukocytosis with regenerative shift. What is the most probable mechanism of
leukocytosis development?
A. Amplification of leucopoiesis and decelerated emigration of leukocytes to
the tissues
B. Decelerated emigration of leukocytes to the tissues
C. Redistribution of leukocytes in the organism
D. Amplification of leucopoiesis
E. Decelerated leukocyte destruction

31. Having helped to eliminate consequences of a failure at a nuclear power


plant, a worker got an irradiation doze of 500 roentgen. He complaines of
headache, nausea, dizziness. What changes in leucocytes quantity can be
expected 10 hours after irradiation?
A. Neutrophilic leukocytosis
B. Leukemia
C. Lymphocytosis
D. Agranulocytosis
E. Leukopenia

47
32. Blood sampling for bulk analysis is recommended to be performed on an
empty stomach and in the morning. What changes in blood count can occur if to
perform blood sampling after food intake?
A. Increased contents of leukocytes
B. Reduced contents of erythrocytes
C. Reduced contents of thrombocytes
D. Increased plasma proteins
E. Increased contents of erythrocytes

33. A man, 30 years, got an radiation in a dose about 3 Gr. What change in
blood will be in 8 hours after radiation?
A. Granulocytopenia
B. Thrombocytopenia
C. Anaemia
D. Limphopeniya
E. Leukopenia

34. Parents of 3 year old child have been given him antibiotics with purpose of
preventing enteric infections for a long time. Month later the child’s condition
changed for the worse. Blood examination revealed apparent leucopenia and
granulocytopenia. What is the probable mechanism of blood changes?
A. Autoimmune
B. Redistributive
C. Age-specific
D. Myelotoxic
E. Hemolytic

35. A 26 year old man is in the torpid shock phase as a result of a car accident.
In blood: 3,2x109/l. What is the leading mechanism of leucopenia development?
A. Intensified elimination of leukocytes from the organism
B. Lysis of leukocytes in the blood-forming organs
C. Leikopoesis inhibition
D. Disturbed going out of mature leukocytes from the marrow into the blood
E. Redistribution of leukocytes in bloodstream

36. Two hours after an exam a student had a blood count done and it was
revealed that he had leukocytosis without significant leukogram modifications.
What is the most probable mechanism of leukocytosis development?
A. Redistribution of leukocytes in the organism
B. Deceleration of leukocyte lyses
C. Leukopoiesis intensification and deceleration of leukocyte lyses
D. Leukopoiesis intensification
E. Deceleration of leukocyte migration to the tissues

48
Correct answers
1-A 2-A 3-A 4-A 5-A 6-A 7-A 8-E 9-A 10-A 11-D 12-B
13-E 14-C 15-A 16-A 17-D 18-C 19-A 20-A 21-C 22-C 23-A 24-A
25-A 26-A 27-A 28-A 29-B 30-D 31-A 32-A 33-D 34-D 35-E 36-A

Situational task:

1. There are a necrotic tonsillitis, fever at a patient. It is known from anamnesis,


that a patient constantly takes amidopyrine in case of headache. Leukocytes -
1,5х109/l. Leukogram: eosinophiles - 4%, metamyelocytes - 0%, stab
neutrophiles - 2%, segmented neutrophiles - 33%, lymphocytes - 50%,
monocytes - 11%.
1) Explain the dicreasing of granulocytes amount.
____________________________________________________________________
____________________________________________________________________
____________________________________________________________________
2) What is the pathology at the patient?
__________________________________________________________________
3) What are the reasons of this pathology development?
____________________________________________________________________
____________________________________________________________________
____________________________________________________________________
4) What are possible consequences?
____________________________________________________________________
____________________________________________________________________

2. Patient suffers from ascaridiasis. Leukocytes - 15,0х109/l.


1) What type of leukocytes will arise in this case?
__________________________________________________________________
2) Explain the mechanisms of increasing of the leukocytes number at this
pathology.
____________________________________________________________________
____________________________________________________________________
____________________________________________________________________
3) Write a leukogram for this pathology.
____________________________________________________________________
____________________________________________________________________
3. The long-term using of cytostatics caused the necrotic tonsillitis development
at a patient.
1) How will change quantitative composition of leukocytes?
__________________________________________________________________
2) Write the leukogram.
____________________________________________________________________
____________________________________________________________________
_________________________________________________________________
49
3) Explain the mechanism of necrotic tonsillitis development.
____________________________________________________________________
____________________________________________________________________

4. Patient was hospitalized in a surgical clinic with the frequent exacerbation of


chronic osteomyelitis of right crus. An operation on the staggered bone was
conducted twice. Remission came only after massive antibacterial therapy. In
the blood: leukocytes - 12,0х109/l, neutrophiles prevaile (segmented, stab,
metamyelocytes in trace).
1) What is the connection between the changes of leukocytes and features of
clinical course?
____________________________________________________________________
____________________________________________________________________
2) What properties of leukocytes should be learned additionally for establishment
of the frequent exacerbation of disease reasons?
__________________________________________________________________

5. Group the next pathological states (peritonitis, hay fever, benzol poisoning,
heart attack, acute radiation disease, ascaridiasis) in 3 groups (for 2 names) by
the same types of leukogram changes and similar mechanisms of development.
Write down results in the following sequence:
1) Variety of leukogram changes (their name).
____________________________________________________________________
____________________________________________________________________
____________________________________________________________________
2) Mechanisms of development
____________________________________________________________________
____________________________________________________________________
____________________________________________________________________

6. Femur phlegmon was diagnosed at a patient. Though operation, the common


state remains grave condition: febrile chill, temperature of body - 39-40ºС, daily
fluctuation of temperature - 3-5ºС, tachicardia, shortness of breathing. Blood
test: haemoglobin - 83 g/l, red corpuscles - 3,0х1012/l, leukocytes - 80х109/l, GV -
0,83, thrombocytes - 200х109/l, ESR - 50 mm/h, basophiles - 0%, eosinophiles -
0%, promyelocytes - 3%, myelocytes - 12%, stab neutrophiles - 33%, segmented
neutrophiles - 40%, lymphocytes - 11%, monocytes - 1%.
1) Explain the changes of blood test.
__________________________________________________________________
2) Reasons, mechanisms of development.
____________________________________________________________________
____________________________________________________________________
____________________________________________________________________
3) Explain the changes of leukogram.
__________________________________________________________________
50
4) What blood pathology these changes of blood have to be differentiated with?
__________________________________________________________________

7.
1) Name the characteristic changes of leukocytes at askaridiasis and acute
purulent appendicitis.
____________________________________________________________________
____________________________________________________________________
____________________________________________________________________
2) Describe the mechanisms of their development.
____________________________________________________________________
____________________________________________________________________
____________________________________________________________________
3) Write the leukogram at these pathological processes.
____________________________________________________________________
____________________________________________________________________
____________________________________________________________________

8. There are followings changes of leukogram: myelocytes - 4%, metamyelocytes


- 16%, stab neutrophiles - 28%, segmented neutrophiles - 36%, lymphocytes -
8%, monocytes - 8%.
1) What kind of the nuclear shift is observed at this leukogram.
__________________________________________________________________
2) What reasons can cause similar changes from the side of blood?
____________________________________________________________________
____________________________________________________________________
____________________________________________________________________
3) What kind of nuclear shifts do you known? Characterize each of them.
____________________________________________________________________
____________________________________________________________________
____________________________________________________________________

51
Practical work: “PATHOLOGY OF WHITE BLOOD.
(Leukocytosis. Leukopenia.)”

Object of work: to show the changes of leucocytes amount at a rabbit ander the
action of chemical factors on the organism.

Description of experiment: leucocytosis at a rabbit is caused by intraperitoneal


introduction of 5 ml of the boiled milk mixed with 5 ml of physiological solution.
Leucopenia is caused by daily subcutaneous introduction of 1 ml of benzol
during 8-10 days.

EXPERIMENT № 1. Count up the initial number of leucocytes at a rabbit. Blood is


taken from the regional vein of ear.
For the calculation of amount of leucocytes take blood in a leucocytal
melangeur to the mark 0,5 (or 1), and then add a 3-5% acetic acid solution painted by
1% solution of methylene-blue to the mark ІІ (thus, blood is diluted in 10 or 20
times).
Thus an acetic acid dissolves erythrocytes fully but doesn’t damage leucocytes.
Shake a mixer for the formation of intimate dredge of leucocytes. Calculation is made
in the chamber of Goryaev. Grind the cover glass. Delete 2 drops of mixture from a
melangeur. Approach the last one to the border of cover glass. The small drop of
liquid passes under the glass. Make calculation by 5-10 min (it is necessary to wait
of leucocytes sedimentation) under the low magnification.
Make a calculation of leucocytes in a chamber on an area of 25 large squares
that corresponds to 400 small squares. A calculation is made on the following
formula:

a x 4000 x b
X=
c

where:
Х- amount of leucocytes in 1 mm3,
a- amount of leucocytes in 400 small squares,
b- dilution of blood,
c- amount of the counted small squares.

For example: in 400 small squares 30 leucocytes are counted at dilution in 20


times. Consequently:
Х = 30  4000  20 = 6000 in 1 мм3, or to increase the amount of the calculated
400
leucocytes on “200”, then Х=6,0х109/л.

52
Enter to the rabbit intraperitoneal 5 ml of the boiled milk at dilution by
physiological solution 1:1. In an hour take blood and again count up the amount of
leucocytes.
.
Amount of leucocytes___________________________

EXPERIMENT № 2. Count up the number of leucocytes at a rabbit after


introduction of benzol.
The count of leucocytes and calculation make on the method and formula
indicated higher.

Amount of leucocytes___________________________

Conclusions__________________________________________________________
____________________________________________________________________
____________________________________________________________________
____________________________________________________________________
____________________________________________________________________
____________________________________________________________________
____________________________________________________________________
____________________________________________________________________

53
THEME: PATHOLOGY OF WHITE BLOOD.
(LEUKEMIA. LEUKEMOID REACTION)

Actuality of theme.
Tumours of the hemopoietic system – hemoblastosis – take a separate place
among the diseases of tumour nature. Leukemia and hematosarcoma belong to
hemoblastosis. Leukemias - are the tumours with the primary defeat of bone marrow.
Hematosarcomas - are the tumours of hemopoietic cells with extramedullar
localization. A blood test is one of basic methods of leukemia diagnostic. Last years
the human organism is more added influencing of chemical, physical and biological
mutagens..
A study of blood picture at the separate types of leukemia is an important step
for early diagnostics of blood diseases, and it is the criterion of leukemia treatment
efficiency estimation also.

General purpose of the lesson. To study etiology, mechanisms of


development and changes of morphological blood composition at the different forms
of leukemia and leukemoid reactions.

For this it is necessary to know (the concrete purposes):


1. To select principal reasons of leukemia and leukemoid reactions origin.
2. To explain the mechanisms of their development.
3. To determine the type of leukemia according to the morphological picture of
blood.
4. To count a leukogram at leukemia, to explain its changes.
5. To define leukemia from leukemoid reactions.

For realization of purposes of lesson it is necessary to have the base


knowledges-skills:
1. Common amount of leukocytes, leukogram. (Normal Physiology Department)
2. Functions of leukocytes. (Normal Physiology Department)
3. Morphological features of different forms of leukocytes (Histology
Department)

The checking of primary level of knowledges.


Give the answers to the following questions:
1. Leukemia, definition.
2. Types of leukemia according to the morphological picture of blood.
3. Types of leukemia according to the type of the damaged sprout of hemopoiesis.
4. Forms of leukemia according to the amount of leukocytes in the peripheral
blood.
5. What is the leukemic form of leukemia?
6. What is the subleukemic form of leukemia?
7. What is the aleukemic form of leukemia?
8. What is the leukopenic form of leukemia?
54
9. What is leukemic gap?
10. What is blast crisis?
11. What is the leukemoid reaction?
12. Types of leukemoid reactions.
13. What is the tumour progression?

Standards of answers:
1. It is the systemic disease of blood of tumour origin with the primary damage of
bone marrow, which is characterized by hyperplasia, metaplasia and anaplasia
of hemopoietic tissue..
2. А) Acute
B) Chronic
3. A) Lymphatic leukemia.
B) Myeloleukemia.
C) Erythremia.
D) Monocytic leukemia.
E) Megakaryocytic leukemia.
F) Undifferentiated leukemia
4. A) Leukemic.
B) Subleukemic.
C) Aleukemic.
D) Leukopenic.
5. Common amount of leukocytes in a peripheral blood more than 50х109/l.
6. Common amount of leukocytes in a peripheral blood from 15 to 50х109/l..
7. Common amount of leukocytes in a peripheral blood in the limits of norm: 4-
9х109/l or a bit higher than norm
8. Common amount of leukocytes in a peripheral blood less than 4х109/l.
9. It is the haematological symptom which is characterized by the presence of
blast and mature cells in default of transitional forms of leukocytes/
10. It is the haematological simptom, which is characterized by disappearance of
transitional forms of leukocytes which were characteristic for a chronic
leukemia, and presence of blast forms of leukocytes in the bone marrow and
peripheral blood.
11. It is the reaction that looks like a leukemia by the picture of blood, but doesn’t
have the tumour origin and never grows into the tumour that it looks like.
12. A) Myeloid (neutrophilic, basophilic, eosinophilic type)
B) Lymphoid
C) Monocytic
D) Mixed
13. It is the process of acquisition by the tumour of greater malignancy.

Theoretical questions for the FMC № 2.


1. Leukemia, definition. Classification of leukemia.
2. Reasons and mechanisms of leukemia development.
3. Morphological picture of blood at acute and chronic leukemia.
55
4. Conception about tumour progression. Main points.
5. Features of leukemia at children.
6. Leukemoid reaction, definition. Reasons, mechanisms of their development.
Kinds. Difference from leukemia.
7. Erythremia, definition. Reasons of origin. Mechanisms of main manifestations
development. Picture of blood. Difference from erythrocytosis.
Literature.
1. Handbook of general and Clinical Pathophysiology/ Edited by
prof.A.V.Kubyshkin, CSMU, 2005.p.209-212
2. Pathophysiology/ Edited by prof.Zaporozan, OSMU, 2005.p.187-191
3. General and clinical pathophysiology/ Edited by Anatoliy V/ Kubyshkin –
Vinnytsia: Nova Knuha Publishers – 2011. p.421-444

Testing according system “Krok-I”


Tests of an open database (2010)

1. A 21-year-old man complains of fatigue and fever up to 38-40°C. Liver and


spleen are enlarged, Hb – 100 g/l, erythrocytes – 2.9 x 1012/l, leukocytes – 4.4 x
109/l, platelets – 48 x 109/l, polymorphonuclear neutrophils – 17%, lymphocytes
– 15%, blast cells – 68%. All cytochemical tests are negative. Provide
hematological conclusion.
A. Undifferentiated leucosis
B. Chronic myeloleukemia
C. Acute myeloblastic leucosis
D. Acute lymphoblastic leucosis
E. Acute erythromyelosis

2. A blood test was taken in a patient with leukaemia. What is the most typical
for acute myeloblastic leukaemia?
A. Leukemic gap
B. Leucocytosis
C. Appearance of myeloblasts in the blood
D. Anaemia
E. Leukocytes degeneration

(2009 – 2004)

3. What blood pathology is the presence of Philadelphia chromosomes in the


blood cells and bone marrow cells typical for?
A. Acute myelogenous leukemia
B. Chronic myelogenous leukemia
C. Hodgkin’s disease
D. Burkitt’s lymphoma
E. Chronic lymphocyte leukemia

56
4. In the patient’s blood analysis the number of leukocytes is 250*10 9/L. What
syndrome does this patient have?
A. Leukemia
B. Leucocytosis
C. Leucopoenia
D. Leucomoid reaction
E. Hyperleucocytosis.

5. Patient M, aged 20 was admitted to the hospital complaining of high


temperature, pain in the bones, and hemorrhage from his gums. Blood analysis
of this patient shows: erythrocytes-2.5x1012/L; Hb-80g/L; leucocytes-2.0x109/L;
thrombocytes-6.0x109/L; differential count: eosinophils-1%; stab neutrophils-
1%; segmented neutrophils-10%; lymphocytes-10%; monocytes-3%; blast cells-
75%. What pathology is this blood analysis typical for?
A. Hodgkin’s disease
B. Burkitt’s lymphoma
C. Acute leukemia
D. Infections mononucleosis
E. Chronic leukemia
6. General amount of leucocytes is 90x109/l. In differential count: eosinophils-
1%; basophils-0%; juvenile neutrophils-0%; stab neutrophils-2%; segmented
neutrophils-20%; prolymphocytes-2%; lymphocytes-70%; Botkin-Gumprecht
cells. Cervical, submandibular lymph nodes are enlarged. What pathology is
such blood picture typical for?
A. Acute lymphoblastic leukemia
B. Hodgkin’s disease
C. Infectious mononucleosis
D. Chronic lymphocytic leukemia
E. Chronic myelogenous leukemia
7. Hemiparesis appeared in a patient with acute promyelocytic leukemia. What
is the main mechanism of the impairment of CNS in this case?
A. Intoxication by leukemic cells decay products;
B. Formation of leukemic infiltrates;
C.Impairment of desintoxicative function of the liver;
D. Cachexia;
E. Increase of thrombogenesis.
8. A patient with leukemia has general number of leukocytes of 120.0x10 9/L.
What kind of leukemia does this patient have?
A. Leukemic
B. Leucopenic
C. Subleukemic
D. Aleukemic
E. Erythremia

57
9. Patient with chronic leukemia has acutely increased temperature,
breathlessness, marked muscular weakness at insignificant physical exertion,
increased sweating, cough. What mechanism of leukemia influence upon
organism underlies complications in this patient?
A. Immunodeficiency due to functional inability of leukocytes
B. Internal bleeding because metastases into vessel wall
C. Anemia
D. Tumor progression
E. Airway obstruction because of development of metastases

10. Blood examination revealed leukocytosis, lymphocytosis, Botkin —Gumpreht


shades, and anemia. What disease is recognized by these findings?
A. Acute myeloleukemia.
B. Chronic lymphoid leukosis.
C. Myelosis.
D. Infectious mononucleosis.
E. Acute leukosis.

11. The test findings of the peripheric blood of a 42-year-old patient are:
hemoglobin — 80.0 g/1, erythrocytes - 3.2 -10 12 /l, leukocytes - 25 -109/l, the
leukocytic formula: basophils — 5 %, eosinophils — 9 %, myeloblasts — 3 %,
promyelocytes — 8 %; neutrophils: myelocytes — 11 %, metamyelocytes — 22
%, stab neutrophile — 17 %, segmentonuclear — 19 %, lymphocytes — 3 %,
monocytes — 3 %. What kind of blood pathology does the patient have?
A. Eritromielosis.
B. Myeloblastsc leukosis.
C. Chronic myeliod leukemia.
D. Promielocytic leukosis.
E. Panmyelophthisis.

12. A patient with acute myeloblast leukosis has developed liver and spleen
enlargement, anemia, myeloblasts in peripheral blood. What principal sign
allows to differ myeloblast leukosis from chronic one?
A. Anemia
B. Pancytopenia
C. Leukemic collapse
D. Leukemic cells in peripheral blood
E. Thrombocytopenia

13. Extraction of a tooth, in a patient with chronic lymphocytic leukemia, was


complicated by prolonged bleeding. What may cause the hemorrhagic syndrome
in this patient?
A. Anemia
B. Lymphocytosis
C. Eosinopoenia
58
D. Neutropoenia
E. Thrombocytopoenia

14. A patient with acute myeloblast leukosis has developed liver and spleen
enlargement, anemia, myeloblasts in peripheral blood. What principal sign
allows to differ myeloblast leukosis from chronic one?
A. Anemia
B. Pancytopenia
C. Leukemic collapse
D. Leukemic cells in peripheral blood
E. Thrombocytopenia
15. Following changes were discovered in peripheral blood of patient:
erhytrocytes - 3,0x1012/l Hb-80g/l, leukocytes - 1,0x109/l. Leukocyte formula:
basophyles - 0%, eosinophyles - 0%, myeloblasts - 64%, promyelocytes - 0%;
myelocytes - 0%, metamyelocytes - 0%, stab - 1%, segmentonuclear - 8%,
lymphocytes - 24%, monocytes- 3%. Make the diagnosis due to blood picture.
A. Chronic myeloid leukemia.
B. Erythremia.
C. Neutrophyle leukocytosis.
D. Acute myeloid leukemia
E. Lymphocytosis
16. Following changes were discovered in peripheral blood of patient:
erhytrocytes- 3,2x1012/l, Hb – 80g/l, leukocytes. - 25x109/l. Leukocyte formula:
basophyles - 5%, eosinophyles - 9%, myeloblasts - 3%, promyelocytes - 8%;
myelocytes - 11%, metamyelocytes - 22%, stab - 17%, segmentonuclear - 19%,
lymphocytes - 3%, monocytes - 3%. Make the diagnosis due to blood picture..
A. Acute myeloid leukemia
B. Erythromyelosis
C. Neutrophyle leukocytosis
D. Eosinophyle leukocytosis
E. Chronic myeloid leukemia
17. Patient marks the promoted fatigueability, general weakness during the last
year. Following changes were discovered in peripheral blood of patient:
erhytrocytes -4.1х1012/l, Hb-119g/l, GV-0.87, leukocytes - 57х109/l, Leukocyte
formula: metamyelocytes -0%, stab -0%, segmentonuclear -9%, eosinophyles -
0%, basophyles -0%, lymphoblastes -2%, prolymphocytes -5%, lymphocytes -
81%, м-3%, platelits-160х109/l, Botkin-Gumpreht bodies in the blood smear.
Make the diagnosis due to blood picture.
A. Chronic lymphoid leukemia.
B. Chronic myeloid leukemia
C. Acute lymphoblast leukemia
D. Acute myeloid leukemia
E. Chronic monoleukemia

59
18. What changes from the side of leukocyte formula can confirm a diagnosisin
the blood at the patient with chronic myeloid leukemia?
A. Leukemic gap
B. Botkin-Gumpreht bodies
C. Hyperregenerative shift of leukocyte formula to the left side
D. Degenerative change of formula to the left
E. Presence of metamyelocytes

19. Following changes were discovered in peripheral blood of patient:


erhytrocytes - 2,8х1012/l, Hb – 80g/l, GV - 0,85, thrombocytes - 160х109/l,
leukocytes - 60х109/l; basophyles - 2%, eosinophyles - 8%, promyelocytes - 5%,
myelocytes - 5%, metamyelocytes - 16%, stab - 20%, segmentonuclear - 34%,
lymphocytes - 7%, monocytes - 3%. Make the diagnosis due to blood picture.
A. Acute myeloid leukemia.
B. Chronic myeloid leukemia
C. Hypopoplastic anaemia.
D. Undifferentiated leukemia.
E. Hemolitic anaemia.

20. At patient with myeloid leukemia neutrophyle leukocytosis is determined in


blood. What variant of nuclear shift of leukocyte formula to the left is most
typical for chronic myeloid leukemia?
A. Hyperregenepative
B. Regenerative
C. Degenerative
D. Regenerative-degenerative
E. -

21. Amount of leucocytes-90х109/l. In leukocyte formula: eosinophyles -1%,


basophyles -0%, metamyelocytes -0%, stab -2%, segmentonuclear -20%,
prolymphocytes -2%, lymphocytes -70%, monocytes -5%, Botkin-Gumpreht
bodies. Lymphatic nodes are ancreased. Make the diagnosis due to blood
picture.
A. Acute lymphoid leukemia.
B. Lymphogranulematosis.
C. Infectious mononucleosis.
D. Chronic lymphoid leukemia
E. Chronic myeloid leukemia.

Correct answers
1-A 2-A 3-B 4-A 5-C 6-D 7-B 8-A 9-A 10-B 11-C
12-C 13-E 14-C 15-D 16-E 17-A 18-C 19-B 20-A 21-D

60
Situate task:

1. Anamnesis of patient: hemorrhage of gums, increase of body temperature,


pain in bones, necrotic tonsillitis. Blood test: RBC - 2,5х1012/l, haemoglobin -
75g/l, leukocytes - 2,0х109/l. Leukogram: eosinophiles - 1%, neutrophiles: stab
neutrophiles - 1%, segmented neutrophiles - 3%, lymphocytes - 3%, monocytes -
1%, blast cells - 91%.
1) What is the pathology at the patient?
__________________________________________________________________
2) What are the reasons for this pathology origin?
____________________________________________________________________
____________________________________________________________________
____________________________________________________________________
3) What are the mechanisms of this pathology development?
____________________________________________________________________
____________________________________________________________________
____________________________________________________________________
4) Explain the decreasing of erythrocytes and thrombocytes number.
____________________________________________________________________
____________________________________________________________________
____________________________________________________________________

2. Patient K., 62 years old, suffers from chronic myeloleukemia during 3 years.
During this time the state of patient was compensated by antileukemic
medicines. There are 80% of blast cells at the last blood smear. Cytoctatic
therapy is noneffective.
1) What is the phenomenon?
__________________________________________________________________
2) Name the main points of this process.
____________________________________________________________________
____________________________________________________________________
____________________________________________________________________

3. Common amount of leukocytes in blood - 8х109/l. Leukogram: basophiles -


0%, eosinophiles - 1%, neutrophile metamyelocytes - 0%, stab neutrophiles -
1%, segmented neutrophiles - 18%, prolymphocytes - 2%, lymphocytes - 74%,
monocytes - 4%. Small lymphocytes prevail. The Botkin-Gumpreht’s bodies
appear in the blood smear. Neck and submaxillary lymphatic nodes are
increased.
1) What is the type of leukemia.
__________________________________________________________________
2) What is Botkin-Gumpreht’s body?
__________________________________________________________________
3) How will reactivity of organism change at such blood test? Explain the
answer.
61
____________________________________________________________________
____________________________________________________________________
4) What is the tumour progression. Name its main points?
____________________________________________________________________
____________________________________________________________________
____________________________________________________________________
____________________________________________________________________
____________________________________________________________________
____________________________________________________________________

4. Patient, 50 years, entered to the clinic with complaints about the promoted
fatigue, insomnia, headache, pain in a heart, itching of the skin, pain in the
bones, hemorrhage of gums. Objectively: cherry-red colour of skin and
mucuses, hyperemia of conjunctiva. Liver and spleen are megascopic. There is
a trophic ulcer of leg. BP - 160/100 mm Hg. Hematocrit - 80/20. Blood test:
haemoglobin - 230g/l, RBC - 8х1012/l, GV - 0,96, leukocytes - 20x109/l,
thrombocytes -600x109/l. In a leukogram formula: basophiles - 0%, eosinophiles
- 3%, metamyelocytes -1%, stab neutrophiles - 6%, segmented neutrophiles -
63%, lymphocytes - 21%, monocytes - 6%, ESR - 2 mm/h.
1) What is the pathology at the patient?
__________________________________________________________________
2) What are the reasons for its development?
____________________________________________________________________
____________________________________________________________________
____________________________________________________________________
3) What are the mechanisms of its development?
____________________________________________________________________
____________________________________________________________________
____________________________________________________________________
4) Explain the mechanism of hypertension for a patient.
____________________________________________________________________
____________________________________________________________________
____________________________________________________________________
5) Explain the picture of blood changes at this pathology.
____________________________________________________________________
____________________________________________________________________
____________________________________________________________________
6) What is the difference between this pathology and erythrocytosis?
____________________________________________________________________
____________________________________________________________________
____________________________________________________________________

62
5. Patient L., 54 years, entered to the clinic with complaints about a weakness,
promoted fatigueability, dizziness, headache, shortness of breathing, periodic
pain in the area of heart, pain in joints, hemorrhages on a skin, nosebleed.
Objectively: a liver and spleen are megascopic, a spleen is dense, sickly at
palpation. Neck and inguinal lymphatic nodes are megascopic. The
temperature of body is promoted. Blood test: haemoglobin - 52g/l, red
corpuscles - 1,6х1012/l, GV - 0,97, leukocytes - 8х109/l, thrombocytes -40х109/l.
Leukogram: basophiles - 13%, eosinophiles - 5%, myeloblasts -14%,
promyelocytes - 10%, myelocytes - 20%, metamyelocytes - 14%, stab
neutrophiles -10%, segmented neutrophiles - 10%, lymphocytes - 2%,
monocytes - 2%.
1) What is the pathology at the patient?
__________________________________________________________________
2) What are the reasons for this pathology origin?
____________________________________________________________________
____________________________________________________________________
____________________________________________________________________
3) What are the mechanisms of this pathology development?
____________________________________________________________________
____________________________________________________________________
____________________________________________________________________
4) Explain the mechanisms of erythrocytes and thrombocytes number changes.
____________________________________________________________________
____________________________________________________________________
____________________________________________________________________

63
Practical work: “MORPHOLOGICAL BLOOD STRUCTURE
AT LEUKEMIC MYELOSIS AND LYMPHADENOSIS”.

Object of work: to study the special features of hemopoiesis at leukemia on the base
of the morphological blood structure. To study the leucocyte formula shifts at
leukemia.

EXPERIMENT № 1. Study the blood smear of the patients with chronic myeloid
leukemia, chronic lymphocytic leukemia, acute leukemia. The smears of the patients
are from the hematologic department of Pirogov memorial regional clinical hospital.
Draw the blood picture at different kind of leukemia.

Acute leukemia Chronic myelocytic Chronic lymphocytic


leukemia leukemia

1_____________________ 1_____________________ 1_____________________


2_____________________ 2_____________________ 2_____________________
3_____________________ 3_____________________ 3_____________________
4_____________________ 4_____________________ 4_____________________
5_____________________ 5_____________________ 5_____________________
6_____________________ 6_____________________ 6_____________________
7_____________________
8_____________________
9_____________________

Conclusions__________________________________________________________
____________________________________________________________________
____________________________________________________________________
____________________________________________________________________
____________________________________________________________________
____________________________________________________________________
____________________________________________________________________

64
EXPERIMENT № 2. Calculation of the leucocyte formula at chronic myeloid and
chronic lymphocytic leukemia.

Chronic myelocytic leukemia.


Neutrophyles

Promyeloblasts

Lymphocytes
Eosinophiles

Myeloblasts

Myelocytes
Basophiles

Monocytes
Segmented
Juvenile

Stab

Chronic myelocytic leukemia.


Neutrophyles
Eosinophiles
Basophiles

Prolymphocytes

Lymphocytes
Segmented

Monocytes
Juvenile

Stab

Conclusions__________________________________________________________
____________________________________________________________________
____________________________________________________________________
____________________________________________________________________
____________________________________________________________________
____________________________________________________________________

65
THEME: HEMORRHAGIC SYNDROME. VIOLATION OF HEMOSTASIS.

Actuality of theme.
It is heavy to name industry of practical medicine, where there is not blood
coagulation disorders. It can be an independent disease and secondary disorders,
promoted by pathology of other functional system (violation of blood coagulation as
pathological syndrome at cardiac diseases, all types of shock, pathology of labor
activity and other). Thus all variety of clinical symptoms of blood coagulation
pathology is predefined development of such pathophysiological syndromes, as
hypocoagulation, hemorrhagic syndrome, hypercoagulation as a thrombosis or
disseminated intravascular coagulation (DIC-syndrome).
Study and knowledge of basic law of hemostasis disorders development is
necessary for their successful prophylaxis and treatment.
Extraordinarily widespread pathology is a hemorrhagic syndrome. It is observed at
many diseases, complicating their motion. Knowledge of basic mechanisms of
hemorrhagic syndrome development is necessary for the doctor of any profession.
General purpose of the lesson - to learn reasons of origin and mechanisms of
basic forms of the system of blood coagulation violations development.

For this it is necessary to know (the concrete purposes):


1. To apply modern achievements of physiology of blood coagulation process for
the analysis of mechanisms of its violations origin and development.
2. To give a concept „hemorrhagic syndrome” and to define his basic varieties.
3. To explain etiology and pathogenesis of thrombocytopathy and vasopathy,
which lie in basis of mechanisms of hemorrhagic syndrome development.
4. To explain the basic manifestations of hemorrhagic syndrome
5. To define the reasons and pathogenesis of haemophilia.
6. To explain directions of pathogenetic treatment of hemorrhagic syndrome
different forms.

For realization of purposes of lesson it is necessary to have the base


knowledges-skills:
1. System of blood coagulation. Modern pictures of structurally functional
organization of coagulative, anticoagulative and fibrinolitic systems
(department of physiology).
2. Description of plasma factors of blood coagulation (department of physiology).
3. Role of thrombocytes in blood coagulation (department of normal physiology).
4. Value of tissues components in blood coagulation (department of physiology).

The checking of primary level of knowledges.


Give the answers to the following questions:
1. Name the plasma factors of blood coagulation.
2. What cells of blood contains the factors of blood coagulation?
3. To expose the value of tissues and cellular factors of blood coagulation.
4. What is the coagulative system of blood consist of?
66
5. Role of liver is in support of hemostasis.
6. Role of endothelium in mechanisms of hemostasis.

Standards of answers at the theoretical questions of initial level of


knowledges:
1. Plasma factors of coagulation of blood:
f.І - fibrinogen
f.ІІ - prothrombin
f.ІV - ions of calcium
f.V - proaccelerin
f.VІІ - proconvertin
f.VІІІ - antihemophilic globulin (AHG)
f.ІХ – Christmas’s factor
f.Х – Stuart-Prayer’s factor
f.ХІ - plasma thromboplastin antecedent
f.ХІІ - Hageman's factor
f.ХІІІ - fibrinostabilize factor.
2. Thrombocytes, red corpuscles, endotheliocytes, basophiles.
3. Tissues factors of blood coagulation:
 components of kallikrein-kinin enzyme system (f.ХІV - plasma prekallikrein,
f.ХV - high-molecular kininogen)
 endothelial Vilebrand’s factor, fibrinolysis activators and inhibitor,
prostacyclin (inhibitor of thrombocytes agregation)
 subendothelial structures (e.g., collagen) which activate a factor ХІІ and
adhesion of thrombocytes.
Cellular factors are the group of thrombocytes fators:
 factor 3 - phospholipid factor
 factor 4 - albuminous antiheparin factor
 tromboksan А2
 erythrocytic analog of factor of 3 thrombocytes - erythroplastin, erythtrocytin.
4. Anticoagulative system of blood consist of:
І. Primary anticoagulants, which are constantly synthesized in an organism and
always contained in blood plasma. Such as:
1) antitrombin ІІІ - universal anticoagulant, is proteases inhibitor, synthesized by
endothelium of vessels; represses activity of all blood proteolytic enzymes
(thrombin, kallikrein, plasmin, f.ХІІа, f.ХІа, f.Ха, f.ІХа, f.VIIа);
2) heparin (antithrombin ІІ) - releases by tissues and blood basophiles;
anticoagulative characteristics has in a complex with antithrombin ІІІ;
3) α1- antitrypsin, α2- macroglobulin, inhibitor of С1-component of complement
– are unspecific protease and factors of blood coagulation inhibitors.
ІІ. Secondary anticoagulants, which are not contained in blood plasma in a norm,
but appear in the process of blood coagulation and fibrinolysis. Such as:
1) antithrombin II-fibrin - inaktivates thrombin;

67
2) products of fibrinolysis - prevents of fibrin polymerization and formation of
fibrin structures.
5. There is a synthesis of prothrombin, factors V, VІІІ, ІХ, Х, fibrinogen in a liver,
and also, reabsorbtion of vitamin K, that is needed for coagulation.
6. The vascular wall takes part in realization of vascular-thrombocytic hemostasis.
The damage of vascular wall results the activation of hemostasis mechanisms:
contact activating of thrombocytes and Hageman's factor (f.ХІІ); release of ADP
from damaged endoteliocytes, that is the strong activator of adgesion and agregation
of thrombocytes; release of tissue thromboplastin (f.ІІІ) that causes the trombin
formation directly at the place of vessel damage; release of Vilebrand’s factor, that
takes part in adgesion of thrombocytes.
Also there is a decline of vascular wall thromboresistence as a result of vascular
wall damage: diminishing of prostacyclin formation (ingibitor of thrombocytes
agregation); diminishing of antithrombin ІІІ secretion (natural anticoagulant);
diminishing of endothelium ability to fix the complex heparin-antithrombin ІІІ on
the its surface; violation of endothelium ability to release activators of fibrinolysis.

Theoretical questions at the base of which the execution of purpose types of


activity is possible.
1. Classification of hemostasis violations by etiology and pathogenesis.
2. Determination of conception „hemorrhagic syndrome”.
3. Reasons of origin and mechanisms of hemorrhagic syndrome development.
4. Etiology and pathogenesis of haemophilia.
5. Hemorrhagic syndrome that is conditioned by vasopathy.
6. Thrombocytopenia, thrombocytopathy - as base of hemorrhagic syndrome
development.
7. Etiology and pathogenesis of DIC- of syndrome.
8. Basic manifestations of hemorrhagic syndrome, mechanisms of their
development.
9. Directions of hemorrhagic syndrome pathogenetic treatment.

Literature.
1. Handbook of general and Clinical Pathophysiology/ Edited by
prof.A.V.Kubyshkin, CSMU, 2005.p.213-220
2. Pathophysiology/ Edited by prof.Zaporozan, OSMU, 2005.p.192-197
3. General and clinical pathophysiology/ Edited by Anatoliy V/ Kubyshkin –
Vinnytsia: Nova Knuha Publishers – 2011. p.444-459

68
Testing according system “Krok-I”
Tests of an open database (2010)

1. A patient with burn disease developed disseminated intravascular coagulation


syndrome (DIC). Time of blood coagulation is less than 3 min. What stage of
DIC might be diagnosed?
A. Hypercoagulation
B. Transitional
C. Hypocoagulation
D. Fibrinolytic
E. Terminal

2. A Chornobyl liquidator developed hemorrhagic syndrome as a sign of acute


radiation sickness. What is the most significant in pathogenesis of the
syndrome?
A. Thrombocytopenia
B. Impairment of the structure of vessels wall
C. Increasing activity of fibrinolytic factors
D. Increasing activity of blood anticoagulation factors
E. Decreasing activity of blood coagulation factors

3. Haemophilia was diagnosed in a child with hemorrhagic syndrome. It is


caused by deficit of what factor?
A. IX (Kristmass)
B. II (prothrombin)
C. VIII (antihaemophilic globulin)
D. XI (prothromboplastin)
E. XII (Hageman's)

4. A hereditary type of coagulopathy with defect of VIII factor of blood


coagulability was diagnosed in a patient. Name the phase of blood coagulability
in which the primary disorders of coagulation take place.
A. Thromboplastin formation
B. Thrombin formation
C. Fibrin formation
D. Clot retraction
E. Clot sedimentation

(2009 – 2004)

1. A patient was admitted to the hospital with abundant hemorrhoid bleeding.


This patient has been suffering from hepatic cirrhosis for a long time. What is
the reason for hemorrhage development under hepatic cirrhosis?
A. Activation of fibrinolysis
B. Plasmin deficiency
69
C. *Prothrombin deficiency
D. Low concentration of thrombostenin in blood
E. Excess of heparin

2. A patient was ill with burn disease that was complicated by DIC syndrome.
What stage of DIC syndrome can be suspected if it is known that the patient΄s
blood coagulates in less that 3 minutes?
A. *Hypercoagulation
B. Hypocoagulation
C. Terminal
D. Fibrinolysis
E. Transition phase

3. Hemorrhagic syndrome connected to disorders of the third phase of


coagulation developed in a patient after hi was operated on pancreas. What is
the possible mechanism of development of hemostasis disorder?
A. Elevation in content of heparin in patient’s blood
B. Reduction in fibrinogen synthesis
C. *Activation of fibrinolysis
D. Deficit of fibrin stabilizing factor
E. Reduction in prothrombin synthesis

4. Shonlein-Henoch disease was diagnosed in a patient. What changes in blood


cells number are characteristic for this disease?
A. Eosinophilia
B. *Thrombocytopenia
C. Erythropoenia
D. Polycytemia
E. Eosinopoenia

5. Patient has hemorrhage from gums, subcutaneous hemorrhages, and frequent


nasal bleedings. Thrombocytopenia was revealed in blood test of this patient.
What is the reason for bleeding development in case of thrombocytopenia?
A. Thrombin deficiency
B. *Reduction of thromboplastin formation
C. Excessive heparin formation
D. Activation of fibrinolysis
E. Excess of prostacyclins

6. A boy has congenital disorder of hemostasis: he has prolonged hemorrhage


even in case of insignificant injuries, subcutaneous bruises, and bleedings in
joint cavities, which restrict the movement activity. Patient’s blood does not
coagulate for a long time if it is taken out from the organism and it does not
contain coagulation factor VIII. Blood cells count including number of platelets
is within norm. What underlies the congenital disease of this boy?
70
A. *Hereditary gene defect linked with X chromosome
B. Toxicosis of pregnancy in boy’s mother
C. Intrauterine infection
D. Intrauterine intoxication
E. Intrauterine immune conflict

7. Hemophilia B was diagnosed in a child who has hemorrhagic syndrome. This


type of hemophilia results from absence of:
A. *Coagulation factor IX (Christmas’s factor)
B. Coagulation factor II (prothrombin)
C. Coagulation factor VIII (antihemophilic globulin)
D. Coagulation factor XI (thromboplastin)
E. Coagulation factor XII (Hageman’s factor)
8. Hemorrhagic syndrome connected to disorders of the third phase of
coagulation developed in a patient after hi was operated on pancreas. What is
the possible mechanism of development of hemostasis disorder?
A. Elevation in content of heparin in patient’s blood
B. Reduction in fibrinogen synthesis
C. *Activation of fibrinolysis
D. Deficit of fibrin stabilizing factor
E. Reduction in prothrombin synthesis
9. A female patient, aged 25, was admitted to the hematological department with
complains of the appearance of hemorrhages of different sizes on the body;
during menstruation there are uterine bleedings. She has been ill for ten years.
Paleness of skin and mucous membranes were determined on examination;
there are hemorrhages of different size and color on the upper and lower
extremities. Pulse = 100 beats/minute, AP 110/70 mmHg. Blood analysis shows:
erythrocytes 3.3*1012 /L, Hb 80g/L, thrombocytes 33*109/L; time of blood
coagulation: beginning is at 2nd minute, end is at 6th minute; time of bleeding
(according to Duke) - 15 minutes. What is the possible diagnosis?
A. Marchiafava-Michelli disease.
B. *Thrombocytopoenic purpura.
C. Glanzman’s thrombasthenia.
D. Willeberandt-Yurgens thrombocytopathia
E. Chronic myelogenous leukemia.
10. Such appearances as petechiae and ecchymoses develop in a boy aged 7, who
often fall ill with acute respiratory diseases. Pathology of internal organs is
absent in this patient. What pathology is present in this case?
A. Hypoplastic anemia
B. *Thrombocytopenia
C. Acute leukemia
D. Hemophilia
E. Chronic leukemia

71
11. A 30-years-old female patient, who is suffering from megrim, often takes
analgin. Hemorrhages on skin and frequent nasal bleedings appear in her at the
recent time. At her blood analysis: number of platelets is 30x10 9/L; bleeding
time is increased. What do these changes result from?
A. *Autoimmune thrombocytopenia
B. Hemorrhagic vasculitis
C. Hemolytic anemia
D. Angiohemophilia
E. Thrombocytopathy

12. Antihemophilic globulin A (factor VIII) is absent in the blood plasma of a


boy with significant hemorrhagic syndrome. What phase of hemostasis is
infringed primarily in this boy?
A. Retraction of blood clot
B. Conversion of fibrinogen to fibrin
C. Conversion of prothrombin to thrombin
D. Extrinsic pathway of prothrombinase (thrombokinase) activation
E. *Intrinsic pathway of prothrombinase (thrombokinase) activation

13. A 12-years-old patient was admitted to the hospital with hemarthrosis of


knee joint (hemorrhage into joint cavity). This patient has been suffering from
hemorrhages since early childhood. What disease does this boy suffer from?
A. *Hemophilia
B. Hemorrhagic vasculitis
C. Iron deficiency anemia
D. Vitamin B12 deficiency anemia
E. Thrombocytic purpura

14. A worker at pharmaceutical plant addresses the doctor with complaints of


general malaise, significant hemorrhages from gums, nasal bleedings, and
numerous subcutaneous hemorrhages. At blood analysis of this patient following
was revealed: number of erythrocytes is 2.2x10 12, content of hemoglobin is 48
g/L, presence of neutropenia with relative lymphocytosis, and number of
platelets is 35x109/L. What is the possible pathogenesis of thrombocytopenia in
this patient?
A. *Decrease of platelet production
B. Enhanced platelet destruction
C. Increase platelet utilization
D. Redistribution of platelets
E. Increased loss of platelets

15. Extraction of a tooth, in a patient with chronic lymphocytic leukemia, was


complicated by prolonged bleeding. What may cause the hemorrhagic syndrome
in this patient?
A. Anemia
72
B. Lymphocytosis
C. Eosinopoenia
D. Neutropoenia
E. *Thrombocytopoenia

16. Changes of some indices of blood were revealed at examination of a patient


suffered from hemophilia. What of enumerated indices corresponds to this
condition?
A. Thrombocytopenia
B. Bleeding time by Duke takes longer
C. Eosinophilia
D. *Time of coagulation takes longer
E. Afibrinogenemia

17. A patient with liver disease revealed the decreasing of prothrombin level in
the blood. It can, first of all, result in the impairment of:
A. The first phase of the coagulatory hemostasis
B. Fibrinolysis
C. Vascular-thrombocytic hemostasis
D. *The second phase of the coagulatory hemostasis
E. Anticoagulative properties of the blood

18. A patient suffers from the haemorrhagic syndrome that shows itself in
frequent nasal bleedings, posttraumatic and spontaneous intracutaneous and
intra-articular haemorrhages. After a laboratory study a patient was diagnosed
with the type B haemophilia. This disease is provoked by the deficit of the
following factor of blood coagulation:
A. IX
B. VII
C. VIII
D. XI
E. V

19. A patient underwent a surgery for excision of a cyst on pancreas. After this
he developed haemorrhagic syndrome with apparent disorder of blood
coagulation.
Development of this complication can be explained by:
A. Reduced number of thrombocytes
B. Activation of Christmas factor
C. Insufficient fibrin production
D .Activation of fibrinolytic system
E. Activation of anticoagulation system

73
Situate task:

1. Point hemorrhages on a body and hemorrhage of gums appeared at a patient


with a leukemia,.
1) What is the mechanism of this syndrome development?
____________________________________________________________________
___________________________________________________________________
3) What changes in the blood picture will be in this case?
____________________________________________________________________
____________________________________________________________________
____________________________________________________________________
4) What treatment can you recommend to this patient?
____________________________________________________________________
____________________________________________________________________
____________________________________________________________________

2. The signs of hemorrhagic syndrome appeared at a patient with the cirrhosis


of liver.
1) What is the mechanism of this syndrome development?
____________________________________________________________________
____________________________________________________________________
____________________________________________________________________

3. Petechias (point hemorrhages) appeared on the skin at a patient with a


mechanical joundise.
1) What link of the hemostasis was violate?
__________________________________________________________________
3) What biochemical blood test must be done, to define this link.
__________________________________________________________________

4. Patient appealed with complaints about widespread point hemorrhages. It is


known that a few blood transfusions were conducted to him.
1) What mechanism of this hemorrhages development?
____________________________________________________________________
____________________________________________________________________
____________________________________________________________________

5. A patient is hospitalized concerning rheumatism. He used nonsteroid


antiinflammatory drug - aspirin in large doses. Extraction of tooth for a patient
was complicated by the bleeding.
1) Explain pathogenesis of hemorrhagic syndrome for a patient.
____________________________________________________________________
____________________________________________________________________
____________________________________________________________________

74
6. A patient suffers from chronic lymfocytic leukemia. During an inspection:
erythrocytes - 2,7х1012/l, haemoglobin - 60 g/l, leukocytes - 23х109/l,
thrombocytes -60х109/l.
1) What reasons can result the violation of blood coagulation?
____________________________________________________________________
____________________________________________________________________
____________________________________________________________________
3) What are the mechanisms of their development?
____________________________________________________________________
____________________________________________________________________
____________________________________________________________________

7. A boy, 5 years, suffers from haemophilia A. The parents of patient are


healthy.
1) What is in the base of this disease?
__________________________________________________________________
2) What is the pathogenesis of hemorrhagic syndrome at haemophilia A?
____________________________________________________________________
____________________________________________________________________
____________________________________________________________________
4) Can the daughters of these parents be ill at haemophilia A?
____________________________________________________________________
____________________________________________________________________
8. Liver cirrhosis is diagnosed at a patient.
1) Explain the development of hemorrhagic syndrome for such patient.
____________________________________________________________________
____________________________________________________________________
____________________________________________________________________

9. A patient suffers from a mechanical joindice.


1) What is the mechanism of hemorrhagic syndrome development at this
pathology?
____________________________________________________________________
____________________________________________________________________
____________________________________________________________________

10. Feelings of fear, pallor of skin, sticky sweat, pain in the epigastria appeared
at a patient after blood transfusion. Objectively: arterial pressure is 70/40 mm
Hg, heart beating - 120/min., pulse of the weak filling, breathing frequency -
34/min. Posttransfusion shock was diagnosed.
1) Name the reasons of this pathology.
____________________________________________________________________
____________________________________________________________________
____________________________________________________________________
_________________________________________________________________
75
2) Expose the pathogenesis of microcirculation disorders at a patient.
____________________________________________________________________
____________________________________________________________________
____________________________________________________________________

11. A patient with the crush syndrome and the clinical picture of acute kidney
insufficiency (anuria, azotemia, intoxication) is hospitalized to the
reanimatological department.
1) What violation of blood coagulation developed at a patient (describe by all
known classifications)?
____________________________________________________________________
____________________________________________________________________
____________________________________________________________________
2) Explain its starting mechanism.
____________________________________________________________________
____________________________________________________________________
____________________________________________________________________

Practical work: “HEMORRAGIC SYNDROME”

Object of work: to show the changes of blood at a hemorragic syndrome


(blood coagulation, amount of thrombocytes).

EXPERIMENT № 1. Determination of hemopexis (clotting) time at a rabbit


with anaemia caused preliminary injection of lead acetate subcutaneosly in the dose
of 0,08-0,17 ml per 1 kg of body weight for 3 days.
Wipe the ear of rabbit by xylol. Puncture of ear regional vein by a needle.
Remove the first 2 drops. Fill a mixer by the blood to the mark 0,5 and blow out the
blood into the glass slide.
Mark this moment on watches as the start of researches. Each two minutes set
the thin tip of the glass stick to the blood drop, do a few spiral movement from a
center to periphery of drop and take out the glass stick from the drop. Each time wash
the glass stick and wipe dry. Consider the moment when the filament of fibrin will be
carried after the tip of glass stick as the beginning of hemopexis. Mark in how many
minutes hemopexis will be finished.
In a norm hemopexis by this method began in 4-5 minutes after taking of blood
drop.
Beginning of hemopexis ______________.
End of hemopexis ______________.
Time of hemopexis_______________.

EXPERIMENT № 2. Determination of thrombocytes amount.


Puncture of ear regional vein by a needle. Remove the first 2 drops. Apply a
drop of 14% solution of sulphuric-sour magnesia into the place of puncture. Mix up
76
blood with magnesia solution in a drop by the paraffined stick and prepare the blood
smear. Dry out it on air, fix by Nikiforov’s mixture and paint by the Romanovskyy’s
method (40 mins). Count 1000 red corpuscles and all plates which met during this
count.
The count is conducted as follows: for example, on 1000 red corpuscles 64
plates are found; number of red corpuscles on the count in a chamber 3500000 in 1
мм3, then
64:3500000 = Х:1000
Х = 64  3500000 = 224000 thrombocytes, or 224х109/l
1000
Number of Thrombocytes____________________________.

Conclusion
______________________________________________________________
____________________________________________________________________
____________________________________________________________________

TASK 2. Analyse some indexes of thromboelastogramme (ТЕG) at rabbit with


the increased and decreased hemopexis.
METHOD. Next constants are measured and calculated at TEG (rate of tape
motion 10 mm/mines)^
1. “R”- constant of thromboplastin, which characterizes time is necessity for
formation of active thromboplastin (1 phase of hemopexis) – is measured from
the beginning of direct ribbon to its dilatation on 2 mm with an addition of the
time marked from the beginning of filling of cuvette to his establishment in a
vehicle. Reductions of “R” are related to hyperthromboplastinemia,
elongation - hypothromboplastinemia.
2. “К”- matches to the 2nd and the beginning of the 3rd phases of hemopexis (to
appearance of filaments of fibrin and thrombin). It is measured from the end of
“R” and to dilatation of curve on 20 mm. “К” is elongated at the insufficiency of
fibrin and thrombin and it is reduced at their surplus.
3. “Ма” (mm)- maximal amplitude of curve, matches to the ending of productive
phase of hemopexis and it characterizes ІІІ phase of hemopexis. Retraction of
clot begins after it. In norm “Ma” is 40-55 mm. Low “Ма” is observed at
diminishing of fibrinogen and thrombocytes or their low functional activity.
4. “Е” (%) is maximal elasticity of clot, which characterizes functional activity of
thrombocytes, amount and quality of fibrinogen.
Е = 100  Ма % (in norm Е = 80-100%)
100  Ма
5. Index of TEG = R x K (mins) / Ma (mm) (in norm it is 0,88-1,28)

77
Compare TEG in a norm and at pathology.

Conclusions__________________________________________________________
____________________________________________________________________
____________________________________________________________________
____________________________________________________________________
____________________________________________________________________

78
Theme: ANALYSIS OF GEMOGRAM.

Actuality of theme.
The system of blood is one of the most dynamic systems of organism.
A study of picture of blood at separate diseases is an important step for early
diagnostics of diseases, and also is the diagnostic criterion of estimation of the state
of the immune system at the different diseases of internal organs.

General purpose of the lesson. To learn reasons of origin and mechanisms of


blood indexes changes at different pathological .

For this it is necessary to know (the concrete purposes):


1. To select the basic changes of blood indexes, explain the mechanisms of their
development.
2. To define pathology of blood by the morphological picture of blood.
For realization of purposes of lesson it is necessary to have the base
knowledges-skills:
1. To know the indexes of blood in a norm. (department of normal physiology)
2. Scheme of hemopoiesis. (department of histology)

Theoretical questions at the base of which the execution of purpose types of


activity is possible.
1. To describe the basic indexes of blood.
2. Indexes of blood at the children of different age.
3. Changes of indexes of blood at different pathological processes, value of these
changes for a practical doctor.
4. Analysis of hemograms at different pathological processes.

Literature.
1. Handbook of general and Clinical Pathophysiology/ Edited by
prof.A.V.Kubyshkin, CSMU, 2005.p.192-220
2. Pathophysiology/ Edited by prof.Zaporozan, OSMU, 2005.p.167-197
3. General and clinical pathophysiology/ Edited by Anatoliy V/ Kubyshkin –
Vinnytsia: Nova Knuha Publishers – 2011. p.371-459

79
HEMOGRAM:

№ 1.
Name. Malinovskyy S.
Haemoglobin 98 g/l
RBC 4,3х1012/l
CI 0,7
Leukocytes 10,6х109/l
Eosinophiles 19%
Basophiles 1%
Neutrophiles
Stab 1%
Segmented 48%
Lymphocytes 18%
Monocytes 13%
ESR 10 mm/h
____________________________________________________________________
____________________________________________________________________
____________________________________________________________________
________________________________________________________________

№2
Name Ivanov F.
Haemoglobin 58 g/l
RBC 2,5х1012/l
CI 0,7
Leukocytes 350х10 9 /l
Eosinophiles 1%
Neutrophiles
Stab 1%
Segmented 7%
Lymphoblasts 7%
Prolymphocytes 3%
Large lymphocytes 12%
Meddle lymphocytes 8%
Small lymphocytes 60%
Monocytes 1%
ESR 20 mm/h

____________________________________________________________________
____________________________________________________________________
____________________________________________________________________
________________________________________________________________

80
№3
Name Sergeev I.
Haemoglobin 110 g/l
RBC 3,5х1012/l
CI 0,9
Leukocytes 150х109/l
Eosinophiles 2%
Basophiles 3%
Myeloblasts 10%
Promyelocytes 10%
Myelocytes 15%
Neutrophiles
Metamyelocytes 15%
Stab 20%
Segmented 24%
Monocytes 1%
ESR 25 mm/h
____________________________________________________________________
____________________________________________________________________
____________________________________________________________________
________________________________________________________________

№4
Name Kats А.
Haemoglobin 88 g/l
RBC 3,7х1012/l
CI 0,7
Leukocytes 9,5х109/l
Eosinophiles 11%
Basophiles 0,5%
Neutrophiles
Stab 2,5%
Segmented 52%
Lymphocytes 30%
Monocytes 4%
ESR 12 mm/h

____________________________________________________________________
____________________________________________________________________
____________________________________________________________________
________________________________________________________________

81
№5
Name Petrov О
Haemoglobin 82 g/l
RBC 3,9х1012 /l
CI 0,6
Leukocytes 26,8х109/l
Eosinophiles -
Basophiles -
Myelocytes 2%
Neutrophiles
Metamyelocytes 26%
Stab 19%
Segmented 39%
Lymphocytes 8%
Monocytes 6%
ESR 32 mm/h

____________________________________________________________________
____________________________________________________________________
____________________________________________________________________
________________________________________________________________

№6
Name Yaremchuk G
Haemoglobin 58 g/l
RBC 1,4х1012/l
CI 1,2
Leukocytes 3,7х109/l
Eosinophiles 4%
Neutrophiles:
Stab 2%
Segmented 57%
Lymphocytes 31%
Monocytes 6%
ESR 22 mm/h
Morphology of blood
Megaloblasts 2 in visual field
Megalocytes 8 in visual field

____________________________________________________________________
____________________________________________________________________
____________________________________________________________________
________________________________________________________________

82
№7
Name Sydoruk I.
Haemoglobin 86 g/l
RBC 4,1х1012/l
CI 0,8
Leukocytes 26,5х109/l
Eosinophiles 2%
Basophiles 1%
Neutrophiles
Metamyelocytes 2,5%
Stab 18,5%
Segmented 31,5%
Lymphocytes 26,5%
Monocytes 18%
ESR 11 mm/h

____________________________________________________________________
____________________________________________________________________
____________________________________________________________________
________________________________________________________________

№8
Name Kavun O.
Haemoglobin 100 g/l
RBC 3,5х1012/l
CI 0,9
Leukocytes 35х109/l
Eosinophiles 2%
Basophiles -
Neutrophiles
Metamyelocytes 1%
Stab 2%
Segmented 29%
Lymphocytes 52%
Monocytes 14%
ESR 32 mm/h

____________________________________________________________________
____________________________________________________________________
____________________________________________________________________
________________________________________________________________

83
№9
Name Smirnov A.
Age 8 years
Haemoglobin 100 g/l
RBC 2,5х1012/l
CI 0,96
Leukocytes 280х10 9 /l
Neutrophiles
Metamyelocytes -
Stab 1%
Segmented 5%
Lymphoblasts 91%
Lymphocytes 3%
ESR 60 mm/h
Complaints on pain in a throat at swallowing. Fever, bleeding of
gums.

____________________________________________________________________
____________________________________________________________________
____________________________________________________________________
________________________________________________________________

№ 10
Name Petrov S.
Age 6 months
Haemoglobin 80 g/l
RBC 3,0х1012/l
CI 0,8
Leukocytes 7,2х109/l
Eosinophiles 2%
Neutrophiles:
Segmented 30%
Lymphocytes 65%
Monocytes 3%
ESR 10 mm/h

____________________________________________________________________
____________________________________________________________________
____________________________________________________________________
________________________________________________________________

84
№ 11
1. Time of bleeding by Duke - 3 minutes
2. Time of blood coagulation - 6 minutes
3. Konchalovsky’s syndrom - positive
4. Amount of Thrombocytes - 250х109 /l

____________________________________________________________________
____________________________________________________________________
____________________________________________________________________
________________________________________________________________

№ 12
1. Time of bleeding by Duke - 8 minutes
2. Time of blood coagulation - 6 minutes
3. Konchalovsky’s syndrome - positive
4. Amount of Thrombocytes - 30х109/l

____________________________________________________________________
____________________________________________________________________
____________________________________________________________________
________________________________________________________________

№ 13
1. Time of bleeding by Duke - 3 minutes
2. Time of blood coagulation -1 hour
3. Konchalovsky’s syndrom - negative
4. Amount of Thrombocytes - 200х109 /l

____________________________________________________________________
____________________________________________________________________
____________________________________________________________________
________________________________________________________________

Situational tasks (name the changes from the side of red and white blood, make
the conclusion according to this changes):

№ 1.
Patient S., age 7. Entered after 5 days of illness with
complaints of expressed malaise, headache, pain at swallow,
vomiting. Objective: body temperature is 39.0 °С. At the view
of the throat - a bright redness of the soft palate, enlarged
tonsils. Supracervical lymph nodes are enlarged and painful on
palpation. On the skin there is bright- rose point rash. There is
pallor of the nasolabial triangle (Filatov's symptom)
85
HEMOGRAME
Erythrocytes 4,7 x1012/l Thrombocytec 220 x109/l
Hemoglobine 156 g/l Hematocrite 0,42
CI ? Serum iron 14,0 mcmol/l
Reticulocytes 3‰ ESR 3 mm/h
9
Leukocytes 15,0 x10 /l
BLOOD SMEAR (Leukograme)
B E Neutrophiles L M
M Y Stab Segmented
0 9 0 0 8 68 10 5
Anisocytosis Poikilocytosis Anisochromia
NOTE. Toxic granulosity of neutrophiles. Knyazkov-Dele
bodies
____________________________________________________________________
____________________________________________________________________
____________________________________________________________________
________________________________________________________________

№ 2.
Patient P., 29 years, had wound of the arm with damage of the
artery and 15% of blood lost (800ml)
HEMOGRAME 1
Erythrocytes 4,5x1012/l Thrombocytec 180x109/l
Hemoglobine 130g/l Hematocrite 0,44
CI ? Serum iron 25,8 mcmol/l
Reticulocytes 3‰ ESR 3mm/h
9
Leukocytes 6,0x10 /l
BLOOD SMEAR (Leukograme) 1
B E Neutrophiles L M
M Y Stab Segmented
0 2 0 0 4 54 32 8
Anisocytosis+ Poikilocytosis+ Anisochromia +
HEMOGRAME 2
Erythrocytes 2.0x1012/l Thrombocytec 250x109/l
Hemoglobine 60g/l Hematocrite 0,20
CI ? Serum iron 13,0 mcmol/l
Reticulocytes 8‰ ESR 15mm/h
9
Leukocytes 12,0 x10 /l
BLOOD SMEAR (Leukograme) 2
B E Neutrophiles L M
M Y Stab Segmented
0 1 0 0 5 64 24 6
Anisocytosis Poikilocytosis++ Anisochromia ++
++
86
HEMOGRAME 3
Erythrocytes 3,8x1012/l Thrombocytec 450x109/l
Hemoglobine 100g/l Hematocrite 0,30
CI ? Serum iron 11,7mcmol/l
Reticulocytes 60‰ ESR 20mm/h
9
Leukocytes 18x10 /l
BLOOD SMEAR (Leukograme) 3
B E Neutrophiles L M
M Y Stab Segmented
0 1 0 9 20 60 8 2
Anisocytosis Poikilocytosis Anisochromia
NOTE. Single polychromatophiles, normocytes
____________________________________________________________________
____________________________________________________________________
____________________________________________________________________

№ 3.
Patient К., 55 years, entered into the clinic with complaints on
general weakness, heart beating, dyspnea, pain in the tongue.
Periodically he has filling of feeble feed, numbness of the
limbs. Last years he has dyspepsia. Objectively: average
fatness, edema of the face. Skin, mucosa and sclera are
yellowness, t - 37,5° C. There is inflammation of the tongue,
atrophy of the lingual papillas, aphtae of the mucosa. Decrease
of the stomach secretion, evacuation, atrophy of the stomach
mucosa, flattening of the folds were founded.
HEMOGRAME
Erythrocytes l,8x1012/l Thrombocytec ІЗОx109/l
Hemoglobine 80g/l Hematocrite 0,25
CI ? Serum iron 11,9 mcmol/l
Reticulocytes 1‰ ESR 30mm/h
9
Leukocytes 3,0x10 /l
BLOOD SMEAR (Leukograme)
B E Neutrophiles L M
M Y Stab Segmented
0 0 0 0 1 40 49 10
Anisocytosis Poikilocytosis+++ Anisochromia +++
+++ hyperchromia
NOTE. Big number of megalocytes, erythrocytes with Cabot's
ring and Jolly's body, neutrophils hypersegmented nucleus.
____________________________________________________________________
____________________________________________________________________
____________________________________________________________________
____________________________________________________________________

87
№ 4.
Patient A., 20 years, admitted to the clinic with complaints of
fatigue, j ; decrease of workability. Objective: patient of
normal fatness. Skin and visible mucous membranes
are pale, easy yellowness of the skin and sclera. Splenomegaly.
HEMOGRAME
Erythrocytes 3,8 x1012/l Thrombocytec 193 x109/l
Hemoglobine 90 g/l Hematocrite 0,44
CI ? Serum iron 44,2 mcmol/l
Reticulocytes 5‰ ESR 5 mm/h
9
Leukocytes 8,0 x10 /l
BLOOD SMEAR (Leukograme)
B E Neutrophiles L M
M Y Stab Segmented
0 2 0 0 5 68 20 5
Anisocytosis Poikilocytosis+++ Anisochromia +++
+++ Target cells
NOTE. Big number of the erythrocytes with basophilic
punctation
____________________________________________________________________
____________________________________________________________________
№ 5.
Patient A., aged 12, was admitted to the clinic with complaints
of increasing hoarseness, barking cough. Objective: body
temperature is 38.0 °С. On examination the throat tonsils are
increased, edematous, mucosa with the presence of thick dirty-
white incrustation, cover to the soft palate. Regional lymph
nodes significantly increased.
HEMOGRAME
Erythrocytes 4,3 x1012/l Thrombocytec 220 x109/l
Hemoglobine 143 g/l Hematocrite 0,42
CI ? Serum iron 14,0 mcmol/l
Reticulocytes 3‰ ESR 3 mm/h
9
Leukocytes 16,0 x10 /l
BLOOD SMEAR (Leukograme)
B E Neutrophiles L M
M Y Stab Segmented
0 0 0 3 6 55 19 7
Anisocytosis Poikilocytosis Anisochromia
NOTE. Toxic granulosity of neutrophiles. Plasmocytes are in
the peripheral blood
____________________________________________________________________
____________________________________________________________________
____________________________________________________________________

88
№ 6.
Patient S., aged 6, became ill with an acute rise of the
temperature. Objective: The temperature - 38.1 °С. On the skin
of the whole body there are papulovesicle rash that occurred
almost simultaneously with the rise of temperature.
HEMOGRAME
Erythrocytes 4,7 x1012/l Thrombocytec 220x109/l
Hemoglobine 157g/l Hematocrite 0,42
CI ? Serum iron 14,0mcmol/l
Reticulocytes 3‰ ESR 3mm/h
9
Leukocytes 3,6x10 /l
BLOOD SMEAR (Leukograme)
B E Neutrophiles L M
M Y Stab Segmente
d
0 0 0 0 1 41 46 12
Anisocytosis Poikilocytosis Anisochromia
____________________________________________________________________
____________________________________________________________________
№ 7.
Patient В., 40 years, entered into the clinic with complaints on
general weakness, dizziness, often syncope. She has increased
brittleness of the hear and nails, disorder of the taste (eats raw
meat, cereals, chock). There is urinary incontinence at cough
and laugh. Hemorrhoid during 15. Objectively: average
fatness, edema of the face. Skin, mucosa are pale with sallow
complexion. Dryness of skin, fissure of skin, flattening of the
nails. In the oral cavity: inflammation of the tongue, atrophy of
the lingual papillas, aphtae of the mucosa.
HEMOGRAME
Erythrocytes 3,5x1012/l Thrombocytec 175x109/l
Hemoglobine 60g/l Hematocrite 0,40
CI ? Serum iron 7,2 mcmol/l
Reticulocytes 2‰ ESR 10mm/h
9
Leukocytes 4,0 x10 /l
BLOOD SMEAR (Leukograme)
B E Neutrophiles L M
M Y Stab Segmente
d
0 1 0 0 3 53 37 6
Anisocytosis+ Poikilocytosis++ Anisochromia ++,
+, microcytosis hypochromia
NOTE. Single polychromatophyles, normocytes.

89
____________________________________________________________________
____________________________________________________________________

№ 8.
Patient Т., aged 40, fell ill acute. Felt a chill. The temperature
is 40°C. Notes weakness, sweating, pain in the muscles, severe
headaches, pain in the eyes, lacrimation, photophobia, dry
cough, sore throat, difficulty of nasal breathing.
Objective: hyperemia of the face and neck, injection of scleral
vessels, skin moist, bradycardia, hypotension. Revealed a
diffuse lesion of the upper respiratory tract (rhinitis,
pharyngitis, laryngitis, tracheitis).
HEMOGRAME
Erythrocytes 4,1x1012/l Thrombocytec 220x109/l
Hemoglobine 137g/l Hematocrite 0,42
CI ? Serum iron 14,0 mcmol/l
Reticulocytes 3‰ ESR 12 mm/h
9
Leukocytes 3,5 x10 /l
BLOOD SMEAR (Leukograme)
B E Neutrophiles L M
M Y Stab Segmented
0 0 0 0 12 36 41 11
Anisocytosis Poikilocytosis Anisochromia
NOTE. Toxic granulosity and nuclei vacuolization of
neutrophiles. Plasmocytes are in the peripheral blood
____________________________________________________________________
____________________________________________________________________

№ 9.
Patient S., aged 28, admitted with complaints of poor appetite,
nausea, vomiting, dull pain in the liver, skin itching. Objective:
low fatness. Skin and visible mucous membranes and sclera
are icteric. On palpation the liver is painful, enlarged in size.
The body temperature is 37.3 °С. Biochemical studies have
shown an increase in aspartate aminotransferase and alanine
aminotransferase. Urine is dark. The feces is discolored.
HEMOGRAME
Erythrocytes 4, 5x1012/l Thrombocytec 220x109/l
Hemoglobine 132g/l Hematocrite 0,42
CI ? Serum iron 14,0 mcmol/l
Reticulocytes 3‰ ESR 20mm/h
9
Leukocytes 3,0x10 /l
BLOOD SMEAR (Leukograme)
B E Neutrophiles L M

90
M Y Stab Segmented
0 0 0 0 6 35 46 13
Anisocytosis Poikilocytosis Anisochromia
____________________________________________________________________
____________________________________________________________________

№ 10.
Patient L, 45 years, entered into the clinic with complaints on
hedache, decrease of the memory, disorders of the sleep, pain
in the arms, decrease of the appetite, episodic pain in the
abdomen, combustions. During last 10 years he was working
on the cable production and had the contact with lead. He
ignored the safety engineering. Objectively: decrease fatness.
Skin, mucosa are pale with sallow complexion. Nystagmus.
Tremor of fingers. There is thin lilac band on the gingival
border.
HEMOGRAME
Erythrocytes 2,l x1012/l Thrombocytec 140 x109/l
Hemoglobine 51 g/l Hematocrite 0,38
CI ? Serum iron 91,5 mcmol/l
Reticulocytes 8‰ ESR 10 mm/h
9
Leukocytes 3,8 x10 /l
BLOOD SMEAR (Leukograme)
B E Neutrophiles L M
M Y Stab Segmente
d
0 0 0 0 3 50 40 7
Anisocytosis Poikilocytosis+++ Anisochromia +++
+++
NOTE. Reaction of the urine on the 6-aminolevulinic acid
deeply positive
____________________________________________________________________
____________________________________________________________________
____________________________________________________________________

№ 11.
Patient 0., aged 33, was admitted to hospital in serious
condition. It is known from the medical history about frequent
bleeding from the nose or gums, menorrhagia (heavy
menstruation). Objective: low fatness, pallor of the skin and
mucous membranes, petechial rash, bruisings. Inflammatory
changes of the mouth, rectum. On auscultation of the heart -
functional systolic murmur. Body temperature
40° C. In the right thigh - avast festering bruise.

91
HEMOGRAME
Erythrocytes 0,7 x1012/l Thrombocytec 25,0 x109/l
Hemoglobine 20g/l Hematocrite 0,28
CI ? Serum iron 34,7 mcmol/l
Reticulocytes 0‰ ESR 45mm/h
9
Leukocytes 0,5 x10 /l
BLOOD SMEAR (Leukograme)
B E Neutrophiles L M
M Y Stab Segmented
0 0 0 0 0 18 74 8
Anisocytosis+ Poikilocytosis++ Anisochromia ++
+
____________________________________________________________________
____________________________________________________________________

№ 12.
Patient P., 29 years, had wound of the arm with damage of the
artery and 15% of blood lost (800ml)
HEMOGRAME
Erythrocytes 4,5x1012/l Thrombocytec 180x109/l
Hemoglobine 130g/l Hematocrite 0,44
CI ? Serum iron 25,8 mcmol/l
Reticulocytes 3‰ ESR 3mm/h
9
Leukocytes 6,0x10 /l
BLOOD SMEAR (Leukograme)
B E Neutrophiles L M
M Y Stab Segmented
0 2 0 0 4 54 32 8
Anisocytosis+ Poikilocytosis+ Anisochromia +
____________________________________________________________________
____________________________________________________________________

№ 13.
Patient P., 29 years, had wound of the arm with damage of the
artery and 15% of blood lost (800ml)
HEMOGRAME
Erythrocytes 3,8x1012/l Thrombocytec 450x109/l
Hemoglobine 100g/l Hematocrite 0,30
CI ? Serum iron 11,7 mcmol/l
Reticulocytes 60‰ ESR 20mm/h
9
Leukocytes 18x10 /l
BLOOD SMEAR (Leukograme)
B E Neutrophiles L M
M Y Stab Segmen
92
ted
0 1 0 9 20 60 8 2
Anisocytosis Poikilocytosis+++ Anisochromia +++
+++
____________________________________________________________________
____________________________________________________________________

№ 14.
Patient S., aged 35, the 8* day of illness. The disease begins
gradually. Increasing of weakness, headaches, every day the
temperature rises. Excrements are detained, meteorism.
HEMOGRAME
Erythrocytes 3,6x1012/l Thrombocytec 160x109/l
Hemoglobine 108g/l Hematocrite 0,42
CI ? Serum iron 14 mcmol/l
Reticulocytes 3‰ ESR 22mm/h
9
Leukocytes 3,0x10 /l
BLOOD SMEAR (Leukograme)
B E Neutrophiles L M
M Y Stab Segmented
0 0 0 0 6 42 50 2
Anisocytosis Poikilocytosis Anisochromia
NOTE. Toxic granulosity of neutrophiles
____________________________________________________________________
____________________________________________________________________
№ 15.
A child, 10 years old, was admitted to the clinic with
complains on acute pain of joints, particularly shoulder and
hip. Objective: average fatness, with a curved spine. Skin and
visible mucous yellowness. Splenomegaly. Joints, feet, hands
swollen. On the legs – the ulcers. High myopia.
HEMOGRAME
Erythrocytes 3,0x1012/l Thrombocytec 500x109/l
Hemoglobine 80g/l Hematocrite 0,38
CI ? Serum iron 15 mcmol/l
Reticulocytes 18‰ ESR 5mm/h
9
Leukocytes 15,5x10 /l
BLOOD SMEAR (Leukograme)
B E Neutrophiles L M
M Y Stab Segmented
0 0 0 5 10 62 20 3
Anisocytosis Poikilocytosis+++ Anisochromia
Sickle-cell
NOTE. Decreased osmotic resistance of the erythrocytes
93
____________________________________________________________________
____________________________________________________________________

№ 16.
Patient R., aged 60, was admitted to the clinic with complaints
of malaise, weakness. From history: during the last 5 years
revealed the no treatable anemia. Objective: low fatness,
unsignificant hepato-and splenomegaly. Body temperature is -
38° C. The lymph nodes are palpable painless, enlarged, not
soldered to the surrounding tissues.
HEMOGRAME
Erythrocytes 2,5 x1012/l Thrombocytec 100x109/l
Hemoglobine 72g/l Hematocrite 0,60
CI ? Serum iron 10,5 mcmol/l
Reticulocytes 0‰ ESR 45mm/h
9
Leukocytes 50,0 x10 /l
BLOOD SMEAR (Leukograme)
B E Neutrophiles L M
M Y Stab Segmented
0 0 0 0 0 38 9 53
Anisocytosis Poikilocytosis+++ Anisochromia +++
+++
NOTE. Big amount of the lysozyme in the blood and urine.
____________________________________________________________________
____________________________________________________________________

№ 17.
Patient Y., aged 52, was admitted to the clinic with the
complains of headaches, painful itching of the skin, worsed
after bathing. From history we know that acute burning pain in
the fingertips (rodonalgia) disappeared at 1-2 days after
ingestion of acetylsalicylic acid. Objective: increased fatness.
Skin and visible mucosa hyperemic, injection of the sclera.
Blood pressure - 150/100 mm Hg. Splenomegaly. At
examination of the oral cavity - severe cyanotic soft palate
(Cooperman's symptom).
HEMOGRAME
Erythrocytes 10,2x1012/l Thrombocytec 800x109/l
Hemoglobine 177g/l Hematocrite 0,54
CI ? Serum iron 10 mcmol/l
Reticulocytes 20‰ ESR 1mm/h
9
Leukocytes 20,0x10 /l
BLOOD SMEAR (Leukograme)
B E Neutrophiles L M

94
M Y Stab Segmented
2 3 3 4 5 66 10 6
Anisocytosis Poikilocytosis Hypochromia
+++
NOTE. Big number of the polychromatophiles, normocytes,
single erythroblasts
____________________________________________________________________
____________________________________________________________________

№ 18.
Patient L, 42, was admitted to hospital in serious condition.
Objective: body temperature - 40° C. On the skin there are
multiple rounded papular infiltrates bluish tint to 1 cm in
diameter. At the view of the oral cavity - gingival is
hyperemic, with areas of hemorrhage, hovering over the teeth,
necrotic angina. Hepatomegaly. In the lungs there are scattered
dry rales. Radiologically pneumonia confirmed.
HEMOGRAME
Erythrocytes 1,5 x1012/l Thrombocytec 75x109/l
Hemoglobine 45g/l Hematocrite 0,62
CI ? Serum iron 6,7 mcmol/l
Reticulocytes 0‰ ESR 60mm/h
9
Leukocytes 150x10 /l
BLOOD SMEAR (Leukograme)
B E Neutrophiles L M
M Y Stab Segmented
0 0 0 0 0 14 7 79
Anisocytosis + Poikilocytosis++ Anisochromia ++
NOTE. Monoblasts-59% Cytochemical reactions:
Activity of a-naphthylacetatesterase +++;
Diffuse disposition of glycogen.
Cytogenical: disorders of 8,h and llIh chromosomes. Big amount
of the lysozyme in the blood and urine.
____________________________________________________________________
____________________________________________________________________
№ 19.
Patient К., 31, entered to the hospital with signs of lesions of
the gastrointestinal tract with complaints of abdominal pain,
which gradually localized in the lower abdomen. Pain is worse
before defecation. Concerned about painful tenesmus.
Defecation is speeded (up to 10 times a day). Objective: body
temperature is 37.5 °С. On palpation of the abdomen marked
spasm and pain of large intestine, especially in the sigmoid
colon. ln the faeces there is a large number of bloody mucus.

95
HEMOGRAME
Erythrocytes 3,6x1012/l Thrombocytec 220x109/l
Hemoglobine 84g/l Hematocrite 0,42
CI ? Serum iron 14 mcmol/l
Reticulocytes 3‰ ESR 20mm/h
9
Leukocytes 18,0x10 /l
BLOOD SMEAR (Leukograme)
B E Neutrophiles L M
M Y Stab Segmented
0 0 2 3 4 65 16 10
Anisocytosis Poikilocytosis Anisochromia
NOTE. Toxic granulosity and nuclei vacuolization of
neutrophiles
____________________________________________________________________
____________________________________________________________________

№ 20.
Patient V., 4 years old, was admitted to hospital in serious
condition. Objective: the child is flaccid, uncommunicative.
Skin and visible mucous membranes are pale with multiple
point and spotty hemorrhages. Peripheral lymph nodes are
enlarged. Temperature - 39.0 °С. Hepato-and splenomegaly,
liver is firm, painless. Occasionally vomiting occurs. Kernig's
symptom is positive. Testicular tumor is palpable.
HEMOGRAME
Erythrocytes 2,9 x1012/l Thrombocytec 75x109/l
Hemoglobine 76g/l Hematocrite 0,50
CI ? Serum iron 10,2 mcmol/l
Reticulocytes 0‰ ESR 39mm/h
9
Leukocytes 80,0x10 /l
BLOOD SMEAR (Leukograme)
B E Neutrophiles L M
M Y Stab Segmented
0 0 0 0 0 15 81 4
Anisocytosis+ Poikilocytosis++ Anisochromia ++
+
NOTE. Lymphoblasts - 70%. Botkin- Hympreht shadows.
Cytochemical reactions:
1. Diffuse disposition of glycogen.
2. Reaction on myeloperoxidase - negative.
Cytogenical: disorders of 8lh and 14lh chromosomes.
____________________________________________________________________
____________________________________________________________________

96
№ 21.
Patient M., 28 years, entered into the clinic with complaints on
general weakness, progressive muscle weakness,
uncomfortable filling in the right subcosteal area, periodical
pain in the abdomen. Moderate hypochromic anemia from the
childhood. Objectively: average fatness. Skin, mucosa and
sclera are pale with yellowness. Hepatomegaly, splenomegaly.
HEMOGRAME
Erythrocytes 3,3 x1012/l Thrombocytec 180x109/l
Hemoglobine 50g/l Hematocrite 0,40
CI ? Serum iron 82,7 mcmol/l
Reticulocytes 1‰ ESR 12mm/h
9
Leukocytes 6,0 x10 /l
BLOOD SMEAR (Leukograme)
B E Neutrophiles L M
M Y Stab Segmented
0 1 0 0 5 55 31 8
Anisocytosis Poikilocytosis+++ Anisochromia +++
+++
NOTE. Single polychromatophiles, normocytes. Increased
number of the sideroblasts in the bone marrow.
____________________________________________________________________
____________________________________________________________________

№ 22.
Boy В., 12 years, entered into the clinic with complaints on
pain in the right subcosteal area. From history: the parents
phenotypically are healthy. In the father's blood smears -
microcytosis of red blood cells. Objective: the patient with a
square tower skull, micro ophthalmia. Little fingers are
shortened. Skin and visible mucous yellowness. At the
examination of the mouth - the gothic palate, incorrect position
of teeth. During abdominal ultrasound revealed gallstones,
splenomegaly
HEMOGRAME
Erythrocytes 3,5 x1012/l Thrombocytec 190x109/l
Hemoglobine 100g/l Hematocrite 0,25
CI ? Serum iron 13,5 mcmol/l
Reticulocytes 15‰ ESR 17mm/h
9
Leukocytes 7,0x10 /l
BLOOD SMEAR (Leukograme)
B E Neutrophiles L M
M Y Stab Segmented
0 2 0 0 5 65 24 4

97
Anisocytosis Poikilocytosis+++, Anisochromia ++
+++, spherocytosis
microcytosis
NOTE. Decrease of osmotic resistance of erythrocytes.
____________________________________________________________________
____________________________________________________________________

№ 23.
Patient S., age 62, was admitted to the hospital with complains
of shortness of breathing, coughing, weakness. Objective: low
fatness. Skin and visible mucous membranes are pale. All
groups of lymph nodes are enlarged, stony-density, painful on
palpation, fused with the surrounding tissues. The lungs are
listened by dry rales. On radiographs - specific infiltration of
the pleura with effusion. Temperature - 38.2 °С. Hepato-and
splenomegaly.
HEMOGRAME
Erythrocytes 3,3 x1012/l Thrombocytec 70x109/l
Hemoglobine 85g/l Hematocrite 0,46
CI ? Serum iron 6,6 mcmol/l
Reticulocytes 2‰ ESR 48mm/h
9
Leukocytes 100,0 x10 /l
BLOOD SMEAR (Leukograme)
B E Neutrophiles L M
M Y Stab Segmented
0 0 0 0 0 15 80 5
Anisocytosis Poikilocytosis++ Anisochromia ++
++
NOTE. Lymphoblasts - 7%. Botkin- Hympreht shadows.
Cytochemical reactions:
1. Diffuse disposition of glycogen.
2. Reaction on myeloperoxidase - negative.
Cytogenical: disorders of 12th chromosomes.
____________________________________________________________________
____________________________________________________________________
____________________________________________________________________

98
Practical training from the topics: “Blood system pathology”.

1. Provide analysis:
 Cause-effective relationships (local and general changes, pathological and
adaptive-compensative changes, specific and nonspecific signs; main and
additional links) in pathogenesis of typical violations of blood system
(anaemia, erytrocytosis, leukocytosis, leukopenia, leukemia; violation of
hemostasis);
 Conformities of the law of violations of peripheral blood cell composition at
acute and chronic leukemias.

2. Ability and skills:


 Decision of situation tasks with definition of typical violations in the blood
system (erytrocytosis, anaemia, leukocytosis, leukopenia, leukemia; violation
of the hemostasis), their main varieties (through application of knowledges
about their classifications principles), reasons of origin and mechanisms of
development.
 On the basis of results of laboratory research (blood test) to determine
maintenance of separate types of leucocytes in a blood, to estimate a result.
 To determine maintenance of haemoglobin in a blood (by Sally); to estimate a
result.
 To count the colour index (globular value) of blood, to estimate a result.
 To identify the regenerative, degenerative, pathological forms of «red» and
«white» blood cells in the peripheral blood smears; to interpret their presence
or absence in a blood.

99
THEORETICAL QUESTIONS FOR THE FINAL MODULE CONTROLE 2:
1. Anemia, definition.
2. Classification of anemias according to pathogenesis, color index, type of
hemopoiesis, ability of the bone marrow to regeneration.
3. Signs of regeneration and degeneration of the bone marrow.
4. Erythrocytosis, definition. Kinds. Reasons and mechanisms of development. Picture
of blood. Difference from erythremia.
5. Etiology, pathogenesis of Iron-deficiency anaemia. Picture of blood. Pathogenetic
treatment.
6. Iron-deficiency states at children. Anaemia of new-born. Green sickness.
7. Iron refractory anemia anaemia. Kinds. Reasons, mechanism of development.
Picture of blood. Differences from Iron-deficiency anaemia.
8. В12-deficiency anaemia. Reasons, mechanisms of development for children and
adults. Picture of blood. Pathogenetic treatment.
9. Hemolytic anaemias. Kinds. Reasons, mechanisms of development of the inherited
and aquired hemolytic anaemias.
10. Hemolytic disease of new-born. Reasons, mechanisms of development. Picture of
blood. Pathogenetic treatment.
11. Leukocytosis, definition. Classification of leukocytosis.
12. Reasons and mechanisms of different forms of leukocytosis development.
13. Shifts of leukogram. Types of shift. Value of the shifts determination. Reasons of
different types of shifts development.
14. Qualitative changes of leukocytes, their value in pathology.
15. Leukopenia, definition. Classification. Reasons and mechanisms of development.
Picture of blood at the different types of leukopenia.
16. Value of leukocytosis and leukopenia for an organism.
17. Agranulocytosis, definition. Kinds. Reasons. Mechanisms of development.
18. Leukemia, definition. Classification of leukemia.
19. Reasons and mechanisms of leukemia development.
20. Morphological picture of blood at acute and chronic leukemia.
21. Conception about tumour progression. Main points.
22. Features of leukemia at children.
23. Leukemoid reaction, definition. Reasons, mechanisms of their development. Kinds.
Difference from leukemia.
24. Erythremia, definition. Reasons of origin. Mechanisms of main manifestations
development. Picture of blood. Difference from erythrocytosis.
25. Classification of hemostasis violations by etiology and pathogenesis.
26. Determination of conception „hemorrhagic syndrome”.
27. Reasons of origin and mechanisms of hemorrhagic syndrome development.
28. Etiology and pathogenesis of haemophilia.
29. Hemorrhagic syndrome that is conditioned by vasopathy.
30. Thrombocytopenia, thrombocytopathy - as base of hemorrhagic syndrome
development.
31. Etiology and pathogenesis of DIC- of syndrome.
32. Basic manifestations of hemorrhagic syndrome, mechanisms of their development.
33. Directions of hemorrhagic syndrome pathogenetic treatment.

100
ZAPOROZHYE STATE MEDICAL UNIVERSITY

THE DEPARTMENT OF INTERNAL DISEASES №3

Test “KROK-2” Tasks in

HEMATOLOGY

ZAPORIZHZHIA
2018
UDC: 616.15(075.8) «RATIFIED»
З-41
By Central methodical advice of
Zaporozhye state medical university

Protocol № ________from ________2018

Authors:

Dotsenko S.Ya., Recalov D.G., Danyuk I.A., Samura B.B., Shekhovtseva T.G.,
Tokarenko I.I., Chornaya I.A., Svistun S.I., Shevchenko M. .V., Sklyarova NP

Reviewers:

Kraydashenko A.O. - Professor, Doctor of Medical Sciences, Head of the Department of


Pharmacology, Pharmacy, Pharmacotherapy and Cosmetology of ZSMU

Svintozevsky A.A. - Associate Professor of the Department of Clinical Pharmacology,


Pharmacy, Pharmacotherapy with a course of cosmetology at Zaporizhzhya State Medical
University, Candidate of Medical Sciences,

Test “KROK-2” Tasks in HEMATOLOGY for independent preparation to practical


studies for English-speaking students of medical faculty of 6 course. Practical work is
represented as collection of test tasks and clinical tasks with standarts of true answers in
hematology.

2
INTRODUCTION
The collection of test tasks “Test “KROK-2” Tasks in HEMATOLOGY” is designed to
improve the quality of training of future general practitioners, including general
practitioners / family doctors, since in recent decades there has been a significant spread
and growth of the pathology of internal organs, especially hematological diseases .
In the presented collection of test questions, questions are considered that will be
useful in practical work of therapists and will provide substantial assistance to students,
interns, general practitioners, family doctors in mastering the problems of diagnosis and
differential diagnosis of hematological diseases.
The collection of test tasks is built on the basis of the curriculum for the discipline
"Internal Medicine", contains a sufficient number of test tasks that reveal the general
provisions and a special part of the pathological states in hematology. The workshop
includes test tasks from recent years from the Testing Center at the Ministry of Health of
Ukraine.
The presented collection of test tasks is a rather relevant form of teaching students
in higher educational institutions of III-IV accreditation levels from the standpoint of the
Bologna process, including preparation for the "KROK-2" licensing exam.

3
1. At the patient S., 68 years old, during examination atrophy of papillae of language,
yellowness of white of the eyes, splenomegaly, and symmetrical paresthesia were found
out, at FGDS - atrophical gastritis, at Рh-metry – achlorhydria. In blood test: anemia,
makrocytosis. What researches can confirm the diagnosis:
А. Sternal puncture
В. US investigation of abdominal cavity
С. Consultation of neurologist
D. Determination of iron of blood serum
Е. Spleen puncture

2. At the patient K., 18 years old, abundant bleeding began after extraction of tooth. In
the anamnesis: haemophilia A. What first aid the patient needs:
A. Ascorutin
B. Aminocapron acid
C. Calcii chloridi
D. Vikasol
E. Cryoprecipitate

3. At the sick M., 42 years old, who was taking mercazolile for a long time, concerning
thyreotoxicosis, agranulotcytosis developed. What changes are possible to expect in
leukoformula:
А. Leukocytosis with lymphotcytosis
В. Leukocytosis with neutrophilia
С. Leukocytosis with lymphopenia
D. Leukopenia with neutrophilia
E. Leukopenia with neutropenia

4. The patient L., 23 years old, complains on increase of temperature of body to 38С,
appearance of hypodermic hemorrhages. Doctor diagnosed aplastic anemia. What
symptom from the below will be observed at the sick:
A. Splenomegaly
B. Lymphadenia
C. Leukopenia
D. Hepatomegaly
E. Hyperthrombotcytosis

5. The sick Z., 68 years old, complains on pain in bones, subfebrile temperature of body,
weight loss. At inspection moderate normochromic anemia is determined, blood
sedimentation-55 mm/h, proteinuria-0,99 g/l. What of research methods is less
informing for clarification of diagnosis:
А. Definition of general protein
В. Definition of albuminous fractions
С. Roentgenography of bones
D. Definition of level of iron of blood serum
Е. Sternal puncture

6. At the patient S., 68 years old, during examination yellowness of white of the eyes,
4
hepatosplenomegaly, symmetrical paresthesia, at additional research - atrophical
gastritis with achlorhydria were discovered. What sign contradicts the clinic of the
described condition:
A. Macrocytosis
В. Gunther`s glossitis
C. Thrombocytopenia
D. Microcytosis
Е. Hypersegmentation of nucleus of neutrophiles

7. Which from diseases transferred below such hematological indexes are characteristic
for: expressed anemia, leukopenia, neutropenia, presence of 15% of plasma cells in bone
marrow:
А. Acute leukemia
В. Chronic myeloleukemia
С. Multiple myeloma
D. Chronic lympholeukemia
E. Lymphogranulomatosis

8. At the 23 years old man, who suffers on acute myeloblastic leukemia, massive
hypodermic hemorrhages, nose bleeding appeared. There are expressed anemia,
thrombotcytopenia, and 30% of blasts in blood test. What first aid the patient needs:
А. Extending of polychemotherapy
В. Transfusion of packet red cells
С. Transfusion of thromboconcentrate
D. Introduction of iron preparations
E. Introduction of vikasol

9. The patient S., 68 years old, immediately hospitalized concerning hard anemia (Hb-
50 g/l, macrocytosis) with complaints on dyspnea in rest, disturbance of step. At
examination: yellowness of white of the eyes, tachycardia, hepatosplenomegaly. What
help the patient needs:
A. Preparations of iron intravenous
B. Vitamin В12, packet red cells
C. Prednizolon
D. Anabolic steroids
E. Packet red cells

10. The patient T., 60 years old, complains on dyspnea. During many years he is
suffering on chronic obstructive pulmonary disease. Hepatosplenomegaly. Blood test: Е-
6,8x1012/l, Нb-190 g/l, L-12x109/l, Т-520x109/l, blood sedimentation-2 mm/h. Define the
reliable diagnosis:
A. Chronic obstructive pulmonary disease. Symptomatic erythrotcytosis
В. Pikvik`s syndrome. Symptomatic erythrotcytosis
С. Hypertonic illness. Symptomatic erythrotcytosis
D. Erythremia. Chronic obstructive pulmonary disease
E. Chronic myeloleukemia. Chronic obstructive pulmonary disease

5
11. At examination of the 70 years old patient with fever and dyspnea, pallor of skin,
moist rales in the lower departments of lungs, tachycardia, diastolic noise in the V point,
AP-140/40, increase of spleen are found out. In blood: Е-2,7x1012/l, Нb-75 g/l, L-
4,2x109/l, blood sedimentation-45 mm/h; in urine - moderate proteinuria,
microhematuria. Define the reliable diagnosis:
А. Chronic myeloleukemia
В. Infectious endocarditis
С. Rheumatic heart disease
D. Pneumonia
Е. Acute myocarditis

12. The sick K., 58 years old, marks increased lymphatic knots of neck and subarm- pits.
At examination: size of lymphatic knots 3х4 sm, painless, soft, mobile, skin above them
is not changed. General analysis of blood: Е-3,2x1012/l, Hb-102 g/l, CI-1,0; L- 235x109/l,
е-2%, r/n-4%, s-12%, l-76%, m-6%, Gumprechts` bodies. Define the reliable diagnosis:
А. Chronic lympholeukemia
В. Reactive lymphadenitis
C. Lymphogranulomatosis
D. Malignant lymphoma
Е. Metastasis of cancer in lymphatic knots

13. The 63 years old participant of liquidation of failure on CHAES, complains on


causeless weakness, feeling of holding apart in the left subcostum. The patient feels itself
sick for a year. Objectively: skin is pale, liver + 3 sm, spleen +10 sm. In blood: Е-
3,1x1012/l, Hb-100 g/l, L-200x109/l, e-6%, b-3%, blast-2%, promiel-10%, miel-18%, r/n
-27%, s-10%, l-12%, m-2%, blood sedimentation-40 mm/h. What diagnosis is the most
credible:
А. Hemolytic anemia
В. Cirrhosis of liver
С. Acute leukemia
D. Chronic myeloleukemia
Е. Chronic lympholeukemia

14. The patient L., 30 years old, complains on general weakness, fragility of nails, hair
fall, considerable and prolonged menstruations. Objectively: pallor of skin, heart rate–
90, AP-100/70. Blood test: Е-3,5x1012/l, Нb-90 g/l, CI-0,7; blood sedimentation-20
mm/h. Define the previous diagnosis:
А. Iron deficiency anemia
B. В12 deficiency anemia
С. Aplastic anemia
D. Acute leukemia
Е. Follic deficiency anemia

15. The sick F., 50 years old, complains on itch of skin after aquatic procedures.
Objectively: skin of red color, liver + 2 sm, spleen + 4 sm. Blood test: Е-6,4x1012/l, Нb-
185 g/l, L-10,0x109/l, е-5%, r/n-8%, s/n-56%, l-26%, m-5%, Т-525x109/l, blood
sedimentation-1mm/h, hematcrit-72%. What research must be conducted for clarification
6
of diagnosis:
А. Definition of В12 in blood
В. Sternal puncture
С. Definition of alkaline phosphatase of blood
D. Iron of blood serum
Е. US investigation of abdominal cavity

16. The sick T., 36 years old, appealed to internist with complaints on pain in throat,
increase of temperature of body to 39С. Objectively: skin is pale, single bruises on thighs,
necrotic changes on tonsils, spleen +3 sm. Blood test: Е-1,9x1012/l, Нb-57 g/l, L-
20,0x109/l, blast -26%, s-25%, l-42%, m-7%, Т-32,0x109/l, blood sedimentation-60
mm/h. What research needs be conducted for clarification of diagnosis:
А. Sternal puncture В.
Smear from fauces
С. US investigation of abdominal cavity
D. Spleen puncture
Е. Computed tomography of abdominal cavity

17. The sick L., 68 years old, complains on increased lymphatic knots, perspiration.
Objectively: skin and mucouses are pale, increased inguinal and subaxillary lymphatic
knots, palpated by diameter of 2-3 sm, soft, unpainful, mobile. Sizes of liver by Curlov
are 18x14x13 sm. Blood test: Е-3,5x1012/l, Нb-100 g/l, CI-0,8, L-380x109/l, е-3%, l-
95%, m-2%, Т-190x109/l. Define the previous diagnosis:
А. Chronic lympholeukemia
В. Acute leukemia
С. Chronic myeloleukemia
D. Leukemoid reaction of lymphoid type
Е. Cirrhosis of liver

18. The sick A., 56 years old, appealed to doctor with complaints on perspiration, weight
loss, heavy feeling in the left half of stomach. Skin and mucouses are pale. Large spleen
is palpated and liver is moderately increased. Blood test: Е-3x1012/l, Нb- 90 g/l, L-
240x109/l, eoz-9%, baz-6%, myeloblast-4%, promyel-3%, myel-23%, metamyel-16%,
r/n-15%, s/n-12%, l-7%, m-5%, blood sedimentation-40 mm/h. Define the previous
diagnosis:
A. Chronic lympholeukemia
B. Acute leukemia
C. Chronic myeloleukemia
D. Leukemoid reaction of myeloid type
E. Cirrhosis of liver

19. The patient O., 50 years old, complains on general weakness, obdormition of lower
extremities. Objectively: skin is pale, liver + 2 sm. Blood test: Е-2,3x1012/l, Нb-95 g/l,
CI-1,3, L-2,4x109/l, r/n-2%, s/n-53%, l-40%, m-5%, blood sedimentation-34 mm/h,
macrocytosis, hypersegmentation of neutrophils. What research needs to be conducted
for definition of diagnosis:
А.US investigation of abdominal cavity
7
В. Sternal puncture
С. Definition of В12 in blood
D. Iron of blood serum
Е. Liver puncture

20. The patient G., 58 years old, complains on headache, dizziness. Objectively: skin is
pletoric, heart rate-82, AP-180/90, liver and spleen are moderately increased. Blood
test: Е-8,0x1012/l, Hb-220 g/l, CI-1,0; L-11,5x109/l, Т-560x109/l.
What research must be conducted for diagnostics of the disease:
A. Echocardioscopy
В. Sternal puncture
С. US investigation of abdominal cavity
D. Iron of blood serum
E. Determination of hematocrit

21. The patient, 19 years old, complains on fever, pain in throat, sickly neck lymphatic
knots. At examination: spleen and liver are increased. In general analysis of blood 32%
of mononucleares are found out. What disease you think about:
А. Lymphatic angina
B. Hepatitis
C. Angina
D. Acute leukemia
E. Sepsis

22. The patient T., 45 years old, complains on general weakness, dyspnea at insignificant
physical load, pain in the right subcostum. During 10 years the patient was misusing of
alcohol. Objectively: reduced feed, skin is pale with icteric tint, systolic noise above all
auscultational points, liver + 6 sm, spleen +2 sm. In blood: Е-1,8x1012/l, Hb-75 g/l, CI–
1,3; L-3,5x109/l, е-3%, r/n-4%, s/n-65%, l-21%, m-7%; Т-110x109/l, blood
sedimentation-50 mm/h. What diagnosis is most credible:
A. В12 deficiency anemia
B. Follic deficiency anemia
С. Hypopolastic anemia
D. Cirrhosis of liver
Е. Autoimmune thrombocytopenia

23. The 35 years old woman, who during two years treats oneself at internist concerning
NCD and at gynaecologist concerning menorrhagia, complains on muscular weakness,
decline of memory, subfebrile temperature.General analysis of blood: Е-3,5x1012/l, Hb-
100 g/l, CI-0,7, anisocytosis, L-3,8х109/l, е-2%, r/n–4%, s/n-60%, l–26%, m-8%, blood
sedimentation-12 mm/h, iron of blood serum-7,8 mkm/l. What treatment needs to be
appointed:
А. Preparations of iron per os
В. Follic acid
C. Preparations of iron intravenous
D. Vitamins of group B
E. Transfusion of packet red cells
8
24. The patient G., 58 years old, complains on headache, dizziness. Objectively: skin is
pletoric, tones of heart are rhythmic, heart rate-82, accent of the second tone above aorta,
AP-180/90, spleen is moderately increased. Blood test: E-8,2x1012/l, Нb-210 g/l, CI-1,2,
L-10,5x109/l, Т-560x109/l. What diagnosis is most credible:
А. Erythremia
В. Chronic myeloleukemia
С. Cerebral insult
D. Tumor of brain
Е. Cirrhosis of liver

25. About what pathology it is possible to think at presence at the sick of ecchymoses,
prolonged menstruations, thrombocytopenia in general analysis of blood and raised
percent of megakariocytes in bone marrow aspirates:
A. Aplastic anemia
B. Acute leukemia
C. Systemic lupus erythematosus
D. Idiopathic thrombocytopenic purpura
E. Chronic hepatitis

26. At the formula of blood: Е-1,3x1012/l, Hb-58 g/l, CI-1,3, megaloblast-2 on 100,
reticulotcyt-0,2%, macrocytosis, L-2,8x109/l, е-3%, r/n-5%, s/n-49%, l-37%, m-6%, Т-
100,0x109/l, blood sedimentation-30 mm/h, formulate the previous diagnosis:
A. Iron deficiency anemia
B. В12 deficiency anemia
С. Aplаstic anemia
D. Acute leukemia
E. Agranulocytosis

27. At the formula of blood: Е-3,5x1012/l, Hb-110 g/l, L-330x109/l, baz-5%, eozin-9%,
promyel-2%, myel-22%, metamyel-21%, r/n-15%, s/n-12%, l-8%, m-6%; thromb-
200,0x109/l, blood sedimentation-45 mm/h. Formulate the previous diagnosis:
А. Acute leukemia
В. Chronic lympholeukemia
С. Chronic myeloleukemia
D. Erythremia
Е. Multiple myeloma

28. The 23 years old patient on the background of angina noticed increase of lymphatic
nodes. At examination phenomena of acute angina, magnification of liver are marked.
At research of blood: Е-4,1x1012/l, Hb-130 g/l, thromb-230,0x109/l, L-6,5x109/l, in
leukocyte formula there are 45% of lymphomonocytic cells, blood sedimentation-35
mm/h. What diagnosis is most credible:
А. Lymphatic angina
В. Chronic lympholeukemia
С. Infectious lymphadenitis
D. Acute hepatitis
9
Е. Acute monoblastic leukemia

29. The 23 years old patient on the background of angina noticed increase of lymphatic
nodes. At examination phenomena of acute angina, magnification of liver are marked.
At research of blood: Е-4,1x1012/l, Hb-130 g/l, Т-230,0x109/l, L-6,5x109/l, in leukocyte
formula there are 45% of lymphomonocytic cells, blood sedimentation-35 mm/h. What
additional research must be conducted for confirmation of diagnosis:
А. Sternal puncture
В. Biopsy of lymphatic knot
С. US Investigation of liver and spleen
D. Smear from fauces
Е. Roentgenography of lungs

30. At the formula of blood: Е-2,5x1012/l, Hb-68 g/l, CI-1,5, megaloblast-5 on 100,
reticulotcyt-0,2%, L-2,8x109/l, е-3%, r/n-4%, s/n-45%, l-33%, m-5%, thromb-
105x109/l, blood sedimentation-30 mm/h. Appoint additional research, which must be
conducted for confirmation of diagnosis:
А. Sternal puncture
В. US Investigation of liver and spleen
С. Roentgenography of lungs
D. Definition of maintenance of В12 in blood
Е. Definition of iron of blood serum

31. At the indicated formula of blood: Е-2,8x1012/l, Hb-80 g/l, CI-0,8, reticulocyt-20%,
L-7,5x109/l, е-2%, r/n-4%, s/n-54%, l-37%, m-3%, Т-200x109/l, blood sedimentation-
35 mm/h, microspherocytosis. Appoint additional research, which is necessary for
confirmation of diagnosis:
А. Sternal puncture
В. Definition of maintenance of В12 in blood
C. Definition of maintenance of iron of blood serum
D. US investigation of liver and spleen
E. Definition of osmotic resistance of erythrocytes

32. At the indicated formula of blood: Е-2,8x1012/l, Hb-80 g/l, microspherocytosis, CI-
0,8, reticul-20%, L-7,5x109/l, е-2%, r/n-4%, s/n-54%, l-37%, m-3%, T-200x109/l, blood
sedimentation-35 mm/h. Define the previous diagnosis:
A. В12 deficiency anemia
В. Follic deficiency anemia
С. Inborn hemolytic anemia
D. Iron deficiency anemia
Е. Acute leukemia

33. The sick K., 72 years old, complains on aversion to meat, weight loss on 12 kg
during 6 months. At examination: skin is pale, above the left collar-bone increased, not
mobile, dense lymphatic knot is palpated. Liver is + 4 sm, dense, painful. General
analysis of blood: Е-2,5x1012/l, Hb-78 g/l, L-11,8x109/l, T-460x109/l, blood
sedimentation-55 mm/h. About what disease it is possible to think:
10
А. Acute leukemia.
B. В12 deficiency anemia
С. Follic deficiency anemia
D. Cancer of stomach with metastasis
Е. Multiple myeloma

34. The sick K., 72 years old, complains on aversion to meat, weight loss on 12 kg during
6 months. At examination: skin is pale, icteric, above the left collar-bone increased, not
mobile, dense lymphatic knot is palpated. Liver is + 4 sm, dense, painful.General analysis
of blood: Е-2,5x1012/l, Hb-78 g/l, L-12,8x109/l, Т-460x109/l, blood sedimentation-55
mm/h. What research needs to be conducted for confirmation of diagnosis:
А. Sternal puncture.
В. US investigation of liver
С. Biopsy of lymphatic knot
D. FGDS
E. Ph-metry

35. The patient G., 57 years old, complains on diarrhea, increase of temperature of body
to 37,5С, obdormition and tingling in lower extremities. The resection of stomach
concerning ulcer was done 4 years ago. At examination: skin is pale, icteric, liver +3 sm,
spleen +2 sm. General analysis of blood: Е-2,3x1012/l, Hb-80 g/l, CI-1,2, L- 2,3x109/l,
formula is not changed, Т-140x109/l, blood sedimentation-45 mm/h, macrocytosis.
About what disease it is possible to think:
А. Iron deficiency anemia
В. Cancer of stomach
C. В12 deficiency anemia
D. Inborn hemolytic anemia
Е. Follic deficiency anemia

36. At the young woman after viral infection acute belly – ache, diarrhea with the
admixtures of blood, pain in knee-joints, increase of temperature of body appeared.
Objectively: skin is pale, micropoint eruption on the skin of shins, at palpation thick
intestine is painful. In blood: leukocytosis, increased blood sedimentation; in urine: low
proteinuria, microhematuria. What additional research must be conducted:
A. Proteinogram
B. С-reactive protein
C. LЕ-cells, antibodies to native DNK
D. Villebrandts` factor, circulating immune complexes
Е. US investigation of abdominal cavity

37. The sick F., 62 years old, appealed to doctor with complaints on general weakness,
headache, itch of skin after contact with water. Objectively: skin of face of redder –
bluish color, AP-180/90, spleen + 4 sm. What diagnosis is most credible:
А. Allergic dermatitis
B. Dermatomyositis
C. Erythremia
D. Hypertonic illness
11
Е. Cirrhosis of liver

38. The patient G., 57 years old, complains on periodic diarrhea, increase of temperature
of body to 37,5С, obdormition and tingling in lower extremities. The resection of stomach
concerning ulcer was done 4 years ago At examination: skin is pale, icteric, liver + 3 sm,
spleen + 2 sm. General analysis of blood: Е-2,3x1012/l, Hb-80 g/l, CI-1,3, L-2,3x109/l,
formula is not changed, Т-140x109/l, blood sedimentation-40 mm/h, macrocytosis. What
additional research must be conducted for clarification of diagnosis: А. US investigation
of liver and spleen
В. Sternal puncture
C. FGDS
D. Research of bilirubin of blood
Е. Roentgenoscopy of stomach

39. The 19 years old youth complains on acute pain and slight swelling in right knee-
joint, limitation of motions in it, which arose up after insignificant trauma. In anamnesis
there is hemophilia. At review hemarthrosis of knee-joint is determined. It is necessary
to use in treatment:
A. Cryoprecipitate
В. Donor blood (at direct transfusion)
C. Decinon
D. Donor blood (ampuled)
Е. Thrombocytic mass

40. The 44 years old man is hospitalized to the infectious separation with the diagnosis
of follicular angina. At examination: temperature of body 38,6С, skin is pale, pulse –
112, AP-90/60. In blood test: Е-2,5x1012/l, HB-90 g/l, CI-0,8; L-38,0x109/l, blast-68%,
r/n-2%, s/n-5%, l-23%, m-2%, blood sedimentation-46 mm/h. What primary research
the patient needs:
А. US investigation of abdominal cavity
В. Sternal puncture.
С. Smear from fauces
D. Seeding of blood on sterility
E. Determination of ferritin of blood

41. The 28 year old woman appealed to doctor with complaints on appearance of
ecchymoses after insignificant traumas or spontaneously. At examination: skin is pied
(fresh and old hemorrhage) on the front surface of trunk and extremities. In blood test:
Т-20x109/l, in bone marrow number of megakaryocytes is increased. What is the most
reliable disease:
А. Hemorrhagic vasculitis
В. Randyu-Oslers` disease
C. Hemophilia
D. Idiopathic thrombocytopenic purpura
E. Systemic lupus erythematosus

42. At the 55 years old man, patient on В12 deficiency anemia, symptoms of funicular
12
myelosis gradually appeared. In blood: anemia, leukopenia, thrombocytopenia, increase
of colour index, blood sedimentation-40 mm/h; hyperbilirubinemia due to indirect. What
is the most credible reason of funicular myelosis:
А. Increased level of bilirubin
В. Accumulating of propion and metilmalon acids
С. Long – lasting hypoxia of nervous system
D. Activating of infection as a result of leukopenia
Е. Deficiency of feed

43. The 44 years old man is hospitalized to the infectious separation with the diagnosis
of follicular angina. At examination: temperature of body of 38,6С, skin is pale, pulse
112, AP-100/60. In blood test: Е-2,5x1012/l, Hb-90 g/l, CI-0,8, L-38x109/l, blast-68%,
r/n-2%, s/n-14%, l-14%, m-2%, blood sedimentation-46 mm/h. What disease it follows
to suspect at the patient:
А. Chronic lympholeukemia
В. Chronic myeloleukemia
С. Acute leukemia
D. Lymphatic angina
Е. Leukemoid reaction

44. At the young woman after viral infection acute belly – ache, diarrhea with the
admixtures of blood, pain in knee-joints, increase of temperature of body appeared.
Objectively: skin is pale, micropoint eruption on the skin of shins, at palpation thick
intestine is painful. In blood: leukocytosis, increased blood sedimetation; in urine: low
proteinuria, microhematuria. What diagnosis is the most reliable:
А. Crohn`s disease
В. Hemorrhagic vasculitis
С. Nodular polyarteriitis
D. Unspecific ulcerative colitis
Е. Systemic lupus erythematosus

45. The patient T., 45 years old, complains on general weakness, dyspnea at insignificant
physical load, pain in the right subcostum. During 10 years the patient was
misusing of alcohol. Objectively: reduced feed, skin is pale with icteric tint, systolic noise
above all auscultational points, liver + 6 sm, spleen +2 sm. In blood: Е-1,8x1012/l, Hb-
75 g/l, CI–1,3; L-3,5x109/l, е-3%, r/n-4%, s-65%, l-21%, m-7%; Т-110x109/l, blood
sedimentation-50 mm/h. What laboratory research must be conducted for clarification
of diagnosis:
A. Sternal puncture
В. Definition of concentration of folates in red corpuscles of blood
C. Definition of ferritin of blood
D. Definition of В12 in blood
Е. Definition of iron of blood serum

46. At the patient, 57 years old, after viral infection subfebrile temperature is saved for a
long time, heavy feeling in the left subcostum is marked. At examination: skin is pale,
spleen +6 sm, liver +3 sm. In blood test: E-2,9x1012/l, Hb-90 g/l, CI-1,0; L-540,0x109/l,
13
eozin-4%, baz-3%, blast-34%, myel-2%, metamyel-3%, r/n-5%, s/n-27%, l-18%, m- 4%,
Т-260x109/l, blood sedimentation-37 mm/h. What disease it follows to suspect at the
patient:
A. Acute myeloblastic leukemia
B. Chronic myeloleukemia, blastic crisis
C. Chronic lympholeukemia.
D. Lymphogranulomatosis
E. Leukemoid reaction of myeloid type

47. The patient F., 29 years old, who during two years treated oneself at gynaecologist
concerning menorrhagia, complains on increased fatigability, palpitation at physical
load, unretaining of urine. Has a desire to eat chalk. In blood test: E-3,5x1012/l, Hb-95
g/l, L-3,8х109/l, е-3%, r/n-5%, s/n-56%, l-26%, m-6%, blood sedimentation-24 mm/h,
hypochromia of red corpuscles, anisocytosis, poikilocytosis, iron of blood serum-5,5
mkm/l. What treatment must be appointed:
А. Follic acid
B. Cyancobalamine
С. Preparations of iron per os
D. Ascorbic acid
Е. Transfusion of packet red cells

48. At the patient, who suffers on chronic lympholeukemia, general weakness increased,
yellowness of white of the eyes and skin appeared. At examination: Hb-65 g/l, reticul-
5%, general bilirubin-80,3 mkmol/l, indirect-65,3 mkmol/l. Urobilin is increased in
analysis of urine. The direct Coombs` test is positive. What pathogenetic mechanism is
lying in the root of anemia:
A. Myelofibrosis
B. Oppressing of erythroid link of hemopoiesis
С. Autoimmune hemolysis
D. Deficiency of follic acid
E. Disturbance of porfirin metabolism

49. The pregnant, 18 years old (20 weeks), complains on weakness, dyspnea at physical
load. At women`s dispensary she was not observed before. Objectively: skin is pale with
lemon tint; face is puffy, language - bright red, liver + 3 sm. In blood test: Е- 3,0x1012/l,
Hb-88 g/l, CI-1,3; L-3,8x109/l, Т-130,0x109/l; е-3%; r/n-4%; s/n-52%; l- 36%; m-5%;
macrocytosis, blood sedimentation-28 mm/h. What diagnosis is the most reliable:
A. Iron deficiency anemia
B. В12 deficiency anemia
C. Follic deficiency anemia
D. Acute leukemia
E. Hepatitis

50. The patient D., 50 years old, complains on dizziness, blinking of "spots" before
eyes. At examination: subicteric skin and mucouses, liver +4 sm, dense, painful, spleen
+8 sm, dense. In blood test: Е-2,2x1012/l, Hb-80 g/l, reticul-30%, L-6,0х109/l, е-3%, r/n-
6%, s/n-62%, l-20%, m-8%; blood sedimentation-30 mm/h; osmotic resistance of red
14
corpuscles 0,52-0,56%; bilirubin of blood is moderately increased due to indirect,
reaction on urobilin is Acutely positive, the Coombs` test is positive. What diagnosis is
the most credible:
A. Minkovskiy – Shoffars` disease
B. Agranulocytosis
С. Night hemoglobinuria
D. Autoimmune hemolytic anemia
Е. Gilber`s disease

51. The patient I., 47 years old, is hospitalized in clinic with complaints on expressed
weakness, increase of temperature of body to 39,2°С, pain in throat. At examination: in
blood test there are anemia, thrombocytopenia, leukocytosis with “leukemic failure”,
blast-34%, blood sedimentation-40 mm/h. What research must be conducted for
clarification of diagnosis:
A. Coagulogram
B. Sciagraphy of bones of skull
C. Unfolded blood test
D. Seeding from tonsils
E. Sternal puncture

52. The patient I., 18 years old, was hospitalized with complaints on nose-bleeding,
hemorrhagic rash as petechias on the skin of lower extremities. Vaccination against flu
was done two weeks ago. In blood test: Е-4,0x1012/l, L-6,7x109/l, Т-30,0x109/l, е-2%,
r/n-4%, s/n-54%, l-32%, m-8%, blood sedimentation-2 mm/h. Your previous diagnosis:
A. Hemorrhagic vasculitis
B. Acute leukemia
С. Аutoimmune thrombocytopenia
D. Randyu-Oslers` disease
E. Systemic lupus erythematosus

53. The patient, 68 years old, complains on general weakness, pain in ribs, lumbar area.
At examination: moderate normochromic anemia, general albumen-107 g/l, diurnal
proteinuria-5,0 g/day. On the sciagrams of bones of skull a lot of shallow rounded hearths
of destruction were discovered by diameter from 0,8 to 2,0 sm. What diagnosis can be
suspected at this patient:
А. Multiple myeloma
B. Osteolytic metastases in bones
C. Amyloidosis of kidneys with nephrotic syndrome
D. Hyperparathyroidic osteodystrophy
E. Chronic glomerulonephritis with nephrotic syndrome

54. The patient S., 64 years old, at examination in policlinic increased lymphatic knots
of neck, arm-pits, groin and liver (+3 sm) were found out. In blood test: Е-3,1x1012/l,
Hb-98 g/l, L-500,0x109/l, е-2%, r/n-1%, s/n-13%, l-80%, m-4%, blood sedimentation-
40 mm/h. What reliable diagnosis the patient has:
A. Chronic myeloleukemia
B. Cancer of liver
15
С. Chronic lympholeukemia
D. Tubercular lymphadenitis
E. Lymphogranulomatsis

55. The patient G., 37 years old, during three months marks frequent nose-bleedings
and menorrhagia, appearance of bruises on skin. Three days ago after considerable
nose-bleeding, dizziness, palpitation appeared. At examination: on the skin of front
surface of trunk and feet - plural petechias, single ecchymoses. In blood test: Е-
3,0x1012/l, Hb-100 g/l, CI-0,7; L-5,3x109/l, Т-40,0x109/l, blood sedimentation-19 mm/h.
What diagnosis can be suspected at this patient:
A. Aplastic anemia
B. Hemophilia
C. Hemorrhagic vasculitis
D. Iron deficiency anemia
Е. Autoimmune thrombocytopenic purpura

56. The patient A., 42 years old, complains on dizziness, appearance of bruises on skin,
nose-bleedings, and weight loss. He is ill for 3 months. At examination: reduced feed,
skin is pale with presence on the front surface of hands, feet and trunk of different
remoteness of plural bruises by diameter from 0,2 to 3,0 sm.
What type of hemorrhage takes place at the patient:
A. Angiomatosal
B. Hematomal
C. Mixed
D. Vasculit-purpural
E. Petechia-macular

57. At the 26 years old patient yellowness of skin, dizziness, palpitation, spleen +2,5 sm
determined. Urine is dark, feces of umber color. In blood test there are normochromic
anemia, reticul-4%. Reaction on urobilin is Acutely positive. What research must be
conducted for establishment of diagnosis:
A. Definition of vitamin В12
B. Definition of iron of blood serum
С. Definition of osmotic resistance of erythrocytes
D. Electrophoresis of proteins of blood serum
E. Sternal puncture

58. The patient A., 20 years old, delivered to inpatient facility concerning pit bleeding
after extraction of tooth. In blood test: Е-2,80x1012/l, Hb-80 g/l, L-4,0x109/l, е-2%, r/n-
3%, s/n-62%, l-28%, m-5%; Т-24,0x109/l: blood sedimentation-25 mm/h. What disease
can be assumed at the patient:
A. Autoimmune thrombocytopenic purpura.
B. Acute leukemia
C. Hemophilia B
D. Agranulocytosis
E. Aplastic anemia

16
59. The patient S, 21 years old, after radial irradiation complains on palpitation, dyspnea,
frequent nose-bleedings, bruises on body. The patient marks frequent acute respiratory
diseases. In blood test: Е-2,0x1012/l, Hb-54 g/l, L-1,7x109/l, е-0%, r/n-0%, s/n-32%, l-
62%, m-6%; Т-30,0x109/l, blood sedimentation-52 mm/h. What treatment is most
expedient in this case:
А. Transfusion of thrombocytic mass
В. Bone marrow transplantation
C. Transfusion of packet red cells
D. Transfusion of whole blood
E. Introduction of antilymphocytic immunoglobulin

60. The patient E., 57 years old, complains on pain in lumbar area, bones of pelvis,
increase of temperature to 37,3С, weight loss. At examination: anemia, blood
sedimentation-70 mm/h, general protein-110 g/l, moderate proteinuria, sediment is
without pathology. At roentgenologic inspection there are destructive changes in the
bones of skull. What is the most credible diagnosis:
A. Multiple myeloma
B. Cancer of stomach
C. Bechterew`s disease
D. Metastases of tumor in bones
E. Chronic glomerulonephritis

61. The patient M., 52 years old, appealed to doctor with complaints on itch of skin after
washing, heavy feeling in head, dizziness. Objectively: face, neck, extremities are of
crimson color, AP-180/100, spleen +4 sm. What is the most reliable diagnosis:
A. Allergic dermatitis
B. Hypertonic illness
C. Chronic myeloleukemia
D. Erythremia
E. Cirrhosis of liver

62. The patient C., 60 years old, complains on dyspnea. During many years he is
suffering on chronic obstructive pulmonary disease. Objectively: diffuse cyanosis,
obesity is expressed, AP-180/110, dry rales, liver and spleen are increased. In blood
test: Е-6,6x1012/l, Нb-190 g/l, L-15x109/l, Т-529,0x109/l, blood sedimentation-2 mm/h.
What research can help to set the diagnosis:
A. Research of function of external breathing
B. US investigation of abdominal cavity
C. Sternal puncture
D. Sciagraphy of thorax
E. Puncture of liver

63. The pregnant, 18 years old (20 weeks), complains on weakness, dyspnea at physical
load. At women`s dispensary she was not observed before. Objectively: skin is pale with
lemon tint; face is puffy, language - bright red, liver +3 sm. In blood test: Е- 3,0x1012/l,
Hb-88 g/l, CI-1,4; L-3,8x109/l, Т-130,0x109/l; е-3%; r/n-4%; s/n-52%; l- 36%; m-5%;
macrocytosis, blood sedimentation-38 mm/h. What research can help to set the diagnosis:
17
А. Sternal puncture.
В. Definition of concentration of folates in red corpuscles of blood
C. Definition of ferritin of blood
D. Definition of В12 in blood
Е. Definition of iron of blood serum

64. At the 65 years old man В12 deficiency anemia is diagnosed. In a week after the
appointed treatment control inspection of peripheral blood is conducted. What index
will be the early criterion for estimation of efficiency of the conducted therapy:
A. Increase of amount of reticulotcytes
B. Increase of level of hemoglobin
C. Decline of colour index of blood
D. Normoblastic blood formation
E. Increase of number of leukocytes

65. The patient, 20 years old, passed regular course of polychemotherapy by the
scheme of "VAMP" concerning acute lymphoblastic leukemia. He has complaints on
weakness, hair fall. In blood test: E-3,5x1012/l, Hb-105 g/l, CI-0,9; L-4,2x109/l, Т-
120,0x109/l. What picture of bone marrow can testify about remission:
А. Content of blastic cells to 5 %
B. Content of blastic cells to 15 %
C. Content of blastic cells to 10 %
D. Content of blastic cells to 1 %
E. Absence of blastic cells
66. The patient O., 62 years old, with increased feed, complains on headache, dizziness,
pressing pain in the area of heart at moderate physical load. Objectively: face and hands
with hyperemia, accent of the second tone above aorta, AP-170/104, liver +3 sm, spleen
+2 sm. In blood test: Е-6,2x1012/l, Hb-І86 g/l, L-11,2x109/l, blood sedimentation-1
mm/h. On ECG there is flatten wave T in V1-V4. What previous diagnosis can be put in
this case:
A. Cushing`s disease
B. Hypertonic illness of II stage
C. Erythremia
D. Secondary erythrocytosis on background of obesity
E. Cirrhosis of liver

67. At the 62 years old woman, who used butadion in connection with pain in joints,
pain in throat, dry cough, and febrile temperature appeared. What changes it is possible
to assume in general analysis of blood:
А. Increase of number of mature granulotcytes
В. Decrease of number or absence of granulotcytes
C. Increase of number of ripening granulotcytes
D. Increase of number of lymphocytes
E. Decrease of number or absence of lymphocytes

68. The youth of 18 years old is hospitalized with complaints on nose bleeding, which
doesn`t succeed to be stopped, and hard pain in right elbow joint. He is ill from
18
babyhood. Objectively: elbow joint is increased, perceptible hot, knee-joints are
deformed, motions in them are limited, heart rate-90, AP-105/70. In blood test: Е-
3,2x1012/l, Нb-110 g/l, CI-0,7, L-5,6x109/l, Т-220,0x109/l, blood sedimentation-14
mm/h. What preparation it is necessary to use in treatment:
A. Packet red cells
B. Calcii chloridi
C. Codginate
D. Aminocapron acid
E. Vikasol

69. At the 65 years old woman, who used butadion in connection with pain in joints,
pain in throat, febrile temperature, chill appeared. Doctor suspected agranulocytosis.
Agranulocytosis is:
А. Decrease of maintenance of granulocytes in blood
В. Increase of maintenance of agranulocytes in blood
С. Decrease of number of neutrophilic granules with the simultaneous increase of their
size
D. Loss of granules in granulocytes
Е. Appearance of granules in agranulocytes

70. The 42 years old man complains on palpitation, nose-bleeding. Objectively: on the
skin of extremities and trunk petechia-spotal hemorrhages, lymphatic knots are not
palpated, pulse-116, liver is not increased, spleen is not palpated. In blood analysis there
is pancyitopenia.About what disease it is possible to think:
A. Verlgoph`s disease
B. Acute leukemia
С. Aplastic anemia
D. Hemorrhagic vasculitis
E. Acute agranulocytosis

71. The 63 years old woman is hospitalized in the hematological separation with
complaints on pressing retrosternal pain, dyspnea, paresthesia of feet. Objectively:
temperature of body 37,6°С, yellowness of white of the eyes, liver +2 sm. In blood test:
Е-1,5x1012/l, Hb-70 g/l, CI-1,3, L-2,6x109/l, Т-132,0x109/l, reticul-0,6%, bilirubin is
moderately increased due to indirect. In myelogram: megaloblastic type of blood
formation.What is the most credible diagnosis:
А. Follic deficiency anemia
B. В12 deficiency anemia
C. Inborn hemolytic anemia
D. Gained hemolytic anemia
E. Iron deficiency anemia

72. The sick T., 24 years old, is hospitalized in the hematological separation with
complaints on pain in lumbar area and right subcostum, acute weakness. In the
anamnesis there was flu a week ago. Objectively: skin is pale-icteric, liver +2 sm,
sensible. In blood: Е-2,0x1012/l, Hb-64 g/l, CI-0,9, L-12,0х109/l; reticul-8%, bilirubin-
38 mkml/l, mainly due to indirect, the direct Coombs` test is positive. What is the
19
previous diagnosis:
A. Hypoplastic anemia
B. Markiafav – Mikelli`s disease
C. Inborn hemolytic anemia
D. Adison – Biermer disease
E. Gained immune hemolytic anemia

73. The patient T., 34 years old, complains on chill, osalgia, nose-bleeding. Objectively:
temperature of body-38,6°С, skin is pale, pulse-120, AP-100/70. In blood test: Е-
2,7x1012/l, Hb-90 g/l, CI-0,9; L-38,0x109/l, blast-68%, r/n-2 %, s/n-8%, l-20%, m-2%,
Т-25x109/l, blood sedimentation-46 mm/h. What disease it follows to suspect at the
patient:
А. Acute leukemia.
B. Leukemoid reaction
C. Chronic lympholeukemia
D. Chronic myeloleukemia
E. Acute agranulocytosis

74. The 63 years old man appealed with complaints on acute general weakness, bad
appetite, weight loss, heavy feeling in the left subcostum. In blood test: Е-3,4x1012/l,
Hb-102 g/l, CI-0,9; L-190x109/l, bas-3%, eozin-8%, blast-1%, promyel-2%, myel-2%,
metamyel-13%, youn-12%, r/n-16%, s/n-31%, l-9%, m-9%, Т-240,0x109/l, blood
sedimentation-30 mm/h. What is the previous diagnosis:
A. Leukemoid reaction of myeloid type
B. Acute leukemia
С. Chronic myeloleukemia
D. Erythromyelosis
E. Chronic lympholeukemia

75. The 65 years old man complains on weakness, dyspnea, numbness of lower
extremities. Objectively: skin is pale with subicteric tint, language is bright red, liver +3
sm. At FGDS there is atrophy of mucous membrane. In blood test: E-2,4x1012/l, Hb-66
g/l, CI-1,4, L-2,8x109/l, е-2%, r/n-4%, s/n-50%, l-42%, m-5%, reticul-0,5%, Т-
120,0x109/l, macrocytosis, blood sedimentation-26 mm/h. What disease it follows to
suspect at the patient:
A. Hemolytic anemia
B. Iron deficiency anemia
C. Hypoplastic anemia
D. В12 deficiency anemia
E. Follic deficiency anemia

76. The 35 years old woman, is delivered in clinic after loss of consciousness on the
street, complains on acute weakness, dizziness. Objectively: pallor of skin, there are
hemorrhages on the skin of forearms and thighs, lymphatic knots are not increased, pulse-
100, AP-90/60, liver and spleen are not increased. In blood test: Е-1,5x1012/l, Hb- 42 g/l,
CI-0,8, reticul-0,1%, L-1,0x109/l, е-1%, r/n-1%, s/n-45%, l-51%, m-2%, Т- 50,0x109/l,
blood sedimentation-45 mm/h. What is the most reliable diagnosis:
20
A. Verlgoph`s disease
B. Aplastic anemia
C. Iron deficiency anemia
D. Hemorrhagic vasculitis
E. Posthemorrhagic anemia

77. The 37 years old man complains on weakness, periodic attacks of pain in the right
subcostum, which appeared 2 years ago. In the anamnesis: from 16 years periodically
icteric colour of skin is marked. Objectively: skin and mucouses are icteric,
hepatosplenomegaly. In blood test: Е-2,4x1012/l, H-84 g/l, CI-1,0, reticul-4%, blood
sedimentation-22 mm/h, osmotic resistance of erythrocytes is reduced,
microspherocytosis, indirect bilirubin-56 mkmol/l, direct-8,2 mkmol/l.What
pathogenesis of anemia the patient has:
А. Genetic defect of membrane of erythrocytes
B. Disturbance of structure of molecule of hemoglobin
C. Disturbance of structure or synthesis of chains of globin
D. Influence of antibodies on erythrocytes
E. Toxic hemolysis

78. The 48 years old man was ill on flu 2 weeks ago, now he complains on dyspnea,
palpitation. Objectively: skin and mucouses are icteric, temperature of body-37,8°С,
pulse-120, AP-105/70, spleen is palpated. In blood test: Е-2,0x1012/l, Hb-70 g/l, CI-1,0,
reticul-18%, osmotic resistance and middle diameter of erythrocytes are norm, general
bilirubin-76 mkmol/l, indirect-63 mkmol/l. What is the most reliable diagnosis:
A. Markiafav – Mikelli`s disease
B. Hereditary microspherocytosis
C. Benign hyperbilirubinemia
D. Autoimmune hemolytic anemia
Е. Cholecystolithiasis

79. The patient P., 58 years old, complains on causeless appearance of bruises on skin,
hemorrhage of gums, dizziness. Objectively: the mucouses and skin are pale, with
numerous hemorrhages of different remoteness, lymphatic knots are not increased, pulse-
100, AP-110/70. In blood test: Е-3,0x1012/l, Hb-92 g/l, CI-0,7, anisocytosis,
poikilocytosis, L-10,0x109/l, е-2%, r/n-12%, s/n-68%, l-11%, m-7%, blood
sedimentation-12 mm/h. What laboratory index it is expedient to define for clarification
of diagnosis:
A. Osmotic resistance of erythrocytes
B. Content of reticulocytes
C. Clotting time of blood
D. Content of thrombocytes
E. Fibrinogen

80. The 30 years old woman first appealed to doctor with complaints on frequent nose-
bleedings, appearance of bruises on body. She is ill for half a year. After examination
the diagnosis of idiopathic thrombocytopenic purpura is set. From what it is necessary
to begin treatment of the sick:
21
A. Transfusion of thrombocyte concentrate
B. Cytostatic preparations
C. Immunoglobulin
D. Splenectomy
E. Glucocorticoids

81. The 60 years old woman during 4 years is under surveillance of doctor-hematologist
concerning chronic lympholeukemia. During last 6 months she had pneumonia twice. In
blood test: Е-3,1x1012/l, Hb-90 g/l, CI-0,9, L-160,0х109/l, е-1%, r/n-2%, s/n-21%, l- 74%,
m-2%, blood sedimentation-20 mm/h, general protein-60 g/l, gamma-globulin- 14%.
What changes in blood are more credible to assist development of complications at the
sick:
А. Decrease of hemoglobin
B. Hypogammaglobulinemia
C. Increase of leucocytes
D. Increase of lymphocytes
E. Hypergammaglobulinemia

82. The 72 years old man complains on rapid fatigability, perspiration, which appeared
two months ago. Objectively: t-37,7°С, liver +2 sm, spleen +8 sm, dense, sensible. In
blood test: Е-3,2x1012/l, Hb-110 g/l, CI-1,1; L-255x109/l, bas-7%, е-9%, promyel-2%,
myel-22%, metamyel-20%, r/-17%, s/n-15%, l-8%, blood sedimentation-15 mm/h, Т-
250,0x109/l. What disease is more reliable stipulated such changes in blood:
A. Erythremia
B. Acute myeloblastic leukemia
C. Non – Hodgkin`s lymphoma
D. Chronic myeloleukemia
E. Leukemoid reaction of myeloid type

83. The 60 years old woman complains on weakness, rapid fatigability for a year. Heavy
feeling in the left subcostum, subfebrile temperature, weight loss joined a month ago.
Objectively: skin is pallor, liver +2 sm, spleen +7 sm, dense, moderately painful. In blood
test: Е-3,0x1012/l, Hb-110 g/l, CI-1,1, L-280,0x109/l, blast cells-23%, bas-6%, е- 6%,
myel-10%, r/n-19%, s/n-7%, l-19%, Т-180x109/l, blood sedimentation-32 mm/h. What is
the most reliable diagnosis:
А. Chronic myeloleukemia, blastic crisis
B. Acute myeloblastic leukemia
C. Acute lymphoblastic leukemia
D. Chronic lympholeukemia
E. Leukemoid reaction of myeloid type

84. The 28 years old woman complains on weakness, periodic increase of temperature of
body to 39,0°С, perspiration at night-time, weight loss. Objectively: skin is pale,
increased cervical, supraclavicular and inguinal lymphatic knots, which are palpated by
size to1,5-2 sm, dense, unpainful. In blood test: E-3,0x1012/l, Hb-90 g/l, CI-0,8, L-
13,0x109/l, е-3%, r/n-9%, s/n-78%, l-7%, m-3%, blood sedimentation-48 mm/h.
Suspicion about lymphogranulomatosis appeared, biopsy of lymphatic knot is appointed
22
to.The presence of what changes is reliable at the research:
A. Proliferation of prolymphocytes and lymphoblasts
B. Proliferation of lymphocytes, lymphoblasts
С. Berezovsky – Shternberg cells
D. Proliferation of prolymphocytes and lymphocytes
E. Proliferation of lymphocytes and plasmocytes

85. At the 68 years old woman in blood test anemia and increase of blood sedimentation
were discovered. In the anamnesis: during last 1,5 years there were fractures of bones
twice. Objectively: pallor of skin, painful senses at percussion of ribs. In blood test: Е-
2,4x1012/l, Hb-76 g/l, CI-0,9, L-4,8x109/l, е-1%, r/n-4%, s/n-60%, l-28%, m-7%, blood
sedimentation-76 mm/h, Т-140x109/l. In analysis of urine: protein-3,3 g/l, L-6-8, Е-8-
10. Conducting of what research is more expedient for confirmation of diagnosis:
A. Bense-Jones protein of urine
B. Biopsy of kidneys
C. Urgent urography
D. Sternal puncture
E. Immunoglobulins of blood

86. The 28 years old woman appealed to doctor with complaints on hemorrhages on the
front surface of trunk and extremities, bleeding from gums. In blood test: Т-20,0х109/l;
in bone marrow number of megakaryocytes is increased. Treatment with glucocorticoids
was conducted and gave positive effect. What disease was diagnosed at the woman:
A. Hemorrhagic vasculitis
B. Hemophilia
C. Randyu-Osler`s disease
D. DIC - syndrome
E. Idiopathic thrombocytopenic purpura

87. The 28 years old sick appealed to internist with complaints on weakness, dizziness,
nose-bleedings. She is ill for 4 months. Objectively: there are hemorrhages in the area
of stomach and thighs by size 1-2 sm of different color. Liver and spleen are not
increased. In blood test: Е-2,6x1012/l, Hb-90 g/l, CI-0,8, iron of blood serum-8,0 mkml/l,
L-4,2x109/l, е-2%, r/n-7%, s/n-40%, m-6%, l-45%, T-47,1x109/l, blood sedimentation-
27 mm/h. Your previous diagnosis:
А. Idiopathic thrombocytopenic purpura
B. Hemolytic anemia
C. Chronic iron deficiency anemia
D. Chronic lympholeukemia
E. Aplastic anemia

88. The patient P., 50 years old, complains on weakness, dizziness, heavy feeling in the
upper half of stomach, paresthesias in finger-tips of hands and feet. Objectively:
yellowness of skin, language of raspberry color, hepatomegaly. At blood: Е-2,3x1012/l,
Hb-90 g/l, reticul-0,2%, CI-1,3, macrocytosis, Jolly`s bodies. Specify, what is not
characteristic for this anemia:
A. Paresthesia
23
B. Ataxia
C. Muscular atrophy
D. Delirium, hallucinations
Е. Decrease of intellect

89. Patient A., 26 years old, complains on fever, itch of skin, perspiration at night.
Objectively: temperature of body-38,6C, right supraclavicular lymphatic knot is
palpated, it is increased, mobile. What research is the most informative for confirmation
of diagnosis:
A. Albuminous fractions of blood
B. General analysis of blood
C. Survey sciagraphy of pectoral cavity
D. Immunogram
E. Puncture of lymphatic knot

90. At the 28 years old sick expressed icterus appeared after self-treatment of flu with
unsteroid anti-inflammatory preparations and antibiotics. At examination increased liver,
oliguria, dark colouring of urine are discovered. Laboratory researches: Е- 2,0x1012/l, Hb-
60 g/l, CI-0,9, L-12x109/l with change of formula to the left, reticul-14%. The increase of
indirect fraction of bilirubin is determined in serum of blood. The Coombs` test is
positive. About what disease it is possible to think:
A. Acute glomerulonephritis
B. Aplastic anemia
C. Toxic hepatitis
D. Acute leukemia
E. Autoimmune hemolytic anemia

91. The patient S., 73 years old, appealed to neurologist with complaints on pain in
lumbar area. The doctor diagnosed radiculitis. After the conducted physical therapy
condition of patient was not improved. After additional research on R-grams of bones of
spine and pelvis osteoporosis and defects of bones were found out. In blood test: moderate
normochromic anemia, in urine - proteinuria, general protein of blood serum - 97 g/l.
About what disease it follows to think:
A. Osteochondrosis with radicular syndrome
B. Senil osteoporosis
C. Multiple myeloma
D. Metastases in bones
E. Lymphogranulomatosis

92. The patient O., 31 years old, appealed to doctor with complaints on fever, weight loss,
itch of skin. At objective inspection increased unpainful lymphatic knot in the left
supraclavicular area is discovered. Liver and spleen are not increased. In blood test: Hb-
80 g/l, L-16,6х109/l, е-2%, r/n-8% s/n-60%, l-24%, m-6 %, blood sedimentation-55
mm/h, Т-190x109/l. What is the most expedient research for confirmation of diagnosis:
A. FGDS
B. Trepanobiopsy
C. Sternal puncture
24
D. Bens-Jones protein of urine
Е. Biopsy of lymphatic knot

93. The patient Y., 18 years old, is hospitalized with bleeding from incised wound of
palm, which lasted 2 days. Considerable bleeding at wounds was observed from
childhood.The similar phenomena are marked at cousin for the line of mother. Skin is
pale, knee and ankle joints are increased, deformed, motions in them are limited.
Laboratory researches: number of thrombocytes-320,0x109/l, bleeding time by Dyuke-3
minutes.What preparations it is necessary to enter the patient:
A. Cryoprecipitate
B. Heparin
C. Prednizolon
D. Thrombocytaric mass
E. Calcii gluconati

94. The sick, 30 years old, complains on weakness, hemorrhage of gums, increase of
temperature of body, pain in throat. There was contact with aniline dyes during 8 years.
At examination: skin is pale, with numerous petechias and ecchymoses, liver and spleen
are not palpated.In blood test: Е-2,5x1012/l, Hb-80 g/l, CI-0,9, L-2,4x109/l, Thr-
50,0x109/l, blood sedimentation-40 mm/h. In myelogram: bone marrow is with acutely
reduced number of cells. What diagnosis can be suspected at the patient:
A. Acute leukemia
B. Hypoplastic anemia
C. В12 deficiency anemia
D. Verlgoph`s disease
E. Agranulotcytosis

95. The sick I., 41 years old, complains on pain in throat, ribs and breastbone. At
examination: t of body - 38,0°С, skin is pale, with presence of petechias and bruises,
pulse 100, insignificant hepatosplenomegaly, there are numerous ulcers with necrotic
edges on the mucous of mouth. In blood test: Е-2,5x1012/l, Hb-70 g/l, CI-0,9, L-
28,0x109/l, blast-78%, s-4%, l-13%, m-5 %, Т-17,5x109/l. blood sedimentation-60
mm/h.What is the most reliable diagnosis:
A. Acute leukemia.
B. Diphtheria
C. Hemorrhagic vasculitis
D. Chronic hepatitis
E. Stomatitis

96. The patient T., 62 years old, complains on permanent pain in breastbone and loin,
general weakness, stuffiness. On the sciagram of spine: clinoid deformation of Thx,
diffuse osteoporosis of almost every vertebra. In blood test: Hb-90 g/l, Е-2,1x1012/l, L-
3,8x109/l, Thr-170,0х109/l, blood sedimentation-78 mm/h. General protein of blood-110
g/l. In urine: protein-2,9 g/l, L-2-4, E-4-6, are changed, hyalin cylinders-2-4.In
myelogram-18% of plasma cells. What is the most reliable diagnosis:
A. Compressional break of vertebra
B. Metastases of tumor in spine
25
C. Amyloidosis of kidneys
D. Multiple myeloma
E. Acute leukemia

97. The 38 years old woman, who suffers on menorrhagias, complains on twinkling of
"spots" before eyes, dizziness, fragility of nails, hair fall. At examination: skin is pale
and dry, pulse – 100, rhythmic. In blood test: Е-3,3x1012/l, Hb-90 g/l, CI-0,7, reticul-
0,8%, L-4,8x109/l, е-2%, r/n-3%, s/n-62%, l-25%, m-10%, hypochromia of erythrocytes,
microcytosis, iron of blood serum - 4,2 mkmol/l. What is the most reliable diagnosis:
A. Hypoplastic anemia
B. Hemolytic anemia
C. В12 deficiency anemia
D. Thalassemia
E. Iron deficiency anemia

98. The sick F., 49 years old, complains on pain in the left subcostum, general weakness,
rapid fatigability, weight loss. Objectively: skin and mucouses are moderately pale, pulse-
92, rhythmic, liver +4 sm, painless, dense, lower edge of spleen is at the level of
umbilicus.In blood test: Е-3,0x1012/l, Hb-90 g/l, CI-0,9, L-540,0x109/l, promyel-10%,
myel-13%, youn-11%, r/n-28%, s/n-22%, е-5%, bas-4%, l-4%, m-3%, Т- 345,0x109/l,
blood sedimentation-38 mm/h. What is the most reliable diagnosis:
А. Leukemoid reaction of myeloid type
В. Chronic myeloleukemia
C. Budd – Chiari syndrome
D. Cirrhosis of liver
E. Chronic lympholeukemia

99. The 45 years old man complains on general weakness, dizziness. During 15 years
there is ulcer of duodenum. Objectively: skin is pale, pulse-100, AP-100/70. At
subsequent inspection anemia, low colour index in blood test are discovered.
Colour index - is:
А. Ratio of the volume of form elements of blood to the volume of blood
В. Degree of admission of every eruthrocyte with hemoglobin
С. Increase of number of erythroblasts in bone marrow
D. Percent correlation of separate forms of leukocytes of blood
E. Blood sedimentation

100. The 60 years old woman complains on general weakness, sense of overfill in
epigastrium, nausea, belch after meal. She is ill over 10 years. Objectively: skin and
mucouses are pale, pulse-98, AP-115/75. In blood test: Е-2,0x1012/l, Hb-100 g/l.
Antibodies to oxyntic cells of stomach are found out. What is the most credible reason
of development of anemic syndrome at the sick:
A. Production of antibodies to gastromucoprotein
B. Disturbance of synthesis of hemoglobin
C. Disturbance of synthesis of erythropoetin
D. Disturbance of suction of iron
E. Increase of charge of iron
26
101. The patient P., 60 years old, complains on general weakness, feeling of weight in
epigastrium and belch after meal. Objectively: skin and of mucouses are pale, pulse- 110,
AP-115/70. In blood test: Е-2,0x1012/l; Hb-100 g/l; CI-1,5, blood sedimentation-28
mm/h. Antibodies to oxyntic cells of stomach are found out. What is the best tactic of
treatment of anemic syndrome at the sick:
A. Preparations of hydrochloric acid
B. Vitamin В12 intramuscular
C. Preparations of iron per os
D. Preparations of iron parenteral
E. Transfusion of packet red cells

102. The 45 years old man complains on general weakness, dizziness. During 15 years
there is ulcer of duodenum. Objectively: skin is pale, pulse-100, AP-90/70. What
primary inspection must be conducted at the patient:
А. General blood test, maintenance of ferritin of blood
В. General blood test, FGDS
C. Content of iron of blood
D. Content of ferritin of blood
E. General analysis of blood, maintenance of iron of blood

103. The 54 years old woman complains on weakness, numbness of finger-tips, shaky
step, heartburn in language. Objectively: skin is pale, pulse-110, systolic noise above the
apex of heart, language of bright red. In blood test: Е-2,3x1012/l, Hb-58 g/l, reticul- 0,2%,
CI-1,3, macrotcytosis, L-2,8x109/l, blood sedimentation-40 mm/h., Т-120,0x109/l. What
pathogenetic factor conducts in development of anemia:
A. Tumor oppression of normal blood formation
B. Intravessal hemolysis
C. Disturbance of synthesis of hemoglobin
D. Disturbance of transport of iron from reticuloendothelial depot
Е. Disturbance of differentiation of erythroid cells

104. At the sick, 42 years old, after acute respiratory disease, fever is saved. At
examination: on the skin of trunk and extremities - petechia-macular rash, lymphatic
knots of arm-pits are increased, soft, painless, tachycardia, systolic noise above all
auscultative points, liver and spleen are increased. In blood test: Hb-100 g/l, Е-
3,1x1012/l, CI-1,0, L-3,5x109/l, blast-33%, r/n-3%, s/n-35%, e-1%, l-20%, m-8%, blood
sedimentation-20 mm/h., Т-55,0x109/l. What diagnosis is most credible:
A. Chronic myeloleukemia
B. Infectious endocarditis
C. Acute leukemia
D. Verlgoph`s disease
E. Lymphogranulomatosis

105. The patient T., 19 years old, marked growing weakness, skin hemorrhages, nose-
bleedings, subfebrile temperature during last 2 months. Lymphatic knots, liver, spleen,
are not increased. In blood test: Е-1,5x1012/l, Hb-50 g/l, reticul-0,1%, CI-0,9, L-
27
1,8x109/l, е-1%, r/n-3%, s/n-58%, l-33%, m-5%, Т-30,0x109/l, blood sedimentation-60
mm/h., iron of blood serum -15 mkmol/l. What is the most credible diagnosis:
A. Hemolytic anemia
B. Acute leukemia
С. Aplastic anemia
D. В12 deficiency anemia
E. Iron deficiency anemia

106. The 37 years old woman complains on muscular weakness, palpitation, hard
swallowing of meal, has a desire to eat chalk. Objectively: satisfactory feed, skin is
pale, pulse-116, AP-90/70. In blood test: Е-3,1x1012/l, Hb-80 g/l, CI-0,7, reticul-0,8%,
L-4,7x109/l, е-2%, r/n-3%, s/n-64%, l-26%, m-5%, blood sedimentation-15 mm/h. Iron
of blood serum-4,3 mkmol/l, general protein-70 g/l. The deficit of what element
stipulated the origin of the disease:
A. Protein
B. Vitamin В6
C. Vitamin В12
D. Iron (Fe++)
E. Follic acid

107. The 23 years old student complains on pain in knee-joints, increase of temperature
of body. In the anamnesis: there was angina 10 days ago. Objectively: t-37,8°С, heart
rate-120, AP-105/70, knee joints, slightly swollen, there are symmetric hemorrhagic rash
on the skin of shins. In blood test: L-8,4x109/l, blood sedimentation-22 mm/h., Т-
190,0х109/l, prothrombin index-90%. Diurnal proteinuria-0,66 g/l., in analysis of urine
by Nechiporenko: L-2000, Е-9000 in 1mkl. What is the most credible diagnosis:
A. Verlgoph`s disease
B. Hemorrhagic vasculitis
C. Acute rheumatic fever
D. Systemic lupus erythematosus
E. Nodular periarteriitis

108. The 38 years old man, who is suffering on hemorrhoid, complains on twinkling of
"spots" before eyes, dizziness. At examination: pallor of skin, pulse-100, AP-90/60,
systolic noise above the apex of heart.In blood test: Hb-95 g/l, E-3,Зx1012/l, CI-0,7, L-
9,8x109/l, e-2%, r/n-3%, s/n-70%, l-24%, m-1 %, blood sedimentation-25 mm/h.,
hypochromia of erythrocytes, iron of blood serum-5,2 mkmol/l. What is the most
reliable reason of sistolic noise:
A. Narrowing of main vessels
B. Myocarditis
C. Tachycardia
D. Low AP
Е. Acceleration of blood flow

109. The sick C., 39 years old, marks weakness, dizziness, dyspnea, muscular weakness
during 4 months. In the anamnesis: fibromyoma of uterus, uterine bleeding. In blood test:
Hb-80 g/l, E-2,2x1012/l, CI-0,7, blood sedimentation-28 mm/h., anisocytosis,
28
poikilotcytosis, iron of blood serum-5,3 mkmol/l.Tactic of conducting of the sick:
A. Dietary nutrition
B. Complex of vitamins
C. Permanent reception of preparations of iron
D. Dietary nutrition, course reception of preparations of iron
E. Dietary nutrition, complex of vitamins

110. The patient, 19 years old, complains on general weakness, pain in bones, fever. At
examination systematic increase of lymphatic knots, hepatolienal syndrome are found
out. In blood test: Е-2,2x1012/l, Hb-67 g/l, L–20x109/l, blast-45%, r/n-3%, s/n-7%, l-
40%, m-5%,Т-45,0x109/l, blood sedimentation-55 mm/h. What diagnosis can be set at
the patient:
А.Acute leukemia
B. Chronic myeloid leukemia
C. Chronic lympholeukemia
D. Hypoplastic anemia
E. Agranulotcytosis

111. At the patient T., 68 years old, systematic increase of lymphatic knots,
hepatosplenomegaly, icterus are found out. In blood test: Е-2,4x1012/l, Hb-65 g/l, reticul-
10%, Т-190x109/l, L-250x109/l, r/n-1%, s/n-7%, l-87%, m-5%, blood sedimentation-55
mm/h. What complication of basic disease can be suspected on clinic- laboratory indexes:
A. Toxic neutropenia
B. Aplastic anemia
C. Autoimmune hemolysis
D. Agranulocytosis
E. Hepatitis

112. The patient with chronic myeloleukemia complains on acute pain in the left
subcostum. Objectively: protective tension of muscles is discovered at palpation in the
projection of the left part of stomach, the pole of spleen is palpated. At auckultation of
subcostum noise of friction is marked. What complication does it follow to suspect:
A. Renal colic
B. Strangulation of diaphragmatic hernia
C. Acute pancreatitis
D. Infarct of spleen
E. Torsion of colon

113. The patient S., 57 years old, at examination widespread osteoporosis of vertebra is
found out. In blood test: Е-3,4x1012/l, Hb-80 g/l, Т-145,0x109/l, L-5,6x109/l, е-3%, r/n-
5%, s/n-57%, l-29%, m-6%, blood sedimentation-55 mm/h. In analysis of urine: protein-
0,264 g/l.General protein of blood is 108 g/l. What research must be done for
confirmation of diagnosis:
A. Definition of circulating immune complexes
B. Definition of level of parathyroid hormone
C. Sternal puncture
D. Densytometry
29
Е. Biopsy of kidneys

114. The sick L., 18 years, is hospitalized with bleeding from incised wound of palm,
which lasted 2 days. Skin is pale, knee and ankle joints are increased, deformed, motions
in them are limited. Laboratory researches: number of thrombocytes - 420,0x109/l,
bleeding time by Dyuke-3 minutes, clotting time by Li-Uayt-27 min.,
prothrombin index-100%, fibrinogen-4 g/l. The prophylaxis of what complication needs
to be conducted at the patient:
A. Pathological fractures
B. Posthemorrhagic anemia
C. Thrombosis
D. Aplastic anemia
E. Infecting of hematoma

115. The 63 years old man marks headache during few years, periodic pressing pain in
the area of heart. In the anamnesis: smoking more than 40 years; two packs of cigarettes
a day. Objectively: face of red color, there are dry, whistling rales above lungs, AP-
185/95. In peripheral blood: Е-6,5x1012/l, Hb-185 g/l, L-6,0x109/l, blood sedimentation-
5 mm/h. Т-190x109/l. What primary research needs to be conducted for clarification of
diagnosis:
A. US investigation of abdominal cavity, research of function of external breathing
B. ECG, ECHOCS
C. Sternal puncture
D. Consultation of oculist
E. US investigation of abdominal cavity

116. At the 23 years old man has fever, perspiration, dizziness, nasal and gingival
bleeding, hemorrhages on the skin of trunk during a week. In blood test: Hb-72 g/l, Е-
2,3x1012/, L-7,6x109/l, blast-86%, s-5%, l-9%, blood sedimentation-23 mm/h. This state
is characterized with all resulted syndromes, except:
A. Anemic
B. Hemorrhagic
C. Ulcerous-necrotic
D. Infectious
E. Bronchospastic

117. The sick D., 56 years old, marks weakness, pain in bones, decline of appetite,
headache during 4 months. Treated oneself at neurologist, passed completed course of
manual therapy. Laboratory researches: anemia, general protein-112 g/l, blood
sedimentation-68 mm/h, moderate proteinuria. On the sciagram of pelvis there are
defects of bone fabric of the rounded form. What diagnosis can be set at the patient: А.
Multiple myeloma
B. Metastases of tumor in bones
C. Systematic osteoporosis
D. Amyloidosis of kidneys
E. Chronic glomerulonephritis

30
118. The patient T., 62 years old, is hospitalized in comma. In blood test: Нb-38 g/l, E-
0,7x1012/l, CI-1,2, macrocytosis, reticul-0,2 %, leukopenia, thrombotcyopenia. In bone
marrow: megaloblastic type of blood formation. Name the preparation for effective
treatment of the sick:
А. Preparations of iron intravenous
B. Packet red cells intravenous drop
C. Cyancobalamin intramuscular, packet red cells intravenous drop
D. Preparations of iron intravenous, packet red cells intravenous drop
E. Cyancobalamin intramuscular

119. The sick S., 50 years old, complains on general weakness, dizziness.
Objectively: pallor of skin, language is as raspberry, at FGDS: atrophy gastritis, Рh-
metry: achilia. In blood test: Е-2,3x1012/l, Hb-90 g/l, CI-1,2, macrotcytosis, Jolly`s
bodies, Cebot`s rings. For this condition is characteristic all signs, except:
A. Pallor of skin with lemon tint
B. Ataxia
C. Paresthesia
D. Exhaustion
E. Subicteric colour of white of the eyes

120. The sick K., 60 years old, appealed with complaints on increased crabbiness,
feeling of "wadding" feet, numbness of tiptoes.Objectively: skin is pale, white of the
eyes are subicteric, liver +2 sm, spleen +2 sm, moderately painful. In blood test: Е-
1,3х1012/l, Hb-58 g/l, CI-1,3, reticul-0,2%, blood sedimentation-30 mm/h.,
macrocytosis. What is needed to conduct for confirmation of diagnosis:
A. Roentgenologic research of stomach
B. Endoskopic research of stomach
С. Sternal puncture
D. Definition of osmotic resistance of erythrocytes
E. Definition of iron of blood serum

121. The patient T., 20 years old, is hospitalized in clinic with complaints on pain in
throat, bleeding from gums, increase of temperature to 38,5°С, osalgia.In blood test: Е-
2,2x1012/l, Hb-60 g/l, L-16,0x109/l, blast-50%, r/n-1%, s/n-12%, l-32%, m -5%, CI-0,8,
Т-76,0x109/l, blood sedimentation-38 mm/h. Name the characteristic changes of blood
at this condition:
A. Anemia
B. Leukotcytosis
C. Reticulocytopenia
D. Appearance of blastic cells
Е. All answers are true

122. The sick N., 54 years old, complains on weakness, palpitation at step, dyspnea. In
blood test: Е-1,8x1012/l; Hb-81g/l; CI-1,3; L-3,2x109/l, macrocytosis, Т-140,0х109/l.
General bilirubin-47,6 mkmol/l, mainly due to indirect. Doctor suspected В12 deficiency
anemia.What disease can not result the increased loss of vitamin В12:
A. Intestinal vermins
31
B. Dysbiosis
C. Ulcer of duodenum
D. Disease of liver
E. Leukemia
123. At the patient C., 52, splenomegaly is found out without special subjective
complaints. In blood test: Hb-132 g/l; L-52,0x109/l, е-5%, bas-2%, myeloblast-6%,
promyel-5%, myel-6%, youn-8%, r/n-4%, s/n-4 %, l-12%, m-6%; blood sedimentation-
19 mm/h. It is expedient to use at treatment:
A. Scheme "5+2"
B. Cyclophosphan
С. Hydrxiourea (hydrea)
D. Prednizolon
E. Treatment is not necessary

124. The sick M., 40 years old, entered clinic with complaints on nasal and uterine
bleeding, presence of bruises on skin. In the anamnesis: viral infection 2 weeks ago.
On the skin of trunk and extremities there are bruises. In blood test: E-2,6x1012/l, Hb-80
g/l, thrombocytes-25,0x109/l, L-6,8х109/l, leukocytar formula is without changes. Blood
sedimentation-30 mm/h. Bleeding time by Dyuke is 13 minutes. Appoint treatment:
A. Preparations of vitamin K
B. Transfusion of packet red cells
C. Transplantation of bone marrow
D. Antihemophyl immunoglobulin
E. Glucocorticoids

125. The girl, 18 years old, complains on acute pain in throat, increase of temperature of
body to 40,0°С. In the anamnesis: acute respiratory infection a week ago, treated with
analgin and biseptol; condition was improved briefly. At examination: there are necrotic
ulcers on mucous of mouth cavity. In blood test: E-3,8х1012/l, Hb-115 g/l, Т- 100,0х109/l,
L-0,8х109/l, е-1%, r/n-1%, s/n-4%, l-77%, m-15%, plasma cells-2%. Blood
sedimentation-46 mm/h. What treatment must be appointed:
A. Antiseptic preparations locally
B. Cytostatic preparations
C. Antihistaminic preparations
D. Stimulators of erythrocytopoiesis
E. Glucocorticoids

126. The 49 years old man marks weakness, palpitation, icterus. Objectively: t of body -
37,8 °С, skin and white of the eyes are subicteric, liver +2 sm, spleen +3 sm. In blood
test: Е-3,0х1012/l, Hb-90 g/l, CI-0,9, reticul-18%, maximal osmotic resistance of
erythrocytes -0,48, general bilirubin-76 mkmol/l, indirect-63 mkmol/l. What additional
research it is expedient to appoint for clarification of diagnosis:
А. Activity of glyco-6-phosphat-dehydrogenase
B. Definition of transaminases
C. Bilious pigments of urine
D. The Coombs` test
Е. Markers of viruses of hepatitis
32
127. The patient B., 48 years old, complains on weakness, palpitation and dyspnea at
the insignificant physical load. There are swelling of stomach, diarrhea, especially after
milk products after toxicoinfection.Objectively: skin and mucouses are pale, at palpation
painful thick intestine.In blood test: Е-3,1x1012/l, Hb-70 g/l, CI-0,65, L- 4,0x109/l, blood
sedimentation-22 mm/h. What additional research it is expedient to conduct:
A. Puncture of bone marrow
B. Definition of vitamin В12 in blood
C. Coprogram
D. The Coombs` test
E. Definition of iron of blood serum

128. The sick M., 65 years old, who is observed at dermatologist concerning obtrusive
itch of skin for a few years, lately markes headache. In the anamnesis: smoking during
40 years. Objectively: redder-cyanotic colour of face, AP-170/100, dry rales in lungs,
splenomegaly. In blood test: Е-7,9x1012/l, Hb-210 g/l, L-12,8x109/l, Т-364,0x109/l,
blood sedimentation-1 mm/h., circulating blood volume-8,1 l. What treatment can be
appointed:
A. Exsanguination
B. Leukeran
C. Roentgenotherapy of spleen
D. Roentgenotherapy of bone marrow
E. Hydrea

129. The sick B., 53 years old, complains on increase of temperature of body, pain in
joints, in stomach. At examination: skin and white of the eyes are pale, icteric, liver +3
sm, the pole of spleen is palpated. In blood test: Hb-72 g/l, CI-0,85, reticul-5%; blood
sedimentation-26 mm/h, bilirubin-52 mkmol/l, indirect-48 mkmol/l; gamma-globulin-
26%; ALT-0,7мкмоль/л; the direct Coombs` test is positive.Your previous diagnosis:
А. Autoimmune hemolytic anemia
B. Chronic hepatitis
C. Hereditary microspherocytic anemia
D. Gilber`s syndrome
E. В12 deficiency anemia

130. At the patient Z., 67 years old, with hepatolienal syndrome during 2 years there are
increased peripheral lymphatic knots, which are soft, unconnected with adjoining fabrics
at palpation. In blood test: L-120,0x109/l, r/n-1%, s/n-9%, l-87%, m-3%. Blood
sedimentation-40 mm/h.In what age this disease is more frequen:
A. At young
B. At mature.
C. At children
D. At teenagers
E. At elderly
131. The sick D., 28 years old, after supercooling noticed fever, pain in muscles and
upper half of stomach. Moderate icterus, dark feces and urine appeared in a day.
Objectively: skin, white of the eyes, mucouses are icteric, hepatosplenomegaly. In blood
test: Е-2,8х1012/l, Hb-80 g/l, CI-0,8, Т-230,0x109/l, L-9,5x109/l, reticul-7%, blood
33
sedimentation-20 mm/h. Biochemical blood test: general bilirubin-67,0 mkmol/l,
indirect-58,3 mkmol/l, direct-8,7 mkmol/l. The Coombs` test is positive. What disease
can be suspected at the sick:
A. Gilber`s syndrome
B. Acute hepatitis
С. Hemolytic anemia
D. Rotor syndrome
E. Cholecystolithiasis

132. The man, 42 years old, complains on headache, increase of arterial pressure to
200/120, itch of skin after soul. In blood test: Е-6,2x1012/l, Нb-200 g/l, Т-650,0x109/l,
L-12,2х109/l, е-7%, r/n-10%, s/n-64%, l-15%, m-4%, blood sedimentation-1 mm/h.
What the itch of skin is linked with:
A. Predominance of maintenance of granulocytes
B. Allergic reaction
C. Thrombocytosis
D. Erythrocytosis
E. All answers are true

133. The 20 years old youth has signs of hemolytic crisis.Similar crises were observed
in 5 and 9 years old. In the anamnesis: operation concerning "wolfish fall" in age of 2
years. Objectively: yellowness of skin and white of the eyes, spleen is increased.In blood
test: Е-2,3x1012/l, Hb-68 g/l, blood sedimentation-38 mm/h, microspherocytosis,
decline of osmotic resistance of erythrocytes. What method of treatment is the most
effective in this case:
A. Plasmapheresis
B. Splenectomy
C. Glucocortikoids
D. Cytostatic preparations
E. Hemosorbtion

134. The woman, 68 years old, hospitalized with complaints on pain in ribs, general
weakness. Objectively: skin is pale; pain is marked at pressure on thorax. In blood test:
Е-2,6x1012/l, Hb-78 g/l, L-6,1x109/l, blood sedimentation-84 mm/h; glucose of blood-
5,2 mmol/l; general protein-100 g/l. In analysis of urine: specific weight-1015, protein-
4,0 g/l, Е-1-3; L-2-3. What research is contra-indicated:
A. Sternal puncture
B. Excretory urography
C. Survey urogram
D. Biopsy of kidney
E. US investigation of kidneys
135. The sick L., 29 years old, appealed to internist concerning growing weakness, pain
in bones, fever. At examination: pallor of skin, there are petechial-macular rash on the
skin of trunk and extremities. Subaxillary lymphatic knots are increased to 1,5x2,0 sm,
soft, painless.In blood test: Е-3,1x1012/l; Hb-80 g/l, CI-0,8, L-2,2x109/l; blast-36%, r/n-
3%, s/n-16%, l-41%, m-4%, Thr-40,0x109/l; blood sedimentation-44 mm/h. Aleukemic
leukemia – is:
34
A. Appearance of blastic cells in blood
B. Absence of blastic cells in blood
C. Number of blastic cells is increased in myelogram
D. Appearance of reticulocytes in blood
E. Decreaese of leukocytes in blood

136. The 40 years old woman, who suffers on menorrhagia, complains on twinkling of
"spots" before eyes, dizziness, fragility of nails, hair fall. At examination: pallor of skin
and mucouses, pulse-100; systolic noise is above all auscultative points. In blood: Е-
3,3x1012/l, Hb-90 g/l, CI-0,7, L-9,8х109/l, hypochromia of erythrocytes, anisocytosis.
What reason of origin of systolic noise:
A. Disturbance of synchronousness of work of valvular apparatus
B. Defeat of myocardium of hypoxic character
С.Acceleration of intracardiac blood stream in conditions of decrease of blood
viscidity
D. Acceleration of intracardiac blood stream in conditions of increase blood viscidity
E. Increase of minute volume of blood

137. The Sick G., 50 years old, complains on weakness, dizziness.At examination: skin
and mucouses are subicteric, liver +4 sm, dense, painful, spleen +8 sm, dense. In blood
test: E-2,4х1012/l, Hb-84 g/l, reticul-25%, L-7,0х109/l, е-3%, r/n-6%, s/n-62%, l-22%,
m-6%; blood sedimentation-30 mm/h; osmotic resistance of erythrocytes is reduced,
maintenance of bilirubin of blood is increased due to indirect, reaction on urobilin is
acutely positive, the Coombs` test is positive. What diagnosis is the most credible:
A. Minkovsky – Shoffar`s disease
B. Rotor disease
С. Night hemoglobinuria
D. Autoimmune hemolytic anemia
Е. Gilber`s disease

138. The patient D., 63 years old, complains on feeling of overfill in epigastrium,
nausea, belch after meal, dyspnea. In blood test: Е-2,0x1012/l, Hb-100 g/l, hyperchromia,
macrocytosis.At FGDS there are signs of gastritis, at Рh-metry - achilia. What is the
most credible reason of development of anemic syndrome:
A. Production of antibodies to gastromucoprotein
B. Disturbance of synthesis of hemoglobin
C. Disturbance of synthesis of erythropoietin
D. Disturbance of suction of iron
E. Increased charges of iron
139. The 30 years old woman entered clinic with complaints on nasal and uterine
bleeding. In the anamnesis: viral infection 2 weeks ago. At examination: on the skin of
trunk and extremities - bruises. In blood test: E-2,8x1012/l, Hb-90 g/l, thrombocyt -
25,0х109/l, L-8,8х109/l, leukocytar formula without changes. Blood sedimentation-30
mm/h, bleeding time by Dyuke is13 minutes. The sick must appoint:
A. Trasfusion of thromboconcentrate
B. Transfusion of packet red cells
C. Transplantation of bone marrow
35
D. Antihemophyl immunoglobulin
E. Glucocorticoids

140. The 39 years old man marks weakness, palpitation.Objectively: t of body - 38,8°С,
skin and white of the eyes are icteric, liver +3 sm, spleen +5 sm. In blood test: Е-
3,2x1012/l, Hb-90 g/l, CI-0,9, reticul-8%, maximal osmotic resistance of erythrocytes -
0,48, general bilirubin-76 mkmol/l, indirect-63 mkmol/l. What additional research is
expedient for clarification of diagnosis:
А. Activity of glyco-6-phosphat-dehydrogenase
B. Definition of transaminases
C. Bilious pigments of urine
D. The Coombs` test
Е. Markers of viruses of hepatitis

141. At the sick O., 23 years old, who suffers on hemophilia A, extraction of tooth is
planned. Introduction of what medical preparation with the purpose of providing
hemostasis is necessary to conduct before and after operation?
A. Cryoprecipitate
В. Fresh frozen plasma
С. Aminocapron acid
D. Vikasol
E. Ascorbic acid

142. The sick N., 22 years old, reduced feed, vegetarian, appealed to policlinic with
complaints on perversion of smell, taste, angular cheilitis. At examination: expressed
bright blue color of white of the eyes. After laboratory research iron deficiency anemia
was diagnosed. What clinical syndrome takes advantage at the patient:
A. Sideropenic
B. Anemic
С. Heart failure
D. Metabolic intoxication
E. Myelodysplastic

143. The patient I., 52 years old, appealed to district doctor with complaints on
weakness, cough with mucous expectoration, weight loss (10 kg during 4 months).
Objectively: t of body - 37,5°С, increased supraclavicular and neck lymphatic knots are
palpated from both sides, there are harsh breathing and dry rales above lungs, pulse-
112, AP–110/70; In blood - lymphocytosis (80%). Choose subsequent tactic of district
doctor:
А. To direct the patient to hematologist
В. To appoint antiinflammatory therapy
С. To direct the patient to oncologist
D. To conduct subsequent ambulatory inspection of patient (R – graphy of organs of
pectoral cavity, computed tomography, biochemical blood tests)
E. To appoint desintoxitcative and symptomatic therapy

144. At the patient, suffering on ischemic heart disease, who used aspirin, weakness,
36
dizziness, more frequent pain in heart appeared. At examination: pale, AP-100/60, heart
rate-100, E-2,6x1012/l, Hb-100 g/l, CI-0,8; L-5,8x109/l, T–288x109/l, iron of blood
serum-11,0 mkmol/l. What disease stipulates unstable stenocardia:
А. Posthemorrhagic anemia
В. Hypertonic illness
C. Myocarditis
D. В12 deficiency anemia
Е. Hemolytic anemia

145. The sick K., 50 years old, complains on decline of appetite, nightly perspiration,
discomfort in stomach, weight loss. Objectively: pallor of skin, hepatosplenomegaly. In
blood test there are anemia, leukocytosis with change of leukocytar formula to the left,
bazophil – eozinophil association. What result of research confirms the previous
diagnosis:
А. Presence of the Philadelphian chromosome
В. Botkin – Gumpercht shadows
С. Increase level of alkaline phosphatase
D. Decrease of cyancobalamin
Е. Total hyperplasia of bone marrow with megacariocytosis

146. The patient C., 28 years old, complains on undulating fever, perspiration.
Objectively: pallor of skin, lymphatic knots are mobile, densely elastic, by the diameter
of 1-2 sm, not painful, not connected with skin. In blood: Е-3,0x1012/l, Hb-100 g/l, L-
14x109/l, change of formula to the left, Т-280x109/l, blood sedimentation-37 mm/h.
What method of research it follows to use for confirmation of diagnosis:
А. Biopsy of lymphatic knot
В. Sternal puncture
С. Biopsy of muscles
D. Sciagraphy of organs of thorax
Е. Lumbar puncture

147. The sick, 40 years old, hospitalized in gynaecological separation with uterine
bleeding. Objectively: skin is pale with superficial hemorrhages in the area of trunk and
extremities. Blood test: Е-2,6•1012/l, Нb-80 g/l, L-4,2•109/l, e-3%, r/n-4%, s/n-58%; l-
30%, m-5%, thromb-50x109/l. Define the type of hemorrhage at this sick :
A. Petechia-macular
B. Hematomal
С. Mixed
D. Vasculit-purpural
E. Angiomatosal

148. The sick T., 28 years old, complains on weakness, dizziness, yellowness of skin.
Objectively: yellowness of skin, liver +3 sm, lower pole of spleen is palpated. In blood
test: anemia, reticulocytosis, leukocytosis with change of leukocytar formula to
promyelocytes. Autoimmune hemolytic anemia is suspected. Choose correct, in relation
to the indicated disease, affirmation:
А. Diagnosis is confirmed by the positive Coombs` test
37
B. Growth of level of direct bilirubin is characteristic
S. Changes of structure of erythrocytar membrane lie in basis of disease
D. Hypoplasia of red link in trepanobioptate
E. Increase of level of alkaline phosphatase

149. At the sick O., 19 years old, on the 7 day of treatment concerning acute rheumatic
fever, yellowness of skin appeared, level of hemoglobin went down, level of
reticulocytes and indirect bilirubin rose. The Coombs` test gave positive result. What
medication is more credible entail appearance of medicine-induced hemolytic anemia:
А. Natrii benzilpenicilini
В. Diclophenac
S. Chlorochin
D. Prednizolon
Е. Ascorbin acid

150. The patient G., 18 years old, complains on periodic appearance of icteric colouring
of skin, heavy feeling in the left subcostum. Objectively: lymphatic knots are not
increased, spleen +3 sm. Blood test: Е-2,7x1012/l, Нb-84 g/l, CI-0,96, reticul-18%,
microspherocytosis. Indirect bilirubin–38 mkmol/l.Your diagnosis:
А. Minkovsky – Shoffar`s anemia
В. Autoimmune hemolytic anemia
С. Sideroachrestic anemia
D. Hypoplastic anemia
E. В12 deficiency anemia

151. The sick G., 42 years old, suffers on menorrhagia during three years. There is a
exacerbation of chronic pancreatitis as pain in the left subcostum, diarrhea during last
two weeks. In blood test there is moderate hypochromic anemia, iron of blood serum –
7,2 mkmol/l. What is necessary for correction of anemia:
А. Preparations of iron parenteral
В. Preparations of iron per os
C. Transfusion of packet red cells
D. Anabolic hormones
Е. Vitamins of group B

152. The patient A., 62 years old, treats oneself at dermatologist during last two years,
concerning itch of skin, which increases after taking a bath. At examination: face of red
color, liver is increased - (+4 sm), spleen - (+5 sm). Blood test: Е-7,1x1012/l, Hb-210 g/l,
L-12,5x109/l, change of formula to the left, thromb-525x109/l, blood sedimentation 1
mm/h. Hematocrit-72%.Your diagnosis:
А. Cirrhosis of liver
В. Acute erythromyelosis
С. Chronic myeloleukemia
D. Subleukemoid myelosis
E. Erythremia

153. The patient K., 66 years old, delivered in hospital in unconscious condition. Last
38
half a year complains on weakness, stuffiness, dizziness. At examination: skin of lemon
colour, tachycardia, systolic noise above all auscultative points, AP-80/60, liver is
increased (+3 sm), spleen is palpated. Blood test: Е-1,0x1012/l, Hb-45 g/l, blood
sedimentation-50 mm/h, glucose of blood-4,2 mmol/l, creatinine of blood-105 mkmol/l,
bilirubin of blood-32,6 mkmol/l due to indirect, negative wave T in V1-V4. Your previous
diagnosis:
А. Diabetic coma
В. Hepatic coma
С. Uremic coma
D. Anemic coma
Е. Infarct of myocardium, cardiogenic shock.

154. A 62-year-old patient complaining of enlargement of cervical, supraclavicular and


axillary lymph nodes, subfebrile temperature for the last 3 months has been admitted to
a hospital. In blood: WBCs - 64 • 109/l, lymphocytes - 72%. What method of study
should be used to specify the diagnosis?
A. Myelogram
B. Lymphography
C. Lymphoscintigraphy
D. X-rays
E. Thermography

155. A 42-year-old patient complains of back pain, darkened urine, general weakness,
dizziness that occurred after treating a cold with aspirin and ampicillin. Objectively: the
patient is pale, with subicteric sclerae. HR - 98 bpm. Liver - +2 cm, spleen - +3 cm. In
blood: RBCs - 2,6 • 1012/l, Hb - 60 g/l, CI - 0,9, WBCs -9,4 • 109/l, basophils - 0,5%,
eosinophils - 3%, stab neutrophils - 6% segmented neutrophils - 58%, lymphocytes -
25%, monocytes - 7%, ESR - 38 mm/hour, reticulocytes - 24%. Total bilirubin - 38 milli-
mole/l. What complication occurred in the patient?
A. Agranulocytosis
B. Paroxysmal nocturnal hemoglobinuria
C. Acquired hemolytic anemia
D. Toxic hepatitis
E. Cholelithiasis

156. A 35-year-old patient has been in the intensive care unit for acute renal failure due
to crush for 4 days. Objectively: the patient is inadequate. Breathing rate -32/min. Over
the last 3 hours individual moist rales can be auscultated in lungs. ECG shows high T
waves, right ventricular extrasystoles. CVP - 159 mm Hg. In blood: the residual nitrogen
- 62 mi-llimole/l, K+- 7,1 millimole/l, Cl-- 78 mi-llimole/l, Na+- 120 millimole/l, Ht -
0,32, Hb - 100 g/l, blood creatinine - 0,9 milli-mole/l. The most appropriate method of
treatment would be:
A. Plasma sorption
B. Hemodialysis
C. Hemosorption
D. Plasma filtration
E. Ultrafiltration
39
157. A 38-year-old patient complains of inertness, subfebrile temperature, enlargement
of lymph nodes, nasal haemorrhages, bone pain. Objectively: the patient’s skin and
mucous membranes are pale, palpation revealed enlarged painless lymph nodes;
sternalgia; liver was enlarged by 2 cm, spleen - by 5 cm, painless. In blood: erythrocytes
-2,7 • 1012/l, Hb- 84 g/l, leukocytes – 58 109/l, eosinophils - 1%, stab neutrophils - 2%,
segmented neutrophils - 12%, lymphocytes - 83%, lymphoblasts - 2%, smudge cells;
ESR- 57 mm/h. What is the most likely diagnosis?
A. Chronic myeloleukemia
B. Acute lymphatic leukemia
C. Acute myeloleukemia
D. Lymphogranulomatosis
E. Chronic lymphatic leukemia

158. A 43-year-old male patient undergoing treatment for peptic ulcer complains of
weakness, dizziness, coffee-ground vomiting, melena. After administration of
haemostatics the patient’s condition has not improved, fresh blood has shown up in the
vomit, skin bruises of different sizes have appeared. In blood: thrombocytes -50 • 109/l,
Lee-White clotting time - 35 minutes, APTT - 80 seconds. In this case it is most rational
to administer the following preparation:
A. Vikasol
B. Fresh frozen plasma
C. Heparin
D. Fibrinogen
E. Rheopolyglucinum

159. A 42-year-old female lives in the basement, is unemployed, undernourished. She


complains of having general weakness, hair loss, brittle nails for six months, likes to eat
chalk. Objectively: the patient is emaciated, pale, has dry skin. Peripheral lymph nodes
are not enlarged. Liver is +1,5 cm. In blood: RBCs -1,8 • 1012/l, Hb- 62 g/l, colour index
- 0,78, reticulocytes - 0,5o/oo, ESR- 18 mm/h. Leukogram exhibits no pathology. What
is a provisional diagnosis?
A. Nutritional iron deficiency anaemia
B. Chronic hepatitis
C. B12-deficiency anaemia
D. Acquired haemolytic anaemia
E. Congenital haemolytic anaemia

160. A 24-year-old patient consulted a doctor about enlarged submandibular lymph


nodes. Objectively: submandibular, axillary and inguinal lymph nodes are enlarged.
Chest radiograph shows enlarged mediastinal lymph nodes. In blood: RBCs - 3,4 •
1012/l, Hb- 100 g/l, colour index - 0,88, thrombocytes - 190 • 109/l, WBCs - 7,5 • 109/l,
eosinophils -8%, stab neutrophiles - 2%, segmented neutrophiles - 67%, lymphocytes -
23%, ESR - 22 mm/h. What study is required to verify the cause of lymphadenopathy?
A. Open biopsy of lymph nodes
B. Ultrasound examination of the abdomen
C. Mediastinal tomography
40
D. Puncture biopsy of lymph nodes
E. Sternal puncture

161. A 27-year-old patient complains of nasal haemorrhages, multiple bruises on the


anterior surface of the trunk and extremities, sudden weakness. In blood: Hb- 74 g/l,
reticulocytes - 16%, RBCs -2,5 • 1012/l, platelets - 30 • 109/l, ESR- 25 mm/h. What is
the most effective measure for the treatment ofthrombocytopenia?
A. Iron preparations
B. Hemotransfusion
C. Cytostatics
D. Splenectomy
E. Vitamin B12

162. A 60 year-old female has been suffering weakness, dizziness, fatigue over the last
year. Recently she has also developed dyspnea, paresthesia. Objectively: skin and
mucous membranes are pale and slightly icteric. The tongue is smooth due to the loss of
lingual papillae. Liver and spleen are located at the costal margin. Blood count: Hb- 70
g/l, RBCs -1, 7- 1012/l, colour index - 1,2, macrocytes. Administer the patient a
pathogenetically justified drug:
A. Vitamin B6
B. Ascorbic acid
C. Iron preparations
D. Vitamin B12
E. Vitamin B1

163. A 22-year-old vegetarian patient with signs of malnutrition consulted a doctor about
smell and taste distortion, angular stomatitis. Objectively: expressively blue sclerae. The
patient was diagnosed with iron deficiency anemia. What is the dominating clinical
syndrome?
A. Anaemic
B. Haemologic
C. Sideropenic
D. Haemolytic
E. Myelodysplastic

164. A 63-year-old female complains of general weakness, a feeling of heaviness,


compression in the epigastrium, postprandial fullness, nausea, belching after meals.
These symptoms have been observed for about 15 years. Objectively: body temperature
is 36,4oC, respiratory rate - 20/min, Ps - 88/min, blood pressure - 115/75 mm Hg. Skin
and mucous membranes are pale. Blood test results: RBC -2,0-1012/l, Hb -100 g/l. Tests
revealed parietalcell antibodies. What is the most likely reason for the development of
anemia in this patient?
A. Disruption of hemoglobin synthesis
B. Production of antibodies to intrinsic factor
C. Disruption of erythropoietin synthesis
D. Impaired iron absorption
E. Increased loss of iron
41
165. On the second day of the disease a 22-year-old male patient complains of high-
grade fever, headache in the region of forehead and superciliary arches, and during eye
movement; aching muscles and joints. Objectively: body temperature is 39oC. Face is
hyperemic, sclerae are injected. The mucous membrane of the soft palate and posterior
pharyngeal wall is bright hyperemic and has petechial hemorrhages. What changes in
the hemogram are typical for this disease?
A. Leukocytosis
B. Neutrocytosis
C. Anemia
D. Accelerated ESR
E. Leukopenia

166. A 30-year-old male patient complains of inertness, low-grade fever, bleeding gums,
frequent quinsies, aching bones. Objectively: the patient has pale skin and mucous
membranes, sternalgia, +2 cm liver, +5 cm painless spleen. Blood test results: RBC-2,7
• 1012/l, Hb- 80 g/l, WBC- 3 • 109/l, eosinophils - 4%, basophils - 5%, blasts -
4segmented neutrophils -17%, lymphocytes -29%, myelocytes - 25%, promyelocytes -
12%, monocytes - 2%, platelets - 80 • 109/l, ESR -57 mm/h. What test should be
performed to verify the diagnosis?
A. Sternal puncture
B. Trephine biopsy
C. Lymph node biopsy
D. Lumbar puncture
E. Chest X-ray

167. A 49-year-old male patient consulted a doctor about difficult swallowing, voice
hoarseness, weight loss. These symptoms have been gradually progressing for the last 3
months. Objectively: the patient is exhausted, supraclavicular lymph nodes are enlarged.
Esophagoscopy revealed no esophageal pathology. Which of the following studies is
most appropriate in this case?
A. Computed tomography of chest and mediastinum
B. X-ray of lungs
C. Multiplanar imaging of esophagus
D. Radioisotope investigation of chest
E. Ultrasound investigation of mediastinum

168. A 19-year-old male patient complains of intense pain in the left knee joint.
Objectively: the left knee joint is enlarged, the overlying skin is hyperemic, the joint is
painful on palpation. Blood test results: RBC- 3,8 • 1012/l, Hb-122 g/l, lymphocytes -
7,4 • 109/l, platelets - 183 • 109/l. ESR- 10 mm/h. Duke bleeding time is 4 minutes, Lee-
White clotting time - 24 minutes. A-PTT is 89 sec. Rheumatoid factor is negative. What
is the most likely diagnosis?
A. Werlhof’s disease
B. Rheumatoid arthritis
C. Thrombocytopathy
D. Hemophilia, hemarthrosis
42
E. Hemorrhagic vasculitis, articular form

169. During an exam, a 22-year-old female student fainted. She grew up in a family with
many children, has a history of frequent acute respiratory infections. Objectively: the
patient has pale skin and mucous membranes, split-end hair, brittle nails. Blood test
results: RBC-2, 7 • 1012/l, Hb- 75 g/l, color index - 0,7, WBC-3,2 • 109/l, platelets - 210
• 109/l, ESR- 30 mm/h. Blood serum iron is 6 mmol/l. What is the most likely diagnosis?
A. Acute leukemia
B. B12-deficiency anemia
C. Iron-deficiency anemia
D. Vegetative-vascular dystonia
E. Aplastic anemia

170. A 43-year-old female complains of significant weakness, sore throat, occurrence of


multiple unexplained bruises on her skin. These symptoms have been present for a week,
the disease is associated with quinsy which she had some time before. Objectively: body
temperature - 38,9oC, respiratory rate -24/min, Ps- 110/min, AP- 100/65 mm Hg. The
patient has pale skin, petechial rash on the extremities, enlarged lymph nodes. Blood test
results: Hb- 80 g/l, RBC- 2,2 • 1012/l; WBC-3, 5 • 109/l; blasts - 52%; eosinophils - 2%;
stab neutrophils - 3%; segmented neutrophils - 19%; lymphocytes - 13%; monocytes -
1%; platelets - 35 • 109/l. ESR - 47 mm/h. What test is required to specify the diagnosis?
A. Protein electrophoresis
B. Immunophenotyping
C. Lymph node biopsy
D. Determination of anti-platelet antibody titer
E. Cytogenetic study

171. A patient complains of fatigue, lack of appetite, pain and burning sensation in the
tongue, numbness of the distal limbs, diarrhea. Objectively: pale skin with lemon-yellow
tint, face puffiness, brown pigmentation in the form of a "butterfly", bright red areas on
the tongue. The liver is 3 cm below the costal margin, soft. Blood count: RBCs -1, 5 •
1012/l, colour index -1,2, WBCs - 3,8109/l, thrombocytes - 180 109/l, eosinophiles -
0%, stab neutrophiles - 1%, segmented neutrophiles - 58%, lymphocytes -38%
monocytes - 3%, RBC macrocytosis. ESR - 28 mm/h. What diagnosis are these
presentations typical for?
A. Iron deficiency anemia
B. Aplastic anemia
C. Acute erythromyelosis
D. B12-deficiency anemia
E. Chronic adrenal failure

172. A 25-year-old female patient complains of marked weakness, sleepiness, blackouts,


dizziness, taste disorder. The patient has a history of menorrhagia. Objectively: the
patient has marked weakness, pale skin, cracks in the corners of her mouth, peeling nails,
systolic apical murmur. Blood test results: RBC - 3,4 • 1012/l, Hb- 70 g/l, colour index
- 0,75, platelets -140 • 109/l, WBC- 6,2 • 109/l. What is the most likely diagnosis?
A. Acute leukemia
43
B. Acute posthemorrhagic anemia
C. B12-deficiency anemia
D. Werlhof’s disease
E. Chronic posthemorrhagic anemia

173. Against the background of angina a patient has developed pain in tubular bones.
Examination revealed generalized enlargement of lymph nodes, hepatolienal syndrome,
sternalgia. In blood: RBCs - 3,6 • 1012/l, Hb-87 g/l, thrombocytes - 45 • 109/l, WBCs -
13 • 109/l, blasts - 87%, stab neutrophiles -1%, segmented neutrophiles - 7%,
lymphocytes -5%, ESR - 55 mm/h. What is the most likely diagnosis?
A. Acute leukemia
B. Erythremia
C. Chronic lymphocytic leukemia
D. Chronic myeloid leukemia
E. Multiple myeloma

174. A 34-year-old patient complains of profuse sweating at night, skin itching, weight
loss (9 kg within the last 3 months). Examination revealed malnutrition, skin pallor.
Palpation of neck and inguinal areas revealed dense elastic lymph nodes of about 1 cm
in diameter, nonmobile, non-adhering to skin. What is the most probable diagnosis?
A. Lymphogranulomatosis
B. Chronic lymphadenitis
C. Lymphosarcoma
D. Burkitt’s lymphoma
E. Cancer metastases

175. A 20-year-old patient was delivered to a surgical unit complaining of an incised


wound on his right forearm that has been bleeding for 1,5 days. Suffers from general
weakness, vertigo, cold sweat, opplotentes. Skin and visible mucous membranes are
pale. Heart rate is 110/min, BP is 100/70 mm Hg. Blood test: Hb is 100 g/l, erythrocytes
2, 5 • 1012/l. What is the cause for the patient’s general condition?
A. Posthemorrhagic anemia
B. Aplastic anemia
C. Wound infection
D. Concomitant disease
E. Acute thrombophlebitis

176. A 22-year-old woman on a reduced diet, vegetarian, attended a hospital with


complaints of distorted smell and taste perception and lesions in the angles of her mouth.
Objectively: sclera is distinctly blue. Diagnosis: iron-deficiency anemia. What clinical
syndrome is expressed primarily?
A. Anemic
B. Hemorrhagic
C. Hemolytic
D. Sideropenic
E. Myelodysplasic

44
177. A woman complains of muscle weakness and general fatigue, dyspnea, vertigo,
brittleness of her hair and nails, an urge to eat chalk. Anamnesis states uterine fibroid.
Common blood analysis: erythrocytes - 2,8 T/l, Hb- 80 g/l, color index - 0,78,
anisocytosis, poikilocytosis, serum iron - 10 mcmol/l. What diagnosis is most likely?
A. Aplastic anemia
B. Hypoplastic anemia
C. Iron-deficiency anemia
D. B12-deficient anemia
E. Autoimmune hemolytic anemia

178. A 24-year-old patient visited a doctor complaining of enlargement of his


submaxillary lymph nodes. Objectively: submaxillary, axillary and inguinal lymph
nodes are enlarged. Chest X-ray shows: enlarged lymph nodes of mediastinum. Blood
test: erythrocytes - 3,4 • 1012/l, Hb- 100 g/l, blood colour index - 0,88, platelets - 190 •
109/l, leucocytes - 7, 5 109/l, eosinophils - 8%, band neutrophils - 2%, segmented
neutrophils -67%, lymphocytes - 23%, ESR- 22 mm/hour. What test must be prescribed
to verify the cause of lymphadenopathy?
A. Puncture biopsy of lymph nodes
B. Open biopsy of lymph nodes
C. Ultrasonography of abdominal cavity
D. Mediastinum tomography
E. Sternal puncture

179. A 56-year-old patient complains of pain in the epigastrium after eating, eructation,
loss of appetite, slight loss of weight, fatigability. The patient smokes; no excessive
alcohol consumption. Objectively: pale mucosa, BP-110/70 mm Hg. The tongue is
’lacquered’.’ The abdomen is soft, sensitive in the epigastric area. Blood test:
erythrocytes - 3,0 T/l, Hb- 110 g/l, color index - 1,1; macrocytosis; leukocytes - 5,5 g/l,
ESR- 13 mm/hour. On fibrogastroduodenoscopy: atrophy of fundic mucosa. What
pathogenesis does this disorder have?
A. H.pylori persistence
B. Alimentary factor
C. Chemical factor
D. Producing antibodies to parietal cells
E. Gastropathic effect

180. A 29-year-old female patient complains of dyspnea, heaviness and chest pain on
the right, body temperature rise up to 37,2oC. The disease is associated with a chest
trauma received 4 days ago. Objectively: skin is pale and moist. Ps- 90/min., regular.
Palpation reveals a dull sound on the right, auscultation reveals significantly weakened
vesicular breathing. In blood: RbCs- 2,8 1012/l, colour index -0,9, Hb- 100 g/l, WBCs-
8,0 • 109/l, ESR- 17 mm/hour. What results of diagnostic puncture of the pleural cavity
can be expected?
A. Chylous liquid
B. Exudate
C. Transudate
D. Purulent punctate
45
E. Haemorrhagic punctate

181. A 58-year-old woman complains of spontaneous bruises, weakness, bleeding gums,


dizziness. Objectively: the mucous membranes and skin are pale with numerous
hemorrhages of various time of origin. Lymph nodes are not enlarged. Heart rate -
100/min., BP- 110/70 mm Hg. There are no changes of internal organs. Blood test
results: RBC-3,0 • 1012/l, Hb- 92 g/l, colour index - 0,9, anisocytosis, poikilocytosis,
WBC - 10 • 109/l, eosinophils - 2%, stab neutrophils - 12%, segmented neutrophils -
68%, lymphocytes - 11%, monocytes - 7%, ESR- 12 mm/hour. What index should be
determined additionally by a laboratory to make a diagnosis?
A. Platelets
B. Reticulocytes
C. Clotting time
D. Osmotic resistance of erythrocytes
E. Fibrinogen

182. A 27-year-old woman complains of bleeding gums, nasal hemorrhages, multiple


hematomas on the skin of her limbs and on the front of her torso, extreme general fatigue.
Blood test: Hb- 64 g/l, erythrocytes -2, 5 • 1012/l, reticulocytes - 16%, platelets - 30 •
109/l, ESR- 22 mm/hour. What approach would be most efficient for treatment of this
pathology?
A. Splenectomy
B. Dicynone (Etamsylate)
C. Platelet concentrate transfusion
D. Cytostatics
E. Group B vitamins

183. A 58-year-old woman undergoing chemotherapy for her oncologic disorder has
developed sore throat. Examination revealed necrotic areas on the mucosa of the pharynx
and tonsils. Many of her teeth are afflicted with caries. In blood: neutrophilic
granulocytes are practically absent against the background of leukopenia. Leukocytes
are represented mainly by lymphocytes and monocytes. What disease can be suspected
in the given case?
A. Agranulocyte tonsillitis
B. Lacunar tonsillitis
C. Pseudomembranous (Vincent’s) tonsillitis
D. Syphilitic tonsillitis
E. Diphtheria

184. A 60-year-old woman developed weakness, vertigo, rapid fatigability during the
last year. Recently there have been dyspnea and paresthesia observed. Objectively: skin
and mucous membranes are pale and icteric. Lingual papillae are smoothed out. Liver
and spleen are situated at the edge of costal arch. Blood test: Hb- 70 g/l, erythrocytes -1,
7• 1012/l, blood color index -1,2, macrocytes. What drug can be prescribed on
pathogenetic grounds?
A. Vitamin B6
B. Ascorbic acid
46
C. Vitamin B12
D. Iron preparations
E. Vitamin B1

185. A 45-year-old man has been suffering from duodenal ulcer disease for 5 years. He
complains of weakness, dizziness, dryness of the skin. Objectively: the skin and visible
mucosa are pale, chapped lips; heart rate is 100/min., BP- 100/70 mm Hg, systolic
murmur at all points on heart auscultation. All other internal organs are unchanged. Fecal
occult blood test is positive. Blood test: erythrocytes - 3,1 • 1012/l, Hb- 88 g/l, color
index - 0,7, leukocytes - 4,6 • 109/l, platelets - 350 109/l, ESR- 21 mm/hour,
anisocytosis, poikilocythemia, serum iron - 9,5 mcmol/l. What treatment tactics would
you choose?
A. Concentrated red cells transfusion
B. Intramuscular introduction of 500 mkg of cyanocobalamin
C. Corticosteroids, cytostatics
D. Iron preparations, balanced diet
E. Ascorbic acid, calcium chloride

186. A 32-year-old welder complains of weakness and fever. His illness initially
presented as tonsillitis one month earlier. On examination: body temperature - 38,9oC,
RR-24/min., HR- 100/min., BP- 100/70 mm Hg, hemorrhages on the legs, enlargement
of the lymph nodes. Complete blood count: Hb- 70 g/l, RBC- 2,2 • 1012/l, WBC- 3,0 •
108 9/l with 32% of blasts, 1% of eosinophils, 3% of band neutrophils, 36% of segments,
20% of lymphocytes, and 8% of monocytes, ESR- 47 mm/hour. What is the cause of
anemia?
A. Chronic lympholeukemia
B. Aplastic anemia
C. B12-deficient anemia
D. Acute leukemia
E. Chronic hemolytic anemia

187. A 37-year-old woman complains of headaches, nausea, vomiting, spasms. The


onset of the disease occurred the day before due to her overexposure to cold. Objectively:
fever up to 40oC; somnolence; rigid neck; Kernig’s symptom is positive on the both
sides; general hyperesthesia. Blood test: leucocytosis, increased ESR. Cerebrospinal
fluid is turbid, yellow- tinted. What changes of the cerebrospinal fluid are most likely?
A. Lymphocytic pleocytosis
B. Blood in the cerebrospinal fluid
C. Xanthochromia in the cerebrospinal fluid
D. Neutrophilic pleocytosis
E. Albuminocytological dissociation

188. A 65-year-old man was diagnosed with B12-deficient anemia and the treatment was
prescribed. A week later control blood test was performed. What would be the early
indicator of the therapy effectiveness?
A. Increased number of reticulocytes
B. Increased hemoglobin level
47
C. Megaloblastic hematopoiesis
D. Normoblastic hematopoiesis
E. Increased erythrocyte number

189. Examination of a 43-year-old man objectively revealed pallor of skin and mucous
membranes, loss of tongue papillae, transverse striation of fingernails, cracks in the
mouth corners, tachycardia. Blood test results: Hb- 90 g/l, anisocytosis, poikilocytosis.
The most likely causative factor of this condition is the inadequate intake of:
A. Zinc
B. Magnesium
C. Selene
D. Iron
E. Copper

190. A 60-year-old woman has been suffering from weakness, dizziness, and fatigue
over the last year. Recently she has also developed dyspnea, paresthesia. Objectively:
the skin and mucous membranes are pale and slightly icteric. The tongue is smooth due
to the loss of lingual papillae. Liver and spleen are located at the costal margin. Blood
count: Hb- 70 g/l, RBC-1, 71012/l, color index -1,2, macrocytes. Administer the patient
a pathogenetically justified drug:
A. Vitamin B12
B. Vitamin B6
C. Ascorbic acid
D. Iron preparations
E. Vitamin B1

191. A 48-year-old man complains of fatigue, excessive sweating, severe skin itching,
undulant fever, enlarged cervical and supraclavicular lymph nodes. Objectively:
paleness of skin and mucosa, cervical lymph nodes are mobile, dense, elastic, walnut-
sized, painless, not attached to the skin. Complete blood count: erythrocytes - 3,0 •
1012/l, Hb-100 g/l, leukocytes - 14 • 109/l, eosinophils -6%, basophils - 3%, band
neutrophils -11%, segmented neutrophils - 69%, lymphocytes -7, monocytes - 4%,
platelets - 280 • 109/l, ESR-37 mm/hour. What method should be applied to verify the
diagnosis?
A. Sternal puncture
B. Muscle biopsy
C. Chest X-ray
D. Lymph node biopsy
E. Lumbar puncture

192. A 22-year-old man suddenly developed extreme weakness, nausea, vomiting with
traces of blood. The patient is known to suffer from peptic ulcer disease of duodenum
and hemophilia A. Objectively: heart rate -102/min., BP-100/60 mm Hg. Complete
blood count: erythrocytes - 3,2 • 1012/l, Hb- 98 g/l, color index - 0,92, leukocytes - 7,4
• 109/l, platelets - 240 • 109/l, ESR-11 mm/hour. What measure would most effectively
decrease hemorrhaging in this case?
A. Platelet concentrate transfusion
48
B. Cryoprecipitate
C. Aminocapronic acid
D. Native plasma
E. Direct transfusion of donor blood

193. A 25-year-old patient has been admitted to the hospital with the following
problems: weakness, sweating, itching, weight loss, enlarged submandibular, cervical,
axillary, inguinal lymph nodes. Objectively: hepatomegaly. Lymph node biopsy
revealed giant Berezovsky-Reed- Sternberg cells, polymorphocellular granuloma
composed of lymphocytes, reticular cells, neutrophils, eosinophils, fibrous tissue, and
plasma cells. What is the most likely diagnosis?
A. Lymph node tuberculosis
B. Lymphoreticulosarcoma
C. Cancer metastases to the lymph nodes
D. Macofollicular reticulosis
E. Lymphogranulomatosis (Hodgkin’s lymphoma)

194. A 25-year-old woman complains of fatigue, dizziness, hemorrhagic rashes on the


skin. She has been presenting with these signs for a month. Blood test: erythrocytes - 1.0
1012/L, Hb- 37 g/L, colour index - 1.1, leukocytes - 1.2 109/L, platelets - 42 109/L.
What analysis would be the most advisable for diagnosis- making in this case?
A. Splenic biopsy
B. Liver biopsy
C. Coagulation studies
D. Sternal puncture (bone marrow biopsy)
E. US of the gastrointestinal tract

195. A 35-year-old man complains of rapidly increasing fatigue, palpitations, ”visual


snow”, dizziness. He has a history of peptic ulcer of the stomach. Objectively the skin is
pale. Vesicular respiration is observed in the lungs. Systolic murmur is detected over the
cardiac apex, heart rate is 100/min., BP is 100/70 mm Hg. The epigastrium is slightly
tender on palpation. Blood test: erythrocytes - 3.2 1012/L, Íb- 100 g/L, color index -
0.94. What type of anemia is the most likely present in this case?
A. Sideroblastic anemia
B. Iron-deficiency anemia
C. Posthemorrhagic anemia
D. Hemolytic anemia
E. Hypoplastic anemia

196. A 57-year-old woman complains of weakness, dyspnea, loss of appetite, and liquid
feces. She has been suffering from this condition for 2 years. Objectively she presents
with pale skin, subicteric sclera, and bright-red fissured tongue. Lymph nodes are not
enlarged. Pulse - 100/min. BP- 105/70 mm Hg. Liver +3 cm, the spleen cannot be
palpated. Blood test: erythrocytes - 1.2 1012/L, Íb- 56 g/L, color index - 1.4, macrocytes,
leukocytes 2, 5 109/L, eosinophils - 1%, juvenile - 1%, metamyelocytes - 1%, band
neutrophils - 8%, segmented neutrophils - 47%, lymphocytes - 38%, monocytes - 4%,
reticulocytes - 0.1%, platelets - 100 109/L, ESR- 30 mm/hour, indirect bilirubin - 26
49
mmol/L. What changes can be expected in the bone marrow puncture material?
A. Presence of blast cells
B. Prevalence of lymphoid tissue
C. Prevalence of megaloblasts
D. Increased number of sideroblasts
E. Erythroid hyperplasia

197. A 28-year-old woman complains of skin hemorrhages after minor traumas and
spontaneous appearance of hemorrhages on the front of her torso and extremities. On
examination: the skin is variegated (old and new hemorrhages), bleeding gums. Blood
platelets - 20 109/L; in the bone marrow there is increased number of megakaryocytes
and no platelet production. Treatment with steroid hormones was effective. What is the
likely diagnosis?
A. Hemophilia
B. Rendu-Osler-Weber disease (Hereditary hemorrhagic telangiectasia)
C. Disseminated intravascular coagulation
D. Idiopathic thrombocytopenic purpura
E. Acute vascular purpura

198. A 60-year-old woman started feeling weakness, vertigo, rapid fatigability during
the last year. Recently she has developed dyspnea and paresthesia observed. Objectively:
skin and mucous membranes are pale and icteric. Lingual papillae are smoothed out.
Liver and spleen are at the edge of costal arch. Blood test: Hb- 70 g/L, erythrocytes - 1.7
1012/L, blood color index - 1.2, macrocytes. What drug can be prescribed on
pathogenetic grounds?
A. Vitamin B12
B. Vitamin B6
C. Ascorbic acid
D. Iron preparations
E. Vitamin B1

199. A 38-year-old patient has been delivered by an ambulance to a surgical department


with complaints of general weakness, indisposition, black stool. On examination the
patient is pale, there are dotted hemorrhages on the skin of his torso and extremities. On
digital investigation there are black feces on the glove. Blood test: Hb- 108 g/L,
thrombocytopenia. Anamnesis states that similar condition was observed 1 year ago.
Make the diagnosis:
A. Thrombocytopenic purpura
B. Hemophilia
C. Bleeding from an ulcer
D. Rectal tumor
E. Nonspecific ulcerative colitis

200. A 35-year-old man complains of persisting enlargement of his peripheral lymph


nodes that cause him no discomfort. The case history states that the first lymph nodes to
enlarge were cervical, supraclavicular, and axillary; new groups of lymph nodes
emerged. Objectively the lymph nodes are soft and elastic on palpation, enlarged,
50
painless, not fixed to the surrounding tissue. What examination method would be the
most informative for early diagnostics of this disease?
A. Magnetic resonance tomography
B. Radioisotope scanning of the skeleton
C. Ultrasound
D. Needle biopsy
E. X-ray

51
Standards of true answers
№ of True № of True № of True № of True
task answer task answer task answer task answer

1 А 51 Е 101 В 151 А
2 Е 52 С 102 В 152 Е
3 Е 53 А 103 Е 153 D
4 С 54 С 104 С 154 A
5 D 55 Е 105 С 155 C
6 D 56 Е 106 D 156 B
7 С 57 С 107 В 157 E
8 С 58 А 108 Е 158 B
9 В 59 В 109 D 159 A
10 D 60 А 110 А 160 A
11 В 61 D 111 С 161 D
12 А 62 С 112 D 162 D
13 D 63 В 113 С 163 C
14 А 64 А 114 В 164 B
15 В 65 А 115 А 165 E
16 А 66 С 116 Е 166 A
17 А 67 В 117 А 167 A
18 С 68 С 118 С 168 D
19 В 69 А 119 D 169 C
20 В 70 С 120 С 170 B
21 А 71 В 121 Е 171 D
22 В 72 Е 122 С 172 E
23 А 73 А 123 С 173 A
24 А 74 С 124 Е 174 A
25 D 75 D 125 Е 175 A
26 В 76 В 126 D 176 D
27 С 77 А 127 Е 177 C
28 А 78 D 128 Е 178 B
29 D 79 D 129 А 179 D
30 А 80 Е 130 Е 180 E
31 Е 81 В 131 С 181 A
32 С 82 D 132 А 182 A
33 D 83 А 133 В 183 A
34 D 84 С 134 В 184 C
35 С 85 D 135 Е 185 D
36 D 86 Е 136 С 186 D
37 С 87 А 137 D 187 D
38 В 88 Е 138 А 188 A
39 А 89 Е 139 Е 189 D
40 В 90 Е 140 D 190 A
41 D 91 С 141 А 191 D
42 В 92 Е 142 А 192 B
43 С 93 А 143 А 193 E
44 В 94 В 144 А 194 D
45 В 95 А 145 А 195 C
46 В 96 D 146 А 196 C
47 С 97 Е 147 С 197 D
48 С 98 В 148 А 198 A
49 С 99 В 149 С 199 A
50 D 100 А 150 А 200 D
52
BASIC LITERATURE
1. Передерій В.Г., Ткач С.М. Основи внутрішньої медицини. В 3 т. Том 1. «Нова
книга», 2009. – 640 с.
2. Передерій В.Г., Ткач С.М. Основи внутрішньої медицини. В 3 т. Том 2. «Нова
книга», 2009. - 784 с.
3. Передерій В.Г., Ткач С.М. Основи внутрішньої медицини. В 3 т. Том 3. «Нова
книга», 2010. - 1006 с.
4. Лекції з гематології / П.М. Перехрестенко, Л.М. Ісакова, Н.М. Третяк, Д.А.
Лисенко, С.В. Бондарчук. – К.: Нора-прінт, 2005. – 128 с.
5. Внутрішня медицина: Порадник лікарю загальної практики: навчальний
посібник. / А.С. Свінціцький, О.О. Абрагамович, П.М. Боднар та ін.; За ред. проф. А.С.
Свінціцького. – ВСВ «Медицина», 2014. – 1272 с. + 16с. кольоров. вкл.
6. Дзяк Г.В., Хомазюк Т.А., Нетяженко В.З. Основи діагностики внутрішніх
хвороб (довідник). – Дн-ск, видавництво ДДМА, 2001.
7. Невідкладна медична допомога: Навч. посібник / К.М.Амосова,
Б.Г.Безродний, О.А.Бур’янов, Б.М.Венцківський та ін.; За ред. Ф.С.Глумчера,
В.Ф.Москаленка. – К.: Медицина, 2006. – 632 с.
8. Практикум з внутрішньої медицини: навч. пос. / К.М. Амосова, Л.Ф.
Конопльова, Л.Л. Сидорова, Г.В. Мостбауер та ін. – К.: Український медичний вісник,
2012 р. – 416 с.
9. Доценко С.Я., Шеховцева Т.Г., Сичов Р.О.та інш. Syndrome of fever of unknown
origin in clinics of internal diseases. (Синдром лихоманки невизначеного ґенезу в клініці
внутрішніх хвороб). Навчально-методичний посібник. Запоріжжя, 2018.- 153 с.

OTHER RECOMMENDED LITERATURE


1. Швец Н.И., Пидаев А.В., Бенца Т.М., Федорова О.А., Миронец В.И..
Неотложные состояния в клинике внутренней медицины: учебное пособие. – Киев,
2006. – 752 с.
2. The Merck Manual of Diagnosis and Therapy, 18th edition /Editors: M.H. Beers,
R.S. Porter, Th.V. Jones //Merck Sharp & Dohme Corp., 2006. –2991 p.
3. Бадюк М.І., Левченко Ф.М., Токарчук В.П., Солярик В.В. та ін. Організація
медичного забезпечення військ: Підручник для студентів вищих медичних закладів
освіти України ІІІ-ІV рівнів акредитації / За ред. проф. Паська В.В. – К.: «МП Леся»,
2005. – 425 с.
4.Анестезіологія та інтенсивна терапія: підручник / Ф.С. Глумчер, Л.П. Чепкий,
Л.В. Усенко та ін.; за ред. Ф.С. Глумчера. – К.: ВСВ «Медицина», 2010. – 336 с.

53
"Step-1" Test Questions on Pathophysiology
(*The correct answer in each question is variant A)
Module 1
1. 27-year-old woman has used penicillin containing eye drops. In a few minutes
itching, skin burning, lips and eyelids edema, whistling cough, decreasing BP
appeared. What antibodies can lead to this allergic reaction?
A. IgE and IgG
B. IgM and IgG
C. IgA and IgM
D. IgM and IgD
E. IgG and IgD
2. An individual is characterized by rounded face, broad forehead, a mongolian
type of eyelid fold, flattened nasal bridge, permanently open mouth, projecting
lower lip, protruding tongue, short neck, flat hands, and stubby fingers. What
diagnosis can be put to the patient?
A. Down’s syndrome
B. Klinefelter’s syndrome
C. Alkaptonuria
D. Supermales
E. Turner’s syndrome
3. A patient visited a dentist with complaints of redness and edema of his mouth
mucous membrane in a month after dental prosthesis. The patient was diagnosed
with allergic stomatitis. What type of allergic reaction by Gell and Cumbs
underlies this disease?
A. Delayed type hypersensitivity
B. Cytotoxic
C. Immunocomplex
D. Anaphylactic
E. Stimulating
4. A patient with clinical signs of immunodeficiency has unchanged number and
functional activity of T and B lymphocytes. Dysfunction’s defect of antigen-
presentation to the immunocompetent cells was found during investigation on the
molecule level. Defect of what cells is the most probable here?
A. Macrophages, monocytes
B. Т-lymphocytes, В-lymphocytes
C. NK-cells
D. Fibroblasts, Т-lymphocytes, В-lymphocytes
E. 0-lymphocytes
5. The preventive radioprotector was given to a worker of a nuclear power station.
What mechanism from the below mentioned is considered to be the main
mechanism of radioprotection?
A. Inhibition of free radicals formation
B. Prevention of tissue’s hypoxia
C. Activation of oxidation reactions
D. Increasing of tissue blood supply
E. Increasing of respiration
6. At the aboratory experiment the eukocyte culture was mixed with staphylococci.
Neutrophile leukocytes engulfed and digested bacterial cells. This processes are
termed:
A. Phagocytosis
B. Pinocytosis
C. Diffusion
D. Facilitated diffusion
E. Osmosis
7. A damage of the atomic power plant reactor resulted in the runout of
radioelements. People in the superstandard radiation zone were radiated with
approximately 250-300 r. and were immediately hospitalized. What changes in the
blood count would be typical?
A. Lymphopenia
B. Leukopenia
C. Anemia
D. Thrombopenia
E. Neutropenia
8. Part of the DNA chain turned about 180 degrees due to gamma radiation. What
type of mutation took place in the DNA chain?
A. Inversion
B. Deletion
C. Doubling
D. Translocation
E. Replication
9. Analysis of blood serum of a patient revealed increase of alanine
aminotransferase and aspartate aminotransferase level. What cytological changes
can cause such a situation?
A. Cellular breakdown
B. Disturbed function of energy supply of cells
C. Disorder of enzyme systems of cells
D. Disturbance of genetic apparatus of cells
E. Disturbance of cellular interrelations
10. In some regions of South Africa there is a spread sickle-shaped cell anemia, in
which erythrocytes have shape of a sickle as a result of substitution of glutamin by
valine in the hemoglobin molecule. What is the cause of this disease?
A. Gene mutation
B. Disturbance of mechanisms of genetic information realization
C. Crossingover
D. Genomic mutations
E. Transduction
11. Having helped to eliminate consequences of a failure at a nuclear power plant,
a worker got an
irradiation doze of 500 roentgen. He complains of headache, nausea, dizziness.
What changes in
leukocytes quantity can be expected 10 hours after irradiation?
A. Neutrophilic leukocytosis
B. Lymphocytosis
C. Leukopenia
D. Agranulocytosis
E. Leukemia
12. Autopsy of a newborn boy revealed polydactylia, microcephalia, cheiloschisis
and uranoschisis
as well as hypertrophy of parenchimatous organs. These defects correspond with
the description of
Patau’s syndrome. What is the most probable cause of this pathology?
A. Trisomy of the 13th chromosome
B. Trisomy of the 18th chromosome
C. Trisomy of the 21st chromosome
D. Nondisjunction of sex chromosomes
E. Partial monosomy
13. A woman has been applying a new cosmetic preparation for a week that
resulted in eye-lid
inflammation accompanied by hyperemia, infiltration and painfulness. What type
of allergic
reaction was developed?
A. IV
B. I
C. II
D. III
E. V
14. In course of histidine catabolism a biogenic amin is formed that has powerful
vasodilatating
effect. Name it:
A. Histamine
B. Serotonin
C. Dioxyphenylalanine
D. Noradrenalin
E. Dopamine
15. An experimental animal was first sensibilized whereupon an antigen dose was
introduced
subcutaneously. This injection resulted in the development of a fibrinous
inflammation with
alteration of vessel walls, basal substance and fibrous structures of connective
tissue in form of
mucoid and fibrinoid swelling and necrosis. What immunological reaction took
place?
A. Immediate hypersensitivity
B. Delayed-type hypersensitivity
C. Reaction of transplantation immunity
D. Normergic reaction
E. Granulomatosis
16. A 38 year old patient suffers from rheumatism in its active phase. What
laboratory
characteristic of blood serum is of diagnostic importance in case of this pathology?
A. C-reactive protein
B. Uric acid
C. Urea
D. Creatinine
E. Transferrin
17. Continuous taking of some drugs foregoing the pregnancy increase the risk of
giving birth to a
child with genetic defects. What is this effect called?
A. Mutagenic effect
B. Embryotoxic effect
C. Teratogenic effect
D. Fetotoxical effect
E. Blastomogenic effect
18. There are several groups of molecular mechanisms playing important part in
pathogenesis of
insult to cells which contributes to the pathology development. What processes are
stimulated by
proteinic damage mechanisms?
A. Enzyme inhibition
B. Lipid peroxidation
C. Phospholipase activation
D. Osmotic membrane distension
E. Acidosis
19. A child was born with cleft palate. Examination revealed aorta defects and
reduced number of
T-lymphocytes in blood. What immunodeficient syndrome is it?
A. DiGeorge
B.Wiskott-Aldrich
C. Chediak-Higashi
D. Louis-Bar
E. Swiss-type
20. Examination of a 12 year old boy with developmental lag revealed
achondroplasia:
disproportional constitution with evident shortening of upper and lower limbs as a
result of growth
disorder of epiphyseal cartilages of long tubal bones. This disease is:
A. Inherited, dominant
B. Inherited, recessive
C. Inherited, sex-linked
D. Congenital
E. Acquired
21. A 30 year old woman has applied a lipstick with a fluorescent substance for a
long time. Then
she got a limited erythema and slight peeling on her lip border, later there appeared
transversal
striae and cracks. Special methods of microscopic examination of the affected area
helped to reveal
sensibilized lymphocytes and macrophages in the connective tissue; cytolysis.
What type of
immunological hypersensitivity was developed?
A. IV type (cellular cytotoxicity)
B. I type (reaginic)
C. II type (antibody cytotoxicity)
D. III type (immune complex cytotoxicity)
E. Granulomatosis
22. As a result of prophylactic medical examination a 7 year old boy was
diagnosed with LeschNyhan syndrome (only boys fall ill with it). The boy’s
parents are healthy but his grandfather by his
mother’s side suffers from the same disease. What type of disease inheritance is it?
A. Recessive, sex-linked
B. Dominant, sex-linked
C. Autosomal recessive
D. Autosomal dominant
E. Semidominance
23. A man suffering from a hereditary disease married a healthy woman. They got
5 children, three
girls and two boys. All the girls inherited their father’s disease. What is the type of
the disease
inheritance?
A. Dominant, X-linked
B. Autosomal recessive
C. Asutosomal dominant
D. Y -linked
E. Recessive, X-linked
24. A female patient suffering from bronchial asthma had got a viral infection that
provoked status
asthmaticus with fatal outcome. Histological examination of lungs revealed spasm
and edema of
bronchioles, apparent infiltration of their walls with lymphocytes, eosinophils and
other leukocytes;
labrocyte degranulation. What mechanism of hypersensitivity underlies the
described alterations?
A. Reagin reaction
B. Inflammatory
C. Autoimmune
D. Immune complex
E. Immune cytolysis
25. Medical examination at the military registration and enlistment office revealed
that a 15-yearold boy was high, with eunuchoid body proportions, gynecomastia,
female pattern of pubic hair
distribution. The boy had also fat deposits on the thighs, no facial hair, high voice,
subnormal
intelligence quotient. Which karyotype corresponds with this disease?
A. 47, XXY
B. 45, XO
C. 46, XX
D. 46, XY
E. 47, XXX
26. During a prophylactic medical examination a 7-year-old boy was diagnosed
with daltonism.
His parents are healthy and have normal colour vision, but his grandfather on his
mother’s side has
the same abnormality. What is the type of the abnormality inheritance?
A. Recessive, sex-linked
B. Dominant, sex-linked
C. Semidominance
D. Autosomal recessive
E. Autosomal dominant
27. A 10-year-old child had the mantoux tuberculin test administered. 48 hours
later a papule up to
8 mm in diameter appeared on the site of the injection. What type of
hypersensitivity reaction
developed after the tuberculin injection?
A. Type IV hypersensitivity reaction
B. Arthus phenomenon
C. Seroreaction
D. Atopic reaction
E. Type II hypersensitivity reaction
28. After the prior sensibilization an experimental animal was given a
subcutaneous injection of an
antigen. The place of injection exhibited a fibrinous inflammation with alteration
of the vessel
walls, basal substance and fibrous structures of the connective tissue in form of
mucoid and
fibrinoid swelling and necrosis. What immunological reaction is it?
A. Immediate hypersensitivity
B. Delayed-type hypersensitivity
C. Reaction of transplantation immunity
D. Normergic reaction
E. Granulomatosis
29. Examination of a patient revealed autoimmune haemolytic anaemia (cytotoxic
type). What
substances act as antigens in the II-type allergic reactions?
A. Modified receptors of cell membranes
B. Antibiotics
C. Hormones
D. Serum proteins
E. Inflammation modulators
30. A 28-year-old female patient consulted a gynecologist about sterility.
Examination revealed
underdeveloped ovaries and uterus, irregular menstrual cycle. Analysis of the sex
chromatin
revealed 2 Barr’s bodies in most somatic cells. What chromosome disease is most
likely?
A. Triple X syndrome
B. Edwards’ syndrome
C. Patau’s syndrome
D. Klinefelter’s syndrome
E. Turner’s syndrome
31. 48 hours after tuberculine test (Mantoux test) a child had a papule 10 mm in
diameter on the
spot of tuberculine injection. What hypersensitivity mechanism underlies these
changes?
A. Cellular cytotoxicity
B. Anaphylaxy
C. Antibody-dependent cytotoxicity
D. Immunocomplex cytotoxicity
E. Granulomatosis
32. A couple had a child with Down’s disease. Mother is 42 years old. This disease
is most
probably caused by the following impairment of prenatal development:
A. Gametopathy
B. Blastopathy
C. Embryopathy
D. Non-specific fetopathy
E. Specific fetopathy
33. Examination of a child who frequently suffers from infectious diseases
revealed that IgG
concentration in blood serum was 10 times less than normal, IgA and IgM
concentration was also
significantly reduced. Analysis showed also lack of B-lymphocytes and
plasmocytes. What disease
are these symptoms typical for?
A. Bruton’s disease
B. Swiss-type agammaglobulinemia
C. Dysimmunoglobulinemia
D. Louis-Bar syndrome
E. Di George syndrome
34. A patient with skin mycosis has disorder of cellular immunity. The most
typical characteristic
of it is reduction of the following index:
A. T-lymphocytes
B. Immunoglobulin G
C. Immunoglobulin E
D. B-lymphocytes
E. Plasmocytes
35. A female patient underwent liver transplantation. 1,5 month after it her
condition became worse
because of reaction of transplant rejection. What factor of immune system plays
the leading part in
this reaction?
A. T-killers
B. Interleukin-1
C. Natural killers
D. B-lymphocytes
E. T-helpers
36. An alcoholic woman has born a girl with mental and physical developmental
lag. Doctors
diagnosed the girl with fetal alcohol syndrome. What effect is the cause of the
girl’s state?
A. Teratogenic
B. Mutagenic
C. Malignization
D. Carcinogenic
E. Mechanic
37. A 50 year old man who was referred to the hospital for treatment of cervical
lymphadenitis
underwent test for individual sensitivity to penicillin. 30 seconds after he went hot
all over, AP
dropped down to 0 mm Hg that led to cardiac arrest. Resuscitation was
unsuccessful. Autopsy
results: acute venous plethora of internal organs; histological examination of skin
(from the site of
injection) revealed degranulation of mast cells (tissue basophils). Degranulation
was also revealed
in myocardium and lungs. What type of hypersensitivity reaction is it?
A. Anaphylactic
B. Delayed-type hypersensitivity
C. Complement-mediated cytotoxic
D. Immunocomplex-mediated
E. –
38. Cytogenetic examination of a patient with reproductive dysfunction revealed
normal karyotype
46ХY in some cells, but most cells have karyotype of Klinefelter’s syndrome - 47
ХХY. Such cell
heterogeneity is called:
A. Mosaicism
B. Inversion
C. Transposition
D. Duplication
E. Monomorphism
39. A 10-year-old child had the mantoux tuberculin test administered. 48 hours
later a papule up to
8 mm in diameter appeared on the site of the injection. What type of
hypersensitivity reaction
developed after the tuberculin injection?
A. Type IV hypersensitivity reaction
B. Arthus phenomenon
C. Seroreaction
D. Atopic reaction
E. Type II hypersensitivity reaction
40. Examination of a pregnant woman having Rh-negative blood revealed high
level of
antierythrocytic antibodies. For its reduction she was implanted with her husband’s
Rh-positive skin
graft. The graft was rejected in two weeks. Its microscopic examination revealed
circulatory
disturbance, edema and cellular infiltration with lymphocytes, neutrophils and
macrophages
predominance. What is the most likely pathology?
A. Graft immunity
B. Immediate hypersensitivity
C. Delayed-type hypersensitivity
D. Granulomatous inflammation
E. Interstitial inflammation
41. A 38-year-old male patient has been ill with systemic lupus erythematosus for
three years. He
was diagnosed with diffuse renal affection accompanied by massive edemata and
expressive
proteinuria. What is the most likely cause of proteinuria development?
A. Autoimmune renal affection
B. Aseptic renal affection
C. Ischemic renal affection
D. Urinary bladder inflammation
E. Urinary tracts inflammation
42. Blood plasma of a healthy man contains several dozens of proteins. During an
illness new
proteins can originate, namely the protein of "acute phase". Select such protein
from the listed
below:
A. C-reactive protein
B. Prothrombin
C. Fibrinogen
D. G immunoglobulin
E. A immunoglobulin
43. During surgical manipulations a patient has been given novocaine injection for
anesthesia. 10
minutes later the patient developed paleness, dyspnea, hypotension. What type of
allergic reaction is
it?
A. Anaphylactic immune reaction
B. Cellulotoxic immune reaction
C. Aggregate immune reaction
D. Stimulating immune reaction
E. Cell-mediated immune reaction
44. A child with a history of frequent angine and pharyngitis has been diagnosed
with
lymphadenopathy and splenomegaly. His appearance is characterised by pastosity
and paleness,
muscular tissue is poorly developed. Lymphocytosis is present. What kind of
diathesis is it?
A. Lymphohypoplastic diathesis
B. Exudative diathesis
C. Gouty diathesis
D. Asthenic diathesis
E. Hemorrhagic diathesis
45. After an immunoassay a child was diagnosed with immunodeficiency of
humoral immunity.
What is the reason for the primary immunodeficiency development in the child?
A. Hereditary abnormality of immune system
B. Embryonal development abnormalities
C. Pathometabolism in mother’s organism
D. Immune responsiveness and resistance disorders
E. Toxic damage of B-lymphocytes
46. A 22-year-old women consulted a gynecologist about sterility. Examination
revealed
underdeveloped ovaries and uterus, irregular menstrual cycle. Analysis of the sex
chromatin
revealed 2 Barr’s bodies in most somatic cells. What chromosome disease is most
likely?
A. Triple X syndrome
B. Edwards’ syndrome
C. Patau’s syndrome
D. Klinefelter’s syndrome
E. Turner’s syndrome
47. Sex chromosomes of a woman didn’t separate and move to the opposite poles
of a cell during
gametogenesis (meiosis). The ovum was impregnated with a normal spermatozoon.
Which
chromosomal disease can be found in her child?
A. Turner’s syndrome
B. Down’s syndrome
C. Patau’s syndrome
D. Edwards’ syndrome
E. Cat cry syndrome
48. A man suffering from a hereditary disease married a healthy woman. They got
5 children, three
girls and two boys. All the girls inherited their father’s disease. What is the type of
the disease
inheritance?
A. Dominant, X-linked
B. Autosomal recessive
C. Asutosomal dominant
D. Y-linked
E. Recessive, X-linked
49. A couple has a son with haemophilia. The parents are healthy but the maternal
grandfather also
has haemophilia. Specify the type of inheritance:
A. Recessive sex-linked
B. Recessive autosomal
C. Dominant sex-linked
D. Semidominance
E. Autosomal dominant
50. 46 chromosomes were revealed on karyotype examination of the 5 year old
girl. One of the
15th pair of chromosomes is longer than usual due to connected chromosome from
the 21 pair.
What type of mutation does this girl have?
A. Translocation
B. Deletion
C. Inversion
D. Insufficiency
E. Duplication
51. Having helped to eliminate consequences of a failure at a nuclear power plant,
a worker got an
irradiation doze of 500 roentgen. He complains of headache, nausea, dizziness.
What changes in
leukocytes quantity can be expected 10 hours after irradiation?
A. Neutrophilic leukocytosis
B. Lymphocytosis
C. Leukopenia
D. Agranulocytosis
E. Leukemia
52. A man with a long-term history of bronchial asthma died from asphyxia.
Histological
examination of his lungs revealed that the lumens of bronchioles and minor
bronchi contained a lot
of mucus with some eosinophils. There was also sclerosis of interalveolar septa,
dilatation of
alveole lumens. What mechanism accounts for the development of hypersensitivity
reaction?
A. Reagine reaction
B. Cytotoxic reaction
C. Immune complex reaction
D. Lymphocyte-mediated cytolysis
E. Granulomatosis
53. A 10 year old child had the mantoux tuberculin test administered. 48 hours
later a papule up to
8 mm in diameter appeared on the site of the injection. What type of
hypersensitivity reaction
developed after the tuberculin injection?
A. Type IV hypersensitivity reaction
B. Arthus phenomenon
C. Seroreaction
D. Atopic reaction
E. Type II hypersensitivity reaction
54. According to the phenotypic diagnosis a female patient has been provisionally
diagnosed with
X-chromosome polysomia. This diagnosis can be confirmed by a cytogenetic
method. What
karyotype will allow to confirm the diagnosis?
A. 47(ХХХ)
B. 48(XXXY)
C. 48(XXYY)
D. 47(XXY)
E. 46(XX)
55. A patient has been diagnosed with acute glomerulonephritis that developed
after he had had
streptococcal infection. It is most likely that the affection of basal glomerular
membrane is caused
by an allergic reaction of the following type:
A. Immune complex
B. Anaphylactic
C. Cytotoxic
D. Delayed
E. Stimulating
56. Examination of patients with periodontitis revealed the interdependence
between the rate of
affection of periodontal tissues and the amount of lysozymes in saliva and gingival
liquid. These
results can be obtained during studying the following protection system of an
organism:
A. Non-specific resistance
B. Humoral immunity
C. Cellular immunity
D. Autoresponsiveness
E. Tolerance
57. A 33 year old male died from disseminated tuberculosis. On autopsy the
symptoms of
tuberculosis were confirmed by both microscopical and histological analyses. All
the affected
organs had epithelioid cell granulomas with caseous necrosis in the centre. What
kind of
hypersensitivity reaction underlies the process of granuloma development?
A. Delayed
B. Antibody-dependent cytotoxicity
C. Complement-dependent cytotoxicity
D. Anaphylactic
E. Immune complex
58. A child with suspected tuberculosis was given Mantoux test. After 24 hours the
site of the
allergen injection got swollen, hyperemic and painful. What are the main
components that
determine such response of the body?
A. Mononuclear cells, T-lymphocytes and lymphokines
B. Granulocytes, T-lymphocytes and IgG
C. Plasma cells, T-lymphocytes and lymphokines
D. B-lymphocytes, IgM
E. Macrophages, B-lymphocytes and monocytes
59. Examination of an 15-year-old girl revealed the following features: hypoplasia
of the ovaries,
broad shoulders, narrow pelvis, shortening of the lower extremities, "sphinx neck".
Mental
development is normal. The girl was diagnosed with Turner’s syndrome. What
kind of chromosome
abnormality is it?
A. Monosomy X
B. Trisomy X
C. Trisomy 13
D. Trisomy 18
E. Nullisomy X
60. A 33-year-old male patient has been referred by an andrologist for the genetic
counselling for
the deviations of physical and mental development. Objectively: the patient is tall,
has asthenic
constitution, gynecomastia, mental retardation. Microscopy of the oral mucosa
cells revealed sex
chromatin (single Barr body) in 30% of cells. What is the most likely diagnosis?
A. Klinefelter syndrome
B. DiGeorge syndrome
C. Down syndrome
D. Recklinghausen’s disease
E. Cushing pituitary basophilism
61. Amniocentesis revealed two sex chromatin bodies (Barr bodies) in each cell of
the sample.
What disease is this character typical for?
A. Trisomy X
B. Klinefelter syndrome
C. Turner’s syndrome
D. Down’s syndrome
E. Patau syndrome
62. A male patient has been diagnosed with acute post-streptococcal
glomerulonephritis. It is most
likely that the lesion of the basement membrane of renal corpuscles was caused by
the following
allergic reaction:
A. Immune complex
B. Anaphylactic
C. Cytotoxic
D. Delayed
E. Stimulating
63. A child cut his leg with a piece of glass while playing and was brought to the
clinic for the
injection of tetanus toxoid. In order to prevent the development of anaphylactic
shock the serum
was administered by Bezredka’s method. What mechanism underlies this method
of desensitization
of the body?
A. Binding of IgE fixed to the mast cells
B. Blocking the mediator synthesis in the mast cells
C. Stimulation of immune tolerance to the antigen
D. Stimulation of the synthesis of antigenspecific IgG
E. Binding of IgE receptors to the mast cells
64. An experiment proved that UV-irradiated skin cells of patients with xeroderma
pigmentosum
restore the native structure of DNA slower than the cells of healthy people due to
the defect in
repair enzyme. What enzyme takes part in this process?
A. Endonuclease
B. RNA ligase
C. Primase
D. DNA polymerase
E. DNA gyrase
65. What condition may develop 15-30 minutes after re-administration of the
antigen as a result of
the increased level of antibodies, mainly IgE, that are adsorbed on the surface of
target cells,
namely tissue basophils (mast cells) and blood basophils?
A. Anaphylaxis
B. Antibody-dependent cytotoxicity
C. Delayed-type hypersensitivity
D. Immune complex hyperresponsiveness
E. Serum sickness
66. 10 days after having quinsy caused by beta-hemolytic streptococcus a 4-year-
old child
exhibited symptoms of glomerulonephritis. What mechanism of glomerular lesion
is most likely in
this case?
A. Immunocomplex
B. Cellular cytotoxicity
C. Anaphylaxis
D. Atopy
E. Antibody-dependent cell-mediated cytolysis
67. A 22-year-old woman ate some seafood. 5 hours later the trunk and the distal
parts of limbs got
covered with small itchy papules which were partially fused together. After one
day, the rash
disappeared spontaneously. Specify the hypersensitivity mechanism underlying
these changes:
A. Atopy (local anaphylaxis)
B. Systemic anaphylaxis
C. Cellular cytotoxicity
D. Immune complex hypersensitivity
E. Antibody-dependent cell-mediated cytolysis
68. A 3-year-old child had eaten some strawberries. Soon he developed a rash and
itching. What
was found in the child’s leukogram?
A. Eosinophilia
B. Hypolymphemia
C. Neutrophilic leukocytosis
D. Monocytosis
E. Lymphocytosis
69. A boy referred to a genetics clinic was found to have 1 drumstick in blood
neutrophils. The boy
is likely to have the following syndrome:
A. Klinefelter’s
B. Down’s
C. Turner’s
D. Edwards’
E. Trisomy X
70. The development of both immune and allergic reactions is based upon the same
mechanisms of
the immune system response to an antigen. What is the main difference between
the immune and
allergic reactions?
A. Development of tissue lesion
B. Amount of released antigen
C. Antigen structure
D. Routes by which antigens are delivered into the body
E. Hereditary predisposition
71. Those organisms which in the process of evolution failed to develop protection
from H2O2 can
exist only in anaerobic conditions. Which of the following enzymes can break
hydrogen peroxide
down?
A. Peroxidase and catalase
B. Oxygenase and hydroxylase
C. Cytochrome oxidase, cytochrome B5
D. Oxygenase and catalase
E. Flavin-dependent oxidase
72. Parents of 5-year-old child report him o have frequent colds that develop into
pneumonias,
presence of purulent rashes on the skin. Laboratory tests have revealed the
following: absence of
immunoglobulins of any type, and naked cells are absent from the lymph nodes
punctate. What kind
of immune disorder is it?
A. X-linked hypogammaglobulinemia (Bruton type agammaglobulinemia)
B. Autosomal recessive agammaglobulinaemia (Swiss type)
C. Hypoplastic anemia
D. Agranulocytosis
E. Louis-Barr syndrome
73. During blood transfusion a patient has developed intravascular erythrocyte
hemolysis. What
kind of hypersensitivity does the patient have?
A. II type (antibody-dependent)
B. I type (anaphylactic)
C. III type (immune complex)
D. IV type (cellular cytotoxicity)
E. IV type (granulomatosis)
74. In the course of puncture biopsy of transplanted kidney the following has been
revealed: diffuse
infiltration of stroma by lymphocytes and plasmocytes and necrotic arteritis. What
pathological
process has developed in the transplant?
A. Immune rejection
B. Ischemic kidney failure
C. Glomerulonephritis
D. Tubular necrosis
E. Pyelonephritis
75. A 2-year-old boy is diagnosed with Down syndrome. What chromosomal
changes may be the
cause of this disease?
A. Trisomy 21
B. Trisomy 13
C. Trisomy X
D. Trisomy 18
E. Monosomy X
76. A 30-year-old patient has dyspnea fits, mostly at night. He has been diagnosed
with bronchial
asthma. What type of allergic reaction according to the Gell-Coombs classification
is most likely in
this case?
A. Anaphylactic
B. Cytotoxic
C. Stimulating
D. Immune complex
E. Delayed-type hypersensitivity
77. Sex chromatin was detected during examination of a man’s buccal epithelium.
It is
characteristic of the following chromosome disease:
A. Klinefelter’s syndrome
B. Down’s disease
C. Turner’s syndrome
D. Triple X syndrome
E. Hypophosphatemic rickets
78. A 12-year-old child has developed nephritic syndrome (proteinuria, hematuria,
cylindruria) 2
weeks after the case of tonsillitis, which is a sign of affected glomerular basement
membrane in the
kidneys. What mechanism is the most likely to cause the basement membrane
damage?
A. Immune complex
B. Granulomatous
C. Antibody-mediated
D. Reaginic
E. Cytotoxic
79. Several minutes after a dentist administered novocaine for local anaesthesia of
a patient’s tooth,
the following symptoms sharply developed in the patient: fatigue, skin itching.
Objectively the
following can be observed: skin hyperemia, tachycardia, BP dropped down to
70/40 mm Hg. What
kind of allergic reaction is this pathology?
A. Anaphylactic
B. Cytotoxic
C. Stimulating
D. Cell-mediated immune reaction
E. Immune complex
80. A 3-year-old boy is diagnosed with Down syndrome. What chromosomal
changes may be the
cause of this disease?
A. Trisomy 21
B. Trisomy 13
C. Trisomy X
D. Trisomy 18
E. Monosomy X
81. The process of metabolism in the human body produces active forms of
oxygen, including
superoxide anion radical O2-
. This anion is inactivated by the following enzyme:
A. Superoxide dismutase
B. Catalase
C. Peroxidase
D. Glutathione peroxidase
E. Glutathione reductase
82. Sex chromatin was detected during examination of a man’s buccal epithelium.
It is
characteristic of the following chromosome disease:
A. Klinefelter’s syndrome
B. Down’s disease
C. Turner’s syndrome
D. Triple X syndrome
E. Hypophosphatemic rickets
83. A 9-year-old child developed nephritic syndrome (proteinuria, hematuria,
cylindruria) 2 weeks
after a case of tonsillitis, which is a sign of affected glomerular basement
membrane in the kidneys.
What mechanism is the most likely to cause the basement membrane damage?
A. Immune complex
B. Granulomatous
C. Antibody-mediated
D. Reaginic
E. Cytotoxic
84. What condition may develop 15-30 minutes after re-administration of an
antigen as a result of
the increased level of antibodies, mainly IgE, that are adsorbed on the surface of
target cells,
namely tissue basophils (mast cells) and blood basophils?
A. Anaphylaxis
B. Antibody-dependent cytotoxicity
C. Delayed-type hypersensitivity
D. Immune complex hyperresponsiveness
E. Serum sickness
Module 2
1. Apatient suffering from trombophlebitis of deep veins suddenly died. The
autopsy has shown
freely lying red friable masses with dim crimped surface in the trunk and
bifurcation of the
pulmonary artery. What pathologic process was revealed by the morbid anatomist?
A. Tromboembolism
B. Thrombosis
C. Tissue embolism
D. Embolism with foreign body
E. Fat embolism
2. Decreased blood supply to the organs causes hypoxia that activates fibroblasts
function. Volume
of what elements is increased in this case?
A. Intercellular substance
B. Vessels of microcircular stream
C. Nerve elements
D. Parenchymatous elements of the organ
E. Lymphatic vessels
3. The pulmonalis embolism has suddenly developed in a 40 year-old patient with
opened fracture
of the hip. Choose the possible kind of embolism.
A. Fat
B. Thrombus-embolus
C. Air
D. Tissue
E. Foreign body
4. A 16-year-old boy was performed an appendectomy. He has been hospitalized
for right lower
quadrant abdominal pain within 18 hours. The surgical specimen is edematous and
erythematous.
Infiltration by what of the following cells is the most typical for the process
occuring here?
A. Neutrophils
B. Eosinophils
C. Basophils
D. Limphocytes
E. Monocytes
5. Necrosis focus was observed in the area of hyperemia and skin edema in a few
hours after burn.
What mechanism strengthens destructive effects in the inflammation area?
A. Secondary alteration
B. Primary alteration
C. Emigration of lymphocytes
D. Diapedesis of erythrocytes
E. Proliferation of fibroblasts
6. Apatient who suffers from pneumonia has high body temperature. What
biologically active
substance plays the leading part in origin of this phenomenon?
A. Interleukin-I
B. Histamine
C. Bradykinin
D. Serotonin
E. Leukotrienes
7. Inflammation of a patient’s eye was accompanied by accumulation of turbid
liquid with high
protein at the bottom of anterior chamber that was called hypopyon. What process
underlies the
8. changes under observation?
A. Disturbance of microcirculation
B. Primary alteration
C. Secondary alteration
D. Proliferation
E. –
9. Utilization of arachidonic acid via cyclooxigenase pathway results in formation
of some
bioactive substances. Name them:
A. Prostaglandins
B. Thyroxine
C. Biogenic amins
D. Somatomedins
E. Insulin-like growth factors
10. While playing volleyball a sportsman made a jump and landed on the outside
edge of his foot.
He felt acute pain in the talocrural joint, active movements are limited, passive
movements are
unlimited but painful. A bit later there appeared a swelling in the area of external
ankle, the skin
became red and warm. What type of peripheral circulation disturbance is the case?
A. Arterial hyperemia
B. Stasis
C. Embolism
D. Venous hyperemia
E. Thrombosis
11. A 17 year old boy fell seriously ill, the body temperature rose up to 38,5 C,
there appeared
cough, rhinitis, lacrimation, nasal discharges. What inflammation is it?
A. Catarrhal
B. Serous
C. Fibrinous
D. Purulent
E. Hemorrhagic
12. A 45 year old woman is ill with breast cancer. Her left arm has symptoms of
lymphatic system
insufficiency – limb edema, lymph node enlargement. What form of lymphatic
circulation
insufficiency is it?
A. Mechanic insufficiency
B. Dynamic insufficiency
C. Resorption insufficiency
D. Combined insufficiency
E. –
13. While playing volleyball a sportsman jumped and then landed across the
external edge of his
foot. This caused acute pain in the talocrural articulation, active movements
became limited, passive
movements remained unlimited but painful. In the region of the external ankle a
swelling appeared,
the skin turned red and became warmer to the touch. What type of peripheral
circulation disorder
has developed in this case?
A. Arterial hyperaemia
B. Stasis
C. Embolism
D. Venous hyperaemia
E. Thrombosis
14. Autopsy of a 73-year-old man who had been suffering from the coronary heart
disease along
with cardiac insufficiency for a long time revealed: nutmeg liver, brown induration
of lungs,
cyanotic induration of kidneys and spleen. What kind of circulation disorder was
the cause of such
effects?
A. General chronic venous congestion
B. Arterial hyperaemia
C. General acute venous congestion
D. Acute anaemia
E. Chronic anaemia
15. After transfusion of 200 ml of blood a patient presented with body temperature
rise up to 37, 9
C. Which of the following substances is the most likely cause of temperature rise?
A. Interleukin-1
B. Interleukin-2
C. Tumour necrosis factor
D. Interleukin-3
E. Interleukin-4
16. A patient who has been abusing tobacco smoking for a long time has got cough
accompanied
by excretion of viscous mucus; weakness after minor physical stress, pale skin.
The patient has also
lost 12,0 kg of body weight. Endoscopic examination of biosy material his illness
was diagnosed as
squamous cell carcinoma. Name a pathological process that preceded formation of
the tumour:
A. Metaplasia
B. Hypoplasia
C. Hyperplasia
D. Necrosis
E. Sclerosis
17. A 17 year old boy fell seriously ill, the body temperature rose up to 38,5 C,
there appeared
cough, rhinitis, lacrimation, nasal discharges. What inflammation is it?
A. Catarrhal
B. Serous
C. Fibrinous
D. Purulent
E. Hemorrhagic
18. Inflammatory processes cause synthesis of protein of acute phase in an
organism. What
substances stimulate their synthesis?
A. Interleukin-1
B. Immunoglobulins
C. Interferons
D. Biogenic amins
E. Angiotensin
19. A 4 year old child complained of pain during deglutition, indisposition.
Objectively: palatine
arches and tonsils are moderately edematic and hyperemic, there are greyish-white
films up to 1
mm thick closely adhering to the subjacent tissues. What pathological process are
these changes
typical for?
A. Inflammation
B. Dystrophy
C. Necrosis
D. Metaplasia
E. Organization
20. Introduction of a big dose of histamine to an experimental animal caused
abrupt drop of arterial
pressure as a result of:
A. Dilatation of resistance vessels
B. Constriction of resistance vessels
C. Increase of heart rate
D. Decrease of heart rate
E. Decrease of heart rate and force
21. A 25 year old man has spent a long time in the sun under high air humidity. As
a result of it his
body temperature rose up to 39oC. What pathological process is it?
A. Hyperthermia
B. Infectious fever
C. Hypothermia
D. Noninfectious fever
E. Burn disease
22. After transfusion of 200 ml of blood a patient presented with body temperature
rise up to
37,9oC. Which of the following substances is the most likely cause of temperature
rise?
A. Interleukin-1
B. Interleukin-2
C. Tumour necrosis factor
D. Interleukin-3
E. Interleukin-4
23. A student failed to answer all the questions of examination paper correctly. As
a result he
blushed, felt hot and lost confidence. What type of arterial hyperemia has
developed in this case?
A. Neurotonic hyperemia
B. Neuroparalytic hyperemia
C. Metabolic hyperemia
D. Pathologic hyperemia
E. Postishemic hyperemia
24. A 41-year-old woman has breast cancer. Her left arm has symptoms of
lymphatic system
insufficiency – limb edema, lymph node enlargement. What form of lymphatic
circulation
insufficiency is it?
A. Mechanic insufficiency
B. Dynamic insufficiency
C. Resorption insufficiency
D. Combined insufficiency
E. –
25. A 23-year-old man has spent a long time in the sun under high air humidity. As
a result of it his
body temperature rose up to 39oC. What pathological process is it?
A. Hyperthermia
B. Infectious fever
C. Hypothermia
D. Noninfectious fever
E. Burn disease
26. A patient complains about dyspnea provoked by the physical activity. Clinical
examination
revealed anaemia and presence of the paraprotein in the zone of gamma-globulins.
To confirm the
myeloma diagnosis it is necessary to determine the following index in the patient’s
urine:
A. Bence Jones protein
B. Bilirubin
C. Haemoglobin
D. Ceruloplasmin
E. Antitrypsin
27. A 56 year old patient suffering from cardiac insufficiency has edema of feet
and shins,
edematous skin is pale and cold. What is the leding mechanism of edema
pathogenesis?
A. Rise of hydrostatic pressure in venules
B. Drop of oncotic pessure in capillaries
C. Increase of capillary permeability
D. Disorder of lymph outflow
E. Positive water balance
28. A female patient has been diagnosed with cervical erosion, which is a
precancerous pathology.
What defense mechanism can prevent the development of a tumor?
A. Increase in natural killer level (NKcells)
B. High-dose immunological tolerance
C. Increase in the activity of lysosomal enzymes
D. Simplification of the antigenic structure of tissues
E. Low-dose immunological tolerance
29. A patient with lobar pneumonia has had body temperature of 39oC with daily
temperature
fluctuation of no more than 1oC for 9 days. This fever can be characterized by the
following
temperature curve:
A. Persistent
B. Hectic
C. Remittent
D. Hyperpyretic
E. Recurrent
30. During the intravenous transfusion of the saline the patient’s condition
deteriorated
dramatically, and the patient died from asphyxiation. Autopsy revealed acute
venous congestion of
internal organs with the dramatic right heart dilatation. When the right ventricle
was punctured
underwater, the bubbles escaped. What pathological process occurred in the
patient?
A. Air embolism
B. Gaseous embolism
C. Adipose embolism
D. Tissue embolism
E. Thromboembolism
31. Deficiency of linoleic and linolenic acids in the body leads to the skin damage,
hair loss,
delayed wound healing, thrombocytopenia, low resistance to infections. These
changes are most
likely to be caused by the impaired synthesis of the following substances:
A. Eicosanoids
B. Interleukins
C. Interferons
D. Catecholamines
E. Corticosteroids
32. As a result of careless handling of an iron, a 34-year-old female patient has got
acute pain,
redness, swelling of her right index finger. A few minutes later, there appeared a
blister filled with a
transparent liquid of straw-yellow color. The described changes are a manifestation
of the following
pathological process:
A. Exudative inflammation
B. Traumatic edema
C. Alternative inflammation
D. Proliferative inflammation
E. Vacuolar degeneration
33. In a patient elevation of body temperature takes turns with drops down to
normal levels during
the day. The rise in temperature is observed periodically once in four days. Specify
the type of
temperature curve:
A. Febris internuttens
B. Febris continua
C. Febris reccurens
D. Febris hectica
E. Febris remitens
34. A patient has acute bronchitis. The fever up to 38,5oC had lasted for a week,
presently there is a
decrease in temperature down to 37,0oC. Specify the leading mechanism in the 3rd
stage of fever:
A. Peripheral vasodilation
B. Increased heat production
C. Development of chill
D. Increased diuresis
E. Increased respiratory rate
35. A 25-year-old patient complains of increasing pain in his leg muscles occurring
during walking
and forcing him to make frequent stops. Objectively: skin of legs is pale, no hair-
covering, toenails
are with trophic changes, no pulsation of pedal arteries. The most probable cause
of these changes
is:
A. Ischemia
B. Venous hyperemia
C. Arterial hyperemia
D. –
E. Embolism
36. This year influenza epidemic is characterised by patients’ body temperature
varying from
36,9oC to 37,9oC. Such fever is called:
A. Subfebrile
B. High
C. Hyperpyretic
D. Apyretic
E. Moderate
37. A patient, having suffered a thermal burn, developed painful boils filled with
turbid liquid in
the skin. What morphological type of inflammation has developed in the patient?
A. Serous
B. Proliferative
C. Croupous
D. Granulomatous
E. Diphtheritic
38. A 7-year-old child has acute onset of disease: temperature rise up to 38oC,
rhinitis, cough,
lacrimation, and large-spot rash on the skin. Pharyngeal mucosa is edematous,
hyperemic, with
whitish spots in the buccal area. What kind of inflammation caused the changes in
the buccal
mucosa?
A. Catarrhal inflammation
B. Suppurative inflammation
C. Fibrinous inflammation
D. Hemorrhagic inflammation
E. Serous inflammation
39. Cellular composition of exudate largely depends on the etiological factor of
inflammation.
What leukocytes are the first to be involved in the focus of inflammation caused by
pyogenic
bacteria?
A. Neutrophil granulocytes
B. Monocytes
C. Myelocytes
D. Eosinophilic granulocytes
E. Basophils
40. This year influenza epidemic is characterized by patients’ body temperature
varying from
36,9oC to 37,9oC. Such fever is called:
A. Subfebrile
B. High
C. Hyperpyretic
D. Apyretic
E. Moderate
41. 30 minutes after drinking mango juice a child suddenly developed a local
swelling in the area
of the soft palate, which impeded swallowing and, eventually, respiration. Mucosa
of the swollen
area was hyperemic and painless. Blood test revealed moderate eosinophilia. Body
temperature was
normal. Anamnesis states that the elder sister of the child has been suffering from
bronchial asthma
attacks. What kind of edema has developed in the child?
A. Allergic
B. Inflammatory
C. Cardiac
D. Alimentary
E. Hepatic
Module 3
1. Galactosemia has been revealed in a child. Concentration of glucose in the blood
has not
considerably changed. What enzyme deficiency caused this illness?
A. Galactose-1-phosphate uridyltransferase
B. Amylo-1,6-glucosidase
C. Phosphoglucomutase
D. Galactokinase
E. Hexokinase
2. The patient with diabetes mellitus has been delivered in hospital in the state of
unconsciousness.
Arterial pressure is low. The patient has acidosis. Point substances, which
accumulation in the
blood results in these manifestations:
A. Ketone bodies
B. Amino acids
C. Monosaccharides
D. High fatty acids
E. Cholesterol esters
3. Some diseases reveal symptoms of aldosteronism with hypertension and edema
due to sodium
retention in the organism. What organ of the internal secretion is affected on
aldosteronism?
A. Adrenal glands
B. Testicle
C. Ovaries
D. Pancreas
E. Hypophysis
4. Inflamation is characterised by increasing penetration of vessels of
microcirculation stream,
increasing of their fluid dynamic blood pressure. Increasing of the osmotic
concentration and
dispersity of protein structures can be found in the intercellular fluid. What kind of
edema are to be
observed in this case?
A. Mixed
B. Hydrodynamic
C. Colloid-osmotic
D. Lymphogenic
E. Membranogenic
5. The B cells of endocrine portion of pancreas are selectively damaged by alloxan
poisoning. How
will it be reflected in blood plasma?
A. The content of sugar increases
B. The content of fibrinogen decrease
C. The level of sugar decreases
D. The content of globulins decreases
E. The content of albumins decreases
6. The concentration of albumins in human blood sample is lower than normal.
This leads to edema
of tissues. What blood function is damaged?
A. Maintaining the oncotic blood pressure
B. Maintaining the Ph level
C. Maintaining the body temperature
D. Maintaining the blood sedimentation system
E. All answers are correct
7. Chronic glomerulonephritis was diagnosed in a 34-year-old patient 3 years ago.
Edema has
developed within the last 6 monthes. What caused the edema?
A. Proteinuria
B. Hyperproduction of vasopressin
C. Liver disfunction of protein formation
D. Hyperosmolarity of plasma
E. Hyperaldosteronism
8. Patient with diabetes mellitus experienced loss of consciousness and convulsions
after an
injection of insulin. What might be the result of biochemical blood analysis for
concentration of
sugar?
A. 1,5 mmol/L
B. 8,0 mmol/L
C. 10,0 mmol/L
D. 3,3 mmol/L
E. 5,5 mmol/L
9. A patient is ill with diabetes mellitus that is accompanied by hyperglycemia of
over 7,2
millimole/l on an empty stomach. The level of what blood plasma protein allows to
estimate the
glycemia rate retrospectively (4-8 weeks before examination)?
A. Glycated hemoglobin
B. Albumin
C. Fibrinogen
D. C-reactive protein
E. Ceruloplasmin
10. A 4 y.o. child with signs of durative proteinic starvation was admitted to the
hospital. The signs
were as follows: growth inhibition, anemia, edemata, mental deficiency. Choose a
cause of edemata
development:
A. Reduced synthesis of albumins
B. Reduced synthesis of globulins
C. Reduced synthesis of hemoglobin
D. Reduced synthesis of lipoproteins
E. Reduced synthesis of glycoproteins
11. Examination of a patient revealed reduced contents of magnesium ions that are
necessary for
attachment of ribosomes to the granular endoplasmatic reticulum. It is known that
it causes
disturbance of protein biosynthesis. What stage of protein biosynthesis will be
disturbed?
A. Translation
B. Transcription
C. Replication
D. Aminoacid activation
E. Termination
12. As a result of exhausting muscular work a worker has largely reduced buffer
capacity of blood.
What acidic substance that came to blood caused this phenomenon?
A. Lactate
B. Pyruvate
C. 1,3-bisphosphoglycerate
D. 3-phosphoglycerate
E. –
13. A patient was delivered to the hospital by an emergency team. Objectively:
grave condition,
unconscious, adynamy. Cutaneous surfaces are dry, eyes are sunken, face is
cyanotic. There is
tachycardia and smell of acetone from the mouth. Analysis results: blood glucose -
20,1
micromole/l (standard is 3,3-5,5 micromole/ l), urine glucose - 3,5%(standard is -
0). What is the
most probable diagnosis?
A. Hyperglycemic coma
B. Hypoglycemic coma
C. Acute heart failure
D. Acute alcoholic intoxication
E. Anaphylactic shock
14. A woman who has been keeping to a clean-rice diet for a long time was
diagnosed with
polyneuritis (beriberi). What vitamin deficit results in development of this disease?
A. Thiamine
B. Ascorbic acid
C. Pyridoxine
D. Folic acid
E. Riboflavin
15. Examination of a man who hadn’t been consuming fats but had been getting
enough
carbohydrates and proteins for a long time revealed dermatitis, poor wound
healing, vision
impairment. What is the probable cause of metabolic disorder?
A. Lack of linoleic acid, vitamins A, D, E, K
B. Lack of palmitic acid
C. Lack of vitamins PP, H
D. Low caloric value of diet
E. Lack of oleic acid
16. A patient who suffers from heart failure has enlarged liver, edemata of lower
extremities,
ascites. What is the leading mechanism in the development of this edema?
A. Hydrodynamic
B. Colloid osmotic
C. Lymphogenous
D. Membranogenic
E. –
17. Examination of a patient revealed hyperkaliemia and hyponatremia. Low
secretion of which
hormone may cause such changes?
A. Aldosteron
B. Vasopressin
C. Cortisol
D. Parathormone
E. Natriuretic
18. A newborn child with pylorostenosis has often repeating vomiting
accompanied by apathy,
weakness, hypertonicity, sometimes convulsions. What disorder form of acid-base
balance is it?
A. Nongaseous alkalosis
B. Gaseous alkalosis
C. Gaseous acidosis
D. Metabolic acidosis
E. Excretory acidosis
19. An infant has apparent diarrhea resulting from improper feeding. One of the
main diarrhea
effects is plentiful excretion of sodium bicarbonate. What form of acid-base
balance disorder is the
case?
A. Metabolic acidosis
B. Metabolic alkalosis
C. Respiratory acidosis
D. Respiratory alkalosis
E. No disorders of acid-base balance will be observed
20. A 65 year old man suffering from gout complains of kidney pain. Ultrasound
examination
revealed renal calculi. The most probable cause of calculi formation is the
strengthened
concentration of the following substance:
A. Uric acid
B. Cholesterol
C. Bilirubin
D. Urea
E. Cystine
21. A chemical burn caused esophagus stenosis. Difficulty of ingestion led to the
abrupt loss of
weight. In blood: 3, 0 · 1012/l, Hb - 106 g/l, crude protein - 57 g/l. What type of
starvation is it?
A. Incomplete
B. Proteinic
C. Complete
D. Water
E. Absolute
22. A patient with nephrotic syndrome has massive edemata of his face and limbs.
What is the
leading pathogenetic mechanism of edemata development?
A. Drop of oncotic blood pressure
B. Increase of vascular permeability
C. Rise of hydrodynamic blood pressure
D. Lymphostasis
E. Increase of lymph outflow
23. A doctor examined a child and revealed symptoms of rachitis. Development of
this desease was
caused by deficiency of the following compound:
A. 1,25 [ОН]-dichydroxycholecalciferol
B. Biotin
C. Tocopherol
D. Naphtaquinone
E. Retinol
24. A 6 year old child was delivered to a hospital. Examination revealed that the
child couldn’t fix
his eyes, didn’t keep his eyes on toys, eye ground had the cherry-red spot sign.
Laboratory analyses
showed that brain, liver and spleen had high rate of ganglioside glycometide. What
congenital
disease is the child ill with?
A. Tay-Sachs disease
B.Wilson’s syndrome
C. Turner’s syndrome
D. Niemann-Pick disease
E. MacArdle disease
25. Apatient ill with enteritis accompanied by massive diarrhea has low water rate
in the
extracellular space, high water rate inside the cells and low blood osmolarity. What
is such
disturbance of waterelectrolytic metabolism called?
A. Hypo-osmolar hypohydration
B. Hyperosmolar hypohydration
C. Osmolar hypohydration
D. Hypo-osmolar hyperhydration
E. Hyperosmolar hyperhydration
26. A 70 year old man is ill with vascular atherosclerosis of lower extremities and
coronary heart
disease. Examination revealed disturbance of lipidic blood composition. The main
factor of
atherosclerosis pathogenesis is the excess of the following lipoproteins:
A. Low-density lipoproteins
B. Cholesterol
C. High-density lipoproteins
D. Intermediate density lipoproteins
E. Chylomicrons
27. A patient has pellagra. Interrogation revealed that he had lived mostly on maize
for a long time
and eaten little meat. This disease had been caused by the deficit of the following
substance in the
maize:
A. Tryptophan
B. Tyrosine
C. Proline
D. Alanine
E. Histidine
28. Nappies of a newborn have dark spots being the evidence of homogentisic acid
formation. This
is caused by the metabolic disorder of the following substance:
A. Tyrosine
B. Galactose
C. Methionine
D. Cholesterol
E. Tryptophan
29. A 50-year-old patient complains about general weakness, appetite loss and
cardiac arrhythmia.
The patient presents with muscle hypotonia, flaccid paralyses, weakened peristaltic
activity of the
bowels. Such condition might be caused by:
A. Hypokaliemia
B. Hypoproteinemia
C. Hyperkaliemia
D. Hypophosphatemia
E. Hyponatremia
30. A 1,5-year-old child presents with both mental and physical lag, decolorizing
of skin and hair,
decrease in catecholamine concentration in blood. When a few drops of 5%
solution of
trichloroacetic iron had been added to the child’s urine it turned olive green. Such
alteration are
typical for the following pathology of the amino acid metabolism:
A. Phenylketonuria
B. Alkaptonuria
C. Tyrosinosis
D. Albinism
E. Xanthinuria
31. A 12-year-old teenager has significantly put off weight within 3 months;
glucose concentration
rose up to 50 millimole/l. He fell into a coma. What is the main mechanism of its
development?
A. Hyperosmolar
B. Hypoglycemic
C. Ketonemic
D. Lactacidemic
E. Hypoxic
32. While determining power inputs of a patient’s organism it was established that
the respiratory
coefficient equaled 1,0. This means that in the cells of the patient the following
substances are
mainly oxidized:
A. Carbohydrates
B. Proteins
C. Fats
D. Proteins and carbohydrates
E. Carbohydrates and fats
33. After taking poor-quality food a patient developed repeated episodes of
diarrhea. On the next
day he presented with decreased arterial pressure, tachycardia, extrasystole. Blood
pH is 7,18.
These abnormalities were caused by the development of:
A. Nongaseous acidosis
B. Gaseous acidosis
C. Nongaseous alkalosis
D. Gaseous alkalosis
E. Metabolic alkalosis
34. Characteristic sign of glycogenosis is muscle pain during physical work. Blood
examination
reveals usually hypoglycemia. This pathology is caused by congenital deficiency
of the following
enzyme:
A. Glycogen phosphorylase
B. Glucose 6-phosphate dehydrogenase
C. Alpha amylase
D. Gamma amylase
E. Lysosomal glycosidase
35. Blood of a 12 year old boy presents low concentration of uric acid and
accumulation of
xanthine and hypoxanthine. This child has genetic defect of the following enzyme:
A. Xanthine oxidase
B. Arginase
C. Urease
D. Ornithine carbamoyltransferase
E. Glycerylkinase
36. A full-term newborn child has yellowish skin and mucous membranes. This
might be probably
caused by temporary deficiency of the following enzyme:
A. UDPglucoronyltransferase
B. Uridine transferase
C. Heme synthetase
D. Heme oxygenase
E. Biliverdin reductase
37. Examination of a patient revealed typical presentations of collagenosis. This
pathology is
characterized by increase of the following urine index:
A. Hydroxyproline
B. Arginine
C. Glucose
D. Mineral salts
E. Ammonium salts
38. Examination of a patient suffering from frequent haemorrhages in the inner
organs and mucous
membranes revealed proline and lysine being included in collagen fibers.
Impairment of their
hydroxylation is caused by lack of the following vitamin:
A. C
B. E
C. K
D. A
E. D
39. A 56 year old patient suffering from cardiac insufficiency has edema of feet
and shins,
edematous skin is pale and cold. What is the leding mechanism of edema
pathogenesis?
A. Rise of hydrostatic pressure in venules
B. Drop of oncotic pessure in capillaries
C. Increase of capillary permeability
D. Disorder of lymph outflow
E. Positive water balance
40. A patient was stung by a bee. Examination revealed that his left hand was hot,
pink, edematic,
there was a big red blister on the site of sting. What is the leading mechanism of
edema
development?
A. Increased vessel permeability
B. Reduced vessel filling
C. Injury of vessels caused by the sting
D. Drop of oncotic pressure in tissue
E. Drop of osmotic pressure in tissue
41. A 50 year old patient has been taking treatment thrice for the last 6 months
because of fractures
caused by domestic accidents. Microscopical examination of bony tissue revealed
foci of lacunar
resolution, giant-cell granulomas in the tumour-like formations, cysts. Bony tissue
was substituted
by fibrous connective tissue. Examination revealed also adenoma of parathyroid
gland and
hypercalcemia. What is the most probable diagnosis?
A. Parathyroid osteodystrophy
B. Myelomatosis
C. Osteomyelitis
D. Osteopetrosis
E. Paget’s disease
42. Patients who suffer from severe diabetes and don’t receive insulin have
metabolic acidosis.
This is caused by increased concentration of the following metabolites:
A. Ketone bodies
B. Fatty acids
C. Unsaturated fatty acids
D. Triacylglycerols
E. Cholesterol
43. A patient was admitted to the infectious department. His symptoms: dry skin,
decreased skin
turgor, rice-water stool. The patient was diagnosed with cholera. What disorder of
water-electrolytic
balance is most often observed in this disease?
A. Isoosmotic hypohydration
B. Hyperosmotic hyperhydration
C. Hypoosmotic hypohydration
D. Hyperosmotic hypohydration
E. Hypoosmotic hyperhydration
44. A 2-year-old child with mental and physical retardation has been delivered to a
hospital. He
presents with frequent vomiting after having meals. There is phenylpyruvic acid in
urine. Which
metabolism abnormality is the reason for this pathology?
A. Amino-acid metabolism
B. Lipidic metabolism
C. Carbohydrate metabolism
D. Water-salt metabolism
E. Phosphoric calcium metabolism
45. Urine analysis of a 12-year-old boy reveals high concentration of all aliphatic
amino acids with
the highest excretion of cystine and cysteine. US of kidneys revealed kidney
concrements. What is
the most likely pathology?
A. Cystinuria
B. Alkaptonuria
C. Cystitis
D. Phenylketonuria
E. Hartnup disease
46. A patient has an increased pyruvate concentration in blood, most of it is
excreted with the
urine. What kind of avitaminosis has this patient?
A. B1
B. E
C. B3
D. B6
E. B2
47. One of the parents is suspected of having phenylketonuria recessive gene.
What is the risk of
giving birth to a child with inborn phenylketonuria?
A. 0%
B. 25%
C. 50%
D. 75%
E. 100%
48. A 14-year-old teenager has significantly put off weight within 3 months;
glucose concentration
rose up to 50 millimole/l. He fell into a coma. What is the main mechanism of its
development?
A. Hyperosmolar
B. Hypoglycemic
C. Ketonemic
D. Lactacidemic
E. Hypoxic
49. A newborn child with pylorostenosis has often repeating vomiting
accompanied by apathy,
weakness, hypertonicity, sometimes convulsions. What disorder form of acid-base
balance is it?
A. Nongaseous alkalosis
B. Gaseous alkalosis
C. Gaseous acidosis
D. Metabolic acidosis
E. Excretory acidosis
50. A patient has been diagnosed with alkaptonuria. Choose an enzyme whose
deficiency can be
the reason for this pathology:
A. Homogentisic acid oxidase
B. Phenylalanine hydroxylase
C. Glutamate dehydrogenase
D. Pyruvate dehydrogenase
E. Dioxyphenylalanine decarboxylase
51. An infant has pylorospasm, weakness, hypodynamia, convulsions as a result of
frequent
vomiting.What kind of acid-base disbalance is it?
A. Excretory alkalosis
B. Excretory acidosis
C. Metabolic acidosis
D. Exogenous nongaseous acidosis
E. Gaseous alkalosis
52. A patient with enteritis accompanied by massive diarrhea has low water rate in
the extracellular
space, high water rate inside the cells and low blood osmolarity. What is such
disturbance of water
electrolytic metabolism called?
A. Hypo-osmolar hypohydration
B. Hyperosmolar hypohydration
C. Osmolar hypohydration
D. Hypo-osmolar hyperhydration
E. Hyperosmolar hyperhydration
53. A newborn child was found to have reduced intensity of sucking, frequent
vomiting, hypotonia.
Urine and blood exhibit increased concentration of citrulline. What metabolic
process is disturbed?
A. Ornithinic cycle
B. Tricarboxylic acid cycle
C. Glycolysis
D. Glyconeogenesis
E. Cori cycle
54. Characteristic sign of glycogenosis is muscle pain during physical work. Blood
examination
reveals usually hypoglycemia. This pathology is caused by congenital deficiency
of the following
enzyme:
A. Glycogen phosphorylase
B. Glucose 6-phosphate dehydrogenase
C. Alpha amylase
D. Gamma amylase
E. Lysosomal glycosidase
55. A month after surgical constriction of rabbit’s renal artery the considerable
increase of
systematic arterial pressure was observed. What of the following regulation
mechanisms caused the
animal’s pressure change?
A. Angiotensin-II
B. Vasopressin
C. Adrenaline
D. Noradrenaline
E. Serotonin
56. A 1,5-year old child with mental and physical retardation has been delivered to
a hospital. He
presents with frequent vomiting after having meals. There is phenylpyruvic acid in
urine. Which
metabolism abnormality is the reason for this pathology?
A. Amino-acid metabolism
B. Lipidic metabolism
C. Carbohydrate metabolism
D. Water-salt metabolism
E. Phosphoric calcium metabolism
57. A nurse accidentally injected a nearly double dose of insulin to a patient with
diabetes mellitus.
The patient lapsed into a hypoglycemic coma. What drug should be injected in
order to help him out
of coma?
A. Glucose
B. Lidase
C. Insulin
D. Somatotropin
E. Noradrenaline
58. A patient has been diagnosed with alkaptonuria. Choose an enzyme whose
deficiency can be
the reason for this pathology:
A. Homogentisic acid oxidase
B. Phenylalanine hydroxylase
C. Glutamate dehydrogenase
D. Pyruvate dehydrogenase
E. Dioxyphenylalanine decarboxylase
59. An infant has pylorospasm, weakness, hypodynamia, convulsions as a result of
frequent
vomiting. What kind of acid-base disbalance is it?
A. Excretory alkalosis
B. Excretory acidosis
C. Metabolic acidosis
D. Exogenous nongaseous acidosis
E. Gaseous alkalosis
60. A 49 year old woman spent a lot of time standing. As a result of it she got leg
edema. What is
the most likely cause of the edema?
A. Increase in hydrostatic pressure of blood in veins
B. Decrease in hydrostatic pressure of blood in veins
C. Decrease in hydrostatic pressure of blood in arteries
D. Increase in oncotic pressure of blood plasma
E. Increase in systemic arterial pressure
61. A 64 year old woman has impairment of twilight vision (hemeralopy). What
vitamin should be
recommended in the first place?
A. A
B. B2
C. E
D. C
E. B6
62. A 4 year old child with hereditary renal lesion has signs of rickets, vitamin D
concentration in
blood is normal. What is the most probable cause of rickets development?
A. Impaired synthesis of calcitriol
B. Increased excretion of calcium
C. Hyperfunction of parathyroid glands
D. Hypofunction of parathyroid glands
E. Lack of calcium in food
63. After taking poor-quality food a patient developed repeated episodes of
diarrhea. On the next
day he presented with decreased arterial pressure, tachycardia, extrasystole. Blood
pH is 7,18.
These abnormalities were caused by the development of:
A. Nongaseous acidosis
B. Gaseous acidosis
C. Nongaseous alkalosis
D. Gaseous alkalosis
E. Metabolic alkalosis
64. A patient with diabetes developed a diabetic coma due to the acid-base
imbalance. Specify the
kind of this imbalance:
A. Metabolic acidosis
B. Metabolic alkalosis
C. Respiratory acidosis
D. Gaseous alkalosis
E. Non-gaseous alkalosis
65. A patient with respiratory failure has blood pH of 7,35. pCO2 test revealed
hypercapnia. Urine
pH test revealed an increase in the urine acidity. What form of acid-base imbalance
is the case?
A. Compensated respiratory acidosis
B. Compensated metabolic acidosis
C. Decompensated metabolic acidosis
D. Compensated respiratory alkalosis
E. Decompensated respiratory alkalosis
66. A comatose patient was taken to the hospital. He has a history of diabetes
mellitus. Objectively:
Kussmaul breathing, low blood pressure, acetone odor of breath. After the
emergency treatment the
patient’s condition improved. What drug had been administered to the patient?
A. Insulin
B. Adrenaline
C. Isadrinum
D. Glibenclamide
E. Furosemide
67. A patient with a pathology of the cardiovascular system developed edemata of
the lower
extremities. What is the mechanism of cardiac edema development?
A. Increased hydrostatic pressure at the venous end of the capillary
B. Increased oncotic pressure
C. Increased hydrostatic pressure at the arterial end of the capillary
D. Reduced osmotic pressure
E. Lymph efflux disorder
68. One of the factors that cause obesity is the inhibition of fatty acids oxidation
due to:
A. Low level of carnitine
B. Impaired phospholipid synthesis
C. Excessive consumption of fatty foods
D. Choline deficiency
E. Lack of carbohydrates in the diet
69. A 67-year-old male patient consumes eggs, pork fat, butter, milk and meat.
Blood test results:
cholesterol - 12,3 mmol/l, total lipids - 8,2 g/l, increased low density lipoprotein
fraction (LDL).
What type of hyperlipoproteinemia is observed in the patient?
A. Hyperlipoproteinemia type IIa
B. Hyperlipoproteinemia type I
C. Hyperlipoproteinemia type IIb
D. Hyperlipoproteinemia type IV
E. Cholesterol, hyperlipoproteinemia
70. A hypertensive patient had been keeping to a salt-free diet and taking
antihypertensive drugs
together with hydrochlorothiazide for a long time. This resulted in electrolyte
imbalance. What
disorder of the internal environment occurred in the patient?
A. Hypochloremic alkalosis
B. Metabolic acidosis
C. Hyperkalemia
D. Hypermagnesemia
E. Increase in circulating blood volume
71. A patient with constant headaches, pain in the occipital region, tinnitus,
dizziness has been
admitted to the cardiology department. Objectively:AP - 180/110 mm Hg, heart
rate - 95/min.
Radiographically, there is a stenosis of one of the renal arteries. Hypertensive
condition in this
patient has been caused by the activation of the following system:
A. Renin-angiotensin
B. Hemostatic
C. Sympathoadrenal
D. Kinin
E. Immune
72. A female patient complains of vision impairment. On examination she was
found to have
obesity, fasting hyperglycemia. What complication of diabetes can cause vision
impairment?
A. Microangiopathy
B. Macroangiopathy
C. Atherosclerosis
D. Neuropathy
E. Glomerulopathy
73. 12-year-old patient was found to have blood serum cholesterol at the rate of 25
mmol/l. The
boy has a history of hereditary familial hypercholesterolemia, which is caused by
the impaired
synthesis of the following protein receptors:
A. Low density lipoproteins
B. High density lipoproteins
C. Chylomicrons
D. Very low density lipoproteins
E. Intermediate density lipoproteins
74. In a young man during exercise, the minute oxygen uptake and carbon dioxide
emission
equalled to 1000 ml. What substrates are oxidized in the cells of his body?
A. Carbohydrates
B. Proteins
C. Fats
D. Carbohydrates and fats
E. Carbohydrates and proteins
75. Increased HDL levels decrease the risk of atherosclerosis. What is the
mechanism of HDL antiatherogenic action?
A. They remove cholesterol from tissues
B. They supply tissues with cholesterol
C. They are involved in the breakdown of cholesterol
D. They activate the conversion of cholesterol to bile acids
E. They promote absorption of cholesterol in the intestine
76. A 46-year-old female patient consulted a doctor about pain in the small joints
of the upper and
lower limbs. The joints are enlarged and shaped like thickened nodes. Serum test
revealed an
increase in urate concentration. This might be caused by a disorder in metabolism
of:
A. Purines
B. Carbohydrates
C. Lipids
D. Pyrimidines
E. Amino acids
77. Diabetic nephropathy with uremia has developed in a patient with pancreatic
diabetes. The
velocity of glomerular filtration is 9 ml/min. What mechanism of a decrease in
glomerular filtration
velocity and chronic renal failure development is most likely in the case of this
patient?
A. Reduction of active nephron mass
B. Decrease in systemic arterial pressure
C. Obstruction of nephron tubules with hyaline casts
D. Tissue acidosis
E. Arteriolar spasm
78. Prolonged fasting causes hypoglycemia which is amplified by alcohol
consumption, as the
following process is inhibited:
A. Gluconeogenesis
B. Glycolysis
C. Glycogenolysis
D. Lipolysis
E. Proteolysis
79. A 39-year-old female patient with a history of diabetes was hospitalized in a
precomatose state
for diabetic ketoacidosis. This condition had been caused by an increase in the
following metabolite
level:
A. Acetoacetate
B. Citrate
C. Alpha-ketoglutarate
D. Malonate
E. Aspartate
80. A patient with homogentisuria has signs of arthritis, ochronosis. In this case,
the pain in the
joints is associated with the deposition of:
A. Homogentisates
B. Urates
C. Phosphates
D. Oxalates
E. Carbonates
81. In case of alkaptonuria, homogentisic acid is excreted in urine in large
amounts. The
development of this disease is associated with a disorder of metabolism of the
following amino
acid:
A. Tyrosine
B. Phenylalanine
C. Alanine
D. Methionine
E. Asparagine
82. Examination of a 56-year-old female patient with a history of type 1 diabetes
revealed a
disorder of protein metabolism that is manifested by aminoacidemia in the
laboratory blood test
values, and clinically by the delayed wound healing and decreased synthesis of
antibodies. Which
of the following mechanisms causes the development of aminoacidemia?
A. Increased proteolysis
B. Albuminosis
C. Decrease in the concentration of amino acids in blood
D. Increase in the oncotic pressure in the blood plasma
E. Increase in low-density lipoprotein level
83. A 67-year-old male patient consumes eggs, pork fat, butter, milk and meat.
Blood test results:
cholesterol - 12,3 mmol/l, total lipids - 8,2 g/l, increased low-density lipoprotein
fraction (LDL).
What type of hyperlipoproteinemia is observed in the patient?
A. Hyperlipoproteinemia type IIa
B. Hyperlipoproteinemia type I
C. Hyperlipoproteinemia type IIb
D. Hyperlipoproteinemia type IV
E. Cholesterol, hyperlipoproteinemia
84. A patient has severe blood loss caused by an injury. What kind of dehydration
will be observed
in this particular case?
A. Iso-osmolar
B. Hyposmolar
C. Hyperosmolar
D. Normosmolar
E. –
85. A patient has the oxyhemoglobin dissociation curve shifted to the left. What
blood changes
induce this condition?
A. Alkalosis, hypocapnia, temperature drop
B. Acidosis, hypercapnia, temperature rise
C. Acidosis, hypercapnia, temperature drop
D. Acidosis, hypocapnia, temperature rise
E. –
86. A patient has been diagnosed with alkaptonuria. Choose an enzyme that can
cause this
pathology when deficient:
A. Homogentisic acid oxidase
B. Phenylalanine hydroxylase
C. Glutamate dehydrogenase
D. Pyruvate dehydrogenase
E. Dioxyphenylalanine decarboxylase
87. An infant has pylorospasm, weakness, hypodynamia, convulsions as a result of
frequent
vomiting. What kind of acid-base disbalance is it?
A. Excretory alkalosis
B. Excretory acidosis
C. Metabolic acidosis
D. Exogenous nongaseous acidosis
E. Gaseous alkalosis
88. A 30-year-old man with diabetes mellitus type I was hospitalised. The patient
is comatose.
Laboratory tests revealed hyperglycemia and ketonemia. What metabolic disorder
can be detected
in this patient?
A. Metabolic acidosis
B. Metabolic alkalosis
C. Respiratory acidosis
D. Respiratory alkalosis
E. Normal acid-base balance
89. A 15-year-old patient has fasting plasma glucose level 4,8 mmol/l, one hour
after glucose
challenge it becomes 9,0 mmol/l, in 2 hours it is 7,0 mmol/l, in 3 hours it is 4,8
mmol/l. Such
parameters are characteristic of:
A. Subclinical diabetes mellitus
B. Diabetes mellitus type 1
C. Diabetes mellitus type 2
D. Healthy person
E. Cushing’s disease
90. A 6-year-old child suffers from delayed growth, disrupted ossification
processes,
decalcification of the teeth.What can be the cause?
A. Vitamin D deficiency
B. Decreased glucagon production
C. Insulin deficiency
D. Hyperthyroidism
E. Vitamin C deficiency
91. A woman complains of visual impairment. Examination revealed obesity in the
patient and her
fasting plasma glucose level is hyperglycemic. What diabetes complication can
cause visual
impairment/blindness?
A. Microangiopathy
B. Macroangiopathy
C. Atherosclerosis
D. Neuropathy
E. Glomerulopathy
92. A patient suffers from disrupted patency of the airways at the level of small
and medium-sized
bronchial tubes. What changes of acid-base balance can occur in the patient?
A. Respiratory acidosis
B. Respiratory alkalosis
C. Metabolic acidosis
D. Metabolic alkalosis
E. Acid-base balance remains unchanged
93. Upon toxic damage of hepatic cells resulting in disruption of liver function the
patient
developed edemas. What changes of blood plasma are the main cause of edema
development?
A. Decrease of albumin content
B. Increase of globulin conten
C. Decrease of fibrinogen conten
D. Increase of albumin conten
E. Decrease of globulin conten
94. Atria of an experimental animal were superdistended with blood, which
resulted in decreased
reabsorption of Na+
and water in renal tubules. This can be explained by the influence of the
following factor on kidneys:
A. Natriuretic hormone
B. Aldosterone
C. Renin
D. Angiotensin
E. Vasopressin
95. A patient with insulin-dependent diabetes mellitus has been administered
insulin. After a
certain period of time the patient developed fatigue, irritability, excessive sweating.
What is the
main mechanism of such presentations developing?
A. Carbohydrate starvation of the brain
B. Increased glycogenolysis
C. Increased ketogenesis
D. Increased lipogenesis
E. Decreased glyconeogenesis
96. Leading symptoms of primary hyperparathyroidism are osteoporosis and renal
damage
resulting in urolithiasis development. What substances are the basis of uroliths in
such cases?
A. Calcium phosphate
B. Uric acid
C. Cysteine
D. Bilirubin
E. Cholesterol
97. A man has suffered multiple bone fractures of his lower extremities during a
traffic accident.
During transportation to a hospital his condition was further aggravated: blood
pressure decreased,
there were signs of pulmonary artery embolism. What kind of embolism is the
most likely in the
given case?
A. Fat embolism
B. Air embolism
C. Gas embolism
D. Tissue embolism
E. Thromboembolism
98. A 7-year-old child suffers from delayed growth, disrupted ossification
processes,
decalcification of the teeth. What can be the cause?
A. Vitamin D deficiency
B. Decreased glucagon production
C. Insulin deficiency
D. Hyperthyroidism
E. Vitamin C deficiency
99. A man complains of visual impairment. Examination revealed obesity in the
patient and her
fasting plasma glucose level is hyperglycemic. What diabetes complication can
cause visual
impairment/blindness?
A. Microangiopathy
B. Macroangiopathy
C. Atherosclerosis
D. Neuropathy
E. Glomerulopathy
Module 4
1. A patient with hypochromic anemia has splitting and loss of hair, increased nail
brittling and
taste alteration. What is the mechanism of the symptoms development?
A. Deficiency of iron-containing enzymes
B. Deficiency of vitamin _12
C. Decreased production of parathyrin
D. Deficiency of vitamin А
E. Decreased
2. In the blood of a 26-year-old man 18%of erythrocytes of the spherical,
ballshaped, flat and
thorn-like shape have been revealed. Other eritrocytes were in the form of the
concavo-concave
disks. How is this phenomenon called?
A. Physiological poikilocytosis
B. Pathological poikilocytosis
C. Physiological anisocytosis
D. Pathological anisocytosis
E. Erytrocytosis
3. There is an inhibited coagulation in the patients with bile ducts obstruction,
bleeding due to the
low level of absorbtion of a vitamin. What vitamin is in deficiency?
A. К
B. А
C. D
D. Е
E. Carotene
4. Patient 54 year-old, 5th day after surgical operation. Blood count: Erythrocytes
3, 6 ∗ 1012/l,
Hemoglobin 95 g/l, Erythrocyte’s hemoglobin content (color index) 0,78;
Leukocytes 16 ∗ 109/l,
Platelets 450 ∗ 109/l Blood picture: anizocytosis, poikilocytosis, reticulocytes-
3,8%. What anemia
does this patient have?
A. Acute posthemorragic anemia
B. Acquired hemolytic anemia
C. Anemia from iron deficiency
D. Hypoplastic anemia
E. Chronic posthemorragic anemia
5. A patient’s blood was analyzed and the decreased erythrocyte’s sedimentation
rate (ESR) was
discovered. What disease from the listed below is accompanied with decreased
ESR?
A. Polycytemia
B. Hepatitis
C. Splenomegaly
D. Vitamin B deficiency
E. Myocardial infarction
6. Substitution of the glutamic acid on valine was revealed while examining initial
molecular
structure. For what inherited pathology is this symptom typical?
A. Sickle-cell anemia
B. Thalassemia
C. Minkowsky-Shauffard disease
D. Favism
E. Hemoglobinosis
7. Punctata hemorrhage was found out in the patient after application of a
tourniquet. With
disfunction of what blood cells is it connected?
A. Platelets
B. Eosinophiles
C. Monocytes
D. Lymphocytes
E. Neutrophiles
8. A 2-year-old child has got intestinal dysbacteriosis, which results in
hemorrhagic syndrome.
What is the most likely cause of hemorrhage of the child?
A. Vitamin K insufficiency
B. Activation of tissue thromboplastin
C. PP hypovitaminosis
D. Fibrinogen deficiency
E. Hypocalcemia
9. In course of an experiment a big number of stem cells of red bone marrow was
in some way
destructed. Regeneration of which cell populations in the loose connective tissue
will be inhibited?
A. Of macrophags
B. Of fibroblasts
C. Of pigment cells
D. Of lipocytes
E. Of pericytes
10. Examination of a 43 y.o. anephric patient revealed anemia symptoms. What is
the cause of
these symptoms?
A. Reduced synthesis of erythropoietins
B. Enhanced destruction of erythrocytes
C. Iron deficit
D. Vitamin B12 deficit
E. Folic acid deficit
11. 24 hours after appendectomy blood of a patient presents neutrophilic
leukocytosis with
regenerative shift. What is the most probable mechanism of leukocytosis
development?
A. Amplification of leukopoiesis
B. Redistribution of leukocytes in the organism
C. Decelerated leukocyte destruction
D. Deceleratied emigration of leukocytes to the tissues
E. Amplification of leukopoiesis and decelerated emigration of leukocytes to the
tissues
12. A patient was ill with burn disease that was complicated by DIC syndrome.
What stage of DIC
syndrome can be suspected if it is known that the patient’s blood coagulates in less
than 3 minutes?
A. Hypercoagulation
B. Transition phase
C. Hypocoagulation
D. Fibrinolysis
E. Terminal
13. A 55 y.o. woman consulted a doctor about having continuous cyclic uterine
hemorrhages for a
year, weakness, dizziness. Examination revealed skin pallor. Hemogram: Hb- 70
g/l, erythrocytes -
3, 2 · 1012/l, color index - 0,6, leukocytes - 6, 0·109/l, reticulocytes - 1%;
erythrocyte hypochromia.
What anemia is it?
A. Chronic posthemorrhagic anemia
B. Hemolytic anemia
C. Aplastic anemia
D. B12-folate-deficiency anemia
E. Iron-deficiency anemia
14. A 23 y.o. patient complains of weakness, temperature rise up to 38-40 C.
Objectively: liver and
spleen are enlarged. Hemogram: Hb-100 g/l, erythrocytes - 2, 9 · 1012/l,
leukocytes - 4, 4 · 109/l,
thrombocytes - 48 · 109/l, segmentonuclear neutrophils - 17%, lymphocytes - 15%,
blast cells -
68%. All cytochemical reactions are negative. Make a hematological conclusion:
A. Undifferentiated leukosis
B. Chronic myeloleukosis
C. Acute myeloblastic leukosis
D. Acute lymphoblastic leukosis
E. Acute erythromyelosis
15. A 3 year old child with fever was given aspirin. It resulted in intensified
erythrocyte
haemolysis. Hemolytic anemia might have been caused by congenital insufficiency
of the following
enzyme:
A. Glucose 6-phosphate dehydrogenase
B. Glucose 6-phosphatase
C. Glycogen phosphorylase
D. Glycerol phosphate dehydrogenase
E. γ-glutamiltransferase
16. A 20 year old patient complains of general weakness, dizziness, quick
fatigability. Blood
analysis results: Hb-80 g/l. Microscopical examination results: erythrocytes are of
modified form.
This condition might be caused by:
A. Sickle-cell anemia
B. Hepatocellular jaundice
C. Acute intermittent porphyria
D. Obturative jaundice
E. Addison’s disease
17. 2 years ago a patient underwent resection of pyloric part of stomach. He
complains of
weakness, periodical dark shadows beneath his eyes, dyspnea. In blood: Hb - 70
g/l, erythrocytes -
3, 0 ·1012/l, colour index - 0,7. What changes of erythrocytes in blood smears are
the most typical
for this condition?
A. Microcytes
B. Megalocytes
C. Schizocytes
D. Ovalocytes
E. Macrocytes
18. A 56 year old patient came to a hospital with complaints about general
weakness, tongue pain
and burning, sensation of limb numbness. In the past he underwent resection of
forestomach. In
blood: Hb-80 g/l; erythrocytes - 2, 0·1012/l; colour index - 1,2, leukocytes - 3, 5 ·
109/l. What
anemia type is it?
A. B12-folate deficient
B. Hemolytic
C. Posthemorrhagic
D. Aplastic
E. Iron-deficient
19. Two hours after an exam a student had a blood count done and it was revealed
that he had
leukocytosis without significant leukogram modifications. What is the most
probable mechanism of
leukocytosis development?
A. Redistribution of leukocytes in the organism
B. Leukopoiesis intensification
C. Deceleration of leukocyte lysis
D. Deceleration of leukocyte migration to the tissues
E. Leukopoiesis intensification and deceleration of leukocyte lysis
20. A 16 year old boy after an illness has diminished function of protein synthesis
in liver as a
result of vitamin K deficiency. It will cause disturbance of:
A. Blood coagulation
B. Erythrocyte sedimentation rate
C. Anticoagulant generation
D. Erythropoietin secretion
E. Osmotic blood pressure
21. A patient was diagnosed with autoimmune hemolitic cytotoxic anemia. What
substances are
antigens in II type allergic reactions?
A. Modified receptors of cell membranes
B. Antibiotics
C. Hormones
D. Serum proteins
E. Inflammation modulators
22. A patient underwent a surgery for excision of a cyst on pancreas. After this he
developed
haemorrhagic syndrome with apparent disorder of blood coagulation. Development
of this
complication can be explained by:
A. Activation of fibrinolytic system
B. Insufficient fibrin production
C. Reduced number of thrombocytes
D. Activation of anticoagulation system
E. Activation of Christmas factor
23. A mother consulted a doctor about her 5-year-old child who develops
erythemas, vesicular rash
and skin itch under the influence of sun. Laboratory studies revealed decreased
iron concentration
in the blood serum, increased uroporphyrinogen I excretion with the urine. What is
the most likely
inherited pathology in this child?
A. Erythropoietic porphyria
B. Methemoglobinemia
C. Hepatic porphyria
D. Coproporphyria
E. Intermittent porphyria
24. A 38-year-old patient with an uterine haemorrhage lasting for 2 days was
delivered to the
admission ward. Which of the following will be revealed in the patient’s blood?
A. Decrease in the haematocrite index
B. Eosinophilia
C. Deceleration in ESR
D. Leukocytosis
E. Increase in the colour index
25. A 70-year-old patient suffers from atherosclerosis complicated by the lower
limb thrombosis
that has caused gangrene on his left toes. What is the most likely cause of the
thrombosis origin?
A. Thrombocyte adhesion
B. Prothrombinase activation
C. Transformation of prothrombin into thrombin
D. Transformation of fibrinogen into fibrin
E. Impaired heparin synthesis
26. As a result of increased permeability of the erythrocyte membrane in a patient
with
microspherocytic anaemia (Minkowsky-Shauffard disease) cells receive sodium
ions and water.
Erythrocytes take form of spherocytes and can be easily broken down. What is the
leading
mechanism of erythrocyte damage in this case?
A. Electrolytic osmotic
B. Calcium
C. Acidotic
D. Protein
E. Nucleic
27. Examination of a patient admitted to the surgical department with symptoms of
acute
appendicitis revealed the following changes in the white blood cells: the total count
of leukocytes is
16 · 109/l. Leukocyte formula: basophils - 0, eosinophils - 2%, juvenile forms -
2%, stabnuclear -
8%, segmentonuclear - 59%, lymphocytes - 25%, monocytes- 4%. The described
changes can be
classified as:
A. Neutrophilia with regenerative left shift
B. Neutrophilia with right shift
C. Neutrophilia with degenerative left shift
D. Neutrophilic leukemoid reaction
E. Neutrophilia with hyperregenerative left shift
28. A patient suffers from the haemorrhagic syndrome that shows itself in frequent
nasal bleedings,
posttraumatic and spontaneous intracutaneous and intra-articular haemorrhages.
After a laboratory
study a patient was diagnosed with the type B haemophilia. This disease is
provoked by the deficit
of the following factor of blood coagulation:
A. IX
B. VIII
C. XI
D. V
E. VII
29. A 34 year old woman was diagnosed with hereditary microspherocytic
hemolytic anemia
(Minkowsky-Shauffard disease). What mechanism caused haemolysis of
erythrocytes?
A. Membranopathy
B. Enzymopathy
C. Hemoglobinopathy
D. Autoimmune disorder
E. Bone marrow hypoploasia
30. A 25 year old Palestinian woman complains of weakness, dizziness, dyspnea.
In anamnesis:
periodically exacerbating anemia. In blood: Hb – 60 g/l, erythrocytes - 2,5 · 1012/l,
reticulocytes –
35 %, anisocytosis and poikilocytosis of erythrocytes, a lot of target cells and
polychromatophils.
What type of anemia is it?
A. Thalassemia
B. Sickle-cell anemia
C. Minkowsky-Shauffard disease
D. Addison-Biermer disease
E. Glucose 6-phosphate dehydrogenasedeficient anemia
31. A 47 year old man with myocardium infarction was admitted to the
cardiological department.
What changes of cellular composition of peripheral blood are induced by necrotic
changes in the
myocardium?
A. Neutrophilic leukocytosis
B. Monocytosis
C. Eosinophilic leukocytosis
D. Thrombocytopenia
E. Lymphopenia
32. A 15 year old girl has pale skin, glossitis, gingivitis. Blood count: erythrocytes
- 3,3 * 1012/l,
hemoglobin - 70 g/l, colour index - 0,5. Examination of blood smear revealed
hypochromia,
microcytosis, poikilocytosis. What type of anemia is it?
A. Iron-deficient
B. B12-folic acid-deficient
C. Sickle-cell
D. Hemolytic
E. Thalassemia
33. A disaster fighter at a nuclear power plant developed hemorrhagic syndrome on
the background
of acute radiation disease. What is the most important factor of syndrome
pathogenesis?
A. Thrombocytopenia
B. Vascular wall damage
C. Increased activity of fibrinolysis factors
D. Increased activity of anticoagulative system factors
E. Decreased activity of coagulative factors
34. A tooth extraction in a patient with chronic persistent hepatitis was complicated
with prolonged
hemorrhage. What is the reason for the haemorrhagic syndrome?
A. Decrease in thrombin production
B. Increase in thromboplastin production
C. Decrease in fibrin production
D. Increase in fibrinogen synthesis
E. Fibrinolysis intensification
35. After a tourniquet application a patient was found to have petechial
haemorrhages. The reason
for it is the dysfunction of the following cells:
A. Platelets
B. Eosinophils
C. Monocytes
D. Lymphocytes
E. Neutrophils
36. After implantation of a cardiac valve a young man constantly takes indirect
anticoagulants. His
state was complicated by hemorrhage. What substance content has decreased in
blood?
A. Prothrombin
B. Haptoglobin
C. Heparin
D. Creatin
E. Ceruloplasmin
37. A patient suffering from chronic myeloleukemia has got the following
symptoms of anemia:
decreased number of erythrocytes and low haemoglobin concentration, oxyphilic
and
polychromatophilic normocytes, microcytes. What is the leading pathogenetic
mechanism of
anemia development?
A. Substitution of haemoblast
B. Intravascular hemolysis of erythrocytes
C. Deficiency of vitamin B12
D. Reduced synthesis of erythropoietin
E. Chronic haemorrhage
38. A 42 year old patient complains of pain in the epigastral area, vomiting; vomit
masses have the
colour of "coffee-grounds", the patient has also melena. Anamnesis records gastric
ulcer. Blood
formula: erythrocytes - 2,8 * 1012/l, leukocytes – 8 * 109
/l, Hb - 90 g/l. What complication is it?
A. Haemorrhage
B. Penetration
C. Perforation
D. Canceration
E. Pyloric stenosis
39. A disaster fighter at a nuclear power plant developed hemorrhagic syndrome on
the background
of acute radiation disease. What is the most important factor of syndrome
pathogenesis?
A. Thrombocytopenia
B. Vascular wall damage
C. Increased activity of fibrinolysis factors
D. Increased activity of anticoagulative system factors
E. Decreased activity of coagulative factors
40. A 26 year old pregnant woman is under treatment at an in-patient hospital.
After a continuous
attack of vomiting she was found to have reduced volume of circulating blood.
What kind of change
in general blood volume is the case?
A. Polycythemic hypovolemia
B. Simple hypovolemia
C. Oligocythemic hypovolemia
D. Polycythemic hypervolemia
E. Oligocythemic hypervolemia
41. Hemoglobin catabolism results in release of iron which is transported to the
bone marrow by a
certain transfer protein and used again for the synthesis of hemoglobin. Specify
this transfer protein:
A. Transferrin (siderophilin)
B. Transcobalamin
C. Haptoglobin
D. Ceruloplasmin
E. Albumin
42. A 3-year-old boy with pronounced hemorrhagic syndrome doesn’t have
antihemophilic
globulin A (factor VIII) in the blood plasma. Hemostasis has been impaired at the
following stage:
A. Internal mechanism of prothrombinase activation
B. External mechanism of prothrombinase activation
C. Conversion of prothrombin to thrombin
D. Conversion of fibrinogen to fibrin
E. Blood clot retraction
43. On the fifth day after the acute blood loss a patient has been diagnosed with
hypochromic
anemia. What is the main mechanism of hypochromia development?
A. Release of immature red blood cells from the bone marrow
B. Impaired iron absorption in the intestines
C. Increased destruction of red blood cells in the spleen
D. Impaired globin synthesis
E. Increased excretion of body iron
44. A 12-year-old patient has been admitted to a hospital for hemarthrosis of the
knee joint. From
early childhood he suffers from frequent bleedings. Diagnose the boy’s disease:
A. Hemophilia
B. Hemorrhagic vasculitis
C. Hemolytic anemia
D. B12 (folic acid)-deficiency anemia
E. Thrombocytopenic purpura
45. In a car accident a man got injured and lost a lot of blood. What changes in
peripheral blood are
most likely to occur on the 2nd day after the injury?
A. Erythropenia
B. Hypochromia
C. Anisocytosis
D. Microplania
E. Significant reticulocytosis
46. After the prolonged vomiting a pregnant 26-year-old woman was found to have
the reduced
volume of circulating blood. What change in the total blood volume can be the
case?
A. Polycythemic hypovolemia
B. Simple hypovolemia
C. Oligocythemic hypovolemia
D. Polycythemic hypervolemia
E. Oligocythemic hypervolemia
47. Examination of a 52-year-old female patient has revealed a decrease in the
amount of red blood
cells and an increase in free hemoglobin in the blood plasma (hemoglobinemia).
Color index is
0,85. What type of anemia is being observed in the patient?
A. Acquired hemolytic
B. Hereditary hemolytic
C. Acute hemorrhagic
D. Chronic hemorrhagic
E. Anemia due to diminished erythropoiesis
48. A patient is diagnosed with hereditary coagulopathy that is characterized by
factor VIII
deficiency. Specify the phase of blood clotting during which coagulation will be
disrupted in the
given case:
A. Thromboplastin formation
B. Thrombin formation
C. Fibrin formation
D. Clot retraction
E. –
49. A 37-year-old female patient complains of headache, vertigo, troubled sleep,
numbness of
limbs. For the last 6 years she has been working at the gas-discharge lamp-
producing factory in the
lead-processing shop. Blood test findings: low hemoglobin and RBC level, serum
iron
concentration exceeds the norm by several times. Specify the type of anemia:
A. Iron refractory anemia
B. Iron-deficiency anemia
C. Minkowsky-Shauffard disease
D. Hypoplastic anemia
E. Metaplastic anemia
50. A patient is diagnosed with iron-deficiency sideroachrestic anemia, progression
of which is
characterized by skin hyperpigmentation, pigmentary cirrhosis, heart and pancreas
affection. Iron
level in the blood serum is increased. What disorder of iron metabolism causes this
disease?
A. Failure to assimilate iron leading to iron accumulation in tissues
B. Excessive iron intake with food
C. Disorder of iron absorption in bowels
D. Increased iron assimilation by body
E. –
51. A blood drop has been put into a test tube with 0,3% solution of NaCl. What
will happen to
erythrocytes?
A. Osmotic haemolysis
B. Shrinkage
C. Mechanical haemolysis
D. Any changes will be observed
E. Biological haemolysis
52. Along with normal hemoglobin types there can be pathological ones in the
organism of an
adult. Name one of them:
A. HbS
B. HbF
C. HbA1
D. HbA2
E. HbO2
53. A patient is diagnosed with hereditary coagulopathy that is characterized by
factor VIII
deficiency. Specify the phase of blood clotting during which coagulation will be
disrupted in the
given case:
A. Thromboplastin formation
B. Thrombin formation
C. Fibrin formation
D. Clot retraction
E. –
54. A 27-year-old girl complains of headache, vertigo, troubled sleep, numbness of
limbs. For the
last 6 years she has been working at a gas-discharge lamp-producing factory in a
lead-processing
shop. Blood test findings: low hemoglobin and RBC level, serum iron
concentration exceeds the
norm by several times. Specify the type of anemia:
A. Iron refractory anemia
B. Iron-deficiency anemia
C. Minkowsky-Shauffard disease
D. Hypoplastic anemia
E. Metaplastic anemia
55. A patient is diagnosed with chronic atrophic gastritis attended by deficiency of
Castle’s
intrinsic factor.What type of anemia does the patient have?
A. B12-deficiency anemia
B. Iron refractory anemia
C. Hemolytic anemia
D. Iron-deficiency anemia
E. Protein-deficiency anemia
56. Biochemical analysis of an infant’s erythrocytes revealed evident glutathione
peroxidase
deficiency and low concentration of reduced glutathione. What pathological
condition can develop
in this infant?
A. Hemolytic anemia
B. Pernicious anemia
C. Megaloblastic anemia
D. Sicklemia
E. Iron-deficiency anemia
57. A 59-year-old woman has been hospitalized in a surgical ward due to
exacerbation of chronic
osteomyelitis of the left shin. Blood test: leukocytes – 15,0·109
/l. Leukogram: myelocytes – 0%,
metamyelocytes – 8%, stab neutrophils – 28%, segmented neutrophils – 32%,
lymphocytes – 29%,
monocytes – 3%. Such blood count would be called:
A. Regenerative left shift
B. Right shift
C. Hyperregenerative left shift
D. Degenerative left shift
E. Regenerative-degenerative left shift
58. A 42-year-old patient complains of pain in the epigastral area, vomiting; vomit
masses have the
color of coffee-grounds; the patient suffers from melena. Anamnesis records
gastric ulcer disease.
Blood formula: erythrocytes – 2,8·1012/l, leukocytes – 8·109
/l, Hb – 90 g/l. What complication is it?
A. Hemorrhage
B. Penetration
C. Perforation
D. Canceration
E. Pyloric stenosis
59. Examination of a 52-year-old woman has revealed a decrease in the amount of
red blood cells
and an increase in free hemoglobin in the blood plasma (hemoglobinemia). Color
index is 0,85.
60. What type of anemia is being observed in the patient?
A. Acquired hemolytic
B. Hereditary hemolytic
C. Acute hemorrhagic
D. Chronic hemorrhagic
E. Anemia due to diminished erythropoiesis
61. Along with normal hemoglobin types there can be pathological ones in the
organism of an
adult. Name one of them:
A. HbS
B. HbF
C. HbA1
D. HbA2
E. HbO2
62. A 50-year-old man suffering from chronic hepatitis frequently observes nasal
and gingival
hemorrhages, spontaneous hemorrhagic rashes on the skin and mucosa. Such
presentations result
from:
A. Decreased synthesis of prothrombin and fibrinogen
B. Increased blood content of aminotransferases
C. Decreased synthesis of serum albumins
D. Increased blood content of macroglobulins and cryoglobulins
E. Decreased blood content of cholinesterase
63. A 30-year-old patient’s blood test has revealed the following: erythrocyte count
is 6·1012/l,
hemoglobin is 10.55 mmol/l. Vaquez’s disease was diagnosed. Name the leading
part of
pathogenesis:
A. Neoplastic erythroid hyperplasia
B. Iron-deficiency
C. B12-deficiency
D. Hypoxia
E. Acidosis
64. A patient is 20 years old, an athlete. He addressed a doctor with complaints of
fatigue, fever up
to 38oC – 40oC. Objectively: the liver and spleen are enlarged, lymph nodes on
palpation are
slightly enlarged, dense, painless. Blood test: Нb – 100 g/l; erythrocytes –
2.9·1012/l; leukocytes –
4.4·109
/l. Leukogram: 68% of blast cells. Cytochemical investigation of blast cells
revealed
negative reactions to glycogen, peroxidase, non-specific esterase, lipids. Name this
disease:
A. Acute undifferentiated leukemia
B. Acute myeloid leukemia
C. Acute monoblastic leukemia
D. Acute lymphoblastic leukemia
E. Acute megakaryoblastic leukemia
Module 5
1. A patient in three weeks after acute myocardial infarction has pain in the heart
and joints and
pneumonia. What is the main mechanism of development of postinfarction
Dressler’s syndrome?
A. Autoimmune inflammation
B. Ischemia of myocardium
C. Resorption of enzymes from necrotized area of myocardium
D. Secondary infection
E. Vessels ’ thrombosis
2. Electrocardiogram of a 45-year-old man showed absence of P-wave in all the
leads.What part of
the conducting system is blocked?
A. Sinu-atrial node
B. Atrioventricular node
C. Common branch of the bundle of His
D. Branches of the bundle of His
E. Purkinje’s fibres
3. A person has steady HR not exceeding 40 bpm. What is the pacemaker of the
heart rhythm in
this person?
A. Atrioventricular node
B. Sinoatrial node
C. His’ bundle
D. Branches of His’ bundle
E. Purkinye’ fibers
4. X-ray examination discovered lungs emphysema in the patient. What is the
reason of short
breath development in this case?
A. Decreased lungs elasticity
B. Increased lungs elasticity
C. Inhibition of respiratory center
D. Excitation of respiratory center
E. Decreasing of alveoli receptors sensitivity
5. A 62-year-old patient was admitted to the neurological department due to
cerebral haemorrage.
His condition is grave. There is evident progression of deep and frequent breath
that turnes into
reduction to apnoea and the cycle repeates. What respiration type has developed in
the patient?
A. Cheyne-Stockes respiration
B. Kussmaul respiration
C. Biot’s respiration
D. Gasping respiration
E. Apneustic respiration
6. After the trauma, the patient’s right n.vagus was damaged. Which violation of
the cardiac
activity is possible in this case?
A. Violation of the automatism of a Kiss-Fleck node
B. Violation of the automatism of a atrioventricular node
C. Violation of a conductivity in the right auricle
D. Block of a conductivity in the atrioventricular node
E. Arrhythmia
7. A patient has a transverse disruption of spinal cord below the IV thoracic
segment. What
changes of respiration will it cause?
A. Respiration will stay unchanged
B. Respiration will stop
C. Respiration will become less frequent
D. Respiration will become deeper
E. Respiration will become more frequent
8. 12 hours after an accute attack of retrosternal pain a patient presented a jump of
aspartate
aminotransferase activity in blood serum. What pathology is this deviation typical
for?
A. Myocardium infarction
B. Viral hepatitis
C. Collagenosis
D. Diabetes mellitus
E. Diabetes insipidus
9. A peripheral segment of vagus nerve on a dog’s neck was being stimulated in
course of an
experiment. The following changes of cardiac activity could be meanwhile
observed:
A. Heart rate fall
B. Heart hurry
C. Enhancement of atrioventricular conduction
D. Heart rate and heart force amplification
E. Increased excitability of myocardium
10. A 12 y.o. boy who suffers from bronchial asthma has an acute attack of
asthma: evident
expiratory dyspnea, skin pallor. What type of alveolar ventilation disturbance is it?
A. Obstructive
B. Restrictive
C. Throracodiaphragmatic
D. Central
E. Neuromuscular
11. A group of mountain climbers went through the blood analysis at the height of
3000 m. It
revealed decrease of HCO3 to 15 micromole/l (standard is 22- 26 micromole/l).
What is the
mechanism of HCO3 decrease?
A. Hyperventilation
B. Intensification of acidogenesis
C. Hypoventilation
D. Decrease of ammoniogenesis
E. Decrease of bicarbonate reabsorption in kidneys
12. A patient who suffers from severe disorder of water-salt metabolism
experienced cardiac arrest
in diastole. What is the most probable mechanism of cardiac arrest in diastole?
A. Hyperkaliemia
B. Hypernatremia
C. Organism dehydratation
D. Hypokaliemia
E. Hyponatremia
13. Examination of coronary arteries revealed atherosclerotic calcific plaques that
close vessel
lumen by 1/3. The muscle has multiple whitish layers of connective tissue. What
process was
revealed in myocardium?
A. Diffuse cardiosclerosis
B. Tiger heart
C. Postinfarction cardiosclerosis
D. Myocarditis
E. Myocardium infarction
14. Examination of a miner revealed pulmonary fibrosis accompanied by
disturbance of alveolar
ventilation. What is the main mechanism of this disturbance?
A. Limitation of respiratory surface of lungs
B. Constriction of superior respiratory tracts
C. Disturbance of neural respiration control
D. Limitation of breast mobility
E. Bronchi spasm
15. A 49 y.o. woman consulted a doctor about heightened fatigue and dyspnea
during physical
activity. ECG: heart rate is 50/min, PQ is extended, QRS is unchanged, P wave
quanity exceeds
quantity of QRS complexes. What type of arrhythmia does the patient have?
A. Atrioventricular block
B. Extrasystole
C. Sinus bradycardia
D. Ciliary arhythmia
E. Sinoatrial block
16. A patient is followed up in an endocrinological dispensary on account of
hyperthyreosis.
Weight loss, tachycardia, finger tremor are accompanied by hypoxia symptoms -
headache, fatigue,
eye flicker. What mechanism of thyroid hormones action underlies the
development of hypoxia?
A. Disjunction, oxydation and phosphorilation
B. Inhibition of respiratory ferment synthesis
C. Competitive inhibition of respiratory ferments
D. Intensification of respiratory ferment synthesis
E. Specific binding of active centres of respiratory ferments
17. A 56 y.o. patient has been suffering from thyreotoxicosis for a long time. What
type of hypoxia
can be developed?
A. Tissue
B. Hemic
C. Circulatory
D. Respiratory
E. Mixed
18. Apatient has extrasystole. ECG shows no P wave, QRS complex is deformed,
there is a full
compensatory pause. What extrasystoles are these?
A. Ventricular
B. Atrial
C. Atrioventricular
D. Sinus
E. –
19. ECG of a patient with hyperfunction of thyroid gland showed heart hurry. It is
indicated by
depression of the following ECG element:
A. R − R interval
B. P − Q segment
C. P − Q interval
D. P − T interval
E. QRS complex
20. An animal with aortic valve insufficiency got hypertrophy of its left heart
ventricle. Some of its
parts have local contractures. What substance accumulated in the myocardiocytes
caused these
contractures?
A. Calcium
B. Potassium
C. Lactic acid
D. Carbon dioxide
E. Sodium
21. Prophylactic medical examination of a 36 year old driver revealed that his AP
was 150/90 mm
Hg. At the end of working day he usually hears ear noise, feels slight indisposition
that passes after
some rest. He was diagnosed with essential hypertension. What is the leading
pathogenetic
mechanism in this case?
A. Neurogenetic
B. Nephric
C. Humoral
D. Endocrinal
E. Reflexogenic
22. An unconscious young man with signs of morphine poisoning entered
admission office. His
respiration is shallow and infrequent which is caused by inhibition of respiratory
centre. What type
of respiratory failure is it?
A. Ventilative dysregulatory
B. Ventilative obstructive
C. Ventilative restrictive
D. Perfusive
E. Diffusive
23. In course of a preventive examination of a miner a doctor revealed changes of
cardiovascular
fitness which was indicative of cardiac insufficiency at the compensation stage.
What is the main
proof of cardiac compensation?
A. Myocardium hypertrophy
B. Tachycardia
C. Rise of arterial pressure
D. Dyspnea
E. Cyanosis
24. In course of an experiment a peripheral section of vagus of an expiremental
animal is being
stimulated. What changes will be observed?
A. Heart rate fall
B. Heart hurry
C. Pupil dilation
D. Increase of respiration rate
E. Bronchi dilation
25. Vagi of an experimental animal were cut on both sides. What respiration
changes will be
observed?
A. It will become deep and infrequent
B. It will become shallow and frequent
C. It will become deep and frequent
D. It will become shallow and infrequent
E. No changes will be observed
26. A patient ill with essential arterial hypertension had a hypertensic crisis that
resulted in an
attack of cardiac asthma. What is the leading mechanism of cardiac insufficiency
in this case?
A. Heart overload caused by high pressure
B. Heart overload caused by increased blood volume
C. Absolute coronary insufficiency
D. Myocardium damage
E. Blood supply disturbance
27. ECG of a 44-year-old patient shows signs of hypertrophy of both ventricles and
the right
atrium. The patient was diagnosed with the tricuspid valve insufficiency. What
pathogenetic variant
of cardiac dysfunction is usually observed in case of such insufficiency?
A. Heart overload by volume
B. Heart overload by resistance
C. Primary myocardial insufficiency
D. Coronary insufficiency
E. Cardiac tamponade
28. After a serious psycho-emotional stress a 45-year-old patient suddenly felt
constricting heart
pain irradiating to the left arm, neck and left scapula. His face turned pale, the cold
sweat stood out
on it. The pain attack was stopped with nitroglycerine. What process has developed
in this patient?
A. Stenocardia
B. Myocardial infarction
C. Stroke
D. Psychogenic shock
E. Stomach ulcer perforation
29. While eating a child choked on food and aspirated it. The child has severe
cough, cyanotic skin
and mucous membranes, rapid pulse, infrequent respiration, prolonged expiration.
The child has
developed the following disorder of the external respiration:
A. Expiratory dyspnea under asphyxia
B. Inspiratory dyspnea under asphyxia
C. Stenotic respiration
D. Alternating respiration
E. Biot’s respiration
30. A 12-year-old adolescent suffering from bronchial asthma has a severe attack
of asthma: he
presents with marked expiratory dyspnea, skin pallor. What type of alveolar
ventilation disorder is
observed?
A. Obstructive
B. Restrictive
C. Thoracodiaphragmatic
D. Central
E. Neuromuscular
31. ECG of a patient shows such alterations: P-wave is normal, P − Q-interval is
short, ventricular
QRST complex is wide, R-wave is double-peak or two-phase. What form of
arrhythmia is it?
A. WPW syndrome (Wolff-Parkinson-White)
B. Frederick’s syndrome (atrial flutter)
C. Atrioventricular block
D. Ventricular fibrillation
E. Ciliary arrhythmia
32. In a healthy adult speed of the excitement conduction through the
atrioventricular node is 0,02-
0,05 m/sec. Atrioventricular delay enables:
A. Sequence of atrial and ventricular contractions
B. Simultaneity of both atria contractions
C. Simultaneity of both ventricles contractions
D. Sufficient force of atrial contractions
E. Sufficient force of ventricular contractions
33. A 49-year-old patient consulted a doctor about increased fatigability and
dyspnea provoked by
physical activity. ECG results: heart rate - 50/min, PQ-interval is prolonged, QRS-
complex is
unchanged, the number of P-waves exceeds the number of QRS-complexes. What
type of
arrhythmia is it?
A. Atrioventricular block
B. Extrasystole
C. Sinus bradycardia
D. Ciliary arrhythmia
E. Sinoatrial block
34. A 58-year-old patient suffers from the cerebral atherosclerosis. Examination
revealed
hyperlipoidemia. What class of lipoproteins will most probably show increase in
concentration in
this patient’s blood serum?
A. Low-density lipoproteins
B. High-density lipoproteins
C. Fatty acid complexes with albumins
D. Chylomicrons
E. Cholesterol
35. A 63 year old male patient who had been suffering from chronic diffuse
obstructive disease,
pulmonary emphysema, for 15 years died from cardiac insufficiency. Autopsy
revealed nutmeg
liver cirrhosis, cyanotic induration of kidneys and spleen, ascites, edemata of lower
limbs. These
changes of internal organs are typical for the following disease:
A. Chronic right-ventricular insufficiency
B. Acute right-ventricular insufficiency
C. Chronic left-ventricular insufficiency
D. Acute left-ventricular insufficiency
E. General cardiac insufficiency
36. A patient staying in the pulmonological department was diagnosed with
pulmonary emphysema
accompanied by reduced elasticity of pulmonary tissue. What type of respiration is
observed?
A. Expiratory dyspnea
B. Inspiratory dyspnea
C. Superficial respiration
D. Infrequent respiration
E. Periodic respiration
37. An unconscious young man with signs of morphine poisoning entered
admission office. His
respiration is shallow and infrequent which is caused by inhibition of respiratory
centre. What type
of respiratory failure is it?
A. Ventilative dysregulatory
B. Ventilative obstructive
C. Ventilative restrictive
D. Perfusive
E. Diffusive
38. Voluntary breath-holding caused increase of respiration depth and frequency.
The main factor
stimulating these changes of external respiration is:
A. Increased tension of CO2 in blood
B. Increased tension of O2 in blood
C. Decreased tension of O2 in blood
D. Decreased tension of CO2 in blood
E. Decreased concentration of H+
in blood
39. 12 hours after an accute attack of retrosternal pain a patient presented a jump of
aspartate
aminotransferase activity in blood serum. What pathology is this deviation typical
for?
A. Myocardium infarction
B. Viral hepatitis
C. Collagenosis
D. Diabetes mellitus
E. Diabetes insipidus
40. A 38-year-old man died in the attempt of lifting weight. He had collaptoid
state. Autopsy
revealed an extensive aneurism rupture of thoracic aorta. He suffered from visceral
syphilis during
his lifetime. What pathological process caused weakness of aortic wall, its
dilatation and rupture?
A. Vanishing of elastic fibers
B. Vanishing of collagen fibers
C. Muscle layer atrophy
D. Intima changes by shagreen leather type
E. Vascularization
41. A patient with coronary disease has been diagnosed with myocardial
hypertrophy, tachycardia
and a decrease in minute blood volume. What is the leading mechanism of cardiac
hystiocyte
damage in this case?
A. Damage of specific membrane pumps
B. Increase in α and β adrenoreceptors quantity
C. Mg2+ loss by cardiac hystiocytes
D. Ca2+ loss by cardiac hystiocytes
E. Cardiac hystiocyte dehydration
42. A 49-year old female patient has limitation of left limbs arbitrary movements.
Muscular tonus
of left hand and leg is overstrained and spasmodic, local tendon reflexes are strong,
pathological
reflexes are presented. What is the most likely development mechanism of
hypertension and
hyperreflexia?
A. Reduction of descending inhibitory influence
B. Motoneuron activation induced by stroke
C. Activation of excitatory influence from the focus of stroke
D. Activation of synaptic transmission
E. Ihibition of cerebral cortex motoneurons
43. A 55-year-old patient was admitted to the cardiological department. ECG data:
negative P wave
overlaps QRS complex, diastolic interval is prolonged after extrasystole. What
type of extrasystole
is it?
A. Atrioventricular
B. Sinus
C. Atrial
D. Ventricular
E. Bundle-branch
44. A patient with marked pneumofibrosis that developed after infiltrating
pulmonary tuberculosis
has been diagnosed with respiratory failure. What is its pathogenetic type?
A. Restrictive
B. Obstructive
C. Dysregulatory
D. Reflex
E. Apneistic
45. A 29-year-old man developed acute heart failure while running for a long time.
What changes
in ionic composition can be observed in the cardiac muscle?
A. Accumulation of Na+ and Ca2+ ions in the myocardium cells
B. Accumulation of K+ and Mg2+ ions in the myocardium cells
C. Reduction of Na+ and Ca2+ ions in the myocardium cells
D. Reduction of K+ and Mg2+ ions in the extracellular space
E. Reduction of Na+ and Ca2+ ions in the extracellular space
46. A patient has a trauma of sternocleidomastoid muscle. This caused a decrease
in value of the
following indicator of external respiration:
A. Inspiratory reserve volume
B. Expiratory reserve volume
C. Respiratory capacity
D. Residual volume
E. Functional residual lung capacity
47. After an attack of bronchial asthma a patient had his peripheral blood tested.
What changes can
be expected?
A. Eosinophilia
B. Leukopenia
C. Lymphocytosis
D. Thrombocytopenia
E. Erythrocytosis
48. A 61-year old patient suffers from essential hypertension. After a physical
stress he
experienced muscle weakness, breathlessness, cyanosis of lips, skin and face.
Respiration was
accompanied by distinctly heard bubbling rales. What mechanism underlies the
development of this
syndrome?
A. Acute left-ventricular failure
B. Chronic right-ventricular failure
C. Chronic left-ventricular failure
D. Collapse
E. Cardiac tamponade
49. From the group of children who were eating sweet sappy watermelon two kids
developed the
signs of poisoning: rapid weakness, dizziness, headache, vomiting, edema,
tachycardia, cyanosis of
mouth, ears, tips of the fingers cyanosis. High concentration of nitrates was
detected. What is the
leading mechanism of the pathogenesis of the poisoning in the two children?
A. Insufficiency of met-Hb-reductase
B. Insufficiency of superoxiddismutase
C. Block cytochrome oxidase
D. Insufficiency glutathione pyroxidase
E. Insufficiency of catalase
50. A 59-year old male patient who had been suffering from chronic diffuse
obstructive disease,
pulmonary emphysema, for 15 years died from cardiac insufficiency. Autopsy
revealed nutmeg
liver cirrhosis, cyanotic induration of kidneys and spleen, ascites, edemata of lower
limbs. These
changes of internal organs are typical for the following disease:
A. Chronic right-ventricular insufficiency
B. Acute right-ventricular insufficiency
C. Chronic left-ventricular insufficiency
D. Acute left-ventricular insufficiency
E. General cardiac insufficiency
51. After a serious psychoemotional stress a 48 year old patient suddenly
developed acute heart
ache irradiating to the left arm. Nitroglycerine relieved pain after 10 minutes. What
is the leading
pathogenetic mechanism of this process development?
A. Spasm of coronary arteries
B. Dilatation of peripheral vessels
C. Obstruction of coronary vessels
D. Compression of coronary vessels
E. Increase in myocardial oxygen consumption
52. The patient with acute myocardial infarction was given intravenously different
solutions during
8 hours with medical dropper 1500 ml and oxygen intranasally. He died because of
pulmonary
edema. What caused the pulmonary edema?
A. Volume overload of the left ventricular
B. Decreased oncotic pressure due to hemodilution
C. Allergic reaction
D. Neurogenic reaction
E. Inhalation of the oxygen
53. Autopsy of a 75 year old patient who had been suffering from disseminated
atherosclerosis and
died under chronic cardiac failure revealed constriction and deformation of
coronary arteries,
tuberous intima whose section appeared to be white and petrosal. Specify the stage
of
atherosclerosis morphogenesis:
A. Atherocalcinosis
B. Lipoidosis
C. Liposclerosis
D. Bilipid
E. Atheromatosis
54. A 67 year old patient complains of periodic heart ache, dyspnea during light
physical activities.
ECG reveals extraordinary contractions of heart ventricles. Such arrhythmia is
called:
A. Extrasystole
B. Bradycardia
C. Tachycardia
D. Flutter
E. Fibrillation
55. A 23-year-old patient has been admitted to a hospital with a craniocerebral
injury. The patient
is in a grave condition. Respiration is characterized by prolonged convulsive
inspiration followed
by a short expiration. What kind of respiration is it typical for?
A. Apneustic
B. Gasping breath
C. Kussmaul’s
D. Cheyne-Stokes
E. Biot’s
56. A patient with bronchial asthma has developed acute respiratory failure. What
kind of
respiratory failure occurs in this case?
A. Obstructive disturbance of alveolar ventilation
B. Restrictive ventilatory defect
C. Perfusion
D. Diffusion
E. Dysregulation of alveolar ventilation
57. Measurements of the arterial pCO2 and pO2 during an attack of bronchial
asthma revealed
hypercapnia and hypoxemia respectively. What kind of hypoxia occurred in this
case?
A. Respiratory
B. Hemic
C. Circulatory
D. Tissue
E. Histotoxic
58. Autopsy of a 78-year-old patient revealed that retroperitoneal tissue was
soaked with blood, the
abdominal aorta had a sacciform protrusion including a defect with irregular edges.
The wall of the
aorta was here and there of stone-like density. This is the complication of the
following disease:
A. Atherosclerosis
B. Essential hypertension
C. Systemic vasculitis
D. Visceral syphilis
E. Marfan syndrome
59. Diseases of the respiratory system and circulatory disorders impair the
transport of oxygen,
thus leading to hypoxia. Under these conditions the energy metabolism is carried
out by anaerobic
glycolysis. As a result, the following substance is generated and accumulated in
blood:
A. Lactic acid
B. Pyruvic acid
C. Glutamic acid
D. Citric acid
E. Fumaric acid
60. Analysis of the ECG revealed the missing of several PQRST cycles. The
remaining waves and
complexes are not changed. Specify the type of arrhythmia:
A. Sinoatrial block
B. Atrial fibrillation
C. Atrioventricular block
D. Atrial premature beat
E. Intra-atrial block
61. Since a patient has had myocardial infarction, atria and ventricles contract
independently from
each other with a frequency of 60-70 and 35-40 per minute. Specify the type of
heart block in this
case:
A. Complete atrioventricular
B. Partial atrioventricular
C. Sino-atrial
D. Intra-atrial
E. Intraventricular
62. A patient with extensive myocardial infarction has developed heart failure.
What pathogenetic
mechanism contributed to the development of heart failure in the patient?
A. Reduction in the mass of functioning myocardiocytes
B. Pressure overload
C. Volume overload
D. Acute cardiac tamponade
E. Myocardial reperfusion injury
63. Analysis of the experimental spirogram of a 55-year-old person revealed a
decrease in tidal
volume and respiratory amplitude compared to the situation of ten years ago. The
change in these
indicators is caused by:
A. Decreased force of respiratory muscle contraction
B. Gas composition of the air
C. Physical build of a person
D. Height of a person
E. Body mass of a person
64. A patient underwent a course of treatment for atherosclerosis. Laboratory tests
revealed an
increase in the antiatherogenic lipoprotein fraction in the blood plasma. The
treatment efficacy is
confirmed by the increase in:
A. HDL
B. VLDL
C. IDL
D. LDL
E. Chylomicrons
65. A patient has increased thickness of alveolar-capillary membrane caused by a
pathologic
process. The direct consequence will be reduction of the following value:
A. Diffusing lung capacity
B. Oxygen capacity of blood
C. Respiratory minute volume
D. Alveolar ventilation of lungs
E. Expiratory reserve volume
66. A 43-year-old-patient has arterial hypertension caused by an increase in cardiac
output and
general peripheral resistance. Specify the variant of hemodynamic development of
arterial
hypertension in the given case:
A. Eukinetic
B. Hyperkinetic
C. Hypokinetic
D. Combined
E. –
67. When studying the signs of pulmonary ventilation, reduction of forced
expiratory volume has
been detected. What is the likely cause of this phenomenon?
A. Obstructive pulmonary disease
B. Increase of respiratory volume
C. Increase of inspiratory reserve volume
D. Increase of pulmonary residual volume
E. Increase of functional residual lung capacity
68. Since a patient has had myocardial infarction, his atria and ventricles contract
independently
from each other with a frequency of 60-70 and 35-40 per minute. Specify the type
of heart block in
this case:
A. Complete atrioventricular
B. Partial atrioventricular
C. Sino-atrial
D. Intra-atrial
E. Intraventricular
69. The resuscitation unit has admitted a patient in grave condition. It is known
that he had
mistakenly taken sodium fluoride which blocks cytochrome oxidase. What type of
hypoxia
developed in the patient?
A. Tissue
B. Hemic
C. Cardiovascular
D. Hypoxic
E. Respiratory
70. A public utility specialist went down into a sewer well without protection and
after a while lost
consciousness. Ambulance doctors diagnosed him with hydrogen sulfide
intoxication. What type of
hypoxia developed?
A. Hemic
B. Overload
C. Tissue
D. Circulatory
E. Respiratory
71. A patient with hypertensic crisis has increased content of angiotensin II in
blood. Angiotensin
pressor effect is based on:
A. Contraction of arteriole muscles
B. Activation of biogenic amine synthesis
C. Prostaglandin hyperproduction
D. Vasopressin production stimulation
E. Activation of kinin–kallikrein system
72. When studying the signs of pulmonary ventilation, reduction of forced
expiratory volume has
been detected. What is the likely cause of this phenomenon?
A. Obstructive pulmonary disease
B. Increase of respiratory volume
C. Increase of inspiratory reserve volume
D. Increase of pulmonary residual volume
E. Increase of functional residual lung capacity
73. Diseases of respiratory system and circulatory disorders impair the transport of
oxygen, thus
causing hypoxia. Under these conditions the energy metabolism is carried out by
anaerobic
glycolysis. As a result, the following substance is generated and accumulated in
blood:
A. Lactic acid
B. Pyruvic acid
C. Glutamic acid
D. Citric acid
E. Fumaric acid
74. Cholesterol content in blood serum of a 12-year-old boy is 25 mmol/l.
Anamnesis states
hereditary familial hypercholesterolemia caused by synthesis disruption of
receptor-related proteins
for:
A. Low-density lipoproteins
B. High-density lipoproteins
C. Chylomicrons
D. Very low-density lipoproteins
E. Middle-density lipoproteins
75. A 30-year-old man has sustained an injury to his thorax in a traffic incident,
which caused
disruption of his external respiration. What type of ventilatory difficulty can be
observed in the
given case?
A. Restrictive extrapulmonary ventilatory impairment
B. Restrictive pulmonary ventilatory impairment
C. Obstructive ventilatory impairment
D. Impaired ventilation regulation dysfunction
E. Cardiovascular collapse
76. A patient demonstrates sharp decrease of pulmonary surfactant activity. This
condition can
result in:
A. Alveolar tendency to recede
B. Decreased airways resistance
C. Decresed work of expiratory muscles
D. Increased pulmonary ventilation
E. Hyperoxemia
77. An uconscious young man in the state of morphine intoxication has been
delivered into an
admission room. The patient’s respiration is slow and shallow due to suppression
of the respiratory
center. What kind of respiratory failure occurred in this case?
A. Ventilatory disregulation
B. Ventilatory obstruction
C. Ventilatory restriction
D. Perfusion
E. Diffusion
78. A 51-year-old patient complains of periodic heartache, dyspnea during light
physical activities.
ECG reveals extraordinary contractions of heart ventricles. Such arrhythmia is
called:
A. Extrasystole
B. Bradycardia
C. Tachycardia
D. Flutter
E. Fibrillation
79. A woman resting in the countryside has been stung by a bee. Immediately after
she developed
pain in the stung area. In a few minutes there developed a vesicle, erythema and
intense itch; later –
urticarial and expiratory dyspnea. What factors resulted in the patient developing
expiratory
dyspnea?
A. Histamine
B. Hageman’s factor
C. Lysosomal enzymes
D. Noradrenaline
E. Adrenaline
80. When ascending to the top of Elbrus, a mountain climber experiences oxygen
starvation,
dyspnea, palpitations, and numbness of the extremities. What kind of hypoxia has
developed in the
mountain climber?
A. Hypoxic
B. Circulatory
C. Hemic
D. Tissue
E. Cardiac
81. A patient complains of palpitations after stress. Pulse is 104/min., P-Q = 0,12
seconds, there are
no changes in QRS complex. What type of arrhythmia does the patient have?
A. Sinus tachycardia
B. Sinus bradycardia
C. Sinus arrhythmia
D. Ciliary arrhythmia
E. Extrasystole
82. An athlete (long-distance runner) during a contest developed a case of acute
cardiac
insufficiency. This pathology resulted from:
A. Cardiac volume overload
B. Disrupted coronary circulation
C. Direct damage to myocardium
D. Pericardium pathology
E. Cardiac pressure overload
Module 6
1. Purulent endometritis developed in a woman after delivery. Treating with
antibiotics inhibitors
of murein synthesis was ineffective. Wide spectrum bactericidal antibiotic was
administered to her.
In 6 hours temperature rapidly increased up to 40 C with shiver. Muscle pains have
appeared. BP
dropped down to 70/40 mmHg. Oligura has developed. What is the main reason for
the
development of this condition?
A. Endotoxic shock
B. Toxic effect of preparation
C. Internal bleeding
D. Anaphylactic shock
E. Bacteremia
2. A 2-year-old child experienced convulsions because of lowering calcium ions
concentration in
the blood plasma. Function of what structure is decreased?
A. Parathyroid glands
B. Hypophysis
C. Adrenal cortex
D. Pineal gland
E. Thymus
3. A patient after hypertension stroke does not have voluntary movements in his
right arm and leg
with the increased muscle tone in these extremites. What type of disfunction of
nervous system is
it?
A. Central paralysis
B. Peripheral paralysis
C. Peripheral paresis
D. Reflex paresis
E. Central paresis
4. The patient has come to the hospital from the smelting workshop in the
condition of
hyperthermia. What is the direct cause of loss of consciousness at the heat stroke?
A. Decreased brain blood supply
B. Arterial pressure drop
C. Increased water loss through sweating
D. Decrease of heart output
E. Dilatation of peripheral vessels
5. A 16 year-old patient got numerous traumas in automobile accident. Now the
patient is haning a
shock. АP - 80/60 mm Hg. daily urine volume 60-80 ml. What pathogenic
mechanism leads to
kidneys function violation?
A. Decreased hydrostatic pressure in glomerular capillaries
B. Increased osmotic pressure in glomerular capillaries
C. Increased pressure in Bowman’s capsule
D. Increased vasopressin blood concentration
E. Trauma of the urinary bladder
6. An autopsy has revealed that kidneys are enlarged, surface is large-granular
because of multiple
cavities with smooth wall, which are filled with clear fluid. What kidney disease
did the patient
have?
A. Polycystic kidney
B. Necrotic nephrosis
C. Pyelonephritis
D. Glomerulonephritis
E. Infarction
7. A 57-year-old patient was admitted to the gastroenterological department with
suspicion of
Zollinger-Ellison syndrom because of rapid increase of gastrin level in the blood
serum. What the
most probable disorder of the secretory function of the stomach here?
A. Hyperacidity hypersecretion
B. Hyperacidity hyposecretion
C. Achylia
D. Hypoacidity hyposecretion
E. Hypoacidity hypersecretion
8. M-r S presents all signs of the hepatic coma: loss of consciousness, absence of
reflexes, cramps,
convulsion, disorder of heart activity, recurrent (periodical) respiration. What are
cerebrotoxical
substances which accumulate in blood under hepar insufficiency?
A. Ammonia
B. IL-1
C. Autoantibody
D. Necrosogenic substances
E. Ketonic body
9. A 19-year-old female suffers from tachycardia in rest condition, weight loss,
excessive sweating,
exophtalmos and irritability. What hormone would you expect to find elevated in
her serum?
A. Thyroxine
B. Cortisol
C. Mineralocorticoids
D. ACTH
E. Insulin
10. A patient with the symptoms of acute alcoholic poisoning was brought to the
hospital. What
carbohydrates metabolism changes are typical for this condition?
A. The gluconeogenesis velocity in liver is decreased
B. The gluconeogenesis is increased in liver
C. The breakage of glycogen is increased in liver
D. The anaerobic glucose metabolism predominates in muscles
E. The anaerobic breakage of glucose is increased in muscles
11. A 59-year-old man has symptoms of parenchymatous jaundice and portal
hypertension.
Histological examination of the puncture of the liver bioptate has revealed an
affected beam-lobule
structure, part of hepatocytes has signs of fat dystrophy, port-portal connective
tissue septa with
formation of pseudo-lobules, with periportal lymphomacrophage infiltrations.
What is the most
probable diagnosis?
A. Liver cirrhosis
B. Alcohol hepatitis
C. Chronic hepatosis
D. Viral hepatitis
E. Toxic dystrophy
12. Glomerular filtration rate (GFR) increased by 20% due to prolonged starvation
of the person.
The most evident cause of filtration changes under this conditions is:
A. Decrease of oncotic pressure of blood plasma
B. Increase of systemic blood pressure
C. Increase of penetration of the renal filter
D. Increase of filtration coefficient
E. Increase of renal plasma stream
13. A 68-year-old woman can not move her upper and lower right extremities after
stroke. Muscle
tone of these extremities and reflexes are increased. There are pathological
reflexes. What form of
the paralysis is it?
A. Hemiplegia
B. Paraplegia
C. Tetraplegia
D. Monoplegia
E. Dissociation
14. Ammonia is a very toxic substance, especially for nervous system. What
substance takes the
most active part in ammonia detoxication in brain tissues?
A. Glutamic acid
B. Lysine
C. Proline
D. Histidine
E. Alanine
15. A patient complains of frequent diarrheas, especially after consumption of
fattening food, and
of body weight loss. Laboratory examination revealed steatorrhea; hypocholic
feces. What can be
the cause of this condition?
A. Obturation of biliary tracts
B. Mucous membrane inflammation of small intestine
C. Lack of pancreatic lipase
D. Lack of pancreatic phospholipase
E. Unbalanced diet
16. A child is languid, apathetic. Liver is enlarged and liver biopsy revealed a
significant excess of
glycogene. Glucose concentration in the blood streamis below normal. What is the
cause of low
glucose concentration?
A. Low (absent) activity of glycogene phosphorylase in liver
B. Low (absent) activity of hexokinase
C. High activity of glycogen synthetase
D. Low (absent) activity of glucose 6-phosphatase
E. Deficit of a gene that is responsible for synthesis of glucose 1-phosphaturidine
transferase
17. After intake of rich food a patient feels nausea and sluggishness; with time
there appeared signs
of steatorrhea. Blood cholesterine concentration is 9,2 micromole/l. This condition
was caused by
lack of:
A. Bile acids
B. Triglycerides
C. Fatty acids
D. Phospholipids
E. Chylomicrons
18. A 2 y.o. child has convulsions as a result of lowered concentration of calcium
ions in blood
plasma. It is caused by reduced function of:
A. Parathyroid glands
B. Hypophysis
C. Adrenal cortex
D. Pineal gland
E. Thymus
19. A 48 y.o. patient was admitted to the hospital with complaints about weakness,
irritability,
sleep disturbance. Objectively: skin and scleras are yellow. In blood: conjugated
bilirubin,
cholalemia. Feces are acholic. Urine is of dark colour (bilirubin). What jaundice is
it?
A. Mechanic
B. Hemolytic
C. Parenchymatous
D. Gilbert’s syndrome
E. Crigler-Najjar syndrome
20. 40 y.o. patient complains of intensive heartbeats, sweating, nausea, vision
impairment, arm
tremor, hypertension. From his anamnesis: 2 years ago he was diagnosed with
pheochromocytoma.
Hyperproduction of what hormones causes the given pathology?
A. Catecholamines
B. Aldosterone
C. Glucocorticoids
D. ACTH
E. Thyroid hormones
21. A 4 y.o. boy has had recently serious viral hepatitis. Now there are such
clinical presentations
as vomiting, loss of consciousness, convulsions. Blood analysis revealed
hyperammoniemia.
Disturbunce of which biochemical process caused such pathological condition of
the patient?
A. Disturbed neutralization of ammonia in liver
B. Disturbed neutralization of biogenic amines
C. Increased putrefaction of proteins in bowels
D. Activation of aminoacid decarboxylation
E. Inhibition of transamination enzyms
22. A patient has a disturbed absorbtion of fat hydrolysates. It might have been
caused by a deficit
in the small intestine cavity:
A. Of bile acids
B. Of bile pigments
C. Of lipolytic enzymes
D. Of sodium ions
E. Of liposoluble vitamins
23. Violation of safety rules resulted in calomel intoxication. Two days later the
daily diuresis was
620 ml. A patient experienced headache, vomiting, convulsions, dyspnea, moist
rales in lungs.
What pathology is it?
A. Acute renal insufficiency
B. Chronic renal insufficiency
C. Uraemic coma
D. Glomerulonephritis
E. Pyelonephritis
24. Autopsy of a 56 y.o. man revealed in the right temporal part of brain a big
focus of softened
grey matter that was semiliquid and light grey. Arteries of cerebral tela contain
multiple whitishyellow thickenings of intima that abruptly narrow the lumen. What
is your diagnosis?
A. Ischemic stroke
B. Brain abscess
C. Hemorrhage
D. Hemorrhagic infarction
E. Brain edema
25. After consumption of rich food a patient has nausea and heartburn, steatorrhea.
This condition
might be caused by:
A. Bile acid deficiency
B. Increased lipase secretion
C. Disturbed tripsin synthesis
D. Amylase deficiency
E. Disturbed phospholipase synthesis
26. An aged man had raise of arterial pressure under a stress. It was caused by
activation of:
A. Sympathoadrenal system
B. Parasympathetic nucleus of vagus
C. Functions of thyroid gland
D. Functions of adrenal cortex
E. Hypophysis function
27. As a result of long-term starvation the glomerular filtration of a man was
accelerated by 20%.
The most probable cause of filtration changes under such conditions is:
A. Fall of oncotic pressure of blood plasma
B. Rise of systemic arterial pressure
C. Increased permeability of renal filter
D. Growth of filtration coefficient
E. Increase of renal plasma flow
28. A patient has yellow skin colour, dark urine, dark-yellow feces. What
substance will have
strengthened concentration in the blood serum?
A. Unconjugated bilirubin
B. Conjugated bilirubin
C. Mesobilirubin
D. Verdoglobin
E. Biliverdin
29. A 44 year old woman complains of general weakness, heart pain, significant
increase of body
weight. Objectively: moon face, hirsutism, AP is 165/100 mm Hg, height - 164 cm,
weight - 103
kg; the fat is mostly accumulated on her neck, thoracic girdle, belly. What is the
main pathogenetic
mechanism of obesity?
A. Increased production of glucocorticoids
B. Reduced production of thyroid hormones
C. Increased insulin production
D. Reduced glucagon production
E. Increased mineralocorticoid production
30. A girl is diagnosed with adrenogenital syndrome (pseudohermaphroditism).
This pathology
was caused by hypersecretion of the following adrenal hormone:
A. Androgen
B. Estrogen
C. Aldosterone
D. Cortisol
E. Adrenalin
31. A cerebral trauma caused increased ammonia generation. What amino acid
participates in the
excretion of ammonia from the cerebral tissue?
A. Glutamic
B. Tyrosine
C. Valine
D. Tryptophan
E. Lysine
32. A 30 year old woman has face edemata. Examination revealed proteinuria
(5,87 g/l),
hypoproteinemia, dysproteinemia, hyperlipidemia. What condition is the set of
these symptoms
typical for?
A. Nephrotic syndrome
B. Nephritic syndrome
C. Chronic pyelonephritis
D. Acute renal failure
E. Chronic renal failure
33. A patient has a decreased vasopressin synthesis that causes polyuria and as a
result of it evident
organism dehydratation. What is the mechanism of polyuria development?
A. Reduced tubular reabsorption of water
B. Reduced tubular reabsorption of Na ions
C. Reduced tubular reabsorption of protein
D. Reduced glucose reabsorption
E. Acceleration of glomerular filtration
34. A driver who got a trauma in a road accident and is shocked has reduction of
daily urinary
output down to 300 ml. What is the main pathogenetic factor of such diuresis
change?
A. Drop of arterial pressure
B. Drop of oncotic blood pressure
C. Increased vascular permeability
D. Decreased number of functioning glomerules
E. Secondary hyperaldosteronism
35. During starvation muscle proteins break up into free amino acids. These
compounds will be the
most probably involved into the following process:
A. Gluconeogenesis in liver
B. Gluconeogenesis in muscles
C. Synthesis of higher fatty acids
D. Glycogenolysis
E. Decarboxylation
36. Rats being under stress have muscular hypertonia and high arterial pressure,
high glucose
concentration in blood and intensified secretion of corticotropin and
corticosteroids. In what stress
phase are these animals?
A. Antishock phase
B. Exhaustion
C. Shock phase
D. Erectile
E. Terminal
37. After a hypertonic crisis a patient presents with lacking spontaneous
movements in his right
arm and leg, muscle tone of these extremities is increased. What type of motor
dysfunction has
developed in this case?
A. Central paralysis
B. Peripheral paralysis
C. Peripheral paresis
D. Reflectrory paresis
E. Central paresis
38. A 46-year-old patient suffering from the diffuse toxic goiter underwent
resection of the thyroid
gland. After the surgery the patient presents with appetite loss, dyspepsia,
increased neuromuscular
excitement. The body weight remained unchanged. Body temperature is normal.
Which of the
following has caused such a condition in this patient?
A. Reduced production of parathormone
B. Increased production of thyroxin
C. Increased production of calcitonin
D. Increased production of thyroliberin
E. Reduced production of thyroxin
39. A 48-year-old patient was admitted to the hospital with complaints about
weakness, irritability,
sleep disturbance. Objectively: skin and scleras are of yellow colour. In blood:
increased
concentration of total bilirubin with prevailing direct bilirubin. The feces are
acholic. The urine is
dark (contains bile pigments). What type of jaundice is it?
A. Mechanic
B. Haemolytic
C. Parenchymatous
D. Gilbert’s syndrome
E. Crigler-Najjar syndrome
40. Two weeks after lacunar tonsillitis a 20-year-old man started complaining
about general
weakness, lower eyelid edemata. After examination the patient was diagnosed with
acute
glomerulonephritis. What are the most likely pathological changes in the urine
formula?
A. Proteinuria
B. Cylindruria
C. Presence of fresh erythrocytes
D. Pyuria
E. Natriuria
41. As a result of continuous starvation the glomerular filtration rate has increased
by 20%. The
most probable cause of the glomerular filtration alteration under the mentioned
conditions is:
A. Decrease in the oncotic pressure of blood plasma
B. Increase in the systemic arterial pressure
C. Increase in the permeability of the renal filter
D. Increase of the filtartion quotient
E. Increase of the renal blood flow
42. Sinoatrial block with android-type obesity had been suffering from arterial
hypertension,
hyperglycemia, glycosuria for a long time and died from the cerebral haemorrhage.
Pathologic
examination revealed pituitary basophil adenoma, adrenal cortex hyperplasia.
What is the most
likely diagnosis?
A. Itsenko-Cushing’s syndrome
B. Diabetes mellitus
C. Acromegalia
D. Pituitary nanism
E. Adiposogenital dystrophy
43. A patient presents with icteritiousness of skin, scleras and mucous membranes.
Blood plasma
the total bilirubin is increased, stercobilin is increased in feces, urobilin is
increased in urine. What
type of jaundice is it?
A. Haemolytic
B. Gilbert’s disease
C. Parenchymatous
D. Obturational
E. Cholestatic
44. As a result of a trauma a patient has developed traumatic shock that led to the
following
disorders: AP is 140/90 mm Hg, Ps is 120 bpm. The patient is fussy, talkative,
pale. Such state
relates to the following shock phase:
A. Erectile
B. Latent period
C. Terminal
D. Torpid
E. –
45. A patient with massive burns developed acute renal insufficiency characterized
by a significant
and rapid deceleration of glomerular filtration. What is the mechanism of its
development?
A. Reduction of renal blood flow
B. Damage of glomerular filter
C. Reduction of functioning nephron number
D. Rise of pressure of tubular fluid
E. Renal artery embolism
46. A 46 year old woman suffering from chololithiasis developed jaundice. Her
urine became darkyellow and feces became colourless. Blood serum will have the
highest concentration of the
following substance:
A. Conjugated bilirubin
B. Unconjugated bilirubin
C. Biliverdin
D. Mesobilirubin
E. Urobilinogen
47. A 30 year old woman has face edemata. Examination revealed proteinuria
(5,87 g/l),
hypoproteinemia, dysproteinemia, hyperlipidemia. What condition is the set of
these symptoms
typical for?
A. Nephrotic syndrome
B. Nephritic syndrome
C. Chronic pyelonephritis
D. Acute renal failure
E. Chronic renal failure
48. A patient has a decreased vasopressin synthesis that causes polyuria and as a
result of it evident
organism dehydratation. What is the mechanism of polyuria development?
A. Reduced tubular reabsorption of water
B. Reduced tubular reabsorption of Na ions
C. Reduced tubular reabsorption of protein
D. Reduced glucose reabsorption
E. Acceleration of glomerular filtration
49. Examination of a 42 year old patient revealed a tumour of adenohypophysis.
Objectively: the
patient’s weight is 117 kg, he has moon-like hyperemic face, redblue striae of skin
distension on his
belly. Osteoporosis and muscle dystrophy are present. AP is 210/140 mm Hg.
What is the most
probable diagnosis?
A. Cushing’s disease
B. Cushing’s syndrome
C. Conn’s disease
D. Diabetes mellitus
E. Essential hypertension
50. A 46-year-old patient suffering from the diffuse toxic goiter underwent
resection of the thyroid
gland. After the surgery the patient presents with appetite loss, dyspepsia,
increased neuromuscular
excitement. The body weight remained unchanged. Body temperature is normal.
Which of the
following has caused such a condition in this patient?
A. Reduced production of parathormone
B. Increased production of thyroxin
C. Increased production of calcitonin
D. Increased production of thyroliberin
E. Reduced production of thyroxin
51. A doctor recommends a patient with duodenal ulcer to drink cabbage and
potato juice after the
therapy course. Which substances contained in these vegetables help to heal and
prevent the ulcers?
A. Vitamin U
B. Pantothenic acid
C. Vitamin C
D. Vitamin B1
E. Vitamin K
52. A 30-year-old male patient with acute pancreatitis has been found to have a
disorder of cavitary
protein digestion. The reason for such condition can be the hyposynthesis and
hyposecretion of the
following enzyme:
A. Tripsin
B. Pepsin
C. Lipase
D. Dipeptidase
E. Amylase
53. A coprological survey revealed lightcolored feces containing drops of neutral
fat. The most
likely reason for this condition is the disorder of:
A. Bile inflow into the bowel
B. Gastric juice acidity
C. Pancreatic juice secretion
D. Intestinal juice secretion
E. Intestinal absorption
54. As a result of continuous starvation the glomerular filtration rate has increased
by 20%. The
most probable cause of the glomerular filtration alteration under the mentioned
conditions is:
A. Decrease in the oncotic pressure of blood plasma
B. Increase in the systemic arterial pressure
C. Increase in the permeability of the renal filter
D. Increase of the filtartion quotient
E. Increase of the renal blood flow
55. Jaundice treatment involves administration of barbiturates inducing the
synthesis of UDPglucuronyl transferase. A medicinal effect is caused by the
production of:
A. Direct reacting (conjugated) bilirubin
B. Indirect reacting (unconjugated) bilirubin
C. Biliverdin
D. Protoporphyrin
E. Heme
56. Blood analysis of a patient with jaundice reveals conjugated bilirubinemia,
increased
concentration of bile acids. There is no stercobilinogen in urine. What type of
jaundice is it?
A. Obstructive jaundice
B. Hepatocellular jaundice
C. Parenchymatous jaundice
D. Hemolytic jaundice
E. Cythemolytic jaundice
57. The patient with complaints of permanent thirst applied to the doctor.
Hyperglycemia, polyuria
and increased concentration of 17-ketosteroids in the urine were revealed. What
disease is the most
likely?
A. Steroid diabetes
B. Insulin-dependent diabetes mellitus
C. Myxoedema
D. Type I glycogenosis
E. Addison’s disease
58. A patient complains of hydruria (7 liters per day) and polydipsia. Examination
reveals no
disorders of carbohydrate metabolism. These abnormalities might be caused by the
dysfunction of
the following endocrine gland:
A. Neurohypophysis
B. Adenohypophysis
C. Islets of Langerhans (pancreatic islets)
D. Adrenal cortex
E. Adrenal medulla
59. A patient presents with icteritiousness of skin, scleras and mucous membranes.
Blood plasma
the total bilirubin is increased, stercobilin is increased in feces, urobilin is
increased in urine. What
type of jaundice is it?
A. Haemolytic
B. Gilbert’s disease
C. Parenchymatous
D. Obturational
E. Cholestatic
60. A patient with massive burns developed acute renal insufficiency characterized
by a significant
and rapid deceleration of glomerular filtration. What is the mechanism of its
development?
A. Reduction of renal blood flow
B. Damage of glomerular filter
C. Reduction of functioning nephron number
D. Rise of pressure of tubular fluid
E. Renal artery embolism
61. A patient has been diagnosed with influenza. His condition became drastically
worse after
taking antipyretic drugs. His consciousness is confused, AP is 80/50 mm Hg, Ps is
140/m, body
temperature droped down to 35,8oC.What complication developed in this patient?
A. Collapse
B. Hyperthermia
C. Hypovolemia
D. Acidosis
E. Alkalosis
62. A child has an acute renal failure. What biochemical factor found in saliva can
confirm this
diagnosis?
A. Increase in urea concentration
B. Increase in glucose concentration
C. Decrease in glucose concentration
D. Increase in concentration of higher fatty acids
E. Decrease in nucleic acid concentration
63. After severe viral hepatitis a 4 year old boy presents with vommiting,
occasional loss of
consciousness, convulsions. Blood test revealed hyperammoniemia. Such
condition is caused by a
disorder of the following biochemical hepatic process:
A. Disorder of ammonia neutralization
B. Disorder of biogenic amines neutralization
C. Protein synthesis inhibition
D. Activation of amino acid decarboxylation
E. Inhibition of transamination enzymes
64. A patient with a history of chronic glomerulonephritis presents with azotemia,
oliguria, hypoand isosthenuria, proteinuria. What is the leading factor in the
pathogenesis of these symptoms
development under chronic renal failure?
A. Mass decrease of active nephrons
B. Intensification of glomerular filtration
C. Tubular hyposecretion
D. Disturbed permeability of glomerular membranes
E. Intensification of sodium reabsorption
65. After the traumatic tooth extraction a patient is complaining of acute, dull,
poorly-localized
pain in gingiva, body temperature rise up to 37,5oC. The patient has been
diagnosed with alveolitis.
Specify the kind of pain in this patient:
A. Protopathic
B. Epicritic
C. Visceral
D. Heterotopic
E. Phantom
66. Blood analysis of a patient with jaundice reveals conjugated bilirubinemia,
increased
concentration of bile acids. There is no stercobilinogen in urine. What type of
jaundice is it?
A. Obstructive jaundice
B. Hepatocellular jaundice
C. Parenchymatous jaundice
D. Hemolytic jaundice
E. Cythemolytic jaundice
67. A 60 year old patient was found to have a dysfunction of main digestive
enzyme of saliva. This
causes the disturbance of primary hydrolysis of:
A. Carbohydrates
B. Fats
C. Proteins
D. Cellulose
E. Lactose
68. Examination of a 42 year old patient revealed a tumor of adenohypophysis.
Objectively: the
patient’s weight is 117 kg, he has moon-like hyperemic face, redblue striae of skin
distension on his
belly. Osteoporosis and muscle dystrophy are present. AP is 210/140 mm Hg.
What is the most
probable diagnosis?
A. Cushing’s disease
B. Cushing’s syndrome
C. Conn’s disease
D. Diabetes mellitus
E. Essential hypertension
69. Toxic affection of liver results in dysfunction of protein synthesis. It is usually
accompanied by
the following kind of dysproteinemia:
A. Absolute hypoproteinemia
B. Relative hypoproteinemia
C. Absolute hyperproteinemia
D. Relative hyperproteinemia
E. Paraproteinemia
70. A patient with jaundice has high total bilirubin that is mainly indirect
(unconjugated), high
concentration of stercobilin in the stool and urine. The level of direct (conjugated)
bilirubin in the
blood plasma is normal. What kind of jaundice can you think of?
A. Hemolytic
B. Parenchymal (hepatic)
C. Mechanical
D. Neonatal jaundice
E. Gilbert’s disease
71. 14 days after quinsy a 15-year-old child presented with morning facial
swelling, high blood
pressure, "meat slops" urine. Immunohistological study of a renal biopsy sample
revealed
deposition of immune complexes on the basement membranes of the capillaries
and in the
glomerular mesangium. What disease developed in the patient?
A. Acute glomerulonephritis
B. Acute interstitial nephritis
C. Lipoid nephrosis
D. Acute pyelonephritis
E. Necrotizing nephrosis
72. An unconscious patient was taken by ambulance to the hospital. On objective
examination the
patient was found to have no reflexes, periodical convulsions, irregular breathing.
After laboratory
examination the patient was diagnosed with hepatic coma. Disorders of the central
nervous system
develop due to the accumulation of the following metabolite:
A. Ammonia
B. Urea
C. Glutamine
D. Bilirubin
E. Histamine
73. A 19-year-old male was found to have an elevated level of potassium in the
secondary urine.
These changes might have been caused by the increase in the following hormone
level:
A. Aldosterone
B. Oxytocin
C. Adrenaline
D. Glucagon
E. Testosterone
74. Due to the use of poor-quality measles vaccine for preventive vaccination, a 1-
year-old child
developed an autoimmune renal injury. The urine was found to contain
macromolecular proteins.
What process of urine formation was disturbed?
A. Filtration
B. Reabsorption
C. Secretion
D. Reabsorption and secretion
E. Secretion and filtration
75. Due to the blockage of the common bile duct (which was radiographically
confirmed), the
biliary flow to the duodenum was stopped. We should expect the impairment of:
A. Fat emulsification
B. Protein absorption
C. Carbohydrate hydrolysis
D. Secretion of hydrochloric acid
E. Salivation inhibition
76. A drycleaner’s worker has been found to have hepatic steatosis. This pathology
can be caused
by the disruption of synthesis of the following substance:
A. Phosphatidylcholine
B. Tristearin
C. Urea
D. Phosphatidic acid
E. Cholic acid
77. A 41-year-old male patient has a history of recurrent attacks of heartbeats
(paroxysms), profuse
sweating, headaches. Examination revealed hypertension, hyperglycemia,
increased basal metabolic
rate, and tachycardia. These clinical presentations are typical for the following
adrenal pathology:
A. Hyperfunction of the medulla
B. Hypofunction of the medulla
C. Hyperfunction of the adrenal cortex
D. Hypofunction of the adrenal cortex
E. Primary aldosteronism
78. 40-year-old female patient has undergone thyroidectomy. Histological study of
thyroid gland
found the follicles to be of different size and contain foamy colloid, follicle
epithelium is high and
forms papillae, there is focal lymphocytic infiltration in stroma. Diagnose the
thyroid gland disease:
A. Basedow’s disease
B. Hashimoto’s thyroiditis
C. Riedel’s thyroiditis
D. De Quervain’s disease
E. Nodular goiter
79. Urine analysis has shown high levels of protein and erythrocytes in urine. This
can be caused
by the following:
A. Renal filter permeability
B. Effective filter pressure
C. Hydrostatic blood pressure in glomerular capillaries
D. Hydrostatic primary urine pressure in capsule
E. Oncotic pressure of blood plasma
80. A patient complaining of pain in the left shoulder-blade region has been
diagnosed with
myocardial infarction. What kind of pain does the patient have?
A. Radiating
B. Visceral
C. Phantom
D. Protopathic
E. Epicritic
81. A patient has a critical impairment of protein, fat and hydrocarbon digestion.
Most likely it has
been caused by low secretion of the following digestive juice:
A. Pancreatic juice
B. Saliva
C. Gastric juice
D. Bile
E. Intestinal juice
82. A 49-year-old male patient with acute pancreatitis was likely to develop
pancreatic necrosis,
while active pancreatic proteases were absorbed into the blood stream and tissue
proteins broke up.
What protective factors of the body can inhibit these processes?
A. α2-macroglobulin, α1-antitrypsin
B. Immunoglobulin
C. Cryoglobulin, interferon
D. Ceruloplasmin, transferrin
E. Hemoplexin, haptoglobin
83. A 16-year-old female patient has fainted after quickly changing her body
position from
horizontal to vertical one. Which process from the ones listed below has caused the
loss of
consciousness in the first place?
A. Decreasing venous return
B. Increasing venous return
C. Increasing central venous pressure
D. Decreasing oncotic pressure of blood plasma
E. Increasing arterial pressure
84. According to the results of glucose tolerance test, the patient has no disorder of
carbohydrate
tolerance. Despite that, glucose is detected in the patients’s urine (5 mmol/l). The
patient has been
diagnosed with renal diabetes. What renal changes cause glucosuria in this case?
A. Decreased activity of glucose reabsorption enzymes
B. Increased activity of glucose reabsorption enzymes
C. Exceeded glucose reabsorption threshold
D. Increased glucose secretion
E. Increased glucose filtration
85. Due to the use of poor-quality measles vaccine for preventive vaccination, a 1-
year-old child
has developed an autoimmune renal injury. The urine was found to contain
macromolecular
proteins. What process of urine formation has been disturbed?
A. Filtration
B. Reabsorption
C. Secretion
D. Reabsorption and secretion
E. Secretion and filtration
86. A 16-year-old adolescent is diagnosed with hereditary UDP (uridine
diphosphate)
glucuronyltransferase deficiency. Laboratory tests revealed hyperbilirubinemia
caused mostly by
increased blood content of the following substance:
A. Unconjugated bilirubin
B. Conjugated bilirubin
C. Urobilinogen
D. Stercobilinogen
E. Biliverdine
87. Poisoning caused by mercury (II) chloride (corrosive sublimate) occurred in
the result of safety
rules violation. In 2 days the patient’s diurnal diuresis became 620 ml. The patient
developed
headache, vomiting, convulsions, dyspnea; moist crackles are observed in the
lungs. Name this
pathology:
A. Acute renal failure
B. Chronic renal failure
C. Uremic coma
D. Glomerulonephritis
E. Pyelonephritis
88. A patient has been diagnosed with influenza. His condition drastically
worsened after taking
antipyretic drugs. He is unconscious, AP is 80/50 mm Hg, Ps is 140/m, body
temperature dropped
down to 35,8oC. What complication developed in this patient?
A. Collapse
B. Hyperthermia
C. Hypovolemia
D. Acidosis
E. Alkalosis
89. After a road accident a victim has tachycardia, arterial blood pressure 110/70
mm Hg,
tachypnoe, the skin is pale and dry, excitation of central nervous system is
observed. What shock
stage is the patient most likely in?
A. Erectile
B. Terminal
C. Torpid
D. Preshock (compensation stage)
E. Agony
90. A 51-year-old patient suffers fromacute pancreatitis with disrupted common
bile duct patency.
What condition can develop in this case?
A. Mechanical jaundice
B. Hemolytic jaundice
C. Hepatocellular jaundice
D. Hepatic coma
E. Portal hypertension
91. A 50-year-old man, who has been suffering from chronic hepatic failure for
several years, has
developed ascites. What is the main mechanism of this disorder development?
A. Increased pressure in portal vein system
B. Decrease of albumin and globulin synthesis in liver
C. Increased content of low-density and very low-density lipoproteins in blood
D. Neurotoxins appearing in blood
E. Increase of blood oncotic pressure
92. A 27-year-old patient with injury to the neck has lost approximately 30% of the
blood volume.
The patient’s condition is severe: blood pressure is 60/40 mmHg, heart rate is
140/min, respiratory
rate is 30/min, conscious. Characterize the condition of the patient’s circulatory
system:
A. Hypovolemic shock
B. Cardiogenic shock
C. Collapse
D. Coma
E. Arterial hypertension
93. An infant born prematurely 2 days ago presents with yellow coloring of skin
and mucosa. Such
a condition in the infant is caused by temporary deficiency of the following
enzyme:
A. UDP-glucuronyl transferase
B. Aminolevulinate synthase
C. Heme oxygenase
D. Heme synthetase
E. Biliverdine reductase
94. A 16-year-old adolescent is diagnosed with hereditary UDP (uridine
diphosphate)
glucuronyltransferase deficiency. Laboratory tests revealed hyperbilirubinemia
caused mostly by
increased blood content of the following substance:
A. Unconjugated bilirubin
B. Conjugated bilirubin
C. Urobilinogen
D. Stercobilinogen
E. Biliverdine
95. A patient with hypertension has developed headache, tinnitus, vomiting, high
BP up to 220/160
mm Hg. On examination: facial asymmetry on the right, volitional mobility is
absent, increased
tendon reflexes and muscle tone of extremities on the right. What motor disorder
of nervous system
occurred in this case?
A. Hemiplegia
B. Paraplegia
C. Tetraplegia
D. Hyperkinesis
E. Monoplegia
96. A 62-year-old patient has been hospitalized due to massive cerebral
hemorrhage. Blood
pressure is 70/30 mm Hg, heart rate is 120/min., respiratory rate is 4/min.,
unconscious, no response
to external stimuli. Such condition can be determined as:
A. Coma
B. Shock
C. Collapse
D. Stress
E. Agony
Krok 2 Medicine
1. Терапевтичний профіль
1
A 47 y.o. woman complains of having paroxysmal headaches for the last 5 years. The pain is
one-sided, intense, localised in frontal region of head, accompanied by nausea and stomach
discomfort, begins one of a sudden. Onset is usually preceded by vision reduction. Anamnesis
gives evidence of periodical AP rise, but at the moment the woman doesn't take any medicines.
Inbetween the onsets of headache her state is satisfactory. Objectively: high-calorie diet (body
weight index - 29), AP- 170/95 mm Hg. Neurologic state has no pecularities. What is the most
probable diagnosis?

A Migraine
B Chronic subdural hematoma
C Epilepsy
D Benign intracranial hypertension
E Hypertensive encephalopathy

2
A 7 y.o. boy suddenly felt pain in his right knee, it became edematic. The day before he took part
in a cross-country race. Family anamnesis has no data about hemophilia and bleeding
sickness. Objectively: body temperature is 37,5oС. The knee is painful, hot to the touch,
edematic with local tissue tension over it. Blood count: Нb- 123 g/L, leukocytes - 5,6*109/L,
thrombocytes - 354*109/L, prothrombin time - 12 seconds (normally 10-15 seconds), partly
activated thromboplastin time - 72 seconds (normally 35-45 seconds). Hemorrhage time is
normal, VIII:C factor is 5% of norm. What is the most probable diagnosis?
A Hemophilia A
B Hemophilia B
C Schoenlein-Henoch disease
D Vitamin K deficiency
E Thrombocytopenia

3
On the 3rd day after the acute anterior myocardial infarction a 55 y.o. patient complains of dull
ache behind his breast bone, that can be reduced by bending forward, and of dyspnea.
Objectively: AP- 140/180 mm Hg, heart sounds are dull. ECG results: atrial fibrillation with
frequence of ventricular contractions at the rate of 110/min, pathological Q wave and S-T
segment raising in the right chest leads. The patient refused from thrombolisis. What is the most
probable diagnosis?
A Acute pericarditis
B Pulmonary embolism
C Tietze's syndrome
D Dissecting aortic aneurysm
E Dressler's syndrome

4
A 54 y.o. man was admitted to the hospital with complaints of sudden intense headache in
occipital region and vomiting. In the medical hystory: moderate arterial hypertension, the patient
was taking hydrochlorothiazide. Three days ago he consulted a therapeutist about intense
headache that was suppressed by an analgetic. Objectively: consciousness is confused, left
pupil is mydriatic. Evident photophobia and tension of neck muscles. Left-side hemiparesis with
increased muscle tonus and reflexes. Body temperature is low, rash is absent. AP- 230/130 mm
Hg, Ps- 50 bpm, BR- 12/min. What is your preliminary diagnosis?
A Acute subdural hematoma
B Myasthenia
C Disseminated sclerosis
D Migraine
E Acute bacterial meningitis
5
A 51 y.o. women was taken to the emergency department in convulsive status epilepticus. The
first means of medical management should be:
A Ensuring that the airway is open and the patient is oxygenating
B Inserting a tongue blade
C Administering an instravenous bolus of 50% dextrose
D Injecting 5 mg of diazepam followed by a loading dose of phenytoin
E Inducing pentobarbital coma

6
A 38 y.o. man complains of having occasional problems with swallowing of both hard and fluid
food for many months. Sometimes he feels intense pain behind his breast bone, epecially after
hot drinks. There are asphyxia onsets at night. He has not put off weight. Objectively: his
general condition is satisfactory, skin is of usual colour. Examination revealed no changes of
gastrointestinal tract. X-ray picture of thorax organs presents esophagus dilatation with level of
fluid in it. What is the preliminary diagnosis?
A Esophagus achalasia
B Myastenia
C Cancer of esophagus
D Esophagus candidosis
E Gastroesophageal reflux

7
A 35 y.o. woman consulted a doctor about occasional pains in paraumbilical and iliac region that
reduce after defecation or passage of gases. Defecation takes place up to 6 times a day, stool
is not solid, with some mucus in it. Appetite is normal, she has not put off weight. First such
symptoms appeared 1,5 year ago, but colonoscopy data reveals no organic changes.
Objectively: abdomen is soft, a little bit painful in the left iliac region. Blood and urine are normal.
What is the preliminary diagnosis?
A Irritable bowels syndrome
B Celiac disease
C Crohn's disease
D Pseudomembranous colitis
E Dispancreatism

8
The physician must undertake measures for primary prophylaxis of iron deficiency anemia.
Which of the following categories of patient are subject to such primary prophylactic measures?
A Pregnant women
B Patients after 60
C All children
D Patients after operation
E Workers of industrial enterprises

9
A patient with unstable angina pectoris was given the following complex treatment:
anticoagulants, nitrates, α-adrenoblockers. However on the third day of treatment the
pain still romains. Which in vestigation shoud be carried out to establish diagnosis?
A Coronarography
B Stress-echocardiogram
C Test with dosed physical exercises
D Esophageal electrocardiac stimulator
E Myocardial scintigraphy

10
The 28 y.o. woman applied to doctor because of limited loss of the hair. In the anamnesis - she
had frequent headache indisposition, arthromyalgia, fever, irregular casual sexual life, drug
user. RW is negative. What examination must be done first?
A Examination for HIV
B Examination for neuropathology
C Examination for gonorrhea
D Examination for fungi
E Examination for trichomoniasis

11
A 35 y.o. woman was admitted to thoracic surgery department with fever up to $40^0C$, onset
of pain in the side caused by deep breathing, cough with considerable quantity of purulent
sputum and blood with bad smell. What is the most likely diagnosis?
A Abscess of the lung
B Complication of liver echinococcosis
C Bronchiectatic disease
D Actinomycosis of lungs
E Pulmonary tuberculosis

12
A 67 y.o. patient complains of palpitation, dizziness, noise in ears, feeling of shortage of air.
Objectively: pale, damp skin. Vesicular respiration, respiratory rate- 22 per min, pulse- 200 bpm,
AP- 100/70 mm Hg. On ECG: heart rate- 200 bmp, ventricular complexes are widened,
deformed, location of segments ST and of wave T is discordant. The wave Р is not changed,
superimposes QRST, natural conformity between Р and QRS is not present. What kind of
arrhythmia is present?
A Paroxismal ventricular tachycardia
B Sinus tachycardia
C Atrial flutter
D Ventricular extrasystole
E Atrial tachycardia

13
Male 30 y.o., noted growing fingers and facial scull, changed face. Complains of poor eyesight,
weakness, skin darkening, loss of body weight. X-ray shows broadening of sella turcica, thinning
of tuberculin sphenoidale, signs of increased intracranial pressure. What diagnosis can you
make?
A Adenoma of hypophysis
B Encephalitis of truncus
C Optico - hiasmatic arachnoiditis
D Adrenal gland tumor
E Tumor of pondo-cerebellar corner

14
A patient complains of a tormental (agonizing) cough with expectoration of up to 600 ml/daily
purulent chocolatecolor sputum with a decay smell. Onset of illness was abrupt, to-
39oC, fever of irregular type. There is the area of darkening with a cavity in a center on
X-ray film, with irregular contours and level of liquid. What disease is the question?
A Gangrene of lung
B Tuberculosis
C Bronchiectatic illness
D Pneumonia complicated by an abscess
E Lobar pneumonia

15
A 24 y.o. patient complains of nausea, vomiting, headache, shortness of breath. He had an
acute nephritis being 10 y.o. Proteinuria was found out in urine. Objectively: a skin is grey-pale,
the edema is not present. Accent of II tone above aorta. BP 140/100-180/100 mm Hg. Blood
level of residual N2- 6,6 mmol/L, creatinine- 406 mmol/L. Day's diuresis- 2300 ml, nocturia.
Specific density of urine is 1009, albumin- 0,9 g/L, WBC- 0-2 in f/vis. RBC.- single in f/vis.,
hyaline casts single in specimen. Your diagnosis?
A Chronic nephritis with violation of kidney function
B Feochromocitoma
C Hypertensive illness of the II degree
D Nephrotic syndrome
E Stenosis of kidney artery

16
A 33 y.o. male patient was admitted to a hospital. A patient is pale, at an attempt to stand up he
complains of strong dizziness. There was vomiting like coffee-grounds approximately hour ago.
BP- 90/60 mm Hg., pulse- 120 b/min. In anamnesis, a patient has suffered from ulcer of the
stomach, painless form during 4 years. An ulcer was exposed at gastrofiberoscopy. Your
diagnosis:
A Ulcer of stomach, complicated with bleeding
B Ulcer of duodenum, complicated with bleeding
C Erosive gastritis
D Acute pleurisy
E Acute myocardial infarction, abdominal form

17
A 48-year-old patient complains of heaviness in the right hypochondrium, itching of the skin.He
had been treated in infectious diseases hospital repeatedly due to icterus and itch. On physical
exam: meteorism, ascitis, dilation of abdominal wall veins, protruded umbilicus, spleen
enlargement. What can be diagnosed in this case?
A Liver cirrhosis
B Cancer of the liver
C Cancer of the head of pancreas
D Gallstones
E Viral hepatitis B

18
A 25-year-old man has facial edema, moderate back pains. His temperature is 37,5oС, BP
180/100 mm Hg, hematuria [up to 100 in v/f], proteinuria [2,0 g/L], hyaline casts - 10 in v/f., specific
gravity -1020. The onset of the disease is probably connected with acute tonsillitis that started 2
weeks ago. What is the most probable diagnosis?
A Acute glomerulonephritis
B Acute pyelonephritis
C Cancer of the kidney
D Urolithiasis
E Chronic glomerulonephritis

19
In the development of the inflammation processes glucocorticoids reduce the level of certain
most important active enzyme. It results also in the reducing of the synthesis of prostaglandins
and leucotrienes which have a key role in the development of inflammation processes. What is
the exact name of this enzyme?
A Phospholipase A2
B Arachidonic acid
C Lipoxygenasе
D Cyclooxygenase – 1
E Cyclooxygenase – 2

20
A 30 y.o. female with rheumatoid arthritis of five years duration complains of pain in the frist
three fingers of her right hand over past 6 weeks. The pain seems especially severe at night
often awakening her from sleep.The most likelly cause is?
A Carpal tunnel syndrome
B Atlanto-axial sublaxation of cervical spine
C Sensory peripheral neuropathy
D Rheumatoid vasculitis
E Rheumatoid arthritis without complication

21
A 19-year-old man has suffered from moderate mental retardation since childhood. The patient
is illiterate, can take care of himself, do simple household work and other kinds of easy work
under supervision. What does his rehabilitation [tertiary prevention] require?
A All the above mentioned
B Supervision of a social worker
C Physical work under supervision
D Supervision of relatives (if any)
E None of the above mentioned

22
A 38 y.o. patient was urgently admitted to the hospital with complaints of sudden weakness,
dizziness, loss of consciousness, body weight loss, nausea, vomiting, severe pain in epigastric
area, diarrhea, skin hyperpigmentation. What is the most probable diagnosis?
A Addisonic crisis
B Acute gastroenteritis
C Meningoencephalitis
D Scleroderma
E Pellagra

23
An unconscious patient presents with moist skin, shallow breathing. There are signs of previous
injection on the shoulders and hips. BP- 110/70 mm Hg. Tonus of skeletal muscles and reflexes
are increased. Cramps of muscles of the extremities are seen. What is the most likely
disorder?
A Hypoglycemic coma
B Hyperglycemic coma
C Hyperosmolar coma
D Hyperlactacidotic coma
E Stroke

24
A patient was admitted to the hospital on the 7th day of the disease with complaints of high
temperature, headache, pain in the muscles, especially in calf muscles. Dermal integuments and
scleras are icteric. There is hemorrhagic rash on the skin. Urine is bloody. The patient was
fishing two weeks ago. What is the most likely diagnosis?
A Leptospirosis
B Yersiniosis
C Salmonellosis
D Brucellosis
E Trichinellosis

25
A 60-year-old woman, mother of 6 children, developed a sudden onset of upper abdominal pain
radiating to the back, accompanied by nausea, vomiting, fever and chills. Subsequently, she
noticed yellow discoloration of her sclera and skin. On physical examination the patient was
found to be febrile with temp of 38,9oC, along with right upper quadrant tenderness. The
most likely diagnosis is:
A Choledocholithiasis
B Benign biliary stricture
C Malignant biliary stricture
D Carcinoma of the head of the pancreas
E Choledochal cyst

26
A 42-year-old woman suffers from bronchial asthma, has an acute attack of bronchial asthma.
What medication from the listed below is contraindicated to render a first aid?
A Euphylinum
B Izardin
C Corazolum
D Morphinum hydrochloride
E Strophanthin hydrochloride

27
4 days ago a 32-year-old patient caught a cold: he presented with sore throat, fatigue. The next
morning he felt worse, developed dry cough, body temperature rose up to 38,2oC, there
appeared muco-purulent expectoration. Percussion revealed vesicular resonance over lungs,
vesicular breathing weakened below the angle of the right scapula, fine sonorous and sibilant
wheezes. What is the most likely diagnosis?
A Focal right-sided pneumonia
B Bronchial asthma
C Acute bronchitis
D Pulmonary carcinoma
E Pulmonary gangrene

28
A 62-year-old patient complaining of enlargement of cervical, supraclavicular and axillary lymph
nodes, subfebrile temperature for the last 3 months has been admitted to a hospital. In blood:
WBCs – 64x109/l, lymphocytes - 72%. What method of study should be used to specify
the diagnosis?
A Myelogram
B Lymphography
C Lymphoscintigraphy
D X-rays
E Thermography

29
A 38 y.o. woman complains of a purulent discharge from the left nostril. The body temperature is
37,5oC. The patient has been ill for a week and associates her illness with common cold.
There are a pain and tenderness on palpation of her left cheek. The mucous membrane in the
left nasal cavity is red and turgescent. The purulent exudate is seen in the middle meatus in
maxillary. What is the most probable diagnosis?
A Acute purulent maxillary sinusitis
B Acute purulent frontitis
C Acute purulent ethmoiditis
D Acute purulent sphenoiditis
E-

30
A 40-year-old female patient has been hospitalized for attacks of asphyxia, cough with phlegm.
She has a 4-year history of the disease. The first attack of asphyxia occurred during her stay in
the countryside. Further attacks occurred while cleaning the room. After 3 days of inpatient
treatment the patient's condition has significantly improved. What is the most likely etiological factor?
A Household allergens
B Pollen
C Infectious
D Chemicals
E Psychogenic
31
The complications of acute cholecystitis which require surgical intervention are as follows
EXCEPT:
A Jaundice
B Empyema of the gall-bladder
C Emphysematous gall-bladder
D Gall-bladder perforation
E Cholangitis conditioned by the presence of stones in the bile tract

32
A 22-year-old girl has been complaining of having itching rash on her face for 2 days. She
associates this disease with application of cosmetic face cream. Objectively: apparent
reddening and edema of skin in the region of cheeks, chin and forehead; fine papulovesicular
rash. What is the most likely diagnosis?
A Allergic dermatitis
B Dermatitis simplex
C Eczema
D Erysipelas
E Neurodermatitis

33
A schizophrenic patient considers himself to be "an outstanding scientist, a brilliant composer
and an unrivalled artist". He complains that "family and friends are always jealous of him and
want to poison him". Determine the psychopathological syndrome:
A Paranoiac
B Paranoid
C Manic
D Paratrophic
E Hebephrenic

34
A 43-year-old alcohol abuser had not consumed alcohol for the last two days. In the evening he
claimed to see rats and feel like they bite his feet. The patient is disoriented, agitated, all the
time attempts to run somewhere. Specify the psychopathological syndrome:
A Delirious
B Amential
C Oneiroid
D Choreatic
E Ganser's syndrome

35
A 42-year-old patient complains of back pain, darkened urine, general weakness, dizziness that
occurred after treating a cold with aspirin and ampicillin. Objectively: the patient is pale, with
subicteric sclerae. HR - 98 bpm. Liver - +2 cm, spleen - +3 cm. In blood: RBCs -
2,6x1012/l, Hb - 60 g/l, CI - 0,9, WBCs - 9,4x109/l, basophils - 0,5%, eosinophils - 3%, stab
neutrophils - 6% segmented neutrophils - 58%, lymphocytes - 25%, monocytes - 7%, ESR - 38
mm/hour, reticulocytes - 24%. Total bilirubin - 38 millimole/l. What complication occurred in the
patient?
A Acquired hemolytic anemia
B Toxic hepatitis
C Cholelithiasis
D Agranulocytosis
E Paroxysmal nocturnal hemoglobinuria

36
A hospital has admitted a 52-year-old patient with disseminated pulmonary tuberculosis,
complaints of acute pain in the right half of chest, that developed after going upstairs to the 3rd
floor; cough, dyspnea at rest, marked cyanosis. What kind of complication should suspected
first of all?
A Spontaneous pneumothorax
B Cardiac failure
C Pulmonary failure
D Pleuritis
E Acute myocardial infarction

37
A 38-year-old male patient has been taking alcohol for 3 years. 3 days after a regular drinking
period he felt anxiety and fear. It appeared to him that he was surrounded by spiders and
worms, pursued by some "condemnatory voices". His behaviour became aggressive. The
patient demonstrated correct self-awareness but impairment of temporal and spatial orientation.
What is the most likely diagnosis?
A Delirium alcoholicum
B Alcoholic paranoia
C Alcoholic hallucinosis
D Alcoholic encephalopathy
E Pathologic intoxication

38
A painter working at a motorcar plant has been diagnosed with moderately severe intoxication
with amide compounds of benzene. The in-patient treatment resulted in a considerable health
improvement. What expert decision should be made in this case?
A The patient should be issued a sick list for out-patient treatment
B The patient may get back to work providing he will keep to hygiene and sanitary regulations
C The patient should be referred to the medio-social expert commission for evaluation of
percentage of work capicty loss
D The patient should be referred to the medio-social expert commission for attributing the
disability group because of an occupational disease
E-

39
A 16-year-old patient who has a history of intense bleedings from minor cuts and sores needs
to have the roots of teeth extracted. Examination reveals an increase in volume of the right knee
joint, limitation of its mobility. There are no other changes. Blood analysis shows an inclination
to anaemia (Hb- 120 g/l). Before the dental intervention it is required to prevent the bleeding by
means of:
A Cryoprecipitate
B Epsilon-aminocapronic acid
C Fibrinogen
D Dried blood plasma
E Calcium chloride

40
A 44-year-old patient complains about difficult urination, sensation of incomplete urinary bladder
emptying. Sonographic examination of the urinary bladder near the urethra entrance revealed
an oval well-defined hyperechogenic formation 2x3 cm large that was changing its position
during the examination. What conclusion can be made?
A Concrement
B Malignant tumour of the urinary bladder
C Urinary bladder polyp
D Prostate adenoma
E Primary ureter tumour
41
A 7-year-old boy had complained of headache, nausea, fatigue for 3 weeks. His condition
gradually deteriorated, headache and general weakness progressed. The boy had bronchitis at
the age of 3. His father has a history of pulmonary tuberculosis. Objectively: body temperature
37,5oC, conscious, lies supine, with the hip and knee flexed to 90 degrees, nuchal rigidity +6
cm, partial ptosis of the right eyelid, the dilated right pupil. General hyperalgesia is present.
Liquor: transparent, pressure - 400 mm of water column, protein - 1,5%, cytosis - 610/3 with
predominant lymphocytes, sugar - 1,22 mmol/l, chlorides - 500 mmol/l. What is the most likely
diagnosis?
A Tuberculous meningitis
B Secondary purulent meningitis
C Epidemic cerebrospinal meningitis
D Serous meningitis
E Pneumococcal meningitis

42
A 35-year-old patient complains about pain and morning stiffness of hand joints and
temporomandibular joints that lasts over 30 minutes. She has had these symptoms for 2 years.
Objectively: edema of proximal interphalangeal digital joints and limited motions of joints. What
examination should be administered?
A Roentgenography of hands
B Complete blood count
C Rose-Waaler reaction
D Immunogram
E Proteinogram

43
A 69-year-old female patient complains of temperature rise up to 38,3oC, haematuria. ESR
- 55 mm/h. Antibacterial therapy turned out to be ineffective. What diagnosis might be suspected?
A Renal cancer
B Polycystic renal disease
C Renal amyloidosis
D Urolithiasis
E Chronic glomerulonephritis

44
Examination of a 35-year-old patient with rheumatism revealed that the right heart border was 1
cm displaced outwards from the right parasternal line, the upper border was on the level with
inferior margin of the 1st rib, the left border was 1 cm in from the left midclavicular line.
Auscultation revealed atrial fibrillation, loud apical first sound, diastolic shock above the
pulmonary artery. Echocardiocopy revealed abnormal pattern of the mitral valve motion. What
heart disease is characterized by these symptoms?
A Mitral stenosis
B Mitral valve prolapse
C Mitral valve insufficiency
D Aortic stenosis
E Tricuspid valve insufficiency

45
A 54-year-old male patient works as an engineer. At the age of 35, he got infected with syphilis
and treated it with "traditional remedies". About 5 years ago, he became forgetful, unable to
cope with work, told cynical jokes, bought useless things, collected cigarette butts in the street.
Objectively: the patient is indifferent, has slow speech, dysarthria, can make only primitive
judgments, is unabe to perform simple arithmetic operations or explain simple metaphors. The
patient is untidy, takes no interest in anything, passive. Considers himself to be completely
healthy. Qualify mental condition of the patient:
A Total dementia
B Lacunar (dysmnestic) dementia
C Somnolentia
D Korsakoff's (amnesic) syndrome
E Hysterical pseudodementia

46
A 47-year-old patient complains of insomnia, heaviness over his entire body, constantly
depressed mood. He considers himself good-for-nothing, inadequate. Believes that he is a
burden to his family, wants to die. The patient is depressed, inactive, has a hypomimic face with
sorrowful expression. He speaks quietly and monotonely,gives short answers. What is the most
likely diagnosis?
A Major depressive disorder
B Atherosclerotic depression
C Initial stage of Alzheimer's disease
D Late-onset schizophrenia
E Neurotic depression

47
A patient, aged 16, complains of headache, mainly in the frontal and temporal areas,
superciliary arch, appearing of vomiting at the peak of headache, pain during the eyeballs
movement, joint's pain. On examination: excited, to- 39oC, Ps- 110/min. Tonic and
clonus cramps. Uncertain meningeal signs. What is the most likely diagnosis?
A Influenza with cerebral edema manifestations
B Influenza, typical disease duration
C Respiratory syncytial virus
D Parainfluenza
E Adenovirus infection

48
A 64 y.o. patient has developed of squeering substernal pain which had appeared 2 hours ago
and irradiated to the left shoulder, marked weakness. On examination: pale skin, cold sweat.
Pulse- 108 bpm, AP- 70/50 mm Hg, heart sound are deaf, vesicular breathing, soft abdomen,
painless, varicouse vein on the left shin, ECG: synus rhythm, heart rate is 100 bmp,
ST-segment is sharply elevated in II, III aVF leads. What is the most likely disorder?
A Cardiogenic shock
B Cardiac asthma
C Pulmonary artery thromboembolia
D Disquamative aortic aneurizm
E Cardiac tamponade

49
A 64-year-old patient complains of severe pain in the right side of chest, dyspnea, dry cough
which appeared suddenly on exertion. Objectively: the right side of the chest lags behind in the
act of breathing. Percussion reveals tympanic sound. Auscultation reveals pronouncedly
diminished breath sounds on the right. Ps- 100/min, weak, arrhythmic. AP- 100/50 mm Hg.
Cardiac sounds are decreased. What disease can be suspected in this patient?
A Right-sided pneumothorax
B Right-sided hydrothorax
C Right-sided dry pleurisy
D Right-sided pleuropneumonia
E PATE

50
A 29-year-old female patient complains of dyspnea, heaviness and chest pain on the right, body
temperature rise up to 37,2oC. The disease is associated with a chest trauma received 4
days ago. Objectively: skin is pale and moist. Ps- 90 bpm, regular. Palpation reveals a dull sound
on the right, auscultation reveals significantly weakened vesicular breathing. In blood: RBCs -
2,8x1012/l, colour index - 0,9, Hb- 100 g/l, WBCs - 8,0x109/l, ESR - 17 mm/h.
What results of diagnostic puncture of the pleural cavity can be expected?
A Haemorrhagic punctate
B Chylous liquid
C Exudate
D Transudate
E Purulent punctate

51
A 54-year-old drowned man was rescued from the water and delivered to the shore. Objectively:
the man is unconscious, pale, breathing cannot be auscultated, pulse is thready. Resuscitation
measures allowed to save the patient. What complications may develop in the near future?
A Pulmonary edema
B Respiratory arrest
C Encephalopathy
D Cardiac arrest
E Bronchospasm

52
An 18-year-old patient since childhood suffers from bleeding disorder after minor injuries. His
younger brother also has bleeding disorders with occasional haemarthrosis. Which laboratory
test will be informative for diagnosis verification?
A Clotting time
B Fibrinogen rate
C Blood clot retraction
D Thrombocyte count
E Determination of prothrombin time

53
A 28-year-old patient complains of periodic compressing heart pain. His brother died at the age
of 34 from a cardiac disease with similar symptoms. Objectively: the patients skin is pale. Heart
borders display no significant deviations. Heart sounds are loud, there is a systolic murmur
above all the points with a peak above the aorta. Echocardioscopy reveals thickening of the
interventricular septum in the basal parts, reduction of left ventricular cavity. What drug should
be administered in order to prevent the disease progression?
A Metoprolol
B Digoxin
C Nitroglycerin
D Captopril
E Furosemide

54
A patient, aged 40, has been ill during approximately 8 years, complains of pain in the lumbar
part of the spine on physical excertion, in cervical and thoracal part (especially when coughing),
pain in the hip and knee joints on the right. On examination: the body is fixed in the forward
inclination with head down, gluteal muscles atrophy. Spine roentgenography: ribs osteoporosis,
longitudinal ligament ossification. What is the most likely diagnosis?
A Ancylosing spondyloarthritis
B Tuberculous spondylitis
C Psoriatic spondyloarthropatia
D Spondyloarthropatia on the background of Reiter's disease
E Spread osteochondrosis of the vertebral column

55
A worker, aged 38, working in the slate production during 15 years, complains of expiratory
exertional dyspnea, dry cough. On examination: deafening of the percutory sounds in
interscapular region, rough breath sounds, dry disseminated rales. On fingers' skin - greyish
warts. Factory's sectorial doctor suspects asbestosis. Which method is the most informative for
diagnosis verification?
A Thorax roentgenography
B Bronchoscopy
C Spirography
D Bronchoalveolar lavage
E Blood gases examination

56
A 37 y.o. woman is suffering from squeezing substernal pain on physical exertion. On
examination: AP- 130/80 mm Hg, heart rate=pulse rate 72 bpm, heart boarders are dilated to
the left side, aortic systolic murmur. ECG- signs of the left venticle hypertrophy. What method of
examination is the most informative in this case?
A Echocardiography
B Phonocardiography
C Coronarography
D Sphygmography
E X-ray

57
A 58-year-old woman complains of osteoarthrosis of knee-joint. For 2 weeks she had been
receiving an in-patient medical treatment. She was discharged from the hospital in satisfactory
condition with complaints of minor pain after prolonged static work. Local hyperemia and
exudative effects in the area of joints are absent. What further tactics is the most expedient?
A Outpatient treatment
B Repeated in-patient treatment
C Conducting arthroscopy
D Refferral to MSEC
E Orthopedist consultation

58
A 36-year-old female has a 7-year history of pollen allergy. Over the last 2 years in August and
September (during ragweed flowering), the patient has had 2-3 asthma attacks that could be
treated with one dose of salbutamol. Objectively: body temperature - 36,5oC, respiratory rate
- 18/min, Ps - 78/min, AP - 115/70 mm Hg. There is vesicular breathing above the lungs.
Cardiac sounds are sonorous, of regular rhythm. What drug would be most effective to prevent
asthma attacks during the critical season for the patient?
A Intalum inhalation
B Berotec inhalation
C Atrovent inhalation
D Suprastin administration
E Theopecum administration

59
A 42-year-old patient applied to hospital with complaints of pain behind the sternum with
irradiation to the left scapula. The pain appears during significant physical work, this lasts for
5-10 minutes and is over on rest. The patient is sick for 3 weeks. What is the preliminary
diagnosis?
A IHD:First established angina pectoris
B IHD:Variant angina pectoris (Prinzmetal's)
C IHD:Stable angina pectoris of effort I FC
D IHD:Stable angina pectoris of effort IV FC
E IHD:Progressive angina pectoris

60
Medical examination of a 19-year-old worker revealed generalized lymphadenopathy mainly
affecting the posterior cervical, axillary and ulnar lymph nodes. There are multiple injection
marks on the elbow bend skin. The man denies taking drugs, the presence of injection marks
ascribes to influenza treatment. Blood count: RBCs- 3,2x1012/l, Hb- 100 g/l, WBCs-
3,1x109/l, moderate lymphopenia. What study is required in the first place?
A ELISA for HIV
B Immunogram
C Sternal puncture
D X-ray of lungs
E Lymph node biopsy

61
A 25-year-old female patient complains of marked weakness, sleepiness, blackouts, dizziness,
taste disorder. The patient has a history of menorrhagia. Objectively: the patient has marked
weakness, pale skin, cracks in the corners of mouth, peeling nails, systolic apical murmur. Blood
test results: RBC - 3,4x1012/l, Hb - 70 g/l, color index - 0,75, platelets – 140x109/l, WBC - 6,2x109/l.
What is the most likely diagnosis?
A Chronic posthemorrhagic anemia
B Acute leukemia
C Acute posthemorrhagic anemia
D B12-deficiency anemia
E Werlhof's disease

62
A 24-year-old patient felt sick in 16 hours after dried fish intake. There was nausea, vomiting,
weakness, flabbiness, double vision. On physical exam, there was decrease of a muscle tone,
anisocoria, flaccid swallowing and tendon reflex. What is the most probable diagnosis?
A Botulism
B Food toxicoinfection
C Acute gastritis
D Acute encephalitis
E Salmonellosis

63
A 72-year-old patient after operation due to holecystectomia was prescribed gentamicin (80 mg
every 8 hours) and cephalothin (2 g every 6 hours) due to fever. In 10 days there was an
increase of creatinine up to 310 mu*mol/L. BP - 130/80 mm Hg, daily quantity of the urine is
1200 mL. Urine tests are without pathology. Ultrasound: the size of kidneys is normal.
What is the most probable reason for renal failure?
A Nephrotoxity of gentamicin
B Acute glomerulonephritis
C Cortical necrosis of kidneys
D Unequal infusion of the liqiud
E Hepatorenal syndrome

64
A 40 y.o. patient was admitted to the gasteroenterology with skin itching, jaundice, discomfort in
the right subcostal area, generalized weakness. On examination: skin is jaundice, traces of
scratches, liver is +5 cm, splin is 6x8 cm. In blood: alkaline phosphatase - 2,0 mmol/(hour*L),
general bilirubin - 60 mkmol/L, cholesterol - 8,0 mmol/L. What is the leading syndrome in the
patient?
A Сholestatic
B Сytolytic
C Mesenchymal inflammatory
D Asthenic
E Liver-cells insufficiency

65
A 55-year-old male had been treated at the surgical department for acute lower-extremity
thrombophlebitis. On the 7th day of treatment he suddenly developed pain in the left part of
chest, dyspnea and cough. Body temperature was 36,1oC, respiratory rate - 36/min. The
patient was also found to have diminished breath sounds without wheezing. Ps- 140/min,
thready. AP- 70/50 mm Hg. The ECG shows QІІІ-S1 syndrome. What is the most likely
diagnosis?
A Pulmonary embolism
B Myocardial infarction
C Cardiac asthma
D Bronchial asthma
E Pneumothorax

66
A 51-year-old female is a weaving factory worker with 15 years of service record. During a
regular preventive examination she complained of frequent headaches, poor sleep, tingling in
the heart, irritability, rapid fatigability, hearing impairment. For years, the noise level has
exceeded the maximum allowable concentration by 10-15 dB. A year ago, the patient underwent
a course of treatment for essential hypertension. Specify the most likely diagnosis:
A Noise disease
B Essential hypertension
C Neurasthenia
D Asthenic-vegetative syndrome
E Arteriosclerotic encephalopathy

67
A 26-year-old patient undergoes a course of treatment due to chronic glomerulonephritis. The
treatment was successful, normalization of all the characteristics was recorded. What
sanitorium and health resort treatment could be recommended?
A The south coast of the Crimea
B Not recommended
C Morshyn
D Myrhorod
E Truskavets

68
After a wasp-bite there was an itching of skin, hoarse voice, barking cough, anxiety. On physical
exam: there is edema of lips, eyelids, cyanosis. What medicine is to be taken first?
A Prednisolone
B Adrenalin
C Euphylin
D Lasix
E Seduxen

69
A 16-year-old adolescent was vaccinated with DTP. In eight days there was stiffness and pain in
the joints, subfebrile temperature, urticarial skin eruption, enlargement of inguinal, cervical
lymph nodes and spleen. What kind of allergic reaction is observed?
A Immunocomplex
B Hypersensitivity of immediate type
C Cytoxic
D Hypersensitivity of delayed type
E-

70
2 weeks after recovering from angina a 29-year-old patient noticed face edemata, weakness,
decreased work performance. There was gradual progress of dyspnea, edemata of the lower
extremities, lumbar spine. Objectively: pale skin, weakening of the heart sounds, anasarca. AP-
160/100 mm Hg. In urine: the relative density - 1021, protein - 5 g/l, erythrocytes - 20-30 in the
field of vision, hyaline cylinders - 4-6 in the field of vision. What is the most likely diagnosis?
A Acute glomerulonephritis
B Essential hypertension
C Acute pyelonephritis
D Infectious allergic myocarditis
E Myxedema

71
A 56-year-old scientist experiences constricting retrosternal pain several times a day while
walking for 100-150 m. The pain lasts for up to 10 minutes and can be relieved by nitroglycerine.
Objectively: the patient is overweight, heart borders exhibit no abnormalities, heart sounds are
rhythmic, Ps- 78 bpm, AP- 130/80 mm Hg. ECG contains low amplitude of T wave in
V4-5. What disease might be suspected?
A Stable FC III stenocardia
B Instable stenocardia
C Stable FC I stenocardia
D Stable FC II stenocardia
E Stable FC IV stenocardia

72
In autumn a 25-year-old patient developed stomach ache arising 1,5-2 hours after having meals
and at night. He complains of pyrosis and constipation. The pain is getting worse after
consuming spicy, salty and sour food, it can be relieved by means of soda and hot-water bag.
The patient has been suffering from this disease for a year. Objectively: furred moist tongue.
Abdomen palpation reveals epigastrial pain on the right, resistance of abdominal muscles in the
same region. What is the most likely diagnosis?
A Duodenal ulcer
B Chronic cholecystitis
C Diaphragmatic hernia
D Stomach ulcer
E Chronic pancreatitis

73
A 51 y.o. woman complains of dull pain in the right subcostal area and epigastric area, nausea,
appetite decline during 6 months. There is a history of gastric peptic ulcer. On examination:
weight loss, pulse is 70 bpm, AP is 120/70 mm Hg. Diffuse tenderness and resistance of
muscles on palpation.There is a hard lymphatic node 1x1cm in size over the left clavicle. What
method of investigation will be the most useful?
A Esophagogastroduodenoscopy with biopsy
B Ultrasound examination of abdomen
C pH-metry
D Ureatic test
E Stomach X-ray

74
On the 20th of June a townsman was brought to clinic. The disease broke out acutely, starting
with fever, rise in temperature to 38-39oC. There also was weakness, acute headache,
nausea, vomiting, pain all over the body, sleep disorder. On physical examination: hyperemia of
skin of face, neck, thorax. Meningeal signs are positive. 12 days ago the patient returned from
the Siberia, from the forest. What is the most probable diagnosis?
A Tick-borne encephalitis
B Influenza
C Omsk hemorrhagic fever
D Pseudotuberculosis
E Epidemic typhus

75
A 37-year-old patient has sudden acute pain in the right epigastric area after having fatty food.
What method of radiological investigation is to be used on the first stage of examining the
patient?
A Ultrasonic
B Roentgenological
C Radionuclid
D Magnetic-resonance
E Thermographic

76
A man, aged 68, complains of tiredness, sweating, enlargement of cervical, submaxillary and
axillary lymph nodes. Blood test: WBC- 35*109/L, lymphocytes - 60%, Botkin and Gumprecht bodies,
level of haemoglobin and quantity of thrombocytes is normal. Myelogram showed 40% of
lymphocytes. What is the most probable diagnosis?
A Chronic lympholeucosis
B Chronic myeloleucosis
C Lymphogranulomatosis
D Acute leucosis
E Tuberculous lymphadenitis

77
A 38-year-old patient is under observation having polyneuritic syndrome with considerable loss
of weight, fever, rise in BP. Blood test:: considerable inflammatory changes. What examination
is the most expedient to make the diagnosis?
A Muscular biopsy with histological investigation of the material
B Determination of antinuclear antibodies
C Electromyography
D Blood culture
E Determination of HLA antigens

78
A 32-year-old male patient has been suffering from pain in the sacrum and coxofemoral joints,
painfulness and stiffness in the lumbar spine for a year. ESR- 56 mm/h. Roentgenography
revealed symptoms of bilateral sacroileitis. The patient is the carrier of HLA B27 antigen. What
is the most likely diagnosis?
A Ankylosing spondylitis
B Coxarthrosis
C Rheumatoid arthritis
D Reiter's disease
E Spondylosis

79
A 58-year-old female patient complains about periodical headache, dizziness and ear noise.
She has been suffering from diabetes mellitus for 15 years. Objectively: heart sounds are
rhythmic, heart rate is 76/min, there is diastolic shock above aorta, AP is 180/110 mm Hg. In
urine: OD- 1,014. Daily loss of protein with urine is 1,5 g. What drug should be chosen for
treatment of arterial hypertension?
A Ihibitor of angiotensin converting enzyme
B $\beta$-blocker
C Calcium channel antagonist
D Thiazide diuretic
E $\alpha$-blocker

80
A 26 y.o. male patient with postoperative hypothyroidism take thyroxine 100 mg 2 times a day.
He has developed tachycardia, sweating, irritability, sleep disorder. Determine further treatment
tactics.
A To decrease thyroxine dosage
B To increase thyroxine dosage
C To administer betablockers
D To add mercasolil to the treatment
E To administer sedatives

81
A 28-year-old man was discharged from the hospital after having an out-of -hospital pneumonia.
He has no complaints. On physical exam: his temperature is 36,6oC, RR-18/min, Ps - 78
bpm, BP- 120/80 mm Hg. During ausculation there is harsh respiration to the right of the lower
part of the lung. Roentgenologically: infiltrative changes are absent, intensification of the
pulmonary picture to the right in the lower lobe. How long should the doctor keep the patient
under observation?
A 12 months
B 1 month
C 3 months
D 6 months
E Permanently

82
A 20-year-old adolescent lives in the nidus of tuberculous infection. The tuberculine Mantoux
test with 2 TU was determined as hyperergic. What signs determine the hyperergic test of this
adolescent?
A 6 mm papula, necrosis
B 20 mm papula
C 24 mm hyperemia
D 4 mm papula
E 12 mm hyperemia

83
A survey radiograph of a miner (24 years of service record, the dust concentration in the
workplace is at the rate of 260-280 mg/m3 with 15% of free silica) shows lung changes that
are typical for pneumoconiosis. What type of pneumoconiosis is it?
A Anthracosilicosis
B Carboconiosis
C Silicatosis
D Anthracosilicatosis
E Silicosis

84
A patient complains of pathological lump, appearing in the right inguinal region on exercise. The
lump is round-shaped, 4 cm in diameter, on palpation: soft elastic consistency, is positioned
near the medial part of Poupart's ligament. The lump is situated inwards from the spermatic
cord. What is the most probable preliminary diagnosis?
A Right-sided direct inguinal hernia
B Right-sided oblique inguinal hernia
C Right-sided femoral hernia
D Varicose veins of the right hip
E Lipoma of the right inguinal area

85
A 35-year-old man was operated on peptic ulcer of the stomach. Mass deficit of the body is 10
kg. The level of glucose after operation in the undiluted cellular blood on an empty stomach is
6,7 mmol. During repeated examination - 11,1 mmol (after meal), level of HbA1c - 10%. Could
you please make an interpretation of the given data?
A Diabetes mellitus
B Disordered tolerance to glucose
C Diabetes mellitus risk group
D Norm
E Postoperative hyperinsulinemia

86
A 52 y.o. woman complains of weakness, painful itching after washing and bathing, sensation of
heaviness in the head. On examination: hyperemia of skin of face, neck, extremities. АP-
180/100 mm Hg. Speeln is 4 cm below the rib arch edge. What is the most probable diagnosis?
A Erythremia
B Essential hypertension
C Dermatomyositis
D Allergic dermatitis
E Systemic sclerodermia

87
A 37-year-old patient was brought to resuscitation unit. General condition of the patient is very
serious. Sopor. The skin is grey, moist. Turgor is decreased. Pulse is rapid, intense. BP -
160/110 mm Hg, muscle tonus is increased. Hyperreflexia. There is an ammonia odor in the air.
What is the presumptive diagnosis?
A Uraemic coma
B Alcoholic coma
C Hyperglycemic coma
D Hypoglycemic coma
E Cerebral coma

88
A 57-year-old man complains of shortness of breath, swelling on shanks, irregularity in cardiac
work, pain in the left chest half with irradiation to the left scapula.Treatment is uineffective. On
physical exam: heart's sounds are diminished, soft systolic murmur on the apex. Ps - 100/min,
arrhythmical, BP - 115/75 mm Hg. The liver is +2 cm, painful. Roentgenoscopy: enlargement of heart
shadow to all sides, pulsation is weak. Electrocardiogram (ECG): leftventricled extrasystolia,
decreased voltage. What method of investigation is necessary to do to determine the diagnosis?
A Echocardiography
B Veloergometria
C X-ray kymography
D ECG in the dynamics
E Coronarography

89
A 55 y.o. patient complains of distended abdomen and rumbling, increased winds evacuation,
liguid foamy feces with sour smell following the diary products consumption. What is the correct
name of this syndrome?
A Syndrome of fermentative dyspepsia
B Syndrome of decayed dyspepsia
C Syndrome of fatty dyspepsia
D Dyskinesia syndrome
E Malabsorption syndrome

90
A 54-year-old patient has an over 20-year history of femoral osteomyelitis. Over the last month
she has developed progressing edemata of the lower extremities. Urine test reveals: proteinuria
at the rate of 6,6 g/l; in blood: dysproteinemia in form of hypoalbuminemia, increase in
α2- and γ-globulin rate, ESR - 50 mm/h. What is the most likely diagnosis?
A Secondary renal amyloidosis
B Acute glomerulonephritis
C Myelomatosis
D Chronic glomerulonephritis
E Systemic lupus erythematosus

91
In an inhabited locality there is an increase of diphtheria during the last 3 years with separate
outbursts in families. What measure can effectively influence the epidemic process of
diphtheria and reduce the morbidity rate to single cases?
A Immunization of the population
B Hospitalization of patients
C Detection of carriers
D Early diagnostics
E Disinfection in disease focus

92
A 14-year-old victim was drawn out of the water in winter after 15 minutes of being in the water.
The victim shows no vital signs. What measures are to be taken?
A To release respiratory tract from water, to create drain position and to take on measures to
restore respiration and blood circulation
B Not to waste time on the release of respiratory tract from water, to take on cardiopulmonary
reanimation
C To transport the victim to the nearest hospital to carry out reanimation measures
D Тo transport the victim to the nearest warm room to carry out reanimation measures
E Not to carry out reanimation measures

93
An electro-gas welding operator working at a machine workshop performs welding and cutting of
metal, which is accompanied by intense UV-radiation. His welding station is equipped with
effective mechanical ventilation. What occupational disease is most likely to develop in an
electro-gas welding operator?
A Photoelectric ophthalmia
B Heatstroke
C Vegetative-vascular dystonia
D Chronic overheating
E Pneumoconiosis

94
A woman complains of high temperature to 38oC, mild pain in the throat during 3 days. On
examination: angle lymphatic nodes of the jaw are 3 cm enlarged, palatinel tonsils are enlarged
and coated with grey plaque which spreads to the uvula and frontal palatinel arches. What is the
most probable diagnosis?
A Larynx dyphtheria
B Infectious mononucleosis
C Vincent's angina
D Agranulocytosis
E Oropharyngeal candidosis

95
A 48-year-old male patient complains of constant pain in the upper abdomen, mostly on the left,
that is getting worse after taking meals; diarrhea, weight loss. The patient is an alcohol abuser. 2
years ago he had acute pancreatitis. Blood amylase is 4 g/h*l. Coprogram shows steatorrhea,
creatorrhea. Blood glucose is 6,0 mmol/l. What treatment is indicated for this patient?
A Panzinorm forte
B Insulin
C Gastrozepin
D Contrycal
E No-spa
96
A 60-year-old female patient had been admitted to a hospital for acute transmural infarction. An
hour later the patient's contition got worse. She developed progressing dyspnea, dry cough.
Respiratory rate - 30/min, heart rate - 130/min, AP- 90/60 mm Hg. Heart sounds were muffled,
there was also diastolic shock on the pulmonary artery. The patient presented with medium
moist rales in the lower parts of lungs on the right and on the left. Body temperature -
$36,4^oC$. What drug should be given in the first place?
A Promedol
B Aminophylline
C Dopamine
D Heparin
E Digoxin

97
A 62-year-old male has been hospitalized in the intensive care unit with a continuous attack of
retrosternal pain that cannot be relieved by nitroglycerin. Objectively: AP- 80/60 mm Hg, heart
rate - 106/min, breathing rate - 22/min. Heart sounds are muffled, a gallop rhythm is present.
How would you explain the AP drop?
A Reduction in cardiac output
B Reduction in peripheral resistance
C Blood depositing in the abdominal cavity
D Adrenergic receptor block
E Internal haemorrhage

98
A 35-year-old female reports heart pain (aching and drilling) occurring mainly in the morning in
autumn and spring and irradiating to the neck, back and abdomen; rapid heartbeat; low vitality.
Occurrence of this condition is not associated with physical activity. In the evening, the patient's
condition improves. Study of somatic and neurological status, and ECG reveal no pathology.
What pathology is most likely to have caused these clinical presentations?
A Somatization depression
B Resting stenocardia
C Pseudoneurotic schizophrenia
D Neurocirculatory asthenia
E Hypochondriacal depression

99
A 18 y.o. male patient complains of pain in knee and ankle joints, temperature elevation to
39,5oC. He had a respiratory disease 1,5 week ago. On examination: temperature-
38,5oC, swollen knee and ankle joints, pulse- 106 bpm, rhythmic, AP- 90/60 mm Hg, heart
borders without changes, sounds are weakened, soft systolic apical murmur. What indicator is
connected with possible etiology of the process?
A Antistreptolysine-0
B 1-antitrypsine
C Creatinkinase
D Rheumatic factor
E Seromucoid

100
A 30 y.o. male patient complains of itching of the skin which intensifies in the evening. He has
been ill for 1,5 months. On examination: there is rash with paired papules covered with bloody
crusts on the abdomen, hips, buttocks, folds between the fingers, flexor surfaces of the hand.
There are traces of line scratches. What additional investigations are necessary to make
diagnosis?
A Examination of rash elements scrape
B Determination of dermographism
C Serologic blood examination
D Blood glucose
E Examination for helmints

101
A 50-year-old locksmith has a long-term record of work under the effect of mercury vapors with
concentration exceeding MPC by 5-10 times. Clinical examination revealed the lability of
vasomotors of skin, pulse and arterial pressure; total hyperhydrosis; asymmetric innervation of
facial and lingual muscles, positive subcortical reflexes, intention tremor. Against the
background of increased emotional excitability the patient presents with lack of self-confidence,
shyness. A dentist found him to have parodontosis, chronic stomatitis. What disease can be
suspected?
A Chronic mercury intoxication
B Residual effects of neuroinfection
C Parkinson's syndrome
D Acute mercury intoxication
E Vascular encephalopathy

102
4 hours after having meals a patient with signs of malnutrition and steatorrhea experiences
stomach pain, especially above navel and to the left of it. Diarrheas take turns with constipation
lasting up to 3-5 days. Palpation reveals moderate painfulness in the choledochopancreatic
region. The amylase rate in blood is stable. X-ray reveals some calcifications located above
navel. What is the most likely diagnosis?
A Chronic pancreatitis
B Chronic gastroduodenitis
C Duodenal ulcer
D Zollinger-Ellison syndrome
E Chronic calculous cholecystitis

103
A 58 y.o. male patient is examined by a physician and suffers from general weakness, fatigue,
mild pain in the left subcostal area, sometimes frequent painful urination. Moderate
splenomegaly has been revealed. Blood test: neutrophilic leukocytosis with the progress to
myelocyte; basophil- 2%; eosinophil- 5%. There is a urate crystales in urine, erythrocyte- 2-3
in the field of vision. What is the preliminary diagnosis?
A Chronic myeloleucosis
B Leukemoid reaction
C Lymphogranulomatosis
D Hepar cirrhosis
E Urolithiasis

104
A 56-year-old patient with diffuse toxic goiter has ciliary arrhythmia with pulse rate of 110 bpm,
arterial hypertension, AP- 165/90 mm Hg. What preparation should be administered along with
mercazolil?
A Propranolol
B Radioactive iodine
C Procaine hydrochloride
D Verapamil
E Corinfar

105
A 54-year-old patient complains of weakness, weight loss despite the unchanged appetite,
frequent urination, skin itch for six months. Some time ago the patient underwent treatment for
furunculosis. She hasn't been examined recently. Objectively: malnutrition, dry skin with signs of
scratching. Small lymph nodes can be palpated in the axillary regions. Changes in the internal
organs are absenr. What testing must be administered in the first place?
A Blood sugar test on an empty stomach
B Complete blood count
C Endoscopy of stomach
D Lymph node biopsy
E Blood sterility testing

106
A 43 y.o. woman complains of severe pain in the right abdominal side irradiating in the right
supraclavicular area, fever, dryness and bitterness in the mouth. There were multiple vomitings
without relief. Patient relates the onset of pain to the taking of fat and fried food. Physical
examination: the patient lies on the right side, pale, dry tongue, tachycardia. Right side of
abdomen is painful during palpation and somewhat tense in right hypochondrium. What is the
most likely diagnosis?
A Perforative ulcer
B Acute cholecystitis
C Acute bowel obstruction
D Acute appendicitis
E Right-sided renal colic

107
Several hours before, a 28-year-old patient suddenly developed acute headache and repeated
vomiting, then lost consciousness. Objectively: focal neurological symptoms were not found.
Pronounced meningeal symptoms were revealed. AP - 120/80 mm Hg. According to clinical and
liquorological findings the patient was diagnosed with subarachnoid haemorrhage. After
administration of dehydrants the patient's condition somewhat improved. What is the main
component of further emergency care?
A Coagulants
B Anticoagulants
C Antiaggregants
D Fibrinolytics
E Corticosteroids

108
On the 5th day of a respiratory disease accompanied by fever a 24-year-old man developed
progressing headaches, systemic dizziness, double vision, facial muscles paresis on the right,
choking from swallowing. He was diagnosed with acute viral encephalitis. Identify the main
tendency of the emergency treatment:
A Zovirax
B Glucocorticoids
C Ceftriaxone
D Lasix
E Haemodezum

109
A 24-year-old man on the 5th day of acute respiratory disease with high grade temperature
started having strong headaches, systemic dizziness, sensation of double vision, paresis of
mimic muscles to the right, tickling by swallowing. Diagnosis: Acute viral encephalitis. Determine
the basic direction of the emergent therapy.
A Zovirax
B Glucocorticoids
C Cephtriaxon
D Lasix
E Hemodesis

110
A 30-year-old patient was delivered to the admission ward of the infectious disease department.
The disease had started acutely on the background of normal temperature with the appearance
of frequent, liquid, profuse stool without pathological impurities. Diarrhea was not accompanied
by abdominal pain. 12 hours later there appeared recurrent profuse vomiting. The patient rapidly
developed dehydration. What is the most likely diagnosis?
A Cholera
B Shigellosis
C Staphylococcal food toxicoinfection
D Salmonellosis
E Campylobacteriosis

111
A 65 y.o. woman complains of complicated mouth opening following foot trauma 10 days ago.
Next day she ate with difficulties, there were muscles tension of back, the back of the head and
abdomen. On the third day there was tension of all muscle groups, generalized convulsions
every 10-15 min. What is the most probable diagnosis?
A Tetanus
B Tetania
C Meningoencephalitis
D Hemorrhagic stroke
E Epilepsy

112
Gastric juice analysis of a 42-year-old male patient revealed absence of free hydrochloric acid
at all stages. Endoscopy revealed pallor, thinning of gastric mucosa, smoothed folds.
Microscopically the atrophy of glands with intestinal metaplasia was found. What disease is this
situation typical for?
A Chronic type A gastritis
B Chronic type B gastritis
C Chronic type C gastritis
D Menetrier disease
E Stomach cancer

113
A 45-year-old female patient complains of frequent liquid stools with a lot of mucus, pus and
blood; pain across the abdomen, loss of 7 kg within 6 months. She has a 1-year history of
non-specific ulcerative colitis. What group of drugs should be preferred for this patient?
A Corticosteroids
B Antibacterial
C Sulfonamides
D Nitrofurans
E Polyenzymes

114
A 54-year-old male patient complains of aching pain in the lumbar region, that is getting worse
after standing in an upright position, physical exercise, supercooling. The patient also reports of
experiencing weakness in the afternoon. Pain in the lumbar region, said about 10 years old.
Objectively: pale skin, to- 37,2oC, AP- 180/100 mm Hg, minor costovertebral angle
tenderness (Pasternatsky symptom). In blood: RBCs - 3,5x1012/l, WBCs - 6,5x109/l, ESR - 22 mm/h.
In urine: the relative density - 1010, leukocytes - 12-15 in the field of vision, erythrocytes - 2-3 in the
field of vision. Urine bacterial count - 100000 in 1 ml. What is the most likely diagnosis?
A Chronic pyelonephritis
B Nephrolithiasis
C Polycystic renal disease
D Chronic glomerulonephritis
E Amyloidosis

115
A 67-year-old male complains of dyspnea on exertion, attacks of retrosternal pain, dizziness.
He has no history of rheumatism. Objectively: pale skin, acrocyanosis. There are rales in the
lower parts of lungs. There is systolic thrill in the II intercostal space on the right, coarse systolic
murmur conducted to the vessels of neck. AP- 130/90 mm Hg, heart rate - 90/min, regular
rhythm. The liver extends 5 cm under the edge of costal arch, shin edemata are present.
Specify the assumed valvular defect:
A Aortic stenosis
B Pulmonary artery stenosis
C Mitral insufficiency
D Ventricular septal defect
E Tricuspid regurgitation

116
A 24-year-old female teacher complains of dizziness and heart pain irradiating to the left nipple.
Pain is not associated with physical activity and cannot be relieved by nitroglycerin, it abates
after taking Valocordin and lasts an hour or more. The patient has a nearly 2-year history of this
disease. Objectively: Ps- 76 bpm. AP- 110/70 mm Hg. Heart borders are normal, heart sounds
are clear. The ECG shows respiratory arrhythmia. Radiograph of the cervicothoracic spine
shows no pathology. Lungs, abdomen are unremarkable. What changes in blood formula can be
expected?
A No changes
B Leukocytosis
C Thrombocytopenia
D Leukemic hiatus
E Increased ESR

117
A 51-year-old female patient complains of frequent defecation and liquid blood-streaked stools
with mucus admixtures, diffuse pain in the inferolateral abdomen, 6 kg weight loss over the
previous month. Objectively: body temperature - 37,4oC, malnutrition, skin is pale and dry.
Abdomen is soft, sigmoid is painful and spasmodic, makes a rumbling sound. Liver is dense,
painful, extends 3 cm below the costal margin. What is the most likely diagnosis?
A Non-specific ulcerative colitis
B Bacillary dysentery
C Sprue
D Intestinal enzymopathy
E Helminthic invasion

118
A 18 y.o. female student complains of dyspnea during the intensive exertion. The condition
became worse half a year ago. On examination: pulse rate is 88 bpm, accelerated, AP- 180/20
mm Hg, pale skin, heart borders are dilated to the left and up. There is systolic-diastolic murmur
in the 2hd intercostal space, S2 at pulmonary artery is accentuated. ECG has revealed both
ventricles hypertrophy. Thoracic X-ray has revealed pulsation and protrusion of the left
ventricle, lung trunk. What doctor's tactics should be?
A Cardiosurgeon consultation
B Dispensary observation
C Administration of therapeutic treatment
D Continuation of investigation
E Exemption from physical exercises

119
A 49-year-old male patient complains of dyspnea of combined nature, cough, shin edemata,
abdomen enlargement due to ascites. He has a 20-year history of chronic bronchitis. For the
last 3 years he has been disabled (group II) because of cardiac changes. Objectively: mixed
cyanosis, edemata. Ps - 92/min, rhythmic, AP - 120/70 mm Hg, respiration rate - 24/min. There
is accentuation of the second sound above the pulmonary artery. Auscultation reveals the box
resonance above the lungs. There are also dry rales over the entire surface of lungs. What is
the mechanism of heart changes development in this patient?
A Euler-Liljestrand reflex
B Kitaev's reflex
C Bainbridge reflex
D Cardiovascular reflex
E Respiratory reflex

120
A 24-year-old man on the second day of the disease with a sudden onset complains of a strong
headache in temples and in the area of orbits, dull pain in the body, dry painful cough. His
temperature is 39oC. Adynamic. Mucous membrane of oropharynx is "flaming", rales are
not ausculated. What is the most probable diagnosis?
A Influenza
B Parainluenza
C Respiratory mycoplasmosis
D Pneumonia
E Meningococcus infection

121
A 32-year-old female complains of dizziness, headache, palpitation, tremor. For the last several
months she has been under outpatient observation for the increased arterial pressure. Since
recently such attacks have become more frequent and severe. Objectively: skin is covered with
clammy sweat, tremor of the extremities is present. HR- 110/min, AP- 220/140 mm Hg. Heart
sounds are muffled. Blood test results: WBCs - 9,8x109/l, ESR - 22 mm/h. Blood
glucose - 9,8 millimole/l. What disease is the most likely cause of this crisis?
A Pheochromocytoma
B Essential hypertension
C Preeclampsia
D Primary hyperaldosteronism
E Diabetic glomerulosclerosis

122
A 76-year-old male consulted a therapist about slow discharge of urine with a small jet. The
patient reported no cardiac problems. Examination revealed atrial fibrillation with a heart rate of
72/min and without pulse deficit. There are no signs of heart failure. ECG confirms the presence
of atrial fibrillation. From history we know that the arrhythmia was detected three years ago.
What tactics for the treatment of atrial fibrillation in the patient should be chosen?
A Does not require treatment
B Digoxin
C Verapamil
D Obzidan
E Ajmaline

123
A 43-year-old female patient complains of unstable defecation with frequent constipations,
abdominal swelling, headache, sleep disturbance. Body weight is unchanged. What disease are
these clinical presentations typical for?
A Irritable colon syndrome
B Chronic enteritis
C Chronic pancreatitis
D Chronic atrophic gastritis
E Colorectal cancer

124
A 43-year-old man who often contacts with ethyl gasoline was admitted to a hospital with
complaints of general weakness, dizziness, memory impairment, sleepiness at daytime and
insomnia at night, sensation of a hair in the mouth, colicky pains in the right subcostal region.
What is the most likely diagnosis?
A Chronic tetraethyl lead intoxication
B Alcoholic delirium
C Chronic mercury intoxication
D Chronic manganese intoxication
E Chronic lead intoxication

125
A 35-year-old patient has been in the intensive care unit for acute renal failure due to crush for
4 days. Objectively: the patient is inadequate. Breathing rate - 32/min. Over the last 3 hours
individual moist rales can be auscultated in lungs. ECG shows high T waves, right ventricular
extrasystoles. CVP - 159 mm Hg. In blood: the residual nitrogen - 62 millimole/l, K+- 7,1
millimole/l, Cl- - 78 millimole/l, Na+- 120 millimole/l, Ht - 0,32, Hb - 100 g/l, blood creatinine
- 0,9 millimole/l. The most appropriate method of treatment would be:
A Hemodialysis
B Plasma sorption
C Hemosorption
D Plasma filtration
E Ultrafiltration

126
A 45-year-old man was brought to clinic with complaints of the pain that started suddenly in the
left chest part and epigastric area, shortness of breath, nausea, one-time vomiting. The acute
pain started after weight-lifting. On physical exam: shallow breathing, RR - 38/min, left chest
part is behind during respiration, by percussion - tympanitic sound, respiration is not ausculated.
Ps - 110 bpm, of weak filling. BP- 100/60 mm Hg, insignificant displacement of heart to the right,
sounds are dull. What examination is the most expedient to do first?
A Roentgenography
B Electrocardiography
C Bronchoscopy
D Esophagogastroscopy
E Ultrasound of the abdominal cavity

127
A 35 y.o. woman is suspected of aplastic anemia. The bone marrow punction has been
administered with the diagnostic purpose. What changes in the marrow punctatum are suggested?
A Replacement of marrow elements with adipose tissue
B Replacement of marrow elements with fibrous tissue
C Prevalence of megaloblasts
D Presence of blast cells
E Absolute lymphocytosis

128
A 58-year-old female patient complains of spontaneous bruises, weakness, bleeding gums,
dizziness. Objectively: the mucous membranes and skin are pale with numerous hemorrhages
of various time of origin. Lymph nodes are not enlarged. Ps is 100/min, AP - 110/70 mm Hg.
There are no changes of internal organs. Blood test results: RBC - 3,0x1012/l, Нb - 92 g/l, colour
index - 0,9, anisocytosis, poikilocytosis, WBC – 10x109/l, eosinophils - 2%, stab neutrophils - 12%,
segmented neutrophils - 68%, lymphocytes - 11%, monocytes - 7%, ESR - 12 mm/h. What laboratory
test is to be determined next for making a diagnosis?
A Platelets
B Reticulocytes
C Clotting time
D Osmotic resistance of erythrocytes
E Fibrinogen

129
A 47-year-old woman underwent a thyroid gland resection on ccount of nodular euthyroid goiter.
What preparations are most likely to prevent the disease recurrence?
A Thyroid hormones
B Mercazolil
C Thyrotropin
D Antistruminum (potassium iodide)
E Radioactive iodine

130
A 55 y.o. male patient complains of weakness during 2 months, pain in the right side of the
thorax, cough, blood-streaked sputum. On X-ray: intensive triangle shadow in the area of lower
lobe that is connected to mediastinum. What is the most likely disorder in the lungs?
A Central cancer of lungs
B Tuberculosis of lungs
C Bronchiectasia
D Pulmonary infarction
E Pleuropneumonia

131
A 60 y.o. patient experiences acute air insufficiency following of the venoectomy due to
subcutaneous vein thrombophlebitis 3 days ago. Skin became cianotic, with grey shade. Marked
psychomotor excitement, tachypnea, substernal pain. What postoperative complication has
occured?
A Thromboembolia of pulmonary artery
B Hemorrhagia
C Hypostatic pneumonia
D Myocardial infarction
E Valvular pneumothorax

132
A 19-year-old woman complains of pain in the abdomen and joints, asks for more analgetics and
somnifacient injections. The patient was examined. Gynecological and urological pathologies
are absent. There are signs of previous punctures along superficial veins of the extremities.
The patient does not explain the origin of punctures. Tendon reflexes of upper and lower
extremities are the same, quick. Photoreaction of the pupil of the eye is weak. The tongue is
grey coated. During communication the patient in affectively not even-tempered. There is
diarrhea without pathologic inclusions. What tactics is necessary to improve the condition of this
patient?
A Consultation of an expert in narcology
B Prescription of medications the patient asks for
C Additional consultation of surgeon
D Treatment with antibiotics
E Consultation of infectious diseases doctor

133
A patient has an over a year-old history of fast progressive rheumatoid arthritis. X-raying
confirms presence of marginal erosions. What basic drug would be the most appropriate in this
case?
A Methotrexate
B Chloroquine
C Prednisolone
D Diclofenac sodium
E Aspirin

134
A female rheumatic patient experiences diastolic thoracic wall tremor (diastolic thrill),
accentuated S1 at apex, there is diastolic murmur with presystolic intensification, opening
snap, S2 accent at pulmonary artery. What rind of heart disorder is observed?
A Mitral stenosis
B Aortic valve insufficiency
C Pulmonary artery stenosis
D Mitral valve insufficiency
E Opened arterial duct

135
A 23-year-old patient complains of a dull ache, sensation of heaviness and distention in the
epigastrium immediately after meals, foul-smelling eructation; dry mouth, empty stomach
nausea, diarrhea. Objectively: the skin is pale, the patient is of thin build. Abdomen is soft on
palpation, there is epigastric pain. The liver does not extend beyond the costal arch. In blood:
Hb - 110 g/l, RBCs - 3,4x1012/l, WBC count is normal. ESR - 16 mm/h. What is the most informative
study that will allow make a diagnosis?
A Esophageal gastroduodenoscopy
B X-ray of digestion organs
C Study of gastric juice
D pH-metry
E Duodenal probing

136
A 49-year-old patient complains of deglutition problems, especially with solid food, hiccups,
voice hoarseness, nausea, regurgitation, significant weight loss (15 kg within 2,5 months).
Objectively: body weight is reduced. Skin is pale and dry. In lungs: vesicular breathing, heart
sounds are loud enough, heart activity is rhythmic. The abdomen is soft, painless on palpation.
Liver is not enlarged. What study is required to make a diagnosis?
A Esophageal duodenoscopy along with biopsy
B Clinical blood test
C X-ray of digestive tract organs
D X-ray in Trendelenburg's position
E Study of gastric secretion

137
A 60-year-old patient has been admitted to a hospital with complaints of dyspnea, tightness in
the right subcostal area, abdomen enlargement. These presentations have been progressing
for a year. Heart auscultation reveals presystolic gallop rhythm. Objectively: swelling of the neck
veins, ascites, palpable liver and spleen. What disease requires differential diagnostics?
A Constrictive pericarditis
B Hepatocirrhosis
C Lung cancer with invasion to the pleura
D Chronic pulmonary heart
E Pulmonary embolism

138
A 40-year-old patient, the forester, complains of severe headache, body temperature rise up to
39,5oC, trembling limbs. From the patient's history we know that he had seriously cut his
hand during the dissection of a killed fox. Objectively: depressed mood. The patient asks not to
turn on the light or open the door. Any noise causes apparent motor excitation. When he saw a
carafe of water, he developed convulsive throat spasms. What tactics should an emergency
doctor choose?
A Deliver the patient to the infectious disease hospital
B Deliver the patient to the resuscitation department
C Deliver the patient to the neurological department
D Deliver the patient to the psychiatric hospital
E Let him stay at home and consult a psychiatrist

139
A 28-year-old woman has a 12-year history of chronic glomerulonephritis with latent course.
Over the past six months she has developed general weakness, loss of appetite, low work
performance, nausea. The patient complains of headache, pain in the joints. On examination:
anemia, blood urea - 34,5 millimole/l, blood creatinine - 0,766 millimole/l, hyperkalemia. What
complication has developed?
A Chronic renal insufficiency
B Acute renal insufficiency
C Nephrotic syndrome
D Renal amyloidosis
E Pyelonephritis

140
A 72-year-old male had had a moderate headache. Two days later, he developed the
progressing speech disorders and weakness in the right extremities. The patient has a history
of myocardial infarction, arrhythmia. Study of the neurologic status revealed elements of motor
aphasia, central paresis of the VII I XII cranial nerves on the right, central hemiparesis on the
same side and hyperaesthesia. What is the most likely diagnosis?
A Ischemic stroke
B Hemorrhagic stroke
C Transient ischemic attack
D Epidural hematoma
E Brain tumor

141
A 70 y.o. male patient with mild headaches complains of speech disorder, weakness in right
limbs. There was a history of miocardial infarction and arrhythmia. On nu eroligical examination
there are elements of motor aphasia, central paresis of VII and XII cranial nerves pairs on the
right side, cental type of hemiparesis and hemihyperesthisia on the same side. What is the most
probable diagnosis?
A Ischemic stroke
B Hemorrhagic stroke
C Transitory ischemic attack
D Epidural hematoma
E Cerebral tumor

142
After treating a field with pesticides a machine operator presents with great weakness,
headache, nausea, vomiting, diarrhea, visual impairment, watery eyes. Objectively: the patient is
excited, hypersalivation, hyperhidrosis, muscle fibrillation of tongue and eyelids are oberved.
Pupils are narrowed, there is tachycardia, lung auscultation reveals moist small and medium
bubbling rales. In blood: changed level of cholinesterase activity. What is the most likely diagnosis?
A Intoxication with organophosphorous pesticides
B Intoxication with organochlorine pesticides
C Intoxication with organomercurial pesticides
D Intoxication with arsenic-containing pesticides
E Intoxication with carbamic acid derivatives

143
A 40-year-old man is ill with autoimmune hepatitis. Blood test: А/G ratio 0,8, bilirubin – 42 mu*mol/L,
transaminase: ALT- 2,3 mmol g/L, АSТ - 1,8 mmol g/L. What is the most effective means in
treatment from the given below?
A Glucocorticoids, cytostatics
B Antibacterial medication
C Hepatoprotectors
D Antiviral medications
E Hemosorbtion, vitamin therapy

144
A farmer hurt his right foot during working in a field and came to the emergency station. He
doesn't remember when he got last vaccination and he has never served in the army.
Examination of his right foot revealed a contaminated wound up to 5-6 cm long with uneven
edges. The further treatment tactics will be:
A To make an injection of tetanus anatoxin and antitetanus serum
B To make an injection of tetanus anatoxin
C To make an injection of antitetanus serum
D Surgical d-bridement only
E To administer an antibiotic

145
A 35-year-old patient has been admitted to a hospital for pain in the left sternoclavicular and
knee joints, lumbar area. The disease has an acute character and is accompanied by fever up
to 38oC. Objectively: the left sternoclavicular and knee joints are swollen and painful. In
blood: WBCs - 9,5x109/l, ESR - 40 mm/h, CRP - 1,5 millimole/l, fibrinogen - 4,8 g/l, uric
acid - 0,28 millimole/l. Examination of the urethra scrapings reveals chlamydia. What is the most
likely diagnosis?
A Reiter's syndrome
B Rheumatic arthritis
C Gout
D Bechterew's disease
E Rheumatoid arthritis

146
A 20 daily y.o. female patient is suffering from chronic bronchitis. Recently there has been
production about 0,5 L of purulent sputum with maximum discharge in the morning. Fingers are
like "drum sticks", there are "watching glass" nails. What is the most probable diagnosis?
A Bronchiectasia
B Pneumonia
C Chronic bronchitis
D Gangrene of lungs
E Tuberculosis

147
Topographic percussion of lungs in a patient who got a serious job-related barotrauma revealed
that the lower lungs borders were located one rib below normal, there was a significant increase
in both lungs height and Kronig's isthmus. What disease should be suspected in the first place?
A Pulmonary emphysema
B Exudative pleuritis
C Chronic bronchitis
D Bronchial asthma
E Pneumothorax

148
An 18 y.o. girl complains of weakness, dizziness, loss of appetite, menorrhagia. There are
many-coloured petechiae on the skin of the upper extremities. Blood test: Hb- 105 g/l; RBC-
3,2x1012/L; C.I.- 0,95; thromb.- 20x109/L. The sedimentation time according to Lee
White is 5'; hemorrhagia duration according to Duke is 8', "pinch and tourniquet" test is
positive. What is the most probable diagnosis?
A Idiopathic thrombocytopenic purpura
B Hemophilia
C Hemorrhagic diathesis
D Iron deficiency anemia
E Marchiafava-Micheli's disease

149
A 28 y.o. male patient was admitted to the hospital because of high temperature 39oC,
headache, generalized fatigue, constipation, sleep disorder for 9 days. There are sporadic
roseolas on the abdomen, pulse- 78 bpm, liver is enlarged for 2 cm. What is the most probable
diagnosis?
A Abdominal typhoid
B Typhus
C Sepsis
D Brucellosis
E Leptospirosis

150
A 50-year-old patient complains about having pain attacks in the right subcostal area for about a
year. He pain arises mainly after taking fattening food. Over the last week the attacks occurred
daily and became more painful. On the 3rd day of hospitalization the patient presented with
icteritiousness of skin and scleras, light-colored feces and dark urine. In blood: neutrophilic
leukocytosis - 13,1x109/l, ESR- 28 mm/h. What is the most likely diagnosis?
A Chronic calculous cholecystitis
B Chronic recurrent pancreatitis
C Fatty degeneration of liver
D Chronic cholangitis, exacerbation stage
E Hypertensive dyskinesia of gallbladder

151
A 20 y.o. patient with bronchial asthma experiences dyspnea attacks 3-4 times a week.
Nocturnal attacks are 1 time a week. FEV1- 50% of necessary figures, during the day it's
variations is 25%. What is the severity of bronchial asthma condition?
A Moderate severity condition
B Mild condition
C Serious condition
D Asthmatic status
E Intermittent flow

152
A 40 y.o. man complains of headache in occipital area. On physical examination: the skin is
pale; face and hand edema, BP- 170/130 mm Hg. On EchoCG: concentric hypertrophy of the
left ventricle. Ultrasound examination of the kidneys reveals thinned cortical layer. Urine
analysis shows proteinuria of 3,5 g/day. What is the probable diagnosis?
A Essential arterial hypertension
B Chronic pyelonephritis
C Chronic glomerulonephritis
D Polycystic disease of the kidneys
E Cushing's disease

153
A 27-year-old patient has a severe headache, nausea and vomiting. Objectively: body
temperature is 38,9oC, there is a haemorrhagic stellate rash on the legs. The patient takes
meningeal pose in bed. Meningeal symptoms are strongly positive. Deep reflexes are brisk,
uniform. Pathological reflexes are absent. It has been suspected that the patient has epidemic
cerebrospinal meningitis. Which of additional tests should be performed in the first place to
verify the diagnosis?
A Lumbar puncture
B Echoencephalography
C Rheoencephalography
D Electroencephalography
E Survey craniogram

154
After a serious nervous stress a 35-year-old patient has developed on the dorsal surface of
hands redness and swelling that were later replaced by small inflammatory nodules, vesicles and
following erosion with a significant serous discharge. The process is accompanied by severe
itching. What is the most likely diagnosis?
A True eczema
B Allergic dermatitis
C Microbal eczema
D Simple contact dermatitis
E Toxicoderma

155
A 36-year-old patient complains of skin rash that appeared a week ago and doesn't cause any
subjective problems. Objectively: palm and sole skin is covered with multiple lenticular
disseminated papules not raised above the skin level. The papules are reddish, dense on
palpation and covered with keratinous squamae. What is the provisional diagnosis?
A Secondary syphilis
B Verrucosis
C Palmoplanar psoriasis
D Palmoplanar rubrophytosis
E Palm and sole callosity

156
In the morning a patient had nausea, abdominal discomfort, single vomiting, dry mouth. In the
evening, the patient presented with the increasing general weakness, double vision, difficult
swallowing of solid food. Objectively: ptosis, mydriasis, anisocoria, absence of gag and
pharyngeal reflex, dry mucous membranes. The previous evening the patient had dinner with
canned food and alcohol. What is the presumptive diagnosis?
A Botulism
B Food toxicoinfection
C Intoxication with unknown poison
D Acute ischemic stroke
E Poliomyelitis

157
A 30-year-old patient complains of paroxysmal abdominal pain, frequent liquid stools up to 10
times a day. Throughout the first 3 days he had a fever, since the 2nd day of disease there
were scant liquid stools mixed with mucus. On palpation: tenderness of all colon segments.
Sigmoid colon was found spastic. What is your provisional diagnosis?
A Acute dysentery
B Intestinal amebiasis
C Salmonellosis
D Cholera
E Balantidiasis

158
A 38-year-old woman experiences episodic increases in arterial pressure up to 240/120 mm
Hg, which is accompanied by nausea, vomiting, tachycardia, increased sweating,
hyperglycemia. The attack is usually followed by the excessive urination. Renal sonography
reveals an additional formation adjacent to the upper pole of the right kidney and possibly
belonging to the adrenal gland. What laboratory test will allow to clarify the diagnosis?
A Determination of urinary excretion of catecholamines and vanillylmandelic acid
B Blood test for insulin and C-peptide
C Estimation of glomerular filtration rate by measuring endogenous creatinine clearance
D Blood test for thyroxine and thyrotrophic hormone
E Blood test for renin level

159
A 32-year-old patient has a 3-year history of asthma attacks, that can be hardly stopped with
berotec. Over a few last months he has experienced pain in the joints and sensitivity disorder of
legs and feet skin. Ps - 80/min, AP - 210/100 mm Hg. In blood: eosinophilia at the rate of 15%.
What disease can be suspected in this case?
A Periarteritis nodosa
B Systemic lupus erythematosus
C Systemic scleroderma
D Dermatomyositis
E Wegener's disease

160
3 hours before, a 68-year-old male patient got a searing chest pain radiating to the neck and left
forearm, escalating dyspnea. Nitroglycerin failed to relieve pain but somewhat reduced
dyspnea. Objectively: there is crimson cyanosis of face. Respiratory rate is 28/min. The patient
has vesicular breathing with isolated sibilant rales. Heart sounds are muffled, with a gallop
rhythm. Ps - 100/min, AP - 100/65 mmHg. ECG shows negative T-wave in V2-V6 leads. What
drug can reduce the heart's need for oxygen without aggravating the disease?
A Isosorbide dinitrate
B Corinfar
C Atenolol
D Streptokinase
E Aminophylline

161
A 46-year-old patient complains of sudden palpitation, which is accompanied by pulsation in the
neck and head, fear, nausea. The palpitation lasts for 15-20 minutes and is over after straining
when holding her breath. What kind of cardiac disorder may be suspected?
A An attack of supraventricular paroxysmal tachycardia
B An attack of ventricular paroxysmal tachycardia
C An attack of atrial flutter
D An attack of ciliary arrhythmia
E An attack of extrasystolic arrhythmia

162
A 5-grade pupil complains about extensive skin rash accompanied by intensive itch, especially
at night. Objectively: there are small red papules set mostly in pairs in the region of interdigital
folds on both hands, on the flexor surface of radicarpal articulations, abdomen and buttock skin
as well as internal surface of thighs. In the centre of some papules vesicles or serohaemorrhagic
crusts can be seen. There are multiple excoriations. What is the most likely diagnosis?
A Scabies
B Dermatitis
C Ringworm of body
D Toxicoderma
E Eczema

163
A welder at work got the first-degree burns of the middle third of his right shin. 5 days later the
skin around the burn became edematic and itchy. Objectively: on a background of a well-defined
erythema there is polymorphic rash in form of papules, vesicles, pustules, erosions with serous
discharge. What is the most likely diagnosis?
A Microbal eczema
B True eczema
C Toxicoderma
D Occupational eczema
E Streptococcal impetigo

164
A 58-year-old patient has a 3-year history diabetes mellitus type II. He has been keeping to a
diet and regularly taking glyburide. He has been delivered to a hospital on an emergency basis
for acute abdomen. Objectively: the patient is of supernutrition type. The skin is dry. In the lungs
vesicular breathing can be auscultated. Heart sounds are regular, 90/min. AP- 130/70 mm Hg.
The symptom of "wooden belly" is visible. Blood sugar - 9,8 millimole/l. The patients has
indication for laparotomy. What is the most appropriate way of further treatment of diabetes?
A To administer short insulin
B To continue taking glyburide
C To administer Semilong to be taken in the morning and insulin - in the evening
D To administer 1 tablet of Glurenorm three times a day
E To administer 1 tablet of Maninil three times a day

165
A 56 y.o. man, who has taken alcoholic drinks regularly for 20 years, complains of intensive
girdle pain in the abdomen. Profuse nonformed stool 2-3- times a day has appeared for the last
2 years, loss of weight for 8 kg for 2 years. On examination: abdomen is soft, painless. Blood
amylase - 12g/L. Feces examination-neutral fat 15 g per day, starch grains. What is the most
reasonable treatment at this stage?
A Pancreatine
B Contrykal
C Aminocapron acid
D Levomicytine
E Imodium

166
A 43-year-old female patiet complains of eruption on her right leg skin, pain, weakness, body
temperature rise up to 38oC. The disease is acute. Objectively: there is an edema on the
right leg skin in the region of foot, a well-defined bright red spot in form of flame tips which feels
hot. There are isolated vesicles in focus. What is your provisional diagnosis?
A Erysipelas
B Microbial eczema
C Contact dermatitis
D Toxicoderma
E Haemorrhagic vasculitis

167
A 45-year-old patient complains of some painless nodular elements tending to peripheral growth
and fusion. He has a 2-year history of this disease. Aggravation takes place mainly in spring. In
anamnesis: the patient's father had similar skin lesions. Objectively: pathological elements looke
like guttate and nummular nodules, plaques covered with white scales. What is your provisional
diagnosis?
A Psoriasis
B Lichen ruber planus
C Neurodermitis
D Pityriasis rosea
E Seborrheic eczema

168
A 47-year-old patient came to see a doctor on the 7th day of disease. The disease developed
very fast: after the chill body temperature rose up to 40oC and lasted up to 7 hours, then it
dropped abruptly, which caused profuse sweat. There were three such attacks occuring once in
two days. Two days ago the patient arrived from Africa. Objectively: pale skin, subicteric sclera,
significantly enlarged liver and spleen. What is the cause of fever attacks in this disease?
A Erythrocytic schizogony
B Tissue schizogony
C Exotoxin of a causative agent
D Endotoxin of a causative agent
E Gametocytes
169
On the 2nd day of disease a 27-year-old patient complains of unbearable headache, repeated
vomiting. Objectively: the patient is in a grave condition. He is conscious but adynamic. Lies in a
forced position with his head thrown back. There is no skin rash. Nuchal muscles are evidently
rigid, there are Kernig's and Brudzinski's signs. to - 39,5oC, Ps -120/min, AP - 130/80 mm Hg. The
leading syndrome of this disease is caused by:
A Liquor hypertension
B Liquor hypotension
C Affection of the cranial nerve nuclei
D Haemorrhages in the adrenal glands
E Hyperthermy

170
On the 2nd day of illness a 27-year-old patient complains of unbearable headache, repeated
vomiting. Objectively: the patient is in a grave condition. He is conscious but adynamic. Lies in a
forced position with his head thrown back. There is no skin rash. Nuchal muscles are evidently
rigid, there are Kernig's and Brudzinski's signs. to- 39,5oC, Ps- 120/min, AP- 130/80 mm Hg. The
leading syndrome of this disease is caused by:
A Liquor hypertension
B Liquor hypotension
C Affection of the cranial nerve nuclei
D Haemorrhages into the adrenal glands
E Hyperthermy

171
A 43 y.o. woman complains of shooting heart pain, dyspnea, irregularities in the heart activity,
progressive fatigue during 3 weeks. She had acute respiratory disease a month ago. On
examination: AP- 120/80 mm Hg, heart rate 98 bpm, heart boarders +1,5 cm left side, sounds
are muffled, soft systolic murmur at apex and Botkin's area; sporadic extrasystoles. Liver isn't
palpated, there are no edema. Blood test: WBC- 6,7x109/L, sedimentation rate- 21 mm/hour. What is
the most probable diagnosis?
A Acute myocarditis
B Climacteric myocardiodystrophia
C Ichemic heart disease, angina pectoris
D Rheumatism, mitral insufficiency
E Hypertrophic cardiomyopathy

172
A 63-year-old female complains of general weakness, a feeling of heaviness, compression in
the epigastrium, postprandial fullness, nausea, belching after meals. These symptoms have
been observed for about 15 years. Objectively: body temperature is 36,4oC, respiratory
rate - 20/min, Ps - 88/min, blood pressure - 115/75 mm Hg. Skin and mucous membranes are
pale. Blood test results: RBC - 2,0x1012/l, Hb - 100 g/l. Tests revealed parietal-cell antibodies. What
is the most likely reason for the development of anemia in this patient?
A Production of antibodies to intrinsic factor
B Disruption of hemoglobin synthesis
C Disruption of erythropoietin synthesis
D Impaired iron absorption
E Increased loss of iron

173
A 37-year-old woman is sick with bronchial asthma for 15 years. Recenlty asthmatic attacks occur
4-5 times per week, night attacks -2-3 times per month. To stop attacks, the patient takes salbutamol.
On physical exam: condition is relatively satisfactory. RR - 20/min, Ps is 76 bpm, BP - 120/80 mm
Hg. Respiration in lungs is vesicular. Cardiac sounds are muted, rhythm is normal. What medication
should be prescribed to prevent attacks of bronchial asthma on the first stage?
A Cromoglycat sodium
B Regular dose of salbutamol
C Inhalation corticosteroids
D Tabletted corticosteroids
E Injection of corticosteroids

174
A 52 y.o. male patient has become ill gradually. There is pain in the left side of the thorax during
2 weeks, elevation of temperature till 38-39oC. On examination: left chest side falls behind
in breathing movement no voice tremor over the left lung. Dullness that is more intensive in lower
parts of this lung. Right heart border is deviated outside. Sharply weakened breathing over the
left lung, no rales. Heart sounds are mufflet, tachycardia. What is the most probable diagnosis?
A Exudative pleuritis
B Spotaneous pneumothorax
C Atelectasis of lung
D Cirrhotic tuberculosis
E Infarction-pneumonia

175
A 50-year-old patient was hospitalized in severe condition with complaints of chills, high grade
temperature, dryness in the mouth, multiple vomiting, pain in the epigastrium, frequent watery,
foamy, dirty green color stool of unpleasant odor. The tongue and the skin are dry. BP - 80/40
mm Hg. What first aid is necessary for the patient?
A Intravenous injection of sodium solutions
B Fresh-frozen plasma transfusion
C To prescribe polyglucin
D Sympathomimetics
E Hemosorbtion

176
A 42-year-old female patient suffers from micronodular cryptogenic cirrhosis. Over the last
week her condition has deteriorated: she developed convulsions, mental confusion, progressing
jaundice. What study may give reasons for such aggravation?
A Determination of serum ammonia
B Determination of cholesterol ethers
C Determination of alpha-phetoprotein
D Determination of ALAT and ASAT
E Determination of alkaline phosphatase

177
A patient has chronic heart failure of the II stage. He takes furosemide regularly three times a
week. He had developed bronchopneumonia and had been administered combined
pharmacotherapy. On the fifth day of therapy the patient complained of hearing impairment.
What drug coadministered with furosemide might have caused the hearing loss?
A Gentamicin
B Linex
C Nystatin
D Tavegil
E Mucaltin

178
A 60-year-old female patient complains of recurrent pain in the proximal interphalangeal and
wrist joints, their periodic swelling and reddening that have been observed for 4 years. X-ray
picture represents changes in form of osteoporosis, joint space narrowing and single usuras.
What is the most likely diagnosis?
A Rheumatoid arthritis
B Osteoarthritis
C Gout
D Pseudogout
E Multiple myeloma

179
A 37-year-old woman complains of generalized fatigue, irritability, dysphagia, chalk hunger. On
physical exam: t- 36,5oC, respirations - 20/min, Ps - 96 bpm, BP - 110/70 mm Hg. Satisfactory
nourishment. The skin and visible mucous membranes are pale. Blood test: Hb -70g/L, erythrocytes -
3,4x1012/L$, CI - 0,7, reticulocytes - 2%, leucocytes - 4,7x109/L, eosinophilis. - 2%, band neutrophils
- 3%, segmented neutrophils - 64%, lymphocytes – 26%, monocytes - 5%, ESR - 15 mm/min. Serum
ferrum - 7,3 mu*mol/L, total protein - 70g/L. Deficit of what factor caused the development of the
disease?
A Ferrum
B Vitamin B6
C Vitamin B12
D Protein
E Folic acid

180
A 28-year-old patient has been hospitalized for the pain in the epigastric region. He has a
10-year history of duodenal ulcer (DU). Recently, the pain character has changed: it became
permanent, persistent, irradiating to the back. There are general weakness, dizziness, fatigue.
The patient has put off weight. Objectively: HR- 68/min, AP- 120/80 mm Hg. What is most likely
cause of deterioration?
A Penetration
B Haemorrhage
C Perforation of duodenal wall
D Exacerbation of duodenal ulcer
E Stenosis development

181
A 57-year-old male patient complains of dyspnea on exertion, heaviness in the right
hypochondrium and shin edemata towards evening. Objectively: temperature - 38,1oC, HR-
20/min, HR=Ps=92/min, AP- 140/90 mm Hg. There is apparent kyphoscoliosis. In the lungs
single dry rales can be auscultated. Heart sounds are muffled, rhythmic. ECG: Rv1+Sv5=15
mm. X-ray picture shows the bulging of pulmonary artery cone, right ventricle enlargement. What
is the most likely cause of this condition?
A Pulmonary heart
B Atherosclerotic cardiosclerosis
C Dilatation cardiomyopathy
D Mitral stenosis
E Primary pulmonary hypertension

182
A 30-year-old female patient has been delivered to a hospital for sudden dyspnea progessing to
asthma, sensation of having a "lump in the throat", hand tremor, fear of death. The attack has
developed for the first time and is associated with a strong emotion. There is no previous
history. Objectvely: respiratory rate - 28/min, Ps - 104/min, rhythmic, AP - 150/85 mm Hg. The
patient has rapid superficial vesicular breathing with extended expiration. Percussion findings:
heart borders are not changed. Cardiac sounds are loud, rhythmic. What is the most likely
diagnosis?
A Neurocirculatory asthenia
B Bronchial asthma
C Hypertensive crisis
D Cardiac asthma
E Thyrotoxic crisis

183
A 24-year-old emotionally-labile woman presents with irritation, depressed mood, palpitation,
shooting pain in the heart area, generalized fatigue following the divorce. On examination: palm
hyperhydrosis, pulse rate- 72-78 bpm, labile, heart without changes. ECG is normal. What is the
most probable pathology in this case?
A Neurasthenia
B Ipochondric neurosis
C Compulsive neurosis
D Schizophrenia
E Depressive neurosis

184
A 30-year-old woman with a long history of chronic pyelonephritis complains about considerable
weakness, sleepiness, decrease in diuresis down to 100 ml per day. AP- 200/120 mm Hg. In
blood: creatinine - 0,62 millimole/l, hypoproteinemia, albumines - 32 g/l, potassium - 6,8
millimole/l, hypochromic anemia, increased ESR. What is the first step in the patient treatment
tactics?
A Haemodialysis
B Antibacterial therapy
C Enterosorption
D Haemosorption
E Blood transfusion

185
A patient had macrofocal myocardial infarction. He is overweight for 36%, AP is 150/90 mm
Hg, blood sugar- 5,9 mmol/L, general cholesterol- 4,9 mmol/L, uric acid- 0,211 mmol/L. Which
risk factor should be urgently eradicated during the secondary prevention?
A Obesity
B Arterial hypertension
C Hyperglycemia
D Hypercholesterolemia
E Hyperuricemia

186
A 36-year-old female patient complains of bruises on the body, gingival haemorrhage, general
weakness. A month ago she had a severe domestic poisoning with some pesticide (the patient
can not remember the name). She has a 7-year record of working in contact with petroleum
products, particularly benzene. In blood: RBCs - 3,2x1012/l, WBCs - 2,7x109/l, thrombocytes –
70x109/l. What is the most likely pathology?
A Benzene intoxication
B Organophosphorus pesticide intoxication
C Organochlorine pesticide Intoxication
D Mercury-containing pesticide intoxication
E Chronic fatigue Syndrome

187
While staying in a stuffy room a 19-year-old emotionally labile girl developed severe weakness,
dizziness, blackout, nausea and loss of consciousness without convulsions. Objectively: the
patient is unconscious, the skin is pale, extremities are cold. AP- 90/60 mm Hg, Ps- 96/min,
deficient, breathing is shallow. Pupillary and tendon reflexes are present. There are no
pathological signs. What is the most likely diagnosis?
A Syncope
B Vegetovascular paroxysm
C Epileptic attack
D Hysterical neurosis
E Transient ischemic attack

188
A patient complains of frequent, bulky, frothy stools with greenish mucus, cramping pain in the
umbilical region, abdominal murmur, body temperature at the rate of 39oC. The patient
associates the disease with consumption of soft-boiled eggs. What is the most likely pathogen?
A Salmonella
B Yersinia
C Shigella
D Enteropathogenic E.Coli
E Vibrio cholerae El Tor

189
A 40-year-old female patient complains of headache, dizziness, muscle weakness, occasional
cramps in the extremities. She has been taking antihypertensive medications for 10 years. AP-
180/100 mm Hg. Blood potassium - 1,8 millimole/l, sodium - 4,8 millimole/l. In urine: alkaline
reaction, the relative density - 1012, protein and sugar are not found, WBCs - 3-4 in the field of
vision, RBCs - 1-2 in the field of vision. Conn's syndrome is suspected. Which drug should be
chosen for the treatment of arterial hypertension?
A Spironolactone
B Propanolol
C Enalapril
D Hydrochlorothiazide
E Clonidine

190
An 18-year-old patient presents no problems. Percussion reveals that heart borders are
displaced to the right and left by 1 cm, there is a coarse systolic murmur with its epicenter within
the 4th intercostal space on the left. What is the most informative examination to confirm the
clinical diagnosis?
A Ventriculography
B ECG
C PCG
D Echocardiography
E Polycardiography

191
A 56-year-old patient complains of having persistent chest pain on the right for the last 2
months. The pain is not associated with respiration. He also complains of cough with
blood-streaked sputum, weakness, decreased performance, fatigue. Chest radiograph shows a
globular shade of 4x6 cm connected to the root of the lung in the lower part of the right lung.
What is the most likely diagnosis?
A Peripheral lung cancer
B Metastasis
C Lung abscess
D Pneumonia
E Tuberculoma

192
A 18-year-old patient had subtotal strumectomy due to malignant capillary cystadenoma of the
thyroid gland. In 2 months there was a suspicion of metastasis presence in the lungs. What
rontgenological method is to be used first?
A Roentgenography of lungs
B Roentgenoscopy of lungs
C Angiopneumonography
D Bronchography
E Bronchoscopy

193
A 58-year-old patient was diagnosed basal-cell skin cancer, 1st stage. Tumor is up to 1 cm in
size and with up to 0,5 cm deep infiltration in tissues. Tumor is localized in the right nasolabial
area. Choose the most optimal method of treatment.
A Short-distance roentgenotherapy
B Long-distance roentgenotherapy
C Long-distance gamma therapy
D Chemotherapy
E Surgical treatment

194
A 27-year-old patient complains of nasal haemorrhages, multiple bruises on the anterior surface
of the trunk and extremities, sudden weakness. In blood: Hb- 74 g/l, reticulocytes - 16%, RBCs
- 2,5x1012/l, platelets – 30x109/l, ESR- 25 mm/h. What is the most effective measure for the
treatment of thrombocytopenia?
A Splenectomy
B Iron preparations
C Hemotransfusion
D Cytostatics
E Vitamin B12

195
2 days ago a patient presented with acute pain in the left half of chest, general weakness, fever
and headache. Objectively: between the 4 and 5 rib on the left the skin is erythematous, there
are multiple groups of vesicles 2-4 mm in diameter filled with transparent liquid. What diease are
these symptoms typical for?
A Herpes zoster
B Pemphigus
C Herpes simplex
D Streptococcal impetigo
E Herpetiform Duhring's dermatosis

196
A woman while working in vegetable garden developed severe pain in the loin. Lasague's and
Nery tension signs are obviously marked on the right. Lumbar lordosis is smoothed, movements
are harshly restrained in lumbar part of the spine. Right ankle (Achilles) reflex is absent. What
kind of disease can it be?
A Lumbar-sacral radiculitis
B Lumbalgia
C Hepatic colic
D Renal colic
E Neuritis of femoral nerve

197
After lifting a load a patient felt undurable pain in the loin. He was diagnosed with acute
lumbosacral radiculitis. Which of the following is contraindicated for this patient?
A Warming procedures
B Dehydrating drugs
C Analgetics
D Vitamins of B group
E Intravenous injection of aminophylline

198
An unconscious 35-year-old patient has been delivered by an ambulance to the intensive care
unit. Objectively: the patient is in semicoma. Moderate mydriasis is present. The reaction of
pupils to light is reduced. The reaction to verbal instructions is missing. AP is150/100 mm Hg,
there is tachycardia. Blood contains methanol. What antidote should be administered?
A Ethanol
B Unithiol
C Thiamine chloride
D Tavegil
E Naloxone

199
After lifting a load, a 36-year-old male patient has experienced a severe pain in the lumbar
region, which spread to the right leg and was getting worse when he moved his foot or coughed.
Objectively: the long back muscles on the right are strained. Achilles jerk is reduced on the right.
There is a pronounced tenderness of paravertebral points in the lumbar region. The straight leg
raise (Lasegue's sign ) is positive on the right. What additional tests should be performed in the
first place?
A Radiography of the spinal column
B Computed tomography
C Magnetic resonance tomography
D Electromyography
E Lumbar puncture

200
A female, aged 20, after smoking notices a peculiar inebriation with the feeling of burst of
energy, elation, irreality and changing of surroundings: the world gets full of bright colours, the
objects change their dimensions, people's faces get cartoon features, loss of time and space
judgement. What is the most likely diagnosis?
A Cocainism
B Morphinism
C Barbiturism
D Nicotinism
E Cannabism

201
A 75 y.o patient can not tell the month, date and season of the year. After long deliberations she
manages to tellher name. She is in irritable and dissatisfied mood. She always carries a bundle
with belongings with her, hides a parcel with bread, shoes in her underwear in her bosom as well
as "invaluable books". What is the most probable diagnosis?
A Senile dementia
B Atherosclerotic (lacunar) dementia
C Presenile melancholia
D Behaviour disorder
E Dissociated personality (psychopathy)

202
A 17-year-old male patient consulted a therapist about malaise, chills, runny nose, aching
muscles and joints, nausea and diarrhea. The patient asks to prescribe him a lot of painkillers
and sedatives (tramadol or solpadein that help the best, and diazepam). Pharyngeal mucosa is
pale pink, clean. Auscultation reveals vesicular breathing. Tachycardia is present. The pupils
are dilated, there is sluggish response to light. There are injection marks on the forearm skin.
During examination, the patient's manner is vulgar, irritable, rude and untruthful. Make a
diagnosis:
A Opioid addiction
B Painkillers addiction
C Sedative drug addiction
D Acute respiratory disease
E Food-born toxic infection

203
While lifting a heavy load a 39-year-old patient suddenly felt a severe headache, pain in the
interscapular region, and started vomiting. Objectively: the pulse is rhythmic, 60/min, AP-
180/100 mm Hg. The patient is agitated. He presents with photophobia, hyperacusis. There are
positive Kernig's and Brudzinski's signs on both sides. In blood: WBCs – 10x109/l. CSF
is bloody, cytosis is 240/3. What is the most likely diagnosis?
A Subarachnoid haemorrhage
B Sympathoadrenal crisis
C Acute hypertonic encephalopathy
D Meningococcal meningitis
E Ischemic stroke

204
A 26-year-old male patient complains of piercing pain during breathing, cough, dyspnea.
Objectively: to- 37,3oC, respiration rate - 19/min, heart rate = Ps- 92/min; AP- 120/80
mm Hg. Vesicular respiration. In the inferolateral parts of chest auscultation in both inspiration
and expiration phase revealed noise that was getting stronger at phonendoscope pressing and
can be still heard after cough. ECG showed no pathological changes. What is the most likely
giagnosis?
A Acute pleuritis
B Intercostal neuralgia
C Subcutaneous emphysema
D Spontaneous pneumothorax
E Pericarditis sicca

205
A 45-year-old patient, a sailor, was hospitalized on the 2nd day of the disease. A week ago he
returned from India. Complains of body temperature of 41oC, severe headache, dyspnea,
cough with frothy rusty sputum. Objectively: the patient is pale, mucous membranes are
cyanotic, breathing rate is 24/min, tachycardia is present. In lungs: diminished breath sounds,
moist rales over both lungs, crepitation. What is the most likely diagnosis?
A Pneumonic plaque
B Miliary tuberculosis
C Influenza
D Ornithosis
E Sepsis

206
HIV displays the highest tropism towards the following blood cells:
A T-helpers
B T-suppressors
C T-killers
D Thrombocytes
E Erythrocytes

207
A 25-year-old patient complains of general weakness, dry cough, sweating, subfebrile
temperature. Objectively: lung auscultation reveals vesicular resiration with no wheezing.
Fluorogram shows focal shadows of high intensity in the 1-2 segments of the right lung. Mantoux
test gave a reaction of 16 mm of induration. What clinical form of tuberculosis is most likely?
A Focal
B Infiltrative
C Disseminated
D Tuberculoma
E Miliary

208
A 22-year-old patient is a clerk. His working day runs in a conditioned room. In summer he was
taken by an acute disease with the following symptoms: fever, dyspnea, dry cough, pleural pain,
myalgia, arthralgia. Objectively: moist rales on the right, pleural friction rub. X-ray picture showed
infiltration of the inferior lobe. In blood: WBC – 11x109/l, stab neutrophils - 6%, segmented
neutrophils - 70%, lymphocytes - 8%, ESR - 42 mm/h. What is the ethiological factor of pneumonia?
A Legionella
B Mycoplasm
C Streptococcus
D Staphylococcus
E Pneumococcus

209
A 43-year-old female patient complains of dyspnea, swelling of legs, abdomen enlargement,
pricking heart pain. She has a history of tuberculous bronchadenitis, quinsies. The patient's
condition deteriorated 6 months ago. Objectively: cyanosis, bulging neck veins, vesicular
breathing. Heart borders are not displaced. Heart sounds are muffled, Ps is 106/min, liver is +4
cm, ascites is present. Low voltage on the ECG has been revealed. Radiograph shows a thin
layer of calcium deposits along the left contour of heart. What treatment should be
recommended to the patient?
A Treatment by a cardiac surgeon
B Digitalis preparations
C Anti-TB drugs
D Diuretics
E Vasodilators, nitrates

210
A 53-year-old female patient complains of cardiac pain and rhythm intermissions. She has
experienced these presentations since childhood. The patient's father had a history of cardiac
arrhythmias. Objectively: the patient is in grave condition, Ps- 220 bpm, AP- 80/60 mm Hg. ECG
results: heart rate - 215/min, extension and deformation of QRS complex accompanied by
atrioventricular dissociation; positive P wave. Some time later heart rate reduced down to
45/min, there was a complete dissociation of P wave and QRST complex. Which of the
following will be the most effective treatment?
A Implantation of the artificial pacemaker
B $\beta$-adrenoreceptor blocking agents
C Cholinolytics
D Calcium antagonists
E Cardiac glycosides

211
A 26-year-old female patient has an 11-year history of rheumatism. Four years ago she
suffered 2 rheumatic attacks. Over the last 6 months there have been paroxysms of atrial
fibrillation every 2-3 months. What option of antiarrhythmic therapy or tactics should be proposed?
A Prophylactic administration of cordarone
B Immediate hospitalization
C Defibrillation
D Lidocaine administration
E Heparin administration

212
A 49-year-old patient complains of dyspnea, cough. There are no sputum discharges. He has
repeatedly used salbutamol and intal but with no effect. Objectively: he is only able to sit while
leaning on the table. Cyanosis of face, acrocyanosis are present. Breathing is shallow,
laboured, in some parts it cannot be auscultated; there are diffuse rales, expiration is
significantly prolonged. Heart sounds are muffled, tachycardia is present. Ps - 112/min, AP-
110/70 mm Hg. Liver is located near the costal arch. There are no peripheral edemata. What is
your provisional diagnosis?
A Status asthmaticus
B Chronic obstructive bronchitis
C Bronchiale asthma, moderate gravity
D Foreign object aspiration
E Cardiac asthma
213
Thrombosis of the coronary artery caused myocardial infarction. What mechanisms of injury will
be the dominating ones in this disease?
A Calcium mechanisms
B Electrolytoosmotic mechanisms
C Acidotic mechanisms
D Protein mechanisms
E Lipid mechanisms

214
On the second day of the disease a 22-year-old male patient complains of high-grade fever,
headache in the region of forehead and superciliary arches, and during eye movement; aching
muscles and joints. Objectively: body temperature is 39oC. Face is hyperemic, sclerae are
injected. The mucous membrane of the soft palate and posterior pharyngeal wall is bright
hyperemic and has petechial hemorrhages. What changes in the hemogram are typical for this
disease?
A Leukopenia
B Leukocytosis
C Neutrocytosis
D Anemia
E Accelerated ESR

215
A female patient consulted a dermatologist about the rash on the trunk and extremities.
Objectively: interdigital folds, flexor surfaces of wrists and navel region are affected with pairs of
nodulo-cystic eruptions and crusts. The rash is accompanied by skin itch that is getting stronger
at night. What external treatment should be administered?
A 20% benzyl benzoate emulsion
B 5% sulfuric ointment
C 2% sulfuric paste
D 5% naphthalan ointment
E 5% tetracycline ointment

216
A 54 y.o. male patient suffers from dyspnea during mild physical exertion, cough with sputum
which is excreted with diffculty. On examination: diffuse cyanosis. Is Barrel-chest. Weakened
vesicular breathing with prolonged expiration and dry whistling rales. AP is 140/80 mm Hg, pulse
is 92 bpm, rhythmic. Spirography: vital capacity (VC)/predicted vital capacity- 65%, FEV1/FVC–
50%. Determine the type of respiratory insufficiency (RI).
A RI of mixed type with prevailing obstruction
B RI of restrictive type
C RI of obstructive type
D RI of mixed type with prevailing resriction
E There is no RI

217
An ambulance had been called to a 48-year-old man. According to his relatives, the patient had
had three attacks of unconsciousness accompanied by convulsions within 24 hours. On
examination the doctor witnessed the following attack: the patient lost consciousness and fell to
the floor, developed tonic, then clonic convulsions of trunk and extremities. The attack lasted 1
minute and ended with involuntary urination. Specify the kind of attack:
A Epileptic seizure
B Syncope
C Vegetative crisis
D Coma
E Attack of hysteria
218
A patient is 60 years old, retired, worked as deputy director of a research institute. Behavioural
changes appeared 2 years ago after the death of her husband: she stopped looking after
herself and leaving the house; then she refused to clean the apartment and cook. Mental status:
temporal disorientation. The patient does not understand many of the questions, is confused;
does not know how to cook soup or fasten a button. Her speech is characterized by stumbling
and logoclonia. She does not recognize doctors, fellow patients. She cries a lot but can not
explain the reason for tears. What is the mechanism of this pathology?
A Atrophy of the cerebral cortex
B Atherosclerotic changes in cerebral vessels
C Serotonin deficiency
D Impaired conversion of dopamine to noradrenaline
E Disorder of melatonin metabolism

219
A 26-year-old patient has abused alcohol since the age of 16, needs a morning-after drink to
cure hangover. He takes alcohol nearly every day, "a little at a time". Twice a week he gets
severely drunk. The patient works as a motor mechanic, over the last 2 years work conflicts
have become more frequent. What medical and tactical actions should be taken in this case?
A Voluntary consultation and treatment at an addiction clinic
B Compulsory treatment
C Referral to treatment at an activity therapy centre
D Referral to medical-social expert commission for assessment of his working ability
E Consultation with a psychologist

220
A 47-year-old female patient has an 8-year history of ulcerative colitis, has been treated with
glucocorticoids. She complains of cramping pain in the umbilical region and left iliac region which
has significantly increased during the past 2 weeks, diarrhea with mucus and blood 4-6 times a
day, elevated body temperature up to 38-39oC, headache and pain in the knee joints.
Objectively: the patient is in moderate condition, Ps - 108/min, AP - 90/60 mm Hg; heart and
lungs are unremarkable; the tongue is moist; abdominal muscle tone is significantly decreased;
peristaltic noises are absent. What complication developed in the patient?
A Toxic dilatation of the colon
B Perforation of the colon
C Enterorrhagia
D Stricture of the colon
E Colon carcinoma

221
A 26-year-old patient with left lower lobe pneumonia experiences an acute chest pain on the left
during coughing. Objectively: diffuse cyanosis, extension of the left side of chest. Percussion
reveals high tympanitis. Auscultation reveals no respiratory murmurs above the left side of
chest. There is a deviation of the right cardiac border towards the midclavicular line. What
examination will be the most informative?
A X-Ray
B Bronchoscopy
C Bronchography
D Pneumotachometry
E Spirography

222
A male patient presents with swollen ankles, face, eyelids, elevated AP- 160/100 mm Hg, pulse-
54 bpm, daily loss of albumine with urine- 4g. What therapy is pathogenetic in this case?
A Corticosteroids
B Diuretics
C NSAID
D Calcium antagonists
E Antibiotics

223
After myocardial infarction, a 50-year-old patient had an attack of asthma. Objectively: bubbling
breathing with frequency of 32/min, cough with a lot of pink frothy sputum, acrocyanosis, swelling
of the neck veins. Ps- 108/min, AP- 150/100 mm Hg. Heart sounds are muffled. Mixed moist
rales can be auscultated above the entire lung surface. What drug would be most effective in
this situation?
A Nitroglycerin intravenously
B Pentamin intravenously
C Strophanthin intravenously
D Dopamine intravenously
E Aminophylline intravenously

224
During dynamic investigation of a patient the increase of central venous pressure is combined
with the decrease of arterial pressure. What process is proved by such combination?
A Increase of bleeding speed
B Developing of cardiac insufficiency
C Shunting
D Depositing of blood in venous channel
E Presence of hypervolemia

225
A male patient complains of heartburn which gest stronger while bending the body, substernal
pain during swallowing. There is a hiatus hernia on X-ray. What disoeder should be expected at
gastroscopy?
A Gastroesophageal reflux
B Chronic gastritis
C Gastric peptic ulcer
D Acute erosive gastritis
E Duodenal peptic ulcer

226
A 43 y.o. male complains of stomach pain, which relieves with defecation, and is accompanied
by abdominal winds, rumbling, the feeling of incomplete evacuation or urgent need for bowel
movement, constipation or diarrhea in alternation. These symptoms have lasted for over 3
months. No changes in laboratory tests. What is the most likely diagnosis?
A Irritable bowel syndrome
B Spastic colitis
C Colitis with hypertonic type dyskinesia
D Chronic enterocolitis, exacerbation phase
E Atonic colitis

227
Against the background of angina a patient has developed pain in tubular bones. Examination
revealed generalized enlargement of lymph nodes, hepatolienal syndrome, sternalgia. In blood:
RBCs - 3,6x1012/l, Hb- 87 g/l, thrombocytes – 45x109/l, WBCs – 13x109/l, blasts - 87%, stab
neutrophils - 1%, segmented neutrophils - 7%, lymphocytes - 5%, ESR - 55 mm/h. What is the most
likely diagnosis?
A Acute leukemia
B Erythremia
C Chronic lymphocytic leukemia
D Chronic myeloid leukemia
E Multiple myeloma
228
A 49-year-old female patient with schizophrenia is all the time listening to something, insists that
"there is a phone in her head" as she hears the voice of her brother who tells her to go home.
The patient is anxious, suspicious, looks around all the time. Specify the psychopathological
syndrome:
A Hallucinatory
B Generalized anxiety disorder
C Paranoiac
D Paraphrenic
E Depressive

229
A 43-year-old male patient undergoing treatment for peptic ulcer complains of weakness,
dizziness, coffee-ground vomiting, melena. After administration of haemostatics the patient's
condition has not improved, fresh blood has shown up in the vomit, skin bruises of different
sizes have appeared. In blood: thrombocytes – 50x109/l, Lee-White clotting time - 35
minutes, APTT - 80 seconds. In this case it is most rational to administer the following
preparation:
A Fresh frozen plasma
B Heparin
C Fibrinogen
D Rheopolyglucinum
E Vikasol

230
A 38-year-old patient complains of inertness, subfebrile temperature, enlargement of lymph
nodes, nasal haemorrhages, bone pain. Objectively: the patient's skin and mucous membranes
are pale, palpation revealed enlarged painless lymph nodes; sternalgia; liver was enlarged by 2
cm, spleen - by 5 cm, painless. In blood: erythrocytes - 2,7x1012/l, Hb- 84 g/l, leukocytes – 58x109/l,
eosinophils - 1%, stab neutrophils - 2%, segmented neutrophils - 12%, lymphocytes - 83%,
lymphoblasts - 2%, smudge cells; ESR- 57 mm/h. What is the most likely diagnosis?
A Chronic lymphatic leukemia
B Chronic myeloleukemia
C Acute lymphatic leukemia
D Acute myeloleukemia
E Lymphogranulomatosis

231
A 30-year-old male patient complains of inertness, low-grade fever, bleeding gums, frequent
quinsies, aching bones. Objectively: the patient has pale skin and mucous membranes,
sternalgia, +2 cm liver, +5 cm painless spleen. Blood test results: RBC - 2,7x1012/l, Нb
- 80 g/l, WBC – 3x109/l, eosinophils - 4%, basophils - 5%, blasts - 4%, stab neutrophils
- 2%, segmented neutrophils - 17%, lymphocytes - 29%, myelocytes - 25%, promyelocytes -
12%, monocytes - 2%, platelets – 80x109/l, ESR - 57 mm/h. What test should be performed to
verify the diagnosis?
A Sternal puncture
B Trephine biopsy
C Lymph node biopsy
D Lumbar puncture
E Chest X-ray

232
A 24-year-old patient complains about putting on weight, limosis. Objectively: the patient's
constitution is of hypersthenic type, body weight index is 33,2 kg/m2, waist circumference is
100 cm. Correlation of waist circumference to the thigh circumference is 0,95. What is the most
likely diagnosis?
A Alimentary constitutional obesity of the I stage, abdominal type
B Hypothalamic Itsenko-Cushing obesity of the II stage, gynoid type
C Alimentary constitutional obesity of the III stage, gynoid type
D Alimentary constitutional obesity of the II stage, abdominal type
E Hypothalamic Itsenko-Cushing obesity of the I stage, abdominal type

233
A 47-year-old male patient has been lately complaining of compressing chest pain that occurs
when he walks a distane of 700-800 m. Once a week, he drinks 2 liters of beer. Rise in arterial
pressure has been observed for the last 7 years. Objectively: Ps - 74/min, AP - 120/80 mm Hg.
The bicycle ergometry performed at workload of 75 watts shows 2 mm ST-segment
depression in V4-V6 leads. What is the most likely diagnosis ?
A Exertional stenocardia, II functional class
B Exertional stenocardia, III functional class
C Exertional stenocardia, IV functional class
D Vegetative-vascular dystonia of hypertensive type
E Alcoholic cardiomyopathy

234
A 58-year-old patient complains about sensation of numbness, sudden paleness of II-IV fingers,
muscle rigidness, intermittent pulse. The patient presents also with polyarthralgia, dysphagia,
constipations. The patient's face is masklike, solid edema of hands is present. The heart is
enlarged; auscultation revealed dry rales in lungs. In blood: ESR- 20 mm/h, crude protein - 85/l,
γ-globulines - 25%. What is the most likely diagnosis?
A Systemic scleroderma
B Dermatomyositis
C Rheumatoid arthritis
D Systemic lupus erythematosus
E Raynaud's disease

235
A 45-year-old man has been exhibiting high activity for the last 2 weeks, he became talkative,
euphoric, had little sleep, claimed being able "to save the humanity" and solve the problem of
cancer and AIDS, gave money the starangers. What is the most likely diagnosis?
A Maniacal onset
B Panic disorder
C Agitated depression
D Schizo-affective disorder
E Catatonic excitation

236
A patient had four generalized convulsive seizures within a day. Between the seizures the
patient did not come to waking consciousness (was in a coma or stupor). Specify his state:
A Status epilepticus
B Frequent generalized seizures
C Frequent jacksonian seizures
D Hysterical attacks
E Frequent complex partial seizures

237
A 35-year-old patient complains of heartburn, sour eructation, burning, compressing retrosternal
pain and pain along the esophagus rising during forward bending of body. The patient hasn't
been examined, takes Almagel on his own initiative, claims to feel better after its taking. Make a
provisional diagnosis:
A Gastroesophageal reflux disease
B Functional dyspepsia
C Cardiospasm
D Gastric ulcer
E Duodenal ulcer

238
As a result of lifting a load a 62-year-old female felt acute pain in the lumbar region, in a buttock,
posterolateral surface of her right thigh, external surface of the right shin and dorsal surface of
foot. Objectively: weakness of the anterior tibial muscle, long extensor muscle of the right toes,
short extensor muscle of the right toes. Low Achilles reflex on the right. Positive Lasegue's sign.
What examination method would be the most effective for specification of the diagnosis of
discogenic compression of L5 root?
A Magnetic resonance scan
B Spinal column X-ray
C Electromyography
D Angiography
E Lumbar puncture

239
A 45-year-old female patient complaining of general weakness, nausea and vomiting hass been
delivered to a hospital by the ambulance. Recently there has been a lack of appetite, weight
loss. Objectively: hyperpigmentation of skin, blood pressure at the rate of 70/45 mm Hg,
bradycardia. Additional studies revealed the reduced concentration of aldosterone and cortisol
in blood, decreased excretion of 17-ketosteroids and 17-oxyketosteroids in the urine,
hyponatremia, chloropenia, hypokalemia. What therapeutic measures are required?
A To administer glucocorticoids, mineralocorticoids, and a diet with a high content of cooking salt
B To prescribe a diet with a high content of cooking salt
C To administer prednisolone
D To administer aldosterone
E To administer insulin

240
A 23-year-old female patient has a mental disease since the age of 18, the course of disease
has no remission periods. At a hospital the patient mostly presents with non-purposeful foolish
excitation: she makes stereotypic grimaces, exposed, masturbating in front of a loud laugh,
repeating the stereotypical abusive shouts. The patient should be assigned:
A Neuroleptics
B Antidepressants
C Tranquilizers
D Nootropics
E Mood stabilizers

241
A 40-year-old patient is registered in a narcological dispensary. Somatically: skin is dramatically
hyperemic, sclera are injected, hyperhidrosis is present. AP- 140/100 mm Hg, heart rate -
100/min. Mental state: autopsychic orientation is intact, allopsychic orientation is distorted. The
patient presents with motor anxiety. There is a look of fear on his face. He refuses to talk about
his problems and asks to release him immediately, because he "may be killed." This state
developed a day after a regular drinking bout. What is your provisional diagnosis?
A Delirium tremens
B Organic delirium
C Paranoia
D Alcoholic hallucinosis
E Alcoholic paranoid

242
During the preventive examination a 17-year-old young man reports no health problems.
Objectively: the patient is undernourished, asthenic; blood pressure is 110/70 mm Hg, Ps -
80/min. Heart borders are within normal range. Auscultation reveals three apical heart sounds,
murmurs are absent. ECG shows no pathological changes, PCG registers the S3 occurring
0,15 seconds after the S2. How can you interpret these changes?
A Physiologic S3
B Fout-ta-ta-rou (three-component rhythm)
C Protodiastolic gallop rhythm
D Presystolic gallop rhythm
E Physiologic S4

243
A patient is being prepared for the operation on account of varix dilatation of lower extremities
veins. Examination of the patient's soles revealed flour-like desquamation along the skin folds.
All the toenails are greyish-yellow, thickened and partially decayed. What dermatosis should be
suspected?
A Rubromycosis
B Pityriasis versicolor
C Candidosis
D Microsporia
E Microbial eczema

244
A 14-year-old patient with signs of internal haemorrhage has been taken to a hospital after a
fight. He has had haemophilia A since childhood. He has been diagnosed with retroperitoneal
hematoma. What should be administered in the first place?
A Cryoprecipitate
B Aminocapronic acid
C Dried plasma
D Platelet concentrate
E Fresh blood

245
A 58-year-old patient complains of a headache in the occipital region, nausea, choking,
opplotentes. The presentations appeared after a physical exertion. Objectively: the patient is
excited. Face is hyperemic. Skin is pale. Heart sounds are regular, the 2nd aortic sound is
accentuated. AP- 240/120 mm Hg, HR- 92/min. Auscultation reveals some fine moist rales in
the lower parts of the lungs. Liver is not enlarged. ECG shows signs of hypertrophy and left
ventricular overload. What is the most likely diagnosis?
A Complicated hypertensic crisis, pulmonary edema
B Acute myocardial infarction, pulmonary edema
C Bronchial asthma exacerbation
D Uncomplicated hypertensic crisis
E Community-acquired pneumonia

246
A 37-year-old patient complains of pain in the lumbar spine that is getting stronger during
walking; restricted mobility, edema of the right side of abdomen. He has a history of focal
tuberculosis. X-ray picture shows the destruction of the adjacent surfaces of the 1-2 vertebral
bodies of the lumbar spine, vertebral body height is decreased, intervertebral foramen is
undetectable. Abdominal ultrasound reveals a 15x20 cm formation in the retroperitoneal space,
there are echo signals of fluid presence. What is the most likely diagnosis?
A Tuberculous spondylitis of the lumbar spine
B Fracture of the 1-2 vertebral bodies of the lumbar spine
C Spinal metastases
D Spondylolisthesis of the lumbar spine
E Osteochondrosis

247
A 43-year-old female patient was delivered to the hospital in grave condition. She has a history
of Addison's disease. The patient had been regularly taking prednisolone but a week before she
stopped taking this drug. Objectively: sopor, skin and visible mucous membranes are pigmented,
skin and muscle turgor is decreased. Heart sounds are muffled, rapid. AP- 60/40 mm Hg, heart
rate - 96/min. In blood: Na - 120 millimole/l, K - 5,8 millimole/l. Development of this complication
is primarily caused by the deficit of the following hormone:
A Cortisol
B Corticotropin (ACTH)
C Adrenaline
D Noradrenaline
E Adrostendion

248
In a cold weather, the emergency room admitted a patient pulled out of the open water. There
was no respiratory contact with the water. The patient is excited, pale, complains of pain,
numbness of hands and feet, cold shiver. Respiratory rate is 22/min, AP - 120/90 mm Hg, Ps -
110/min, rectal temperature is 34,5oC. What kind of warming is indicated for this patient?
A Passive warming
B Infusion of 37oC solutions
C Hot compresses
D Warm bath
E Hemodialysis with blood warming

249
Survey radiograph of a 52-year-old worker of an agglomeration plant (28 years of experience,
the concentration of metal dust is 22-37 mg/m3) shows mildly pronounced interstitial fibrosis
with diffused contrast well-defined small nodular shadows. The patient has no complaints.
Pulmonary function is not compromised. What is the provisional diagnosis?
A Siderosis
B Silicosis
C Anthraco-silicatosis
D Silicatosis
E Anthracosis

250
A 60-year-old patient complains of nearly permanent sensation of heaviness and fullness in the
epigastrium, that increases after eating, foul-smelling eructation, occasional vomiting with food
consumed 1-2 days ago, weight loss. 12 years ago he was found to have an ulcer of pyloric
channel. The patient has taken ranitidine for periodic hunger pain. The patient's condition has
been deteriorating over the last 3 months. Objectively: splashing sound in the epigastrium is
present. What kind of complication is it?
A Pyloric stenosis
B Penetration of gastric ulcer
C Functional pyloric spasm
D Foreign body in the stomach (bezoar)
E Malignization of gastric ulcer

251
A 52-year-old patient works as a secretary and has 30 year record of service. She complains
of spasms in her right hand during working and inability to type and write. Up to 80% of her work
involves hand load. The patient has been presenting with these symptoms for 2 years.
Objectively: the right hand is tense, there is an increase in muscle tone, attempts to write cause
spasms. Examination revealed no pathological changes of CNS. What is the most likely diagnosis?
A Spastic form of coordination neurosis
B Neuralgic form of coordination neurosis
C Paretic form of coordination neurosis
D Hysteric neurosis
E Chronic manganese intoxication
252
Examination of an electric welder with 15 years of service record revealed dry rales in the lower
lung fields. Radiograph shows diffuse nodules sized 3-4 mm in the middle and lower lung fields.
What disease can be suspected?
A Heavy-metal coniosis
B Silicosis
C Silicatosis
D Carbon pneumo coniosis
E Bronchitis

253
A 22-year-old vegetarian patient with signs of malnutrition consulted a doctor about smell and
taste distortion, angular stomatitis. Objectively: expressively blue sclerae. The patient was
diagnosed with iron deficiency anemia. What is the dominating clinical syndrome?
A Sideropenic
B Anaemic
C Haemologic
D Haemolytic
E Myelodysplastic

254
A patient complains of retrosternal pain, difficult swallowing, over 10 kg weight loss within three
months, general weakness. In blood: hypochromic anaemia, neutrophilic leukocytosis. In feces:
weakly positive Gregersen's reaction. On esophagram a filling defect with ill-defined serrated
edges shows up along a large portion of the esophagus. What is the most likely diagnosis?
A Esophageal carcinoma
B Benign tumour
C Esophageal achalasia
D Peptic ulcer
E Sideropenic dysphagia

255
A 12-year-old boy periodically has short episodes (10-15 seconds) of a brief loss of awareness
with a dazed look and eyes stare in an upright position, blank expression of face, absence of
motions and subsequent amnesia. Specify the described state:
A Absence seizure
B Obnubilation
C Trance
D Fugue
E Sperrung

256
A 19-year-old male patient complains of intense pain in the left knee joint. Objectively: the left
knee joint is enlarged, the overlying skin is hyperemic, the joint is painful on palpation. Blood test
results: RBC - 3,8x1012/l, Hb - 122 g/l, lymphocytes - 7,4x109/l, platelets – 183x109/l. ESR - 10 mm/h.
Duke bleeding time is 4 minutes, Lee-White clotting time - 24 minutes. A-PTT is 89 s. Rheumatoid
factor is negative. What is the most likely diagnosis?
A Hemophilia, hemarthrosis
B Werlhof's disease
C Rheumatoid arthritis
D Thrombocytopathy
E Hemorrhagic vasculitis, articular form

257
Explosion of a tank with benzene at a chemical plant has killed and wounded a large number of
people. There are over 50 victims with burns, mechanical injuries and intoxication. Specify the
main elements of medical care and evacuation of population in this situation:
A Sorting, medical assistance, evacuation
B Sorting, evacuation, treatment
C Medical assistance, evacuation, isolation
D Isolation, rescue activity, recovery
E Sorting, recovery, rescue activity

258
An emergency doctor has diagnosed a 32-year-old woman with generalized convulsive status
epilepticus. The deterioration in the patient's condition is caused by a sudden gap in the epilepsy
treatment. Specify the doctor's further tactics:
A Hospitalization in the intensive care unit
B Hospitalization in the department of neurology
C Hospitalization in the department of neurosurgery
D Outpatient monitoring by a neuropathologist
E Outpatient monitoring by a neurosurgeon

259
A 63-year-old male patient with persistent atrial fibrillation complains of moderate dyspnea.
Objectively: peripheral edemata are absent, vesicular breathing is present, heart rate - 72/min,
AP - 140/90 mm Hg. What combination of drugs will be most effective for the secondary
prevention of heart failure?
A Beta-blockers, ACE inhibitors
B Beta-blockers, cardiac glycosides
C Cardiac glycosides, diuretics
D Cardiac glycosides, ACE inhibitors
E Diuretics, beta-blockers

260
A 57-year-old male patient had an attack of retrosternal pain that lasted more than 1,5 hours.
Objectively: the patient is inert, adynamic, has pale skin, cold extremities, poor volume pulse,
heart rate - 120/min, AP - 70/40 mm Hg. ECG shows ST elevation in II, III, aVF leads. What
condition are these changes typical for?
A Cardiogenic shock
B Arrhythmogenic shock
C Perforated gastric ulcer
D Acute pericarditis
E Acute pancreatitis

261
A 42-year-old female lives in the basement, is unemployed, undernourished. She complains of
having general weakness, hair loss, brittle nails for six months, likes to eat chalk. Objectively: the
patient is emaciated, pale, has dry skin. Peripheral lymph nodes are not enlarged. Liver is +1,5
cm. In blood: RBCs - 1,8x1012/l, Hb- 62 g/l, colour index - 0,78, reticulocytes - 0,5 o/oo, ESR- 18
mm/h. Leukogram exhibits no pathology. What is a provisional diagnosis?
A Nutritional iron deficiency anaemia
B Chronic hepatitis
C B12-deficiency anaemia
D Acquired haemolytic anaemia
E Congenital haemolytic anaemia

262
A 20-year-old patient complains of breath shortness, continuous dull heart pain, irritability.
Objectively: general condition is satisfactory, the pulse is labile, AP- 130/60 mm Hg. ECG shows
repolarization disorder. The patient has been diagnosed with cardiac-type neurocirculatory
dystonia. The patient should receive treatment under the following conditions:
A Outpatient treatment
B Inpatient treatment at the therapeutic department
C Inpatient treatment at the cardiology department
D Inpatient treatment at the cardiac surgery department
E Inpatient treatment at the psychiatric department

263
A 45-year-old male patient complains of acute pain in his right side irradiating to the right thigh
and crotch. The patient claims also to have frequent urination with urine which resembles a meat
slops. The patient has no previous history of this condition. There is costovertebral angle
tenderness on the right (positive Pasternatsky's symptom). What is the most likely diagnosis?
A Urolithiasis
B Acute appendicitis
C Acute pyelonephritis
D Acute cholecystitis. Renal colic
E Acute pancreatitis

264
A 38-year-old male works within the range of ionizing radiation. At a routine medical examination
he presents no problems. In blood: RBCs - 4,5x1012/l, Hb- 80 g/l, WBCs - 2,8x109/l, thrombocytes –
30x109/l. Decide if this person can work with sources of ionizing radiation:
A Working with radioactive substances and other sources of ionizing radiation is contraindicated
B The patient is allowed to work with radioactive substances
C The patient can only work with radioactive substances of low activity
D The patient can be allowed to work after an extended medical examination
E The patient is allowed to work with radioactive substances for the limited period of time

265
A patient who undergoes treatment at a tuberculosis clinic has complained of having
progressing headache for the last 3 weeks. Neurological examination reveals rigidity of occipital
muscles, no focal symptoms. What is your provisional diagnosis?
A Tuberculous meningitis
B Chorea minor
C Brain tumour
D Myelitis
E Convexital arachnoiditis

266
A patient with chronic suppurative otitis has developed severe headache, vomiting, body
temperature rise. The meningeal symptoms are present. There are no focal neurological
symptoms. The further tactics of a doctor should be:
A Urgent hospitalization and diagnostic lumbar puncture
B Skull radiography
C Regular medical check-up
D Administration of anti-inflammatory drugs
E Referral for a consultation with otolaryngologist

267
A 28-year-old male patient complains of sour regurgitation, cough and heartburn that occurs
every day after having meals, when bending forward or lying down. These problems have been
observed for 4 years. Objective status and laboratory values are normal. FEGDS revealed
endoesophagitis. What is the leading factor in the development of this disease?
A Failure of the lower esophageal sphincter
B Hypersecretion of hydrochloric acid
C Duodeno-gastric reflux
D Hypergastrinemia
E Helicobacter pylori infection
268
Routine examination of a 16-year-old boy revealed the presence of three heart sounds on
auscultation. The third sound is low and occurs in early diastole, there is no additional murmur.
In history: pneumonia six months ago. The patient presents no problems. Examination revealed
hyposthenia, underdevelopment of muscles. Laboratory and instrumental studies reveald no
peculiarities. What is the origin of the additional heart sound?
A Physiological III sound
B The sound of the mitral valve opening
C Protodiastolic gallop rhythm
D Pericardial diastolic sound
E The sound of the tricuspid valve opening

269
A patient's condition is getting worse towards evening: she becomes excited, complains of
"internal anxiety", "a weight on her heart", foreboding of evil - "something evil will happen to me
or my family". The patient is sad, melancholic, has poor appetite and sleep disorders. Specify
the kind of mental disorder:
A Anxious depression
B Somatized depression
C Endogenous depression
D Hypochondriac depression
E Agitated depression

270
A 30-year-old male patient had been admitted to the TB hospital because of the following
changes detected by fluorography: an ill-defined shadow of low intensity up to 1 cm in diameter
in the S1 of the right lung. CT scan showed a destruction area in the center of the shadow.
Sputum analysis revealed MTB. The patient was diagnosed with focal tuberculosis. What
phases of tuberculosis are the identified changes typical for?
A Infiltration and disintegration
B Infiltration and dissemination
C Resorption and scarring
D Disintegration and dissemination
E Calcification and resorption

271
A 43-year-old female complains of significant weakness, sore throat, occurrence of multiple
unexplained bruises on her skin. These symptoms have been present for a week, the disease is
associated with quinsy which she had some time before. Objectively: body temperature -
38,9oC, respiratory rate - 24/min, Ps - 110/min, AP - 100/65 mm Hg. The patient has pale
skin, petechial rash on the extremities, enlarged lymph nodes. Blood test results: Hb - 80 g/l,
RBC - 2,2x1012/l; WBC - 3,5x109/l; blasts - 52%; eosinophils - 2%; stab neutrophils - 3%; segmented
neutrophils - 19%; lymphocytes - 13%; monocytes - 1%; platelets – 35x109/l. ESR - 47 mm/h. What
test is required to specify the diagnosis?
A Immunophenotyping
B Protein electrophoresis
C Lymph node biopsy
D Determination of anti-platelet antibody titer
E Cytogenetic study

272
A 47-year-old male patient complains of compressive chest pain that occurs both at rest and
during light physical activity; irregular heartbeat. These problems arose 3 months ago. The
patient's brother died suddenly at the age of 30. Objectively: Ps - 84/min, arrhythmic, AP -
130/80 mm Hg. ECG confirms signs of left ventricular hypertrophy, abnormal Q-waves in V4-V6
leads. EchoCG reveals that interventricular septum is 1,7 cm, left ventricular wall thickness is
1,2 cm. What is the most likely diagnosis?
A Hypertrophic cardiomyopathy
B Neurocirculatory asthenia
C Exertional angina
D Myocarditis
E Pericarditis

You might also like